You are on page 1of 504

US-MLE Step 2 CK – UW Explanations/Tables

SURGERY
//////////////////////////////GENERAL PRINCIPLES////////////////////////////////
POSTOPERATIVE
FEVER

Postoperative fever, defined as temperature >38 C (100.4 F), is common following


major surgery and is generally mediated by the release ofpyretic cytokines (eg, IL-
1, IL-6, TNF-α, IFN-γ) in response to tissue
trauma, blood cell lysis, or bacterial
endotoxins/exotoxins. The underlying
etiology can often be differentiated based on
how much time has elapsed since surgery.
Immediate postoperative fever occurs within
hours of the operation. Most cases are caused
by tissue damage incurred during a major
procedure (eg, open cholecystectomy); fever
and leukocytosis generally last <3 days and
are managed symptomatically (eg,
acetaminophen) and observed. Other causes
of immediate fever include blood transfusion
and drug reactions; these are often associated
with hypotension (due to vasodilation) and
rash.
Acute fever (1-7d) occurs within the first week after surgery and is most often caused
by nosocomial or SSI or certain noninfectious causes such as PE. Subacute (7-28 days
postoperatively) fever is usually caused by bacterial infection. Fever is delayed with
infection because several days of bacterial replication are generally necessary to
produce enough endotoxin/exotoxin to generate fever. Most acute postoperative
fevers are caused by nosocomial infections such as UTI and pneumonia, whereas
subacute postoperative fevers are often due to abscess or infection of a central line or
surgical site.

Factors that favor infection over contamination include:

• Systemic signs, such as fever, hypotension, or leukocytosis


• Erythema and tenderness at the catheter entry site (the absence of local signs
does not rule out an infection)
• Culture growth within 48 hours and in both aerobic and anaerobic bottles
• Two or more blood culture samples with the same organism and drug
susceptibility

Fever occurring immediately (within a few hours) in the operative or postoperative


period is typically due to prior infection or trauma, inflammation due to surgery,
malignant hyperthermia, or medications (eg, anesthetics) or blood products given
during or prior to surgery. Acute fever (within the first week after surgery) is most
often caused by nosocomial infections (eg, pneumonia, urinary tract infection) or other
noninfectious causes such as pulmonary embolus (PE). Subacute fever (>1 week after
surgery) is usually due to drug fever, surgical site infection, or PE. Delayed
postoperative fever is typically caused by an infection (eg, viral infections from blood
products, infective endocarditis).

Drug fever is typically a diagnosis of exclusion. It is often associated with use of


anticonvulsants, antibiotics (beta-lactams, sulfonamides), or allopurinol. Most cases of
drug fever occur 1-2 weeks after initiation of therapy and are ofthen accompanied by
rash and peripheral eosinophilia.

Malignant hyperthermia is caused by inhaled anesthetics and typically occurs


intraoperatively or in the immediate postoperative period. Patients usually have fever,
tachycardia, hypercarbia, tachypnea, severe muscle rigidity, acidosis, and
rhabdomyolysis and are at high risk for cardiac arrest and death. Urgent treatment with
dantrolene (a skeletal muscle relaxant) and cooling measures is required to prevent
death.

Nosocomial pneumonia usually occurs 48hrs after hospital admission. It is not likely if
the patient has no cough or other respiratory signs or symptoms.

SSI is possible during this time frame; however, this patient's wound is intact with no
signs of infection.
Indwelling urinary catheters increase the risk for urinary tract infection with enteric
organisms such as Escherichia coli, Klebsiella pneumoniae, and Proteus mirabilis

Postoperative atelectasis is not generally considered an independent cause of fever


but can predispose patients to developing pneumonia. Atelectasis is usually
managed/prevented with incentive spirometry, not positive airway pressure and chest
physiotherapy (these measures are primarily reserved for those with excessive
secretions that lead to respiratory plugging).

Febrile nonhemolytic and acute hemolytic transfusion reactions may cause fever, but
these reactions typically occur immediately (within 1-6 hours) after the transfusion, not
days later. When red cells and plasma are separated from whole blood, small amounts
of residual plasma and/or leukocyte debris may remain in the red cell
concentrate. During blood storage, these leukocytes release cytokines that when
transfused can cause transient fevers, chills, and malaise (without hemolysis) within 1-6
hours of transfusion. Management includes stopping the transfusion to rule out other
serious causes of fever (eg, acute hemolytic reaction) and antipyretics (avoid aspirin in
thrombocytopenic patients).

Immunologic blood transfusion reactions

Transfusion
Onset* Cause Key features
reaction

Recipient anti-IgA • Angioedema, HoTN,


Within
antibodies RD/wheezing,
Anaphylactic seconds to
directed against shock
minutes
donor blood IgA • IgA deficient recipient

ABO • Fever, flank pain,


Within incompatibility hemoglobinuria
Acute hemolytic 1 hr (often clerical • DIC
error) • Positive Coombs test

Febrile Cytokine
Within accumulation
nonhemolytic (MC 1-6 hr during blood
• Fever & chills
reaction) storage

Recipient IgE
Within against blood
Urticarial 2-3 hr product
• Urticaria
component

• Respiratory distress
Donor anti-
Within • Noncardiogenic pulmonary
TRALI 6 hr
leukocyte
edema with bilateral
antibodies
pulmonary infiltrates
• Often asx
• Laboratory evidence of
Within
Anamnestic hemolytic anemia
Delayed hemolytic days to
antibody response • Positive Coombs test,
weeks
positive new antibody
screen

Within Donor T- • Rash, fever, GI sx,


GVH weeks lymphocytes pancytopenia

*Time after transfusion initiation.

TRANSFUSION
Transfusion reactions associated with hypotension
REACTIONS
Reaction Onset* Cause Clinical features

Seconds to Recipient anti-IgA • Shock, angioedema/urticaria


Anaphylaxis minutes antibodies & respiratory distress

• Respiratory distress
& noncardiogenic
Minutes to Donor antileukocyte
TRALI hours antibodies
pulmonary edema
• Bilateral pulmonary
infiltrates

Acute Minutes to • Fever, flank


ABO incompatibility
hemolysis hours pain, hemoglobinuria & DIC

Bacterial
• Fever, chills, septic shock
Bacterial sepsis Hours contamination of
& DIC
donor product

*Time after transfusion initiation.


DIC = disseminated intravascular coagulation.

This patient developed respiratory distress, hypotension, and pulmonary infiltrates


during blood transfusion, findings concerning for transfusion-related acute lung
injury (TRALI). TRALI is an uncommon but potentially fatal transfusion reaction in
which neutrophils activated by a component in the transfused blood (eg, anti-leukocyte
antibodies) cause damage to pulmonary microvasculature. Risk factors for TRALI
include smoking, alcohol abuse, and critical illness.

Damage to the capillary endothelium results in inflammatory pulmonary edema.


Hypoxia and dyspnea develop within 6 hours after transfusion
initiation. Tachycardia, hypotension, and fever may also be present. Chest x-ray
reveals bilateral pulmonary infiltrates.

Treatment of TRALI includes immediate transfusion cessation and respiratory


supportive care; most patients require ventilatory support. Some patients recover
within 24-48 hours, but up to 50% of previously critically ill patients expire.
………..

Diuretic therapy is the treatment for transfusion-associated circulatory overload


(TACO). Patients have respiratory distress within 6 hours after transfusion initiation;
however, hypertension due to fluid overload is typically present. In addition, if a
patient has not received a significant amount of fluid, it makes TACO less likely.

In addition to intramuscular epinephrine, intravenous antihistamine administration is


indicated in anaphylactic transfusion reactions,

DISSEMINATED
Disseminated intravascular coagulation
INTRAVASCULAR
COAGULATION • Sepsis
(DIC) • Severe traumatic injury
Major causes • Malignancy
• Obstetric complications

• Procoagulant excessively triggers coagulation cascade →


Pathophysiology • Formation of fibrin-/platelet-rich thrombi & fibrinolysis →
• Bleeding & organ damage (eg, kidneys, lungs)

• Thrombocytopenia
• Prolonged PT & PTT
Laboratory
• ↓ Fibrinogen
findings • ↑ D-dimer
• Microangiopathic hemolytic anemia (schistocytes)

A patient's oozing from venipuncture and surgical sites and reduced urine output after
a serious traumatic injury raises strong suspicion for DIC, a consumptive
coagulopathy. Traumatic injury increases the risk of DIC due to endothelial (exposes
tissue factor) and tissue damage (releases procoagulant proteins and phospholipids);
other common causes of DIC include sepsis, malignancy, and obstetrical complications.
DIC is marked by:

• Overactivation of the coagulation cascade, leading to the formation of fibrin-


and platelet-rich thrombi and the consumption of coagulation factors
(prolonged PT/PTT), platelets (thrombocytopenia), and fibrinogen.
• Subsequent fibrinolysis (to break up the clots), which increases fibrin
degradation products (eg, D-dimer). Anticoagulation proteins (eg, protein
C/S) are also consumed.

Patients with acute DIC usually develop bleeding from venipuncture/surgical sites,
ecchymosis, and petechiae. Organ damage (eg, renal insufficiency [reduced urine
output in the setting of intravascular fluids]) is also commonly seen.

ABO incompatibility can cause DIC, but severe symptoms (eg, fever, chills, respiratory
difficulty) typically develop within seconds or a few minutes of transfusion initiation;
manifestations a day later would be atypical.
Acute liver failure can cause prolonged PT/PTT and mild
thrombocytopenia. Although patients are sometimes mildly hypocoaguable (eg, easy
bruising), clinically significant bleeding and oozing from venipuncture sites is atypical.
Complement-mediated microangiopathy (also called nondiarrheal HUS) is a
hereditary disorder marked by unregulated complement activation and the formation
of platelet-rich thrombi. Although renal insufficiency is common, it does not typically
cause oozing from venipuncture sites. In addition, it is not typically associated with
trauma (unlike DIC).

Drug-induced thrombocytopenia is usually due to the formation of antibodies against


platelets or the suppression of platelet production in the bone marrow. Most cases
manifest several days (~1 week) after drug exposure (not within a day).

Vitamin K is necessary for the formation of several clotting factors (eg, factors II, VII, IX,
and X). Deficiency is rare in healthy individuals and takes several weeks or months
(not a day) to develop. Most cases arise in patients with disorders of the
pancreas/biliary system (because vitamin K is fat-soluble) or in those who take
prolonged courses of antibiotics.

Vitamin K Cofactor in the enzymatic carboxylation of glutamic acid residues on prothrombin


deficiency complex proteins. Vitamin K deficiency is MCly due to inadequate dietary intake,
intestinal malabsorption, or hepatocellular disease causing loss of storage sites. The
liver can normally store a 30-day supply of vitamin K, but an acutely ill person with
underlying liver disease can become vitamin K deficient in as little as 7-10 days.

This patient received no enteral nutrition postoperatively and was given a broad-
spectrum antibiotic, and so both natural sources of vitamin K were compromised. In
addition, he also likely has alcoholic liver disease, further limiting his vitamin K
stores. Vitamin K deficiency decreases plasma levels of all the prothrombin complex
proteins (factors 2, 7, 9, 10, and protein C and S). This initially increases the
prothrombin time (PT), followed by prolongation of the partial thromboplastin time
(PTT). Administration of vitamin K rapidly replenishes the stores in 8-10 hours, and
fresh frozen plasma may be used for the management of acute hemorrhage in the
interim.

DIC causes depletion of clotting factors and secondary fibrinolysis. This causes
bleeding, acute kidney and liver injury, hypotension, and tachycardia. Testing shows
thrombocytopenia, prolonged PT and PTT, decreased fibrinogen, and schistocytes on
peripheral smear.

Lupus anticoagulant (LAC) is an antiphospholipid antibody that prolongs the PTT


during diagnostic testing. However, patients with LAC are actually hypercoagulable
with increased risk for venous thrombosis. Factor V Leiden causes a hypercoagulable
state due to activated protein C resistance.
Factor VIII deficiency is an X-linked inherited plasma coagulation disorder that primarily
prolongs the PTT. It would usually be apparent earlier in life.

Splenic sequestration leads to thrombocytopenia, and patients rarely develop clinical


bleeding or prolonged PT/PTT.

TTP is a rare form of consumptive thrombocytopenia caused by widespread platelet


thrombi that form in the microcirculation. It typically presents with thrombocytopenia,
microangiopathic hemolytic anemia, fever, and neurologic findings (eg, altered mental
status).

HEPARIN-
INDUCED
THROMBOCYTOP
ENIA (HIT)

Type 2 heparin-induced
thrombocytopenia (HIT) is a life-
threatening complication of heparin
therapy. A drop in the platelet count
by >50% or a new thrombus within 5-10
days of initiating heparin should raise
suspicion for HIT (HIT may manifest sooner
in patients previously exposed to
heparin). HIT is antibody mediated and
causes a mild to moderate
thrombocytopenia (rarely <20,000/mm3)
with minimal bleeding risk. However, venous and arterial thrombotic risk is
significant, as high as 50% in untreated HIT.

If HIT is suspected, all heparin products (including heparin flushes) should


be discontinued immediately, and anticoagulation should be initiated with a non-
heparin medication (eg, argatroban, fondaparinux).

…………
Although the diagnosis of HIT is confirmed with a high titer immunoassay or a
functional assay of the blood (eg, serotonin release assay [gold
standard]), discontinuing heparin and initiating nonheparin anticoagulation should not
be delayed for laboratory confirmation due to the high risk of thrombus (up to 50% if
untreated). Platelet counts typically normalize 2-7 days after
stopping heparin; transfusions are not required unless the patient is overtly bleeding
(which is uncommon in HIT).

Warfarin is used for anticoagulation maintenance in patients with HIT but only after the
patient has received another anticoagulant and the platelet count is
>150,000/mm3. Initial treatment with warfarin is contraindicated in patients with HIT as
it rapidly lowers protein C levels, which may transiently increase the risk of thrombus.

All heparin products, including low molecular weight heparin, should be avoided in
patients with HIT.

Cholesterol embolization can occur due to plaque embolization into small arteries
following coronary angiography. Cholesterol embolization classically causes skin
findings (eg, livedo reticularis [mottled erythema], gangrene, cyanosis) that most
commonly affect the lower extremities

ANTERIOR The ddx for an anterior mediastinal mass includes the "4 T's": thymoma, teratoma,
MEDIASTINAL thyroid neoplasm, and terrible lymphoma. Within the category of teratoma, one must
MASS also include other germ cell tumors. Teratomas can often be distinguished from other
germ cell tumors on imaging by the presence of fat or calcium, particularly if in the
form of a tooth. Serum hormone levels may be helpful in differentiating seminomatous
germ cell tumors from nonseminomatous variants. Serum β-HCG can be elevated in
1/3 of patients with a seminoma, although the AFP is essentially always normal.
Nonseminomatous forms of germ cell tumors include yolk sac tumor, choriocarcinoma,
and embryonal carcinoma. A mixture of different cell types is also possible and is
referred to as a mixed germ cell tumor. Most patients with a nonseminomatous germ
cell tumor have an elevated AFP, with a considerable amount also having an elevated
β-HCG.
UPPER
Upper extremity deep venous thrombosis
EXTREMITY DEEP
VENOUS • Central catheter or PICC line
THROMBOSIS (UE Epidemiology • Young, athletic males (spontaneous)
DVT) • Thoracic outlet obstruction

• Unilateral arm or forearm edema


• Pain/heavy sensation
Manifestations
• Erythema
• Dilated subcutaneous collateral veins

Diagnosis • Doppler or duplex ultrasonography


Treatment • 3 months of anticoagulation

PICC = peripherally inserted central catheter.


A patient's arm swelling, pain, and mild erythema developed in the setting of
a peripherally inserted central catheter (PICC), raising strong suspicion for upper
extremity deep venous thrombosis (DVT). All upper extremity CVCs (eg, internal
jugular, subclavian) are associated with increased risk of DVT due to endothelial trauma
during insertion and improper positioning of the catheter tip; the risk of DVT is
somewhat higher with PICCs because they travel long distances through narrow
peripheral veins (often occupying >50% of total vein caliber). Hospitalized patients,
particularly those with a hypercoagulable state (eg, malignancy), are most likely to
develop PICC-related DVT.

Upper extremity DVT is MC within 7-14 days of PICC insertion. The primary
manifestation is arm or forearm swelling. Pain and mild erythema may also occur. In
contrast to PICC infection, patients with DVT usually have a normal appearing catheter
insertion site (eg, no purulence). The diagnosis is made with compression or duplex
USG, which visualizes the subclavian, axillary, and brachiocephalic veins. Although
upper extremity DVT is associated with a relatively low risk of pulmonary embolism
(~6%), treatment with 3 months of anticoagulation is recommended. The PICC line
can often remain in place.
…..

Chest MRI can be used to diagnose superior vena cava (SVC) syndrome, which is often
caused by lung malignancy. The SVC is not well visualized with ultrasound, although
indirect findings (eg, waveforms) can suggest SVC occlusion. Although SVC syndrome
is associated with increased risk of DVT, patients usually have dyspnea, chest pain,
and/or face or neck swelling.

Superficial phlebitis can be treated with warm compresses and NSAIDs. It is generally
caused by peripheral, not central, venous access and marked by redness along the
peripheral vein. Arm swelling in the setting of PICC is far more likely to indicate DVT.

GRAFT-VERSUS- This patient's profuse, watery diarrhea and maculopapular rash raise strong suspicion
HOST DISEASE for acute graft-versus-host disease (GVHD), an immune-mediated, multisystem
(GVHD) inflammatory condition that occurs in 35%-50% of patients who undergo
allogeneic hematopoietic stem cell transplantation (HSCT). Acute GVHD arises when
donor T cells (particularly cytotoxic T cells) identify antigens on host epithelial cells as
foreign and subsequently generate a strong proinflammatory response.

Patients with acute GVHD generally develop symptoms within 100 days of
transplantation, including the following:

• Maculopapular rash that is often painful and can become confluent


(resembling Stevens-Johnson syndrome)
• Profuse, watery diarrhea that has a secretory pattern (eg, persistent, unrelated
to eating, occurs at night) and is often associated with crampy abdominal pain,
nausea, and vomiting
• Liver inflammation with damage to the biliary tract epithelium, leading to
elevated bilirubin, alkaline phosphatase, and transaminases

Acute GVHD is usually diagnosed by biopsy (eg, colonoscopy with biopsy) after
common infections such as Clostridioides difficile and cytomegalovirus are ruled
out. Glucocorticoids are the mainstay of therapy.

……..
GVHD is almost entirely mediated by T cells; studies have found that removal of donor
T cells almost completely eliminates the risk of GVHD (but increases the risk of graft
rejection and disease recurrence). B cells and the humoral antibody response play a
small role in the pathogenesis of GVHD. A classic example of ADCC is natural killer cell
activation by antibodies (which is used in some forms of cancer immunotherapy).

Cryptosporidiosis is a common cause of profuse secretory diarrhea and most often


manifests in those with impaired cellular or humoral immunity. However, a
maculopapular rash would be atypical, and this patient, who has been hospitalized
since transplant and has been eating a low-microbial diet, is at a lower risk.

Conditioning chemotherapy is administered prior to transplantation to destroy active


immune cells (immunoablative) or to destroy hematopoietic stem cells in the bone
marrow to create room for transplanted stem cells (myeloablative). Although
chemotherapy is often associated with a few days of nausea and vomiting,
gastrointestinal side effects generally resolve within a week; symptoms 3 weeks later
would be very atypical.

Patients who undergo solid organ or HSCT are at increased risk for lymphoproliferative
disorders. Most cases present many years (not weeks) after transplantation with
extranodal or CNS lesions.
CERVICAL Malignancy is extremely likely in
LYMPHADENOPA a patient with a persistent (>2
THY (LAD) weeks), palpable (>1.5 cm), firm
neck mass, who has a smoking
history and no history of a
preceding infection. By far
the MC malignancy to manifest
in an upper cervical lymph node
is mucosal head and neck
SCC. Head and neck SCC
is common in patients with a
significant history of alcohol and
tobacco use. The first (and only)
apparent manifestation is often a
palpable cervical lymph node,
which represents regional nodal
metastasis.
Identification of the primary source of SCC is of paramount importance because it
directs all treatment decisions. Although SCC can originate from many sites (eg, skin,
uterine cervix, upper esophagus), the vast majority of cervical nodal SCC arises from the
mucosal surfaces of the head and neck (ie, nasopharynx, oral cavity, oropharynx,
larynx). Thorough examination of these structures must include endoscopic
visualization using laryngopharyngoscopy.

………..

Abdominal malignancies (eg, stomach, pancreas, colon, ovaries) can spread via the
thoracic duct to the left supraclavicular lymph nodes (Virchow node)—an ominous
sign. However, most of these abdominal malignancies are adenocarcinomas rather
than SCC

Most types of breast cancer are adenocarcinomas rather than SCC.

Hepatomegaly can sometimes reflect liver neoplasm, either primary (eg, HCC) or
metastatic. HCC typically metastasizes to the lungs, portal vein, and portal lymph
nodes and is unlikely to cause cervical adenopathy. Conversely, head and neck SCC
does not commonly spread to the liver. This patient's hepatomegaly is likely due to
nonalcoholic fatty liver disease.

Thyroid cancer metastasizes to cervical nodes. However, thyroid cancers are papillary,
follicular, medullary, or anaplastic, not squamous cell.

……..

Malignancy is extremely likely in a patient with a persistent (>2 weeks), palpable (>1.5
cm), firm neck mass; a smoking history; and no preceding infection. By far the most
common malignancy in an upper cervical node is mucosal head and neck squamous
cell carcinoma (SCC). Indeed, the first (and only) apparent manifestation may be a
palpable cervical lymph node, representing regional nodal metastasis. Referred
otalgia is another common presenting symptom, facilitated by either the
glossopharyngeal nerve (CN IX) (innervates both the base of tongue and the external
auditory canal [EAC]) or the vagus nerve (CN X; innervates parts of the
larynx/hypopharynx and the EAC).

Identification of the primary source of head and neck SCC is essential to direct
treatment. Thorough examination includes endoscopic visualization
using laryngopharyngoscopy as well as neck imaging (CT with contrast) to evaluate
the primary site and characterize the cervical nodal disease. Fine-needle aspiration of
the lymph node is advised over open biopsy to avoid tumor seeding.

…..

CLL may present in older patients with painless cervical lymphadenopathy. However, it
typically causes waxing and waning lymphadenopathy. It would not cause ear pain,
and leukocytosis would also be expected.

Granulomatous polyangiitis can present with head and neck symptoms, including ear
pain and subcutaneous nodules. However, it is a systemic disease, often with life-
threatening renal (eg, glomerulonephritis) and pulmonary (eg, nodules, hemorrhage)
manifestations. A solitary lymph node in the neck with no constitutional symptoms
would be unlikely.

Hodgkin lymphoma can also cause painless, cervical lymphadenopathy. However,


many patients have constitutional "B symptoms" (eg, fever, night sweats, weight
loss). Hodgkin lymphoma would not explain the ear pain in this patient, and head and
neck SCC is much more common in adult smokers.
Infectious mononucleosis can cause adenopathy, but it is typically bilateral with
multiple lymph nodes involved. Additionally, it is typically seen in adolescents or
young adults and presents with fever, pharyngitis, and fatigue.
Medullary thyroid carcinoma (MTC) is a rare neuroendocrine carcinoma of the thyroid
parafollicular C cells that can be sporadic or associated with a germline RET mutation
(multiple endocrine neoplasia type 2). Although MTC can metastasize to lateral neck
nodes, a thyroid nodule would also be expected.

Mycobacterial lymphadenitis, often caused by Mycobacterium avium complex, presents


as a solitary, enlarging cervical lymph node. However, it typically presents in children
age 1-5 years, and the lymph node becomes fluctuant with an overlying violaceous skin
discoloration. Tuberculous lymphadenitis is unlikely in a patient with no risk
factors. Cervical lymphadenitis is unlikely to cause ear pain.

METHEMOGLOBI This patient has new-onset hypoxia by pulse oximetry (85%) during an endoscopic
NEMIA (MetHgb) procedure, with a significant difference between the oxygen saturation value estimated
on blood gas analysis and that obtained on pulse oximetry (ie, large oxygen saturation
gap). He likely has acquired methemoglobinemia due to anesthetic use.

Some medications - most commonly topical anesthetics (eg, benzocaine), dapsone,


and nitrates (in infants) - cause the iron component of hemoglobin to be oxidized,
thereby forming methemoglobin, which cannot bind oxygen. The remaining normal
hemoglobin also has an increased affinity for oxygen, resulting in less oxygen delivery
to tissues. Because methemoglobin absorbs light at distinct wavelengths, pulse
oximetry commonly is ~85% (as seen in this patient) regardless of the true oxygen
saturation. In parallel, blood gas analysis frequently returns a falsely elevated result
for oxygen saturation (eg, 99% in this patient) as it provides an estimate based only on
the PaO2, not on effective hemoglobin-oxygen binding. These inaccurate readings
create the large oxygen saturation gap.

Cyanosis can occur when methemoglobin comprises ~10% of total hemoglobin, but
hypoxia symptoms (eg, headache, lethargy) occur only when levels surpass 20%. At
levels >50%, there is risk of severe symptoms (eg, altered mental status, seizures,
respiratory depression) and death. Treatment involves discontinuing the causative
agent and administering methylene blue, which helps reduce iron to its normal state.
…………

Anaphylaxis (eg, due to medication) can lead to respiratory compromise and hypoxia
but is associated with symptoms such as wheezing, stridor, and dyspnea.

Although atelectasis and mucus plugging can lead to transient hypoxemia due to a
collapsed portion of lung, the hypoxemia is commonly reversible with supplemental
oxygen and would not explain the oxygen saturation gap. Procedural hypoventilation
should also correct with supplemental oxygen.
This patient has clear lung fields and normal heart sounds, making pulmonary edema
less likely. Pulmonary edema also typically improves with supplemental oxygen and is
not characterized by a large oxygen saturation gap.

Patients with liver disease can have large-volume ascites and severe muscle wasting,
which may lead to decreased diaphragmatic excursion and resultant hypoxia. However,
this patient has only mild ascites on ultrasound and no evidence of muscle wasting.

Hepatopulmonary syndrome results from arteriovenous shunting in the lungs and can
occur after years of liver disease. Patients frequently exhibit platypnea (induced
dyspnea in the upright position) and orthodeoxia (hypoxia in the upright
position). This syndrome would not explain the patient's large oxygen saturation gap.
ANTICOAGULANT
S

This patient has a provoked deep vein thrombosis (DVT) due to recent
surgery. Treatment with anticoagulation can prevent extension of the thrombus, acute
pulmonary embolism, and possible development of future thrombus. Provoked DVT
due to recent surgery will likely require at least 3 months of treatment.

Warfarin is the preferred long-term oral anticoagulant in end-stage renal


disease patients. It inhibits the synthesis of the vitamin K-dependent clotting factors II,
VII, IX, and X and anticoagulant proteins C and S. Warfarin takes several days to
become therapeutic and first acts on proteins C and S, causing a transient
prothrombotic state; as a result, it generally cannot be started alone. This patient
therefore will need to be started on intravenous unfractionated heparin for
immediate treatment of the DVT (as it acts quickly) and then on warfarin (typically in
the evening of the same day). Heparin is continued for 4-5 days until the INR is at
therapeutic levels (goal 2–3).

…………..
Aspirin has no role in the treatment of confirmed DVT as it is a platelet inhibitor and
does not modify the coagulation cascade.
Starting anticoagulation in a hemodynamically stable patient 48-72 hours after
surgery is generally safe and does not significantly increase the risk of bleeding. IVC
filters can be used in patients in whom anticoagulation is contraindicated (eg, massive
GI bleed, hemorrhagic stroke).

Both LMWH (eg, enoxaparin) and rivaroxaban are NOT recommended in ESRD; they
are metabolized by the kidney, so their use in patients with ESRD is associated with
increased bleeding risk. IV unfractionated heparin (UFH) is not contraindicated in
ESRD.

UFH would generally not be started alone as it is an IV medication and needs to be


continued until the INR is therapeutic, which can take several days. The goal is to
discharge the patient on oral warfarin quickly.

NECK A patient developed a hematoma after core needle biopsy for an asymptomatic thyroid
HEMATOMA nodule. She has no personal or family history of easy bleeding or bruising, making
AFTER THYROID vitamin deficiency and most hereditary bleeding disorders unlikely. Therefore, despite
BIOPSY a core needle biopsy being a fairly low-risk procedure, the most likely explanation for
the hematoma is insufficient hemostasis following tissue injury from the biopsy.

……..
Platelet dysfunction can be seen in patients who take antiplatelet agents (eg, aspirin) or
have certain comorbid conditions (eg, liver disease, uremia). Von Willebrand disease
can affect platelet function. This otherwise healthy individual (with no family history of
bleeding) is unlikely to have platelet dysfunction. Furthermore, most cases would
manifest immediately with oozing at the biopsy site.

Factor VIII deficiency is seen in hemophilia A, which usually presents in childhood with
bruising and mucocutaneus or joint bleeding. Because it is transmitted in an X-linked
recessive fashion, male patients are affected; a female patient with no history of
bleeding is unlikely to have factor VIII deficiency.

Clinically significant thrombocytopenia (eg, primary immune thrombocytopenia)


usually manifests with a history of bruising and mucocutaneous bleeding.

Although vitamin C deficiency (scurvy) can increase the risk of bleeding due to blood
vessel fragility, and vitamin K deficiency can increase the risk of bleeding due to
coagulation factor (eg, factor II, VII, IX, X) deficiency, this patient's lack of
mucocutaneous bruising or bleeding makes these conditions unlikely. In addition,
healthy individuals who consume a relatively normal diet are at very low risk for
vitamin deficiency.

HYPOVOLEMIA Despite initial resuscitation (2 L of crystalloid), a patient has continued hemodynamic


instability (eg, SBP <90 mm Hg), which is consistent with shock. Because hemorrhagic
shock is the most common type of shock in the trauma setting, particular attention is
given during trauma survey to areas where large blood loss can occur ("blood on the
floor and 4 more"):
• External bleeding ("the floor"): up to the ENTIRE blood volume
• Chest: up to 40% of the blood volume/hemithorax (up to 80% in both)
• Abdomen (ie, peritoneal cavity): up to the ENTIRE blood volume
• Pelvis: up to the ENTIRE blood volume; blood loss often hidden within
the retroperitoneum
• Thigh: up to 1-2 L/thigh (2-4 in both)

This patient has no evidence of external bleeding or major hemorrhage into the chest
(bilateral breath sounds), abdomen (nondistended, soft, no intraperitoneal free fluid),
or thighs (normal muscle tone). Therefore, pelvic/retroperitoneal bleeding should be
suspected. Pelvic fracture can cause tearing of the thin-walled VENOUS
plexus (presacral, lumbar), leading to life-threatening hemorrhage. For early detection
of this potentially lethal injury, pelvic x-ray is typically performed as an adjunct to the
trauma primary survey.

…………..

Adrenal crisis can be precipitated by trauma and cause shock refractory to fluid
resuscitation. However, this occurs much more commonly in patients with underlying
adrenal insufficiency (eg, autoimmune-mediated adrenalitis, adrenal suppression from
chronic glucocorticoid use) rather than in previously healthy patients. Bilateral adrenal
infarct/hemorrhage from trauma is extremely rare.

Spinal fracture can cause spinal cord injury leading to neurogenic shock. However,
neurogenic shock is typically associated with flaccid paralysis (vs normal muscle
tone), as well as HoTN with bradycardia (vs tachycardia) from sudden loss of
sympathetic tone. Severe ICH may also cause neurogenic shock or, alternately,
Cushing triad (from increased intracranial pressure). However, Cushing triad is
characterized by HTN (vs hypotension), bradycardia (vs tachycardia), and irregular
respiration.
Pancreatic injury can occur with blunt abdominal trauma, especially when the
pancreas is rapidly compressed (eg, steering wheel impact) against the vertebral
column. However, even severe injuries (eg, transection with pancreatic fluid leakage)
do not typically cause acute hemodynamic instability. They more commonly result in
symptoms similar to those of pancreatitis (eg, abdominal pain, nausea, emesis).

ADRENAL
Acute adrenal insufficiency (adrenal crisis)
INSUFFICIENCY
• Adrenal hemorrhage or infarction
Etiology • Illness/injury/surgery in patient with chronic AI
• Pituitary apoplexy

• HoTN & shock


Clinical features • N, V, abdominal pain
• Fever, generalized weakness
• Hydrocortisone or DXM
Treatment • Rapid IV volume repletion

AI = adrenal insufficiency.
A patient's presentation is most consistent with primary adrenal insufficiency (PAI)
complicated by adrenal crisis. Her
history of fatigue, weight loss,
abdominal pain, anorexia, and GI
disturbance likely reflects PAI. PAI
usually involves autoimmune
destruction of all 3 layers of
the adrenal cortex, leading to
deficiency of mineralocorticoids (eg,
aldosterone), glucocorticoids (eg,
cortisol), and androgens. Therefore,
common findings in chronic PAI
include hyponatremia
(hypovolemia-induced ADH
secretion), hypoglycemia (cortisol
deficiency), and peripheral
eosinophilia (eosinophils are normally inhibited by corticosteroids).
Acutely stressful events (eg, surgery, endoscopy, injury, infection) in patients with
chronic adrenal insufficiency can precipitate adrenal crisis, manifesting as severe
HoTN and shock. Adrenal crisis in PAI appears to be mainly a manifestation of
mineralocorticoid deficiency (hypoaldosteronism), given that it can occur even in
patients taking physiologic doses of glucocorticoids. However, the resulting severe
HoTN, often refractory to volume resuscitation and poorly responsive to
vasopressors, is likely exacerbated by glucocorticoid deficiency because cortisol is
needed to potentiate the effect of alpha-1 stimulation on peripheral vascular tone.

Treatment requires rapid volume repletion and glucocorticoid replacement (eg, IV


dexamethasone or hydrocortisone); DXM is often preferred because it does not
interfere with plasma cortisol level that may be needed to confirm the diagnosis.
Mineralocorticoid replacement (eg, fludrocortisone) requires several days to exert its
sodium-retaining effects and is typically started after initial resuscitation.

…………

GN sepsis can cause profound vasodilation and severe hypotension; however, it is less
likely in the absence of fever and would not explain this patient's prior abdominal pain,
anorexia, and weight loss, or current eosinophilia. N. meningitis (a GN coccus) has an
affinity for the adrenal glands and can cause massive adrenal hemorrhage with
adrenal crisis (ie, Waterhouse-Friderichsen syndrome), but the presentation would be
acute without preceding chronic symptoms.
Propofol can cause cardiac suppression and hypotension, at least partly due to
inhibition of sympathetic drive. However, propofol-induced hypotension cannot
account for eosinophilia and hypoglycemia and would likely have improved with
epinephrine administration.

Acute PE can cause severe hypotension and shock but would also cause significant
hypoxemia.

Thyrotoxicosis (eg, excess T3 and/or T4) usually presents with HTN and HrGlc.

//////////////////////////////INFECTIOUS DISEASES///////////////////////////////
VENTILATOR-
ASSOCIATED
PNEUMONIA
(VAP)

VAP is a type of nosocomial pneumonia that develops ≥48 hours after endotracheal
intubation and is MCly caused by aerobic GN bacilli (eg, P aeruginosa, E coli, K
pneumoniae) and GP cocci (eg, MRSA, Streptococcus). Patients usually have fever,
purulent secretions, difficulty with ventilation (eg, increased RR, decreased TV), and
leukocytosis.

The first step is to obtain a CXR:


1. Patients with a normal x-ray are unlikely to have VAP and should be
evaluated for other causes.

2. Those with an abnormal x-ray (eg, alveolar infiltrates, air bronchograms,


silhouetting of adjacent solid organs) require LRT sampling (ie,
tracheobronchial aspiration) for Gram stain and culture. Patients should
receive empiric antibiotics (usually GP, antipseudomonal, and GN
coverage) until culture susceptibility results return as treatment delay can
increase mortality. However, respiratory tract sampling should be
done prior to starting antibiotics as treatment can decrease the sensitivity
of both Gram stain and culture.

………..

CRP may be elevated in VAP, but it is a nonspecific marker of inflammation and cannot
identify the underlying diagnosis.
CT scan of the chest can identify complications (eg, empyema) in patients that do not
improve clinically with therapy. When Gram stain and culture are not suggestive of
VAP, CT scan can also be considered to evaluate for alternate causes (eg, PE). VAP is
much more likely than pulmonary embolism in a patient with fever, leukocytosis, and
dense infiltrates on chest x-ray. In addition, routine prophylactic anticoagulation
(eg, subcutaneous heparin) is standard of care in patients in the ICU, and it is likely
that a patient who underwent aortic valve replacement and has atrial fibrillation is
already receiving therapeutic anticoagulation therapy (eg, IV heparin). All these factors
make pulmonary embolism less likely than VAP.
D-dimer is used as an initial test for deep venous thrombosis and pulmonary
embolism. However, it is nonspecific and may be elevated in inflammatory or
infectious conditions (eg, VAP). This patient's fever, leukocytosis, and lobar infiltrates
are more consistent with VAP.

ANIMAL BITES Cats have long, sharp teeth that can inoculate oral flora deep into skin, reaching soft-
tissue structures (eg, nerves, tendon sheaths). Therefore, cat bites are much more likely
to cause serious infection than dog or human bites, andantibiotic prophylaxis is
recommended in addition to routine wound care (eg, copious irrigation). Oral flora of
cats includes Pasteurella multocida (gram-negative coccobacilli) and oral
anaerobes. Amoxicillin/clavulanate is the agent of choice for prophylaxis; amoxicillin
has activity against P multocida, and the addition of clavulanate provides coverage
against oral anaerobes.
For cat bites, a booster dose of tetanus-
containing vaccine should be
administered to incompletely vaccinated
children or those whose last vaccine
was >5 years ago.
Due to the high risk for infection
following cat bites, wound closure
should be avoided unless there is a
major cosmetic implication (eg, facial wound).
……..

Observation and close follow-up without antibiotic prophylaxis are appropriate for
immunocompetent individuals with minor human or dog bites that are not located on
hands, feet, or genitalia. This treatment would be inappropriate management for cat
bites as the majority will become infected without antibiotic prophylaxis.
Azithromycin has activity against Bartonella henselae, the causative agent of cat-
scratch disease. However, cat-scratch disease is uncommon and does not require
antibiotic prophylaxis in immunocompetent individuals.
Clindamycin treats gram-positive skin flora (eg, Staphylococcus aureus, Streptococcus
pyogenes) and oral anaerobes. Although clindamycin is commonly used to treat
cellulitis and abscesses unrelated to bites, it has no activity against P multocida and
therefore should not be used as monotherapy for prophylaxis following a cat bite.

HUMAN BITES A patient experienced a bite wound from a clenched fist ("fight-bite") injury. Human
bite wounds are prone to polymicrobial infection with aerobic and anaerobic oral
flora. The MC organisms include strep, S aureus, E corrodens, H influenzae, and beta-
lactamase–producing anaerobic bacteria.

Patients often initially ignore a wound until pain, swelling, or purulent discharge
develops. Blood and wound cultures should be sent; empiric antibiotics are
required. Although large studies on empiric antibiotic selection for human bite
wounds are scarce, amoxicillin-clavulanate is often the treatment of choice due to
excellent coverage of GP, GN, and beta-lactamase–producing oral anaerobic
organisms. Surgical debridement is usually necessary, and wounds are typically left
open to drain and heal by secondary intention (due to high infection risk with
closure). Tetanus vaccination should be administered to those who are not up to
date.

………
Ampicillin can be used for URTI (eg, pharyngitis) and is effective against many GP and
GN organisms. However, it does not provide coverage for beta-lactamase–producing
flora and is often combined with the beta-lactamase inhibitor sulbactam in an IV
formulation.
Ciprofloxacin covers many GN organisms and some GP organisms (excluding most
streptococci), but it does not provide reliable coverage for anaerobic organisms. It is
often used for GU and GI infections.
Clindamycin is effective against GP bacteria and anaerobes but not against most GN
organisms (eg, E corrodens). It is used for some lung abscesses, skin and soft-tissue
infections, and female upper reproductive tract infections (anaerobic coverage in
combination with other agents).

Erythromycin is sometimes used as a second-line agent in the treatment of


gonococcal or chlamydial urethritis. Erythromycin covers some atypical organisms
(eg, Mycoplasma), but overall it does not provide adequate GP or anaerobic
coverage.

NOSOCOMIAL Central venous catheters (CVCs) are the MCCs of nosocomial bloodstream
BLOODSTREAM infections as they create a direct pathway for colonized skin organisms to access the
INFECTIONS circulatory system. Coagulase-negative staphylococci and Staphylococcus
aureus cause the majority of infections; however, Candida species are isolated in
approximately 10% of cases.

Candida is a budding yeast that frequently colonizes the skin, mucous membranes, and
GIT of humans. Active infections are rare in healthy individuals and are usually limited
to superficial structures (eg, vaginal candidiasis). However, patients
with immunocompromise (eg, postchemotherapy) or intravascular catheter (especially
in the intensive care unit setting) are at high risk for invasive disease.

Candida bloodstream infections may


cause fever, sepsis, and/or multiorgan
failure.

Diagnosis is made primarily with blood


cultures (or biopsy), but serum beta-D-
glucan antigen can be used as a rapid
indicator of possible infection.

Treatment usually requires several weeks


of iv antifungals.

…………….

Candida in a blood culture should always prompt a search for a nidus of infection and
should never be considered a contaminant.

Indwelling urinary catheters, especially those obstructed by a fungus ball, may


occasionally cause candidemia; however, this is a far less common source of infection
than an intravascular catheter.
Mucous membrane infections are a rare cause of invasive disease.
Pressure ulcers may cause bacteremia with a wide range of skin and fecal organisms;
however, skin wounds are a rare cause of invasive candidal infections.

PROSTHETIC
Prosthetic joint infection (PJI)
JOINT INFECTION
(PJI) Early onset Delayed onset Late onset

Time to
onset
<3 months 3-12 months >12 months
after
surgery

• Acute pain • Chronic joint pain • Acute sx in previously


Present • Wound infection or • Implant loosening asx joint
ation breakdown • Sinus tract • Recent infection at
• Fever formation distant site

• S. aureus • CNS • S. aureus


MC
• GN rods • Propionibacterium s • GN rods
organis
• Anaerobes pecies • Beta-hemolytic
ms • Enterococci streptococci

This patient has subacute pain in his prosthetic knee 6 months after arthroplasty. The
synovial fluid analysis shows a mildly elevated leukocyte count with a predominance of
neutrophils. This is consistent with an inflammatory process, most likely a prosthetic
joint infection (PJI). The leukocyte count in the synovial fluid in PJI is usually elevated
to >1000/mm3 but is often lower than in septic native joints (usually >50,000/mm3).

PJI can be acquired by perioperative contamination of the joint or by extension from


an overlying wound infection:

• Infections due to virulent organisms (eg, Staphylococcus


aureus, Pseudomonas aeruginosa) typically present within the first 3 months
after surgery (early onset infection) with acute pain, fever, leukocytosis, and
overt local signs of infection (eg, erythema, purulent drainage), not seen in this
patient.
• Infections due to less virulent organisms
(eg, CNS, Propionibacterium species, are likely to have a delayed onset (3-12
months) and present with chronic pain, implant loosening, gait impairment, or
sinus tract formation. Fever and leukocytosis are usually
absent. Staphylococcus epidermidis is a coagulase-negative staphylococcus
commonly implicated in delayed-onset PJI.
• Late-onset infections presenting >12 months after surgery are unlikely to have
been acquired perioperatively and are usually due to hematogenous spread of
a distant infection (eg, urinary tract infection).

………….
Atypical mycobacteria are a rare cause of PJI. They usually cause late-onset infections
and are most commonly seen in patients with immunosuppressive disorders (eg, HIV)
or local trauma.
Late Lyme disease can present with acute intermittent attacks of monoarticular or
oligoarticular arthritis, but PJI with chronic, persistent joint pain is rare.

Chlamydia trachomatis and Salmonella species can be associated with reactive arthritis
following an initial genitourinary or gastrointestinal infection. Reactive arthritis is
characterized by asymmetric oligoarthritis with urethritis/enteritis and
conjunctivitis. Subacute PJI is an uncommon presentation.

Disseminated Neisseria gonorrhoeae infection most commonly causes a syndrome


characterized by migratory polyarthritis, tenosynovitis, and rash. Acute septic
monoarthritis can also be seen, but synovial fluid cell counts are usually higher
(>50,000/mm3).

SURGICAL SITE
**** Risk of surgical site infection
INFECTION (SSI)
///////////////// • Malnutrition/hypoalbuminemia, obesity
• Glucocorticoids, immunosuppressive drugs, CTX, radiation
**** Patient • Smoking
factors • Uncontrolled DM, PAD, venous insufficiency
• Concurrent infection at another site
• Advanced age

• Emergency surgery
Procedural • Surgery for malignancy
factors • Open surgical approach
• Inadequate site preparation & antibiotic ppx (as appropriate)

Personnel • Inadequate hand hygiene


factors • Inappropriate attire/gloving
Hernia repair: Emergency repair is indicated for patients with bowel obstruction, acute
incarceration, or hernia strangulation, but otherwise elective repair is usually
appropriate.

Besides perioperative pulmonary complications, smoking is associated with an


increased risk of surgical site infection (SSI). Smoking causes alterations in tissue
perfusion, inflammatory response, and collagen synthesis that can promote infection
and contribute to poor wound healing. Current smoking (especially smoking on the
day of surgery) is associated with greater risk than past smoking, but patients who
quit smoking are still at a moderately increased risk compared to those who have
never smoked. Smoking cessation interventions prior to elective surgery are
recommended, especially if cessation can be achieved at least 4-6 weeks before
surgery.
Obese patients are at increased risk of SSI, especially related to abdominal procedures
(eg, ventral hernia repair). Underweight patients may also be at increased risk,
although this risk is more specifically attributable to malnutrition and
hypoalbuminemia.

Certain comorbid conditions, especially uncontrolled diabetes and PAD, are


associated with increased risk of SSI. Hypothyroidism may blunt fever and mask
signs of SSI but is not associated with a significant increase in infection risk.
……………

A patient has the following signs and symptoms suggestive of necrotizing surgical site
infection:

• Pain, edema, or erythema spreading beyond the surgical site


• Systemic signs such as fever, tachycardia, or HoTN
• Paresthesia or anesthesia at the edges of the wound
• Purulent, cloudy-gray discharge ("dishwater drainage")
• Subcutaneous gas or crepitus

Necrotizing surgical site infections occur more commonly in patients with


diabetes and are usually polymicrobial. These infections are considered emergencies
if they involve the fascial plane and develop into necrotizing fasciitis. The most
important step in management of this condition is early surgical exploration to assess
the extent of the process and debride necrotic tissues. Adjunctive therapies, including
broad-spectrum antibiotics, adequate hydration, and tight glycemic control, are also
important but are secondary to surgical exploration.

………..

Appropriate wound dressing and tight glucose control play a key role in adequate
surgical wound healing in diabetics. However, once infection is established, definitive
therapy with surgical exploration and antibiotic therapy is needed.
Negative-pressure wound therapy (ie, vacuum-assisted wound closure) is a wound-
dressing system that applies sub-atmospheric pressure to a wound to accelerate the
healing process. It is reserved for healthy, granulating wounds. It is not used initially
when the wound is infected or necrotic.

IV antibiotics alone are sufficient therapy for wound infections limited to cellulitis, but
surgical debridement is required when the infection penetrates the deeper skin layers
and adjacent tissue.
Topical antimicrobial agents do not have a clear role in surgical site infections. They
are NOT useful for prevention as they may inhibit wound healing, and they are not a
substitute for parenteral therapy once infection has been established.
**** Antibiotics for SSI prevention

Wound
Antibiotic
classifica Procedure examples Typical contaminants
prophylaxis
tion

1st-line: IV
cefazolin
Cardiac, neurological, Skin flora: Streptococcus, S
Clean* orthopedic, vascular aureus & CoNS Alternatives:
vancomycin,
clindamycin

Based on
Clean- surgical site,
GI, GU,
Skin flora, GN bacilli, enterococci broader
contami gynecologic/obstetric,
& endogenous flora of the viscus coverage often
nated** H&N, thoracic
indicated (e.g.,
metro)

*Uninfected, uninflamed, the viscus is not entered.


**Viscus (eg, alimentary, genitourinary, respiratory systems) is entered under controlled conditions.
SSI = surgical site infection.

In addition to preoperative antiseptic skin preparation and sterile surgical


technique, prophylactic antibiotics can help reduce surgical site infections (SSIs).
Administered prior to incision, prophylactic antibiotics reduce the microorganism
burden at the surgical site and are indicated when there is a high risk of infection (eg,
traumatic wounds) or when infection would cause significant morbidity/mortality (eg,
mediastinitis after cardiac surgery).

The specific antibiotic is chosen based on several factors, including the type of
surgical wound and its expected degree of microbial contamination:

• Clean – without infection or viscus entry


• Clean-contaminated – with controlled viscus entry
• Contaminated – inflamed, acutely traumatic, or with viscus spillage
• Dirty – infected, necrotic, or fecally contaminated

Cardiac procedures are typically considered clean, and most associated SSIs are due to
skin flora. Therefore, only GP skin flora (eg, Streptococcus, S aureus, CoNS) coverage is
required. 1st- and 2nd-G cephalosporins (eg, cefazolin) typically provide good
prophylactic coverage. However, this patient has a history of a severe IgE-mediated
allergic reaction (ie, anaphylactic shock) to penicillin, with which first- and second-
generation cephalosporins have structural similarities (ie, shared allergenic epitopes).
Therefore, an alternative antibiotic (eg, vancomycin, clindamycin) with good GP
coverage should be selected.
…………..

Azithromycin has a broad spectrum of activity against many GP and some GN


bacteria including atypical bacteria (eg, Mycoplasma, Legionella); it is commonly used
for treatment of CAP. It has inferior coverage against Staph and is not typically
used for SSI prophylaxis.

Ciprofloxacin has antimicrobial activity against aerobic, enteric, GN bacilli and is


used for prophylaxis in some GU procedures. Metronidazole has antimicrobial activity
against anaerobes (eg, Bacteroides, Clostridium) and protozoa (eg, Trichomonas
vaginalis) and is used as part of a prophylaxis regimen in some GI, gynecologic, and
GU procedures. However, neither antibiotic provides adequate coverage against
typical GP skin flora for this cardiac procedure.

Piperacillin-tazobactam has antimicrobial activity against GP, GN, and anaerobic


bacteria including P aeruginosa; it is commonly used in broad-spectrum, empiric
coverage for severe infections. However, such broad coverage is unnecessary (following
principles of antimicrobial stewardship) for prophylaxis against a narrower spectrum of
gram-positive skin flora. In addition, piperacillin-tazobactam should be avoided given
this patient's severe penicillin allergy.

Vibrio vulnificus

• GN, free-living in marine environments


Epidemiology • Ingestion (oysters) or wound infection
• ↑ Risk in those with liver disease* (cirrhosis, hepatitis)

• Rapidly progressive (often <12 hours)


Manifestations • Septicemia - septic shock, bullous lesions
• Cellulitis - hemorrhagic bullae, necrotizing fasciitis

Diagnosis • Blood & wound cultures

• Empiric in those with likely illness as highly fatal


Treatment • IV ceftriaxone + doxycycline

*Hereditary hemochromatosis is particularly high risk as iron acts as a growth catalyst.


Vibrio vulnificus is a free-living, gram-negative bacterium that grows in brackish
coastal water and marine environments. Levels are greatest in the summer months,
reaching as high as 8% of total bacteria in some areas. Infections are primarily
acquired through the consumption of raw oysters (which concentrate the bacteria) or
through wound contamination during recreational activities (eg, sailing) or raw
seafood handling. Most patients who become ill have liver disease (eg, alcoholic
cirrhosis, viral hepatitis); those
with hemochromatosis are at
particularly great risk as free iron acts
as an exponential growth catalyst.

Patients with wound contamination


typically develop a mild cellulitis, but
those with liver disease or certain
comorbidities (eg, DM) are at risk
for necrotizing fasciitis with
hemorrhagic bullous lesions and septic
shock.

Diagnosis is made by blood and wound cultures, but treatment (intravenous


antibiotics) should not be delayed due to the high risk of death.
…………..

Pasteurella infections are typically associated with dog or cat bites/scratches and may
cause rapid-onset cellulitis and necrotizing fasciitis
P aeruginosa skin infections commonly occur due to hot tub (not ocean) water, nail
puncture wounds, and ear piercings. Folliculitis and cellulitis are more common than
necrotizing fasciitis.
S aureus and Streptococcus pyogenes are leading causes of cellulitis and
erysipelas. Although these organisms may occasionally cause necrotizing fasciitis
(particularly S pyogenes), lesions typically develop over a few days (not hours).

NECROTIZING This patient has fever, hypotension, erythema, and swelling. Notably, she has pain
FASCITIS out of proportion to the physical examination findings. In addition, her CT scan is
suggestive of air in the deep tissue. This constellation of findings is concerning
for necrotizing fasciitis. Necrotizing fasciitis is a rapidly spreading infection involving
the subcutaneous fascia, generally following trauma. It can also result from significant
peripheral vascular disease (ie, diabetes). gas is the most frequently recovered
pathogen, although necrotizing fasciitis is usually polymicrobial. Gas production by
microbes leads to air in the soft tissues, which results in crepitus on examination in
about 50% of cases.
Patients with
necrotizing fasciitis
have pain and swelling
of the affected site.
The pain is frequently
more severe than
expected based on the
degree of swelling and
erythema and
usually precedes
systemic signs such as
fever or HoTN.
Untreated necrotizing
fasciitis progresses to
rapid discoloration of
the affected site,
purulent discharge,
bullae, and necrosis.
Imaging can reveal the extent of the infection and identify air in the tissue bed.
However, if necrotizing fasciitis is strongly suspected, therapy should not be delayed to
pursue additional imaging. Broad-spectrum antibiotic therapy and resection of
necrotic tissue are necessary for treatment. Because of its rapid progression,
necrotizing fasciitis causes significant morbidity and mortality even with aggressive
treatment.

……….
Abscesses are localized to
a tender, fluctuant area
and do not spread
aggressively as does
necrotizing
fasciitis. Systemic signs
such as hypotension and
high fever are unusual.

Cellulitis is an acute skin


infection that presents
with warmth, erythema,
edema, and tenderness. Evidence of underlying tissue necrosis, including severe pain
or crepitus or systemic signs such as hypotension, should raise concern for a more
aggressive process such as necrotizing fasciitis or other deep-tissue infection.
Hematomas may be tender to palpation, but signs of infection such as erythema, fever,
and hypotension are not consistent with hematoma.
Pyomyositis (muscle abscess) may have a very similar clinical presentation compared to
necrotizing fasciitis, with fever, erythema, swelling, and pain. However, pyomyositis is
generally limited to one muscle group and does not spread rapidly. This patient also
has air in the tissue planes on CT scan, which is consistent with necrotizing fasciitis.

FLUOROQUINOLO In patients w/ aortic aneurysm, the use of fluoroquinolones (eg, levofloxacin,


NES moxifloxacin, ciprofloxacin) is relatively contraindicated. Fluoroquinolones upregulate
cell matrix metalloproteases, leading to increased collagen degradation; this
mechanism is likely responsible for several associated adverse effects, including
Achilles tendon rupture, retinal detachment, and aortic aneurysm rupture.

The increased risk of aortic aneurysm rupture with fluoroquinolones is small overall, but
it warrants serious consideration due to the high morbidity and mortality of
rupture. Hence, when possible, these drugs should be avoided in patients with
known aortic aneurysm or substantial risk factors for aortic aneurysm (eg, Marfan
syndrome, EDS, advanced atherosclerotic disease, uncontrolled HTN).

Other adverse effects of fluoroquinolones that are likely unrelated to collagen


degradation include encephalopathy, peripheral neuropathy, and QT-interval
prolongation.
…………

Amoxicillin is sometimes used for outpatient treatment of CAP. Adverse effects


include hypersensitivity reactions (eg, urticaria, acute interstitial nephritis) and
nonallergic skin rash (eg, erythema multiforme).

Macrolides (eg, azithromycin, clarithromycin) can be used as monotherapy for


outpatient treatment of CAP or can be combined with a beta-lactam antibiotic for
inpatient treatment. Adverse effects include GI disturbance (eg, nausea) and QT-
interval prolongation.

Ceftriaxone is usually the beta-lactam of choice for inpatient treatment of


CAP. Like most beta-lactams, ceftriaxone is associated with hypersensitivity
reactions. It is also associated with cholestasis in some patients.

Doxycycline monotherapy is appropriate for outpatient treatment of CAP, and the


drug is sometimes combined with a beta-lactam for inpatient treatment. Adverse
effects include skin photosensitivity and medication-induced esophagitis.

TOXIC SHOCK
Clinical features of toxic shock syndrome (TSS)
SYNDROME (TSS)
• Fever usually >38.9 C (102 F)
• HoTN with systolic BP ≤90 mm Hg
• Diffuse macular erythroderma
• Skin desquamation, including palms & soles, 1-2 weeks after illness onset
• Multisystem involvement (3 or more systems)
o GI (vomiting &/or diarrhea)
o Muscular (severe myalgias or elevated creatine kinase)
o Mucous membrane hyperemia
o Renal (BUN or serum creatinine >1-2x upper limit of normal)
o Hematologic (platelets <100,000/mm3)
o Liver (ALT, AST & total bilirubin >2x upper limit of normal)
o CNS (altered mentation without FNDs)

ALT = alanine aminotransferase; AST = aspartate transaminase; BP = blood pressure;


BUN = blood urea nitrogen.

A patient's clinical presentation (recent nasal packing followed by rapid onset of rash,
fever, HoTN, diarrhea, and thrombocytopenia) is consistent with toxic shock
syndrome (TSS) due to S aureus. TSS is caused by TSST-1, which acts as a superantigen
and causes T cell activation and release of large amounts of cytokines. Approximately
50% of TSS cases are related to menstruation (tampon use); the remainder are
nonmenstrual (eg, surgical wound infections, sinusitis, septorhinoplasty).

The clinical symptoms and signs of TSS develop rapidly (usukky w/in 2-3
days). Signs and symptoms include fever, diffuse myalgias, vomiting, profuse
diarrhea, HoTN, and a diffuse macular erythroderma similar to sunburn. Leukocytosis
may not be present, but immature neutrophils are elevated. Thrombocytopenia is
common instead. Management includes supportive therapy (eg, IV fluids), removal of
foreign materials from surgery, and broad-spectrum anti-staphylococcal antibiotics.

………….
Skin findings in acute meningococcemia include a petechial rash that progresses to
ecchymosis, bullae, vesicles, and, ultimately, gangrenous necrosis. Patients also
usually present with fever, nausea, vomiting, severe myalgias, meningeal signs, and
shock.
The rash associated with RMSF is typically petechial, starts in the extremities, and
usually occurs a few days after fever onset.

Scarlet fever is primarily a disease of children. Typically, a GAS infection occurs


earlier at another site (eg, tonsillitis, pharyngitis). Skin findings are typically preceded
by a prodrome of fever, headache, V, and sore throat. 12-48 hours later, fine pink
blanching papules appear on the neck and upper trunk and quickly generalize with
flexural accentuation (rough, sandpaper-like texture).

SJS is a severe skin reaction to certain medications and is occasionally due to


infections in children. It is characterized by a prodrome of fever and influenza-like
symptoms, followed by the development of mucocutaneous erythematous and
purpuric macules that progress to necrosis and sloughing of the epidermis.

LYMPHANGITIS
Lymphangitis

• Cutaneous injury → pathogen invasion of lymphatics in deep


Epidemiology dermis
• S pyogenes & MSSA

• Tender, erythematous streaks proximal to wound


Manifestations • Regional tender lymphadenopathy (lymphadenitis)
• Systemic symptoms (eg, fever, tachycardia)

Treatment • Cephalexin

A patient had a skin abrasion and subsequently developed tender, erythematous


streaks extending towards her knee and regional lymphadenopathy, raising strong
suspicion for acute infectious lymphangitis. Most cases develop in the setting
of cutaneous wound, which creates a portal for skin pathogens to enter the deep
dermis and invade lymphatic channels. Patients generally have tender, erythematous
streaks extending from the wound toward the draining lymph nodes; regional tender
lymphadenopathy (lymphadenitis) and systemic symptoms (eg, fever) are often
present.

The leading causes of lymphangitis are S pyogenes and MSSA; therefore, empiric
treatment with cephalexin is generally curative.

………..
Azithromycin is often used for URI due to excellent activity against a number of GP
upper respiratory pathogens, including S pyogenes. However, it is not regularly used
for skin and soft tissue infection due to limited activity against MSSA.

Ciprofloxacin is primarily used for gram-negative bacterial infections, particularly


those that arise below the level of the diaphragm (eg, urinary tract, gastrointestinal). It
has limited activity against gram-positive bacteria and is not commonly employed for
skin and soft tissue infections.

Griseofulvin is an antifungal agent that is primarily used to treat cutaneous fungal


infections (eg, ringworm). This patient with tender erythematous streaks and regional
lymphadenitis following a cutaneous injury is far more likely to have a bacterial
infection.

Itraconazole is an antifungal agent that is often used to treat sporotrichosis, the


leading cause of nodular lymphangitis. In contrast to acute infectious lymphangitis,
nodular lymphangitis is marked by the formation of nodular swellings along the
proximal lymphatic channel over the course of weeks (not days). Gardeners are
classically affected.
Although penicillin V has activity against S pyogenes, it is not typically employed for
skin and soft tissue infections because many strains of MSSA are resistant.

OSTEOMYELITIS S aureus and P aeruginosa are responsible for most deep infections following
puncture wounds. Pseudomonas infections are particularly prevalent after puncture
wounds through the sole of a shoe as the warm, moist environment is quite hospitable
to this microorganism.

Many puncture wound injuries remain asx, but it is estimated that >50% develop
superficial cellulitis or deeper infections such as osteomyelitis. Evidence of an active
infection is increased in those with skin findings of infection (edema, warmth, erythema,
tenderness), leukocytosis, and fever.

Radiographs are usually required to evaluate for underlying osteomyelitis when deep-
penetrating injuries occur; however, bone changes consistent with osteomyelitis often
take >2 weeks to form. Blood cultures are usually drawn, and bone biopsy with culture
may be required to identify the microorganism.

Treatment includes IV antibiotics (eg, ciprofloxacin, piperacillin-tazobactam) and


surgical debridement. If the pt’s vaccination status is out of date, tetanus vaccine and
TIG should be administered.

………….
β-hemolytic streptococci and CNS may cause wound infections but rarely cause
osteomyelitis in the absence of predisposing factors such as DM.

Candida osteomyelitis is sometimes seen in injection drug users but is rare following a
puncture wound.
E coli and K pneumoniae may cause osteomyelitis (especially in patients with DM), but
both microorganisms are far less common than Pseudomonas in patients with
puncture wounds.
M tb is a rare cause of osteomyelitis in the US but is common in developing
countries. The MCly affected bone is the spine (Pott disease).

HOT TUB This patient developed a tender papulopustular eruption on his lower extremities after
FOLLICULITIS swimming in a heated hotel pool, raising strong suspicion for P
aeruginosa folliculitis ("hot-tub" folliculitis). Most cases develop within hours or a
few days after swimming in an inadequately chlorinated swimming pool or hot
tub. The rash that forms is generally tender and may appear as papules, pustules, or
nodules. Low-grade fever is often present. The eruption is usually self-limited and
does not require treatment. However, patients should be advised to avoid the
contaminated water that led to this condition.

If lesions persist, an PO fluoroquinolone may be considered.


………..
Although S aureus is the leading cause of purulent skin infection (and folliculitis), most
cases are sensitive to TMP-SMX and are marked by a purulent lesion with
surrounding erythema and induration. The presence of several papulopustular
lesions that did not respond to TMP-SMX make this pathogen less likely.

GAS (eg, S pyogenes) is the leading cause of cellulitis. Most cases present with
spreading erythema, warmth, and tenderness at a site of skin breakdown (eg, fungal
infection, wound). Purulent lesions are less common and generally indicate an
alternate pathogen.

Sporothrix schenckii is a dimorphic fungus found in soil that can occasionally cause
cutaneous infection after being introduced into the skin during gardening or other
outdoor activities. However, infections usually take weeks to develop and generally
present with an ulcerative papule at the site of inoculation; subsequent lesions form
along the proximal lymphatic chain. Bilateral papulopustular eruption on the lower
extremities makes Pseudomonas folliculitis far more likely.

V vulnificus is a estuarine bacteria that causes cellulitis following inoculation into a


wound. This organism lives in marine environments, not hotel pools, and usually
causes a spreading cellulitis similar to S pyogenes.

DIABETIC FOOT Foot infections are common in patients with DM. Risk is increased in those with poor
INFECTIONS glycemic control (impaired immunity), neuropathy (impaired recognition),
and/or PAD (impaired healing). Infections typically develop in areas of skin trauma or
defect; less noticeable portals of entry such as skin cracks/fissures, fungal infections, or
hangnails may make early recognition particularly problematic.
Symptoms usually start with localized skin erythema, warmth, tenderness, and
edema. Deeper infections should be suspected in those with long-standing wounds
(>1-2 weeks), systemic symptoms (fever, chills), large ulcer size (>2 cm), elevated
ESR, and the presence or palpation of bone in the ulcer base. Patients with any of
these features almost always have polymicrobial infections with a mixture of GP
(eg, S aureus, S pyogenes), GN (eg, P aeruginosa), and anaerobic organisms.

A patient with a long-standing ulcer, fever, and x-ray evidence of osteomyelitis likely
has a polymicrobial bone infection due to contiguous spread from the overlying
ulcer. He requires wound debridement, evaluation of his arterial insufficiency, and
empiric, IV antibiotics (eg, piperacillin-tazobactam PLUS vancomycin). Bone
biopsy with microbial culture is sometimes performed prior to antibiotic therapy, but
many patients are treated empirically.

Superficial diabetic foot infections may be monomicrobial, but deeper infections


are almost always polymicrobial. This patient has many signs (eg, size, chronicity,
osteomyelitis) of deeper infection. In adults, underlying osteomyelitis with an overlying
wound is almost always the result of contiguous spread. Hematogenous osteomyelitis
accounts for 20% of osteomyelitis cases in adults (it is much more common in children)
and primarily affects the vertebrae, pelvis, and hips. Lymphatic spread is not a
common route for the development of osteomyelitis.

……….

Patients with diabetes have multiple risk factors for the development of infected foot
ulcers, including:

• peripheral neuropathy (eg, decreased lower extremity sensation, as in this


patient): leading to impaired recognition of minor damage
• hyperglycemia (eg, elevated hemoglobin A1c, as in this patient): leading to
impaired immunity
• peripheral arterial disease: further contributing to impaired healing once a
wound is present

Contiguous (nonhematogenous) extension of infection from the ulcer to underlying


bone causes osteomyelitis. Because this complication may be asymptomatic, patients
with diabetic foot ulcers should be assessed for extent of infection. As seen in this
patient, the presence of a positiveprobe-to-bone test (palpating bone with a sterile,
blunt wooden or metal tool), a large ulcer (eg, >2 cm2), or an ulcer lasting ≥1 week
increase the likelihood of osteomyelitis. Fever, pain, an elevated ESR, and sinus tract
drainage (eg, purulent drainage) may be present.

The most specific diagnostic test for osteomyelitis is bone biopsy and culture. Bone
biopsy, preferably performed before antibiotic initiation, allows for identification of
the causative pathogens and tailoring of antibiotic therapy. Positive superficial
wound cultures do not reliably predict the culprit organisms; even among patients with
diabetes and foot ulcers who do not have osteomyelitis, wound base culture should be
performed following debridement and curettage (ie, aseptically scraping the wound
base with a blade or curette) rather than following a simple swab (which can be
contaminated).

……
Management of a foot ulcer complicated by osteomyelitis includes surgical
debridement of necrotic material and prolonged (ie, multiple weeks) antimicrobial
therapy. Amputation is the option of last resort and is used in instances of significant
complications (eg, ischemia, necrosis) or when medical management fails.
Although an elevated ESR (eg, >70 mm/hr) is suggestive of osteomyelitis in the
appropriate clinical setting, it can be increased by any significant inflammatory process
and therefore is nonspecific. In osteomyelitis, an ESR is helpful in following the
response to antibiotic treatment; however, the diagnosis should ideally be established
via tissue biopsy and the culprit organism identified via microbiologic examination.
Although patients with diabetes are at increased risk for superficial fungal
infections, invasive fungal infections are an uncommon cause of osteomyelitis, so
fungal cultures are not a part of the standard evaluation. Routine blood cultures can
be helpful in patients with osteomyelitis due to hematogenous spread (eg, infective
endocarditis) but are less helpful in disease due to direct extension of local infection.

LEMIRRE Lemierre syndrome (LS) is a severe life-threatening infection that affects young
SYNDROME (LS) immunocompetent patients and is usually caused by the GN anaerobic
/////////////////// bacillus Fusobacterium necrophorum. F. necrophorum is part of the normal oral
***** flora. LS usually begins with an oropharyngeal infection, usually tonsillitis, but can also
arise as a complication from dental work or mastoiditis. The bacterium invades the
lateral pharyngeal space through the lymphatic system and affects the neurovascular
structures, causing internal jugular vein thrombosis (neck pain along SCM
border) and infection.

The classic presentation is a prolonged (eg, weeklong) duration of sore throat with
high fever, rigors, dysphagia, and neck pain and swelling along the
sternocleidomastoid muscle. Once the internal jugular vein is infected, septic
thromboemboli can seed other organs, particularly the lungs, causing respiratory
symptoms and leading to nodules (± cavitations) on CXR. LS should be considered in
any toxic-appearing patient with respiratory difficulties and significant neck swelling
or tenderness in the week following an oropharyngeal infection.

Microbiologic diagnosis requires culture from blood or pus. In addition to


supportive (eg, airway) management, treatment is with IV antibiotics and, possibly,
surgery (eg, incision and drainage, vein excision) in patients with no response to
antibiotics.

…………….
C. perfringens is a GP bacillus that commonly causes food poisoning and gas gangrene.

Prior to the use of the conjugate vaccine against H influenzae serotype b (Hib), the
organism was an important cause of epiglottitis, which can present with febrile
pharyngitis and progress to respiratory compromise. Hib infection can also lead to
bacteremia.

K pneumoniae is a GN rod that is a rare cause of CAP. A classic presentation is


pneumonia with thick "currant jelly" sputum in alcoholics or patients with DM.

GAS causes many cases of bacterial pharyngitis. However, tonsillar exudates are
classically present and patients are not typically severely toxic-appearing.

CATHETER- Catheter-associated urinary tract infection (CA-UTI) is a common complication of


ASSOCIATED urinary catheter use and can be caused by:
URINARY TRACT
INFECTION (CA- • Extraluminal ascent of microorganisms due to the ability of some pathogens
UTI) (eg, E coli, Enterococcus spp, Candida spp, P aeruginosa) to form biofilm
(slime-enclosed bacterial aggregates) along the catheter wall, allowing them to
reach the bladder within 24 hours of catheter insertion
• Intraluminal infection (less common) due to impaired urinary catheter drainage
or contamination of a urinary collection bag

CA-UTI is most effectively prevented by avoiding unnecessary catheter use and


minimizing the duration of catheterization. However, in patients with neurogenic
bladder, long-term catheter use is required. In these patients, clean intermittent
catheterization (CIC), which involves periodic insertion and removal (eg, every 4-6
hours) of a clean urinary catheter and can often be performed by the patient, is usually
the initial treatment. CIC interrupts the extraluminal and intraluminal mechanisms of
infection and, compared with the use of indwelling catheters, is associated with a
significantly lower risk of CA-UTI. Indwelling catheters (changed monthly) can be
considered if patients or their caregivers cannot perform CIC but are associated with an
increased risk of UTI, stricture, and bladder spasm; suprapubic tube placement is
another option.

………..

Bladder irrigation with antibacterial solutions can lead to the emergence of resistant
bacteria and may increase the risk of UTI due to the killing of nonpathogenic bacteria.

The application of antibacterial creams to the urethral meatus or antibacterial washes


to the external genitalia has not been found to be helpful in decreasing the risk of CA-
UTI.
In patients using a urinary catheter, antibiotics should be administered only in the
setting of a proven UTI. Prophylactic antibiotics may increase the risk of development
of resistant organisms and have not been reliably demonstrated to decrease the risk of
CA-UTI.

SPLEEN RUPTURE
***** Atraumatic splenic rupture
///////////////////
• Hematologic malignancy (eg, leukemia, lymphoma)
***** • Infection (eg, CMV, EBV, malaria)
Risk factors • Inflammatory disease (eg, SLE, pancreatitis)
• Splenic congestion (eg, cirrhosis, PREGNANCY)
• Medications (eg, anticoagulation, G-CSF)

• Diffuse or LUQ abdominal pain, peritonitis


Clinical
• Referred left shoulder pain (Kehr sign)
presentation • Hemodynamic instability

• Acute anemia
Diagnosis • Intraperitoneal free fluid on imaging

• Catheter-based angioembolization (stable patients)


Treatment • Emergency splenectomy (unstable patients)

CMV = cytomegalovirus; EBV = Epstein-Barr virus; G-CSF = granulocyte-stimulating colony


factor; LUQ = left upper quadrant; SLE = systemic lupus erythematosus.
A patient with acute-onset abdominal pain,
shock, and acute anemia likely has splenic
rupture, an uncommon but life-threatening
complication of chronic lymphocytic
leukemia (CLL). Most cases of rupture are
associated with BAT; however, atraumatic
rupture can also occur. Common
predisposing factors for atraumatic rupture
include hematologic malignancy (eg,
leukemia, lymphoma), infection (eg, EBV
mononucleosis), and inflammatory disease
(eg, SLE). These conditions can lead to
splenomegaly, derangement of the splenic
architecture, and/or thinning of the
splenic capsule, increasing the risk of rupture. In the setting of concomitant
anticoagulation (eg, apixaban for atrial fibrillation) the chances of rupture are greatly
increased.
Examination may reveal hemodynamic instability, diffuse abdominal pain, and signs
of peritonitis (eg, rebound, guarding). On occasion, pain may be referred from the
diaphragm and phrenic nerve to the left shoulder (Kehr sign). Focused abdominal
ultrasound or CT scan demonstrates free intraperitoneal fluid.
Splenectomy is the management of choice in hemodynamically unstable patients,
although catheter-based angioembolization may be attempted in stable patients.

………..
Bowel perforation can present with diffuse abdominal pain, signs of peritonitis,
and shock; however, it would not explain an acute drop in hemoglobin.

Acute colonic ischemia presents with abdominal pain, but the pain is typically sudden,
cramping, and localized. Tenesmus and rectal bleeding or bloody diarrhea may also
follow.
Mesenteric artery occlusion causes abdominal pain, but examination is usually
unrevealing with pain being out of proportion to findings. Occlusion can occur either
from ruptured plaque with in situ thrombus or from a thromboembolic event (eg, due
to atrial fibrillation). The diagnosis is less likely in a patient who is taking apixaban as
prescribed (coagulation parameters [eg, PT/INR, PTT] are not reliable indicators of
direct oral anticoagulant activity).
Necrotizing pancreatitis, similar to other severe inflammatory states, can predispose
to splenic rupture, but patients usually first have signs and symptoms of acute
pancreatitis such as fever, tachypnea, and severe epigastric pain radiating to the
back. Syncope and acute anemia are uncommon.
Septic shock can present with hemodynamic instability and leukocytosis, but
a neutrophil-predominant leukocytosis is expected. This patient's lymphocyte-
predominant leukocytosis is consistent with underlying CLL.

SPLENIC ABSCESS This patient's


presentation is consistent
with a likely splenic
abscess, which classically
causes the triad of fever,
leukocytosis, and left
upper-quadrant
abdominal pain (due to a
splenic fluid collection).
Patients can also develop
left-sided pleuritic chest
pain, left pleural effusion,
and splenomegaly. Risk
factors for splenic abscess
include infection (eg,
infective endocarditis) with
hematogenous spread,
immunosuppression, IV
drug use, trauma, and hemoglobinopathies.

Diagnosis is usually confirmed by CT scan of the abdomen.


Splenic abscess is typically due to infective endocarditis (or seeding from another site
of infection). Likely mechanisms include hematogenous seeding or septic emboli to
the spleen. Common causative organisms are Staph, Strep, and Salmonella.
Antibiotics alone for treating splenic abscess have a high mortality (up to 50% in
some studies). As a result, splenectomy is recommended for all
patients. Percutaneous drainage may be an option in poor surgical candidates.

Patients with preexisting structural cardiac conditions are at increased risk of


endocarditis. There should be strong suspicion for endocarditis in a patient who has
MVP (associated with a fivefold increased risk of endocarditis at least), fever with
systemic manifestations (eg, malaise, weight loss), and splenic abscess (likely due to
septic emboli); therefore, to complete the evaluation for Duke criteria, blood cultures
should be obtained and echocardiography should be performed, particularly if blood
culture results are positive.

……….

Malaria can cause fever, chills, and fatigue; anemia, thrombocytopenia, and jaundice
are also common. Splenic abscess is not seen with malaria although splenomegaly
can be present.
Hodgkin lymphoma can present with fever, chills, and weight loss. Lymphoma can
also be associated with splenomegaly but typically does not cause splenic abscess or
infarction.
Infectious mononucleosis is due to the Epstein-Barr virus. Patients usually develop
LAD, fever, and possible HSM. However, infectious mononucleosis is usually not
associated with left pleural effusion or splenic abscess. It is also uncommon in this age
group.

Lung adenocarcinoma can present with fever, weight loss, dyspnea, and pleural
effusion, as seen in this patient. However, lung cancer usually metastasizes to the liver,
bone, brain, and adrenal glands. Splenic abscess is not seen in the absence of an
additional infectious process.

Tuberculosis can present with fever and weight loss. Gastrointestinal tuberculosis
most commonly involves the liver, intestine, and peritoneum. However, splenic
involvement is less common, and symptoms are more chronic in nature.

LUDWIG ANGINA Ludwig angina is a rapidly progressive cellulitis of the submandibular space. Most
cases arise from dental infections in the mandibular molars that spread contiguously
down the root into the submylohyoid (and then sublingual) space. The infection is
usually polymicrobial with a mixture of oral aerobic (eg, viridans streptococci) and
anaerobic bacteria.
Patients develop symptoms rapidly with systemic (fever, chills, malaise) and local
compressive (eg, mouth pain, drooling, dysphagia, muffled voice, airway
compromise) manifestations. Physical examination findings are often striking due to
mass effect from edema. The submandibular area is usually tender and indurated, and
the floor of the mouth is often elevated, displacing the tongue. Anaerobic, gas-
producing bacteria may cause crepitus.
CT scan of the neck confirms the diagnosis and rules out an abscess.

Most patients are treated with:

- IV antibiotics (eg, ampicillin-sulbactam, clindamycin) and

- Removal of the inciting tooth.

- Drainage and surgery are rarely required as the process is cellulitic and
typically nonsuppurative.

- Impaired respiratory status requires prompt attention and early intervention


with a mechanical airway.

……..

Hematogenous spread is a common cause of endocarditis, osteomyelitis, and septic


joints.
Epiglottitis can cause life-threatening airway obstruction with sore throat, fever,
muffled voice, and drooling. However, this patient has swelling on the floor of the
mouth and in the submandibular area (not in the supraglottic structures or laryngeal
tracheal area), making Ludwig angina more likely.

LAD is typically absent in Ludwig angina. Lymph nodes can rarely cause airway
obstruction in tuberculosis lymphadenitis (scrofula).

Tonsillitis may rarely cause airway obstruction with hoarseness and stridor. However,
the palatine tonsils would not result in inflammation beneath the tongue and in the
submandibular area.

The parotid gland is located on the lateral face superficial to the masseter muscle. It is
not affected in Ludwig angina. Parotid gland edema may be seen with mumps or
secondary bacterial infection after parotid duct obstruction.

CYROMEGALOVIR Patients who undergo renal transplantation receive high-dose immunosuppressant


US (CMV) medication for several months after surgery, putting them at risk for opportunistic
infections. One of the MC opportunistic infections following renal transplantation
is cytomegalovirus (CMV). CMV is a widely prevalent herpesvirus that establishes a
latent infection after initial inoculation. Most patients remain asx, but infection may
reemerge in patients who are severely
immunocompromised.
CMV reactivation results in viremia and/or
tissue-invasive disease. The GI tract is the MC
organ system affected, and patients usually
manifest symptoms of colitis or enteritis,
including fever, malaise, vomiting, bloody
diarrhea, and abdominal pain. Laboratory
studies often show cytopenias due to BM
involvement, and peripheral blood smear
typically reveals atypical lymphocytes.

As end-organ disease can occur in the absence of viremia, BIOPSY of the affected
organ is usually required for diagnosis. Patients with symptoms of CMV colitis
typically have multiple, large, shallow erosions or ulcers on colonoscopy.

Treatment with antivirals (eg, ganciclovir) and a reduction of immunosuppressant


medication is usually required.
…….
Initial infections with both EBV and CMV may cause infectious mononucleosis, and
both viruses establish latent infections; however, EBV reactivation is typically
associated with malignancies (eg, nasopharyngeal carcinoma, NHL), not colitis.

Although graft rejection can occur after kidney transplantation, GVHD is quite rare.
GVHD is typically seen in patients who have undergone allogeneic stem-cell
transplantation.

Patients who have undergone transplantation are at risk for herpes simplex
reactivation with resultant tracheobronchitis, esophagitis, pneumonia, and/or
hepatitis; colitis is uncommon. This patient is much more likely to have CMV
reactivation.

Ischemic colitis can cause abdominal pain and bloody stools; however, atypical
lymphocytes are not a feature of this disease and endoscopy usually reveals pale
mucosa with petechial bleeding, hemorrhagic nodules, or cyanotic mucosa with
hemorrhagic ulcers.

////////////////////ENDOCRINE, DIABETES, & METABOLISM//////////////////////


PHEOCHROMOCY
Von Hippel-Lindau (VHL) disease
TOMA
Etiology • Germline mutation in the VHL tumor suppressor gene on chr. 3

• Cerebellar & retinal hemangioblastomas


Manifestation
• Pheochromocytoma
s
• RCC (clear cell subtype)

• Surveillance screening for associated malignancies


Management
• Resection of identified tumors
A patient has vision loss and headaches due to retinal and CNS hemangioblastomas,
which are benign but heavily vascular tumors that can lead to hemorrhage or mass
effect symptoms in the brain and spinal cord. Most hemangioblastomas are sporadic;
however, this patient has multiple hemangioblastomas and a FHx of ICH, which
suggests an inherited disorder, likely von Hippel-Lindau disease (vHL).
vHL is an autosomal dominant disorder associated with mutations in
the VHL gene. Besides hemangioblastomas, vHL is associated with clear cell renal cell
carcinoma, pancreatic neuroendocrine tumors (PNET), endolymphatic sac tumors of
the middle ear (hearing loss), and pheochromocytomas. Pheochromocytomas (arising
in the adrenal medulla) and paragangliomas (in the sympathetic ganglia) are
neuroendocrine tumors (NET) associated with excess production of catecholamines,
leading to headache, palpitations, and potentially severe HTN.

As with hemangioblastomas, most pheochromocytomas and paragangliomas are


sporadic. However, a significant minority occur as a manifestation of a familial
disorder. In addition to vHL, major genetic syndromes associated with catecholamine-
secreting tumors include MEN type 2 (2A and 2B) and NF type 1.

……………

Primary hyperaldosteronism (Conn syndrome) can be caused by an adrenal adenoma


or bilateral adrenal hyperplasia. It typically presents with HTN but is not associated
with hemangioblastomas.

ADPKD can cause HTN due to local renal ischemia and increased renin release. It is
associated with an increased risk of ICH due to cerebral aneurysms (not
hemangioblastomas). Although vHL may be associated with the formation of renal
cysts, this is not a major contributor to HTN.

All pheochromocytomas should undergo surgical resection due to the risk for
complications caused by cahecolamine surges and the potential for malignant disease
(approximately 10% of pheochromocytomas are malignant). Surgical removal of a
pheochromocytoma is a high-risk procedure because intraoperative cahecolamine
surges can precipitate hypertensive crisis, pheochromocytoma crisis (a condition
characterized by labile BP, high fever, and MOF), and fatal arrhythmias. To help
prevent these complications, appropriate adrenergic blockade is needed prior to
surgery.

The order of administration of adrenergic blockade is critical. Beta-blocker therapy


in the absence of alpha-adrenergic blockade can lead to lethal hypertensive crisis due
to unopposed alpha-1 stimulation. Therefore, alpha-adrenergic blockade should be
administered first, usually 7-14 days prior to surgery. Phenoxybenzamine, an
irreversible and nonselective alpha blocker, is often preferred; however, selective
alpha-1 blockers (eg, terazosin) are also sometimes used. Patients should also be
encouraged to increase sodium intake and stay well-hydrated to minimize possible
side effects of alpha-adrenergic blockade (eg, orthostasis, hypotension).
Once adequate alpha-adrenergic blockade is established, beta-adrenergic blockade
(eg, propranolol) can be safely initiated, usually 2-3 days prior to surgery.

………….

Octreotide is a somatostatin analogue used in carcinoid syndrome (eg, diarrhea,


flushing, elevated urinary 5-hydroxyindoleacetic acid [5-HIAA]). It has unclear effects
on slowing the growth of malignant pheochromocytomas, which are histologically
identical to benign pheochromocytomas but show evidence of metastasis or
localized tissue invasion.

Pheochromocytoma

• Classic triad: episodic headache, sweating & tachycardia


Indications for
• Resistant HTN or HTN accompanied by unexplained ↑ glucose
testing • Family history or familial syndrome (eg, MEN2, NF1, VHL)

• Urine or plasma metanephrine levels


Diagnostic approach • Confirmatory abdominal imaging for ↑ metanephrines

• 10% bilateral (unilateral), 10% extraadrenal (adrenal), 10%


Notable features malignant (benign)

• Preoperative alpha blockade prior to beta blockade


Management • Laparoscopic or open surgical resection

HTN = hypertension; MEN2 = multiple endocrine neoplasia type 2; NF1 = neurofibromatosis type
1; VHL = von Hippel-Lindau syndrome.

Hypertension, episodic headaches, and unexplained hyperglycemia should raise


suspicion for pheochromocytoma. Pheochromocytomas are rare neuroendocrine
tumors (NET) that arise from chromaffin cells and release excess catecholamines (ie,
dopamine [DA], epinephrine [E], norepinephrine [NE]) into the bloodstream, resulting
in characteristic manifestations. Headaches are often episodic and present in 90% of
patients. Paroxysmal HTN occurs in approximately half of patients and may be
associated with orthostasis (possibly reflecting low plasma volume). Vision changes,
coinciding with episodes of headache and hypertension, can also occur.
Because catecholamines inhibit insulin secretion, hyperglycemia is also
common. Therefore, pheochromocytoma should be considered in patients who
develop hyperglycemia that is atypical for both type 1 (ie, usually presents with DKA)
and type 2 (ie, rarely occurs in patients age <35 with normal BMI) DM, especially
when accompanied by other suggestive features, such as onset of HTN (particularly at
a young age).

Other indications for screening include: the classic triad of headache, sweating, and
tachycardia; resistant hypertension; or a familial syndrome (eg, MEN type 2).
The initial step in evaluation for pheochromocytoma involves testing for urine or
plasma levels of metanephrines, which are products of catecholamine breakdown.
Normal levels typically rule out the disease; elevated levels are nonspecific and
warrant additional testing, such as CT scan of the abdomen, to confirm the presence
and localization of the suspected pheochromocytoma (usually on one or both adrenal
glands).

………..

CT scan of the head is typically indicated for headaches only when red-flag features
(eg, FNDs, fever, new-onset headache in patient age >50) are present.

DXM suppression testing is indicated for suspected hypercortisolism (ie, Cushing


syndrome). Although HTN and HrGlc are common, patients are usually overweight
and have characteristic findings on physical examination (eg, purple abdominal
striae, fat pad on the upper back).
Renal vascular imaging may help diagnose FMD, which typically presents with
headaches and can cause HTN. However, FMD usually occurs in women and would not
explain the hyperglycemia.

A serum glucagon level is indicated for suspected glucagonoma. Although a


glucagonoma would cause hyperglycemia, it would not explain HTN, episodic
headaches, and vision changes.

ADRENAL
Acute adrenal insufficiency (adrenal crisis)
INSUFFICIENCY
• Adrenal hemorrhage or infarction
Etiology • Illness/injury/surgery in patient with chronic AI
• Pituitary apoplexy

• Hypotension & shock


Clinical features • N, V, abdominal pain
• Fever, generalized weakness

• Hydrocortisone or DXM
Treatment • Rapid IV volume repletion

AI = adrenal insufficiency.

Acute adrenal insufficiency (adrenal crisis) is characterized by severe and often


refractory HoTN, V, abdominal pain, and fever. It can be triggered by acute illness or
other major stressor (eg, surgery) and occurs in patients with underlying primary
adrenal insufficiency (PAI; Addison disease) or suppression of the hypothalamus-
pituitary-adrenal (HPA) axis due to chronic glucocorticoid use. Those with PAI usually
have HoNa and HrK due to concurrent mineralocorticoid insufficiency. By contrast,
potassium is normal in patients with central adrenal insufficiency although HoNa
can be present due to increased release of ADH. Treatment for adrenal crisis includes
IV hydrocortisone or DXM with aggressive fluid support and should be started
empirically without waiting for laboratory confirmation.
Patients on long-term glucocorticoid therapy who have Cushingoid features (eg,
central obesity, moon facies) are at very high risk of adrenal crisis. However, HPA
suppression can be seen in as little as 3 weeks in patients taking prednisone >20
mg/day or equivalent. These patients may not respond appropriately to acute stressors
and should be given higher doses ("stress doses") of glucocorticoids during the
acute condition. Low-dose glucocorticoids (<5 mg/day) have minimal risk and
usually do not require stress dosing. Intermediate doses (5-20 mg/day) can cause
HPA suppression and require preoperative evaluation with an early-morning cortisol
level to determine risk.

……………

Lactic acidosis can be seen in patients taking metformin but would also show an
elevated anion gap.

Hypoglycemia is common in adrenal crisis. Insulin-induced HoGlc can cause


dizziness but typically causes systolic HTN with widened pulse pressure.
Pelvic abscess usually presents several days to weeks after surgery.
Postoperative hemorrhage can cause acute HoTN and tachycardia but would not
explain this patient's hypoglycemia, nausea, and vomiting.

Postoperative ileus can cause abdominal pain and vomiting but symptoms are
initially mild and nonspecific, and evolve over several hours to days.

HYPONATREMIA
Syndrome of inappropriate antidiuretic hormone (SIADH)
(HoNa)
• CNS disturbance (eg, stroke, hemorrhage, trauma)
• Medications (eg, carbamazepine, SSRIs, NSAIDs)
Etiologies • Lung disease (eg, pneumonia)
• Ectopic ADH secretion (eg, small cell lung cancer)
• Pain &/or nausea

• Mild/moderate hyponatremia: nausea, forgetfulness


Clinical features • Severe hyponatremia: seizures, coma
• Euvolemia (eg, moist mucous membranes, no edema, no JVD)

• Hyponatremia
Laboratory • Serum osmolality <275 mOsm/kg H2O (hypotonic)
findings • Urine osmolality >100 mOsm/kg H2O
• Urine sodium >40 mEq/L

• Fluid restriction ± salt tablets


Management
• Hypertonic (3%) saline for severe hyponatremia

ADH = antidiuretic hormone; JVD = jugular venous distension; NSAIDs = nonsteroidal anti-
inflammatory drugs; SSRIs = selective serotonin reuptake inhibitors.

A patient's acute nausea and vomiting are likely due to hyponatremia induced by
recent use of intranasal desmopressin. Desmopressin is an analogue of antidiuretic
hormone (ADH) commonly used to treat central diabetes insipidus (CDI); it also
promotes the release of von Willebrand factor (vWF), and can treat mild to moderately
heavy menstrual bleeding associated with von Willebrand disease (vWD). However,
since desmopressin induces physiologic effects of ADH (eg, renal water reabsorption,
concentration of urine), it can cause hyponatremia and excess ADH stimulation,
manifesting as syndrome of inappropriate ADH secretion (SIADH). Therefore,
serum electrolytes should be obtained first in this patient to check for hyponatremia
and rule in the diagnosis of SIADH.

Expected laboratory findings in patients with SIADH include hypotonic


hyponatremia (due to ADH-mediated water retention and reabsorption), high urine
osmolality (ie, concentrated urine), and high urine sodium (due to increased secretion
of natriuretic peptides). As seen in this patient, SIADH is also associated
with euvolemia (eg, absence of edema or JVD) on physical examination. At first, ADH-
mediated water reabsorption slightly increases extracellular volume; however, increased
natriuretic peptide secretion leads to sodium and water excretion, ultimately
normalizing extracellular volume.

………
MRI of the brain can be used to assess for CNS pathology (eg, bleeding, acute stroke)
that could manifest with headache or dizziness. However, this patient lacks focal
neurologic signs; therefore, a metabolic workup with serum electrolytes is a more
appropriate initial test.

Plasma desmopressin is not routinely measured. It may be high in this patient, given
her recent medication use with the onset of menses, but serum electrolytes are a more
rapid initial step for confirming hyponatremia and SIADH.

THYROID Thyroid nodules are common


NODULES and may be diagnosed on
physical examination or noted
incidentally when imaging
studies are obtained for other
reasons. Once a thyroid nodule
is found, cancer risk factors
(eg, family history, radiation
exposure) should be assessed
by history. Physical examination
should evaluate the size,
mobility, and firmness of the
thyroid nodule and check for
the presence of enlarged
cervical lymph nodes.
Serum TSH levels should be
obtained, along with a thyroid
ultrasound, to determine nodule size and sonographic features. Certain sonographic
features (eg, microcalcifications, irregular margins, internal vascularity) carry a
much higher risk of malignancy than others (eg, cystic or spongiform lesions). Thyroid
nodules >1 cm with these high-risk sonographic features—and all noncystic thyroid
nodules >2 cm—should undergo FNA.
If the TSH is low, the patient should be evaluated with radionuclide thyroid scan using
I-123:

• A hypofunctioning ("cold") nodule (decreased isotope uptake compared to


surrounding tissue) is associated with a higher risk of cancer.
• A hyperfunctioning ("hot") nodule (increased isotope uptake in the nodule
with decreased surrounding uptake) is associated with a low cancer risk;
therefore, FNA is not necessary.

A patient who has a thyroid nodule with no concerning sonographic features and a low
TSH should receive a radionuclide thyroid scan.

………….

A CT scan of the neck is inferior to ultrasound for characterizing thyroid nodules. Even
if a thyroid nodule is detected on CT scan, a thyroid ultrasound is still needed.
Anti-thyroperoxidase antibody (TPO-Ab) testing can identify chronic autoimmune
(Hashimoto) thyroiditis, but it is used for patients with high TSH levels (eg,
hypothyroid).

Calcitonin is an important tumor marker in medullary thyroid carcinoma (MTC). In


patients with a thyroid nodule, testing for calcitonin should be performed if there is a
family history of MTC or if cytologic examination (eg, FNA) of the nodule is
consistent with MTC.

Graves disease typically causes a diffusely enlarged goiter rather than a focal nodule.

THYROID Surgical resection is the primary treatment for papillary thyroid cancer. The extent of
CANCER surgery depends on the tumor size, extrathyroidal invasion, patient age, and LN
involvement. Small tumors (<1-2 cm) without evidence of LN involvement or
contralateral thyroid abnormalities can usually be managed with partial
thyroidectomy/lobectomy, whereas larger tumors usually warrant total
thyroidectomy. In addition, regional LN or central compartment LN dissection is
usually performed for papillary cancer with LN metastasis or extrathyroidal
invasion.
Following surgery, adjuvant therapy with radioiodine ablation is warranted for
patients with increased risk of tumor recurrence (eg, large tumors, extrathyroidal
invasion, LN metastasis, incomplete resection). TSH can stimulate growth of occult
residual or metastatic disease. For this reason, patients at increased risk of recurrence
should also receive adequate doses of thyroid replacement to suppress TSH
secretion.

……….

Mutations of several genes (eg, BRAF, RET) are common in papillary thyroid
cancer. Genetic mutation profiling may be performed when the cytology results
from FNA biopsy are indeterminate but are not needed if cytology confirms papillary
cancer.

HYPERTHYROIDIS
M

Patients with a thyroid


nodule and a low
TSH undergo a radioiodine
scan, or thyroid
scintigraphy, to determine
how thyroid nodules are
functioning based on iodine
uptake. The scan measures
the amount of uptake (ie, the amount of thyroid hormone production) in the nodule
compared to surrounding normal thyroid tissue.
This patient's scan shows a hyperactive or "hot" nodule, which indicates that the
nodule has increased synthesis of thyroid hormones and therefore increased uptake
of radioactive iodine compared to the rest of the thyroid. This pattern is consistent
with a toxic thyroid adenoma, a benign tumor that produces thyroid hormones
autonomously (ie, regardless of changes in serum TSH). In patients with toxic
adenomas, the increased thyroid hormone levels exert negative feedback to decrease
TSH secretion. The normally functioning thyroid gland responds to suppressed levels
of TSH by reducing thyroid hormone synthesis (and takes up much less iodine). This
difference in uptake between the metabolically active nodule and the downregulated
normal thyroid tissue leads to the characteristic "hot" nodule.
……….

Factitious ingestion of thyroid hormone would cause a low serum


TSH; however, the radioiodine scan would show homogeneous
decreased uptake throughout the gland.

 Graves disease
Medullary thyroid carcinoma (MTC): These cells do not produce
T3 or T4, and they do not participate in the regulation of TSH
secretion; therefore, patients with MTC typically have normal
thyroid function tests. Further, MTC does not uptake iodine
and would appear "cold" on a radioiodine scan.

 Although papillary thyroid cancers usually uptake iodine


and produce thyroid hormone, it is typically at a level less than
normal thyroid tissue → cold nodule

Thyroid storm, a life-threatening


thyrotoxicosis usually triggered by a
specific event (eg, thyroid or non-
thyroid surgery, trauma, infection)
in patients with undiagnosed or
inadequately treated
hyperthyroidism. Proposed
mechanisms include a rapid
increase in serum thyroid
hormone levels or increased
sensitivity to thyroid hormone.
Characteristic features include
tachycardia, hypertension, cardiac
arrhythmias (eg, atrial fibrillation),
and fever up to 40-41 C (104-106
F). Other findings include anxiety,
AMS, seizure, severe N, V, hepatic
dysfunction, tremor, lid lag, and
goiter.

Diagnosis is suspected clinically and confirmed by thyroid function studies


documenting hyperthyroidism. Treatment includes BBs (eg, propranolol) for symptom
control, thionamides (eg, PTU) to block new hormone synthesis, iodine solution to
block thyroid hormone release (given at least an hour after PTU to prevent excess
iodine incorporation into thyroid hormone), and glucocorticoids to decrease
peripheral conversion of T4 to T3.
………..

Fluid resuscitation
and intravenous dexamethasone
should be given to patients
with suspected acute adrenal
insufficiency. These patients will
have hypotension often
associated with tachycardia,
abdominal pain, vomiting, and
weakness.

IV dantrolene can treat malignant


hyperthermia (MH), a
hypermetabolic syndrome due to
volatile anesthetics. MH typically
presents with hypercarbia, sinus
tachycardia, muscle rigidity,
elevated CK, hyperkalemia,
and hyperthermia. This patient's
lid lag, normal potassium, and
absence of muscle rigidity are more consistent with hyperthyroidism.

IV lorazepam can treat alcohol withdrawal, which usually presents with tachycardia,
HTN, fever, and AMS. A history of moderate alcohol use makes withdrawal less
likely.

Pheochromocytoma is usually not associated with high fever.

HYPERPARATHYR
Primary hyperparathyroidism
OIDISM (HPTH)
• Parathyroid adenoma (most common), hyperplasia, carcinoma
Etiology • Increased risk in MEN types 1 & 2A

• Asymptomatic (MC)
Symptoms • Mild, nonspecific symptoms (eg, fatigue, constipation)
• Abdominal pain, renal stones, bone pain, neuropsychiatric symptoms

• HrCa
Diagnostic
• ↑ or inappropriately N PTH
findings • Elevated 24-hour urinary calcium excretion

• Age <50
Indications for • Symptomatic hypercalcemia
• Complications: Osteoporosis (T-score <−2.5, fragility fracture),
parathyroidecto nephrolithiasis/calcinosis, CKD (GFR <60 mL/min)
my • Elevated risk of complications: Calcium >1 mg/dL above
normal, urinary calcium excretion >400 mg/day
CKD = chronic kidney disease; GFR = glomerular filtration rate; MEN = multiple
endocrine neoplasia; PTH = parathyroid hormone.

A patient has hypercalcemia, hypophosphatemia, and an elevated parathyroid


hormone (↑ PTH) level consistent with primary hyperparathyroidism (PHPT). PTH
causes increased reabsorption of calcium from the distal tubule, but net urinary calcium
excretion is increased due to excess resorption of calcium from bones. By contrast,
familial hypocalciuric hypercalcemia is characterized by hypercalcemia with an elevated
PTH but low urinary calcium excretion (often <100 mg/24 hr).

Parathyroidectomy is recommended for patients with symptomatic hypercalcemia.


It is also recommended for patients with complications (eg, osteoporosis,
nephrocalcinosis, nephrolithiasis, impaired renal function) or at significant risk for
complications (eg, serum calcium ≥1 mg/dL above normal, urinary calcium excretion
>400 mg/24 hr). In addition, younger patients (age <50) are likely to have
complications during their lifetime and should be offered surgery.
Parathyroid imaging (eg, sestamibi scan, USG) helps optimize the surgical approach by
potentially determining the affected side and evaluating for the possibility of a
minimally invasive intervention.
……………..

Parathyroidectomy is the treatment of choice for PHPT. However, bisphosphonate


therapy can be used in patients who decline surgery and have osteopenia or
osteoporosis.

Imaging to evaluate for MEN syndromes is recommended only if there is evidence of


multiglandular HPTH or additional endocrine tumors.
Continued observation is appropriate in older asymptomatic patients with near-
normal calcium levels and normal bone density.

Vitamin D supplementation is not recommended in patients with PHPT who do not


have overt vitamin D deficiency as it may exacerbate hypercalcemia and
hypercalciuria.
HYPERCALCEMIA
(HrCa)

A patient with quadriparesis has a corrected calcium level of 12.9 mg/dL:

corrected calcium = measured calcium + 0.8 × (4 – albumin)


12.1 + 0.8 × (4 – 3) = 12.1 + 0.8 = 12.9 mg/dL
with symptomatic hypercalcemia (nausea, polyuria) and suppressed parathyroid
hormone (PTH) level likely due to immobilization. Hypercalcemia of immobilization is
likely due to increased osteoclastic bone resorption. The risk is increased in patients
with a pre-existing high rate of bone turnover (eg, younger individuals, Paget
disease). The onset of hypercalcemia is usually around 4 weeks after immobilization,
although patients with chronic renal insufficiency may develop hypercalcemia in as
little as 3 days.

The onset of hypercalcemia due to immobilization is often insidious, and the


presenting symptoms can be nonspecific. Bisphosphonates inhibit osteoclastic bone
resorption and are effective in treating hypercalcemia of immobilization and reducing
the associated bone loss.

……

Hypercalcemia in malignancy can be due to osteolytic metastasis, secretion of PTH-


related protein (PTHrP), or increased formation of 1,25-dihydroxyvitamin D.
Acute rhabdomyolysis causes hypocalcemia due to precipitation of calcium and
phosphorus in damaged muscles. Hypercalcemia can occur due to remobilization of
calcium during the diuretic/recovery phase of the illness, often with concurrent
hyperphosphatemia. However, this patient recovered from rhabdomyolysis several
weeks before the onset of hypercalcemia.

HYPOPARATHYR Post-surgical hypoparathyroidism can occur during thyroidectomy or from the


OIDISM (HoPTH) removal of 3 1/2 parathyroid glands due to parathyroid hyperplasia. Hypocalcemia is
the resultant condition of the hypoparathyroidism and the MC complication of
thyroidectomy. Patients with hypocalcemia can be asx at initial presentation (found
incidentally) or may complain only of nonspecific symptoms (eg, fatigue, anxiety,
depression). Involuntary contractions (tetany) involving the lips, face, and
extremities and seizures are seen in severe hypocalcemia. ECG may show QT-interval
prolongation.

………

Persistent hypothyroidism (not hyper) can be seen post-thyroidectomy and can


cause hyponatremia without thyroid hormone supplementation. Prolonged QT
interval is not seen with hypothyroidism/hyponatremia.
Vitamin D toxicity is associated with hypercalcemia, which results in shortened QT
interval.

HYPOCALCEMIA
Acute hypocalcemia

• Neck surgery (parathyroidectomy)


• Pancreatitis
• Sepsis
Causes • TLS
• Acute alkalosis
• Chelation: blood (citrate) transfusion, EDTA, foscarnet

• Muscle cramps
• Chvostek & Trousseau signs
Clinical features • Paresthesias
• Hyperreflexia/tetany
• Seizures

Treatment • IV calcium gluconate/chloride

EDTA = ethylenediaminetetraacetic acid; IV = intravenous.


A patient has paresthesias, carpal spasm (Trousseau sign), and hyperreflexia. In the
setting of high-volume transfusion, these neurologic features are most likely due
to hypocalcemia. Other potential features of severe hypocalcemia include
tetany, laryngospasm, seizures, encephalopathy, and heart failure. Patients
with chronic hypocalcemia may have only mild symptoms, but patients with
acute hypocalcemia will have more
severe manifestations at the same
calcium concentration.

Citrate in transfused
blood binds ionized calcium, which is
the biologically active fraction (total
calcium levels will not be significantly
affected). Hypocalcemia is uncommon
following blood transfusion in patients
with normal liver function as citrate is
rapidly metabolized by the liver;
however, this patient likely has hepatic
dysfunction due to the liver laceration as well as ischemic liver injury due to
HoTN. Other infused substances that can chelate calcium in the blood
include lactate, foscarnet, and sodium ethylenediaminetetraacetic acid (EDTA).

…………
Alcohol withdrawal is most pronounced 12-48 hours following the last drink. Clinical
features may include hyperreflexia, tremor, diaphoresis, and gastrointestinal upset, but
carpal spasm and paresthesias are more consistent with hypocalcemia.
Hyperventilation due to anxiety can cause acute respiratory alkalosis, increasing the
binding of calcium to albumin and reducing ionized calcium concentrations.
However, this patient's respiratory rate is only 12/min.

Manifestations of fat embolism include respiratory insufficiency, hypoxemia, altered


mental state, and petechial rash.
Hypomagnesemia (not hypermagnesemia) can present similarly to hypocalcemia.
Severe hypermagnesemia causes hyporeflexia, paralysis, apnea, and cardiac
arrest. Rarely, very severe hypERmagnesemia can cause hypocalcemia by inhibiting
PTH release, but this would typically be seen with infusions of magnesium, such as in
the treatment of severe preeclampsia and eclampsia.
Symptoms of hypocalcemia usually
stem from neuromuscular
irritability and manifest as perioral
tingling and numbness, muscle
cramps, tetany, carpopedal spasms,
and seizures. Prolongation of the
QT interval can also occur. If the
hypocalcemia develops rapidly, as in
surgical patients, the symptoms are
typically more severe, even with
modestly reduced serum calcium
levels. However, chronic
hypocalcemia may be asymptomatic even with markedly reduced serum calcium levels.

The causes of primary hypoparathyroidism include:

1. Post-surgical (most common cause)


2. Autoimmune
3. Congenital absence or maldevelopment of the parathyroid glands (eg,
DiGeorge syndrome)
4. Defective CaSR on the parathyroid glands
5. Non-autoimmune destruction of the parathyroid gland due
to infiltrative diseases such as hemochromatosis, Wilson disease, and neck
irradiation

Post-surgical hypoparathyroidism can occur during thyroidectomy and sub-


total parathyroidectomy (ie, removal of 3½ parathyroid glands for parathyroid
hyperplasia). Autoimmune hypoparathyroidism is the MC nonsurgical syndrome.

……..
Drugs such as phenytoin, carbamazepine, and rifampin cause vitamin D
deficiency by inducing the P450 cytochrome system in the liver, which degrades
vitamin D to inactive metabolites.

Serum phosphorus is usually low in vitamin D deficiency.

///////////////////FEMALE REPRODUCTIVE SYSTEM & BREAST///////////////////


FIRST AID NOTES Only presentations nonsuspicious of cancer:
- Lactating woman with local erythematous warm swelling.
- Cyclical changing mass in young woman with clear aspirate.
Everything else is extensively worked up.
A 42-year-old woman presents with an undiagnosed breast mass. Think: Evaluate
without delay. Observation is not an option I age > 30. If age < 30, serial physical
examination with observation or 2–4 weeks or until next menstrual period is an option.

supernumerary nipples (polythelia) more than the normal number of nipples, may be
on the breast or other parts of the body-found along the "milk line" from axilla to
groin. These accessory nipples resemble raised nevi (commonly called "moles").

in upper outer quadrant -----> Most common site of Ca Breast. Non-motile & ill-
defined borders-----> rules out Fibroadenoma

Painless ------>rules out painful breast disorder Mastitis, fibrocystic changes etc.

Plus inc, age points towards, Malignancy.

Lymph node involvement would be in the axilla as lymph from breast goes to the
pectoral group of the axillary lymph nodes.

Metastatic tumors of the breast are rare & would present as a well-circumscribed
tumor with multiple satellite foci, the absence of an intraductal component, and the
presence of many lymphatic emboli.
In adults, the most frequent types of tumors metastasizing in the breast are malignant
melanoma and neuroendocrine-like tumors, especially small cell carcinoma and
carcinoid. In children, rhabdomyosarcoma is the most common.

Mastitis: Lactational and non-lactational.


1. Tx: prevent breast engorgement (pump or lactation). US to localize an abscess
→ Aspirate fluid for gram stain and cx, if abscess→ I&D + Abx. If a nonlactating
women presents with similar picture, consider inflammatory carcinoma.
2. Mammary duct ectasia: characterized by ductal dilation, inspissated breast
secretion, and chronic granulomatous inflammation in the periductal and
interstitial area.
3. Breast trauma by a seat belt, Radiation → fat necrosis (persistent,
irregular, firm, painless mass). Often indistinguishable from carcinoma by
clinical exam or mammography. Dx and Tx: Excisional bx.
4. Hidradenitis sappurativa (image on the Lt): accessorya reolar
glands of Montgomery. ↑ in women w/ acne. Skin involvement can be
contigiuously (like paget dz) or multifocally. Tx: Abx PLUS I&D, rarely,
skin loss→ graft.

Benign dz:
1. Fibroadenoma: a well-circumscribed,
round to ovoid, or macrolobulated smooth,
mobile mass with generally uniform
hypoechogenicity. Intralesional sonographically
detectable calcification may be seen in ~10% of
cases. Sometimes a thin echogenic rim
(pseudocapsule) may be seen sonographically.
MC breast lesion in adolescents and young
females (tenderness ↑ w/ pregnancy). Dx: FNA, if
diagnostic, may observe if asx and size < 2 cm. If
> 2 cm, pt is >40 y/o, or symptomatic offer surgical excision. If nondiagnostic →
excise
2. Mondor’s dz: Local trauma, surgery, in ection, repetitive movements of UL.
A. Presentation: Acute pain in axilla or superior aspect of lateral breast.
B. Confirm dx w/ US
C. If clear dx w/ US, tx w/ ASA, warm compresses, limit UL motion (affected
side). Resolves w/in 2-6 ww, if persists → surgery
D. If unclear US or associated mass → Excisional bx.
3. FCC: Present with premenstrual breast pain or nodularity (no discrete mass);
often multifocal and b/l. Nonproliferative lesions include simple cysts (fluid-flled
duct dilation, blue dome), papillary apocrine change/metaplasia, stromal fibrosis.
Risk of cancer is usually not increased. Not associated with an increased risk or
breast cancer unless biopsy reveals lobular or ductal hyperplasia with atypia.
Subtypes include:
A. Sclerosing adenosis—acini and stromal fibrosis, associated with
calcifications. Slight (1.5–2 ×) risk for cancer.
B. Epithelial hyperplasia—cells in terminal ductal or lobular epithelium. 5% risk
of carcinoma with Atypical cells.
Tx: NSAIDs, OCPs, Danazol, or Tamoxifen. Avoid xanthines (caffeine, cola,
tobacco). If single dominant cyst, aspirate fluid; if green or cloudy → discard. if
bloody→ cytology.
Atypical ductal or lobular hyperplasia: must be excised.
4. Mammary duct ectasia (plasma cell mastitis): perimenopausal women,
Noncyclical, non-bloody Nipple discharge, lumps behind nipples. Excision biopsy
required to rule out cancer. Tx: Excision of affected ducts.
5. Phylloides: Malignant in <10%, present in >30 y/o (older than fibroadenoma).
Only way to distinguish from Fibroadenoma is bx (mitosis in phylloides).
Treatment:
A. Smaller tumors: Wide local excision with at least a 1-cm margin.
B. Larger tumors: Simple mastectomy.
C. No need for sentinel lymph node biopsy (No LN mets, if mets→
hematogenous)
6. Intraductal papilloma: MCC of U/L nipple discharge.
BREAST MASS
Benign breast disease

Diagnosis Clinical features

• Solitary, well-circumscribed, mobile mass


Breast cyst • ± Tenderness

• Multiple, diffuse nodulocystic masses


Fibrocystic changes • Cyclic PREmenstrual tenderness

• Solitary, well-circumscribed, mobile mass


Fibroadenoma • Cyclic PREmenstrual tenderness

• Post trauma/surgery
Fat necrosis • Firm, irregular mass
• ± Ecchymosis, skin/nipple retraction

Fat necrosis of the breast: This benign condition is associated with breast surgery (eg,
breast reduction/reconstruction) and trauma (eg, seatbelt injury). Fat necrosis can
mimic breast cancer in its clinical and radiographic presentation because it commonly
presents as a fixed mass with skin or nipple retraction and gives the appearance
of calcifications on mammography. US can demonstrate a hyperechoic mass, which
often correlates with a benign etiology. Biopsy is diagnostic and typically shows fat
globules and foamy histiocytes.

Despite benign biopsy results, the entire mass is often excised due to concerning
findings of calcifications on mammography and a fixed irregular mass on clinical
examination. Once the diagnosis is confirmed with pathologic analysis, routine
annual screening is sufficient as the risk of breast cancer is not increased.
………

Radiation and surgery are not indicated for this nonmalignant condition. Prophylactic
mastectomy can be considered in patients with a strong family history of breast
cancer, such as a patient with >1 first-degree relatives with breast cancer before age
50, or a hereditary oncogenic mutation.
MRI of the breast is used for the screening of high-risk patients (eg, BRCA carrier,
first-degree relative of known BRCA carrier) and the evaluation of disease extent
and chemotherapy response in breast cancer patients. Further imaging is not
necessary with this patient as excisional biopsy confirmed a benign condition.

Palpable breast masses are common in adolescents and women age <30, especially
immediately prior to menses due to estrogen stimulation. The ideal time to examine
the breast is 5-10 days after menses (eg, follicular phase) when hormonal stimulation
is minimal. Some masses may be re-evaluated in the next menstrual cycle if not well
palpated in the luteal phase.

Small size, regular texture, round shape, and mobility characterize a benign process
presenting as a breast cyst (eg, soft, fluid-filled, mobile structure) or fibroadenoma
(eg, firm, solid, mobile mass). Both cysts and fibroadenomas may increase in size
with estrogen use (eg, COCs) and are associated with premenstrual discomfort. These
benign structures may occur anywhere in the breast but are usually in the upper outer
quadrants.

Despite this patient being premenstrual, her breast lesion is easily palpated. As
fibroadenoma is most common before age 30 and cyst incidence peaks after age
30, fibroadenoma is the most likely diagnosis based on this patient's age.

………….
Fibrocystic breast changes present as multiple, small, cyclically tender masses (diffuse
breast nodularity) on physical examination. A predominant cyst may occur.

A galactocele is a milk-retention cyst resulting from a plugged duct, usually due to


cessation of breastfeeding.
An intraductal papilloma presents with bloody or serosanguineous nipple
discharge. Breast examination may show no palpable mass or a small (eg, <1 cm)
palpable mass adjacent to the nipple.
A palpable breast mass may
be evaluated using the "triple
diagnostic" (clinical breast
examination, imaging
studies, biopsy) approach,
based on the patient's age
and risk factors for breast
cancer. A young age and
palpable mass with benign
features are most suggestive
of fibroadenoma. Reassurance
may be appropriate for an
adolescent with benign
findings that include a
decrease in the size of a mass
after a menstrual period.
However, further evaluation
is needed to confirm a
diagnosis of a new breast mass in an adult as the incidence of cancer increases with
age.

Although imaging tests (eg, US and/or mammogram) are appropriate next steps in
management, only a biopsy will confirm the diagnosis. A needle aspiration is
performed for small cystic or solid lesions, while core biopsy is preferred for larger
solid masses.
……..

Excision of a breast mass (lumpectomy) is necessary if imaging studies and/or biopsy


results suggest malignancy. Large, growing fibroadenomas may require
lumpectomy for symptomatic relief, but a fine needle or core biopsy should be
performed first to confirm the diagnosis.
In a patient age <30, ultrasound is the best imaging tool to distinguish between a
cystic and a solid mass. If ultrasound is concerning → mammography can be an
adjunctive study to evaluate the entire breast tissue for features of malignancy.
However, a mammogram is not a useful initial diagnostic modality in women
age <30 due to increased density of breast tissue. However, only a biopsy will confirm
what is suspected based on imaging results.
MRI of the breast provides clear imaging of breast tissue and vasculature but is not the
first-line option for evaluation of breast masses. It is reserved for specific indications,
such as presurgical evaluation of breast cancer, evaluation of silicone breast implant
integrity, and breast cancer screening in BRCA-positive women.
Although ultrasound may assist in differentiating a cystic mass from a solid mass, only
a biopsy will provide confirmation of what is suspected on imaging.

A palpable breast mass requires further evaluation as physical examination alone is


insufficient for excluding malignancy. Mammography is the first-line imaging study
for women age >30 with a palpable mass. Although screening mammograms are
used in asymptomatic women, diagnostic (eg, multiple-view) mammograms are used
to evaluate a palpated mass or an abnormal screening result. Targeted
ultrasound is used with mammography to further characterize the mass (eg, solid
versus cystic). In women <30, US is preferred for a palpable abnormality, although
mammography can be used for further characterization if an abnormality (eg,
irregular borders, complex cyst, indistinct borders) is seen on ultrasound.

………….

Biopsy, often ultrasound guided, is required to confirm the diagnosis of a palpable


mass. Core biopsy is used for solid, acellular (eg, stromal) masses, whereas excisional
biopsy is used for large or suspicious ones. FNA may be used for suspected cystic or
small masses. However, the first step in management is imaging to further characterize
the mass and guide biopsy approach.
Although reassurance and reevaluation after 1 or 2 menstrual cycles (not 6 months)
are appropriate for adolescents, all palpable breast masses in adult women require
further evaluation (eg, imaging, possible biopsy) due to the increasing risk of breast
cancer with age.

A patient has a painful, palpable


breast mass. Because history
and physical examination alone
are insufficient for breast cancer
evaluation, all breast masses in
women age <30 require
evaluation with USG. USG can
help in the assessment of cancer
risk based on the composition
and characteristics of the mass.
Patients with a symptomatic,
simple breast cyst can
undergo FNA for pain relief.
Further management depends
on the results of the fine-needle
aspiration:

• If the aspirated fluid is clear (ie, not bloody) and the breast mass resolves
completely, patients are at low risk for cancer and may undergo observation
with a repeat breast examination in 4-6 weeks.
• In contrast, if the fluid is bloody or the mass does not resolve (ie, remains
palpable), patients are at increased risk for breast cancer and require core
needle biopsy, which can provide a definitive histologic diagnosis.

………….
Breast MRI is used for screening in high-risk patients (eg, BRCA carrier) or assessing for
metastatic disease in patients with diagnosed breast cancer. It is not used for
evaluating a new breast mass because breast MRI has high sensitivity but low
specificity for cancer (ie, both benign and malignant masses appear enhanced).
COCs can cause breast pain, particularly in the first few months after initiation, but
they do not typically cause breast cysts. Therefore, this patient does not need to
discontinue her COCs.

Thermography is the measurement of skin temperature in an attempt to detect


underlying cancer (ie, increased temperature may suggest higher metabolic activity and
increased risk of cancer). However, thermography has not been proven to identify
breast cancer and is not recommended.

BREAST ABSCESS A patient has a breast abscess based Breast abscess


on the presence of signs of mastitis
• Maternal age >30
(fever, localized erythema /pain)
Risk factors • First pregnancy
accompanied by a unilateral • Tobacco use
fluctuant, tender, palpable mass.
Breast abscesses typically develop • Fever
Clinical
from untreated mastitis, which is • Focal inflammation
features
• Fluctuant, tender mass
typically due to incomplete
emptying of the breast (eg, poor Diagnosis • Breast ultrasound
latch, alternating of breast and bottle
feeds) and resultant milk stasis. In • Antibiotics
Management
• Drainage
turn, persistent or severe mastitis can
lead to a focal collection of purulent fluid (abscess).
Mastitis and breast abscess are typically caused
by S aureus, which enters the breast through nipple
trauma (eg, chafing/blistering).

The diagnosis of breast abscess is made clinically,


but ultrasound may be required to differentiate
severe mastitis from an abscess if a mass is deep
within the tissue.
Management includes drainage of the abscess with
either needle aspiration (usually under US
guidance) or I&D.
Empiric antibiotics against MSSA (eg, dicloxacillin, cephalexin) are added for
treatment of the surrounding mastitis. Continued breastfeeding is recommended for
continued milk drainage.
………

Breast binding is not recommended for breast engorgement, mastitis, or abscess


because it increases pain and facilitates ductal blockage.

Core needle and skin punch biopsies are indicated in the evaluation of inflammatory
breast cancer, which can present similar to mastitis or abscess (eg, unilateral breast
pain, axillary LAD). However, patients typically have peau d'orange skin thickening and
a solid (rather than fluctuant) mass.

Ice packs and a supportive bra are indicated in the management of breast pain (ie,
mastalgia). Patients with mastalgia have bilateral breast pain with no associated mass.

Warm compresses and massage are indicated in the management of a clogged


lactiferous duct, which can present with unilateral breast pain; however, the mass is
typically located in the subareolar region and has no associated axillary LAD.

OVARIAN CYST
Acute abdominal/pelvic pain in women

Ultrasound
Diagnosis Clinical presentation
findings

• Recurrent mild & unilateral mid-cycle pain


Mittelschm
prior to ovulation NOT indicated
erz • Pain lasts hours to days

• Amenorrhea, abdominal/pelvic pain & vaginal


Ectopic No intrauterine
bleeding
pregnancy • Positive β-hCG
pregnancy

• Sudden-onset, severe, unilateral lower Enlarged ovary with


Ovarian
abdominal pain; nausea & vomiting decreased or absent
torsion • Unilateral, tender adnexal mass on examination blood flow

• Sudden-onset, severe, unilateral lower


Ruptured
abdominal pain immediately following Pelvic free fluid
ovarian cyst strenuous or sexual activity

• Fever/chills, vaginal discharge, lower abdominal


PID pain & CMT
± TOA

A patient presents with hemoperitoneum due to a ruptured ovarian cyst. Corpus


luteum cysts form in the second half of the menstrual cycle, after ovulation has
occurred. Although hemoperitoneum does not typically occur with ovarian cyst
rupture, patients who are on anticoagulation can bleed intra-abdominally and
become hemodynamically unstable. Typical presentation of a ruptured cyst is sudden
onset of unilateral lower abdominal pain. Symptoms of hemoperitoneum include
diffuse severe abdominal pain, pleuritic chest pain, and shoulder pain (due to
phrenic nerve irritation). Physical examination with a ruptured ovarian cyst shows
unilateral lower quadrant tenderness; with hemoperitoneum, diffuse abdominal
rigidity with rebound and guarding is present.

Laboratory testing shows a decreased hematocrit due to intra-abdominal blood


loss. Pelvic ultrasound confirms intra-abdominal and pelvic free fluid and a possible
adnexal mass (if the cyst is incompletely drained). Treatment of a hemodynamically
unstable patient with hemoperitoneum due to a ruptured ovarian cyst is surgery to
stop the bleeding.
……….

Acute appendicitis causes right-sided lower abdominal pain with leukocytosis.


Although physical examination findings of an acute abdomen may be similar to those
from hemoperitoneum, hematocrit does not decrease with appendicitis.
Acute mesenteric ischemia results from inadequate intestinal circulation (eg,
thromboembolism). The resultant intestinal infarction classically presents as a relatively
normal abdominal examination despite excruciating pain.

Diverticulitis typically presents as left-sided lower abdominal pain; it does not lead to a
diffuse acute abdomen or a drop in hematocrit.

Ovarian torsion may present with sudden lower abdominal pain; it does not typically
result in an acute abdomen on physical examination and does not cause a drop in
hematocrit.
Splenic rupture may cause hemoperitoneum, but it typically presents as left-sided
upper abdominal pain.

//////////////////PREGNANCY, CHILDBIRTH, & PEUERPERIUM///////////////////


WOUND
DEHISCENCE AND
EVISCERATION
Abdominal surgery can result in a variety of postoperative wound complications (eg,
infection, dehiscence). These complications are more common in obese patients, those
who are immunocompromised (eg, chronic corticosteroids, cancer), and those with
increased intraabdominal straining (eg, excessive lifting, cough).

Postoperative abdominal wounds can be categorized based on fascial involvement:

• Superficial wound dehiscence is a separation of the skin and subcutaneous


tissue with an intact rectus fascia. It typically develops within the first
postoperative week and occurs secondary to an abnormal subcutaneous fluid
buildup (eg, seroma), resulting in a scant serosanguineous fluid drainage.

• Deep (fascial) wound dehiscence involves the rectus fascia (ie, nonintact) and
results in exposure of the intraabdominal organs to the external environment.
Patients with a superficial dehiscence and no signs of infection (eg, induration,
erythema, purulent drainage) are conservatively managed with regular dressing
changes. These regular changes help with delayed (secondary) closure by removing
excessive fluid and closing physiologic dead space, which debrides nonviable tissue
and eliminates potential bacterial reservoirs. In contrast, fascial dehiscence is a
surgical emergency because of the risk of bowel evisceration and strangulation.
…………..

CT scan of the abdomen and pelvis may be used in patients with suspected
necrotizing fasciitis or abscess. These patients typically have signs of infection (eg,
edema, induration, purulent drainage); those with necrotizing fasciitis may also have
associated crepitus. In both cases, treatment is with systemic antibiotics and wound
debridement.

Abdominal binders may be used temporarily in patients with fascial dehiscence (but
no evisceration) in preparation for emergency surgery. They are not used for
superficial dehiscence because they do not improve outcomes.

//////////////////////////MALE REPRODUCTIVE SYSTEM//////////////////////////


MALE For centuries, male circumcision (ie, removal of the penile foreskin) has been performed
CIRCUMCISION for a variety of cultural, religious, and medical reasons. Until recently, most boys and
men in the United States were circumcised. However, US neonatal circumcision rates
have been decreasing, and the rates in many other developed nations have historically
been lower. Because the procedure is elective, parents expecting boys often seek
clarification about the medical implications of neonatal circumcision, including its risks
and benefits.

Health benefits associated with neonatal circumcision include the following:

• Significant decrease (~10-fold) in UTIs in the first year of life


• Prevention of pathologic phimosis (ie, foreskin constriction preventing
retraction) and modest reduction in the risk of penile cancer (likely due to
reduction in phimosis, which is a risk factor).
• Reduction in certain penile inflammatory disorders (eg, balanitis)
• Reduction in the risk of acquiring some STIs (eg, HPV) and HIV through penile-
vaginal sex, especially in countries where HIV is endemic

Circumcision, like any surgical procedure, has associated risks and complications (eg,
infection, inadequate skin removal, urethral injury); however, the complication rate is
low (<1%). Although most professional medical societies have refrained from
recommending routine neonatal circumcision for male infants in developed nations,
they support a thorough presentation of these health implications to deciding parents.

………….

Circumcision can be helpful for maintaining penile hygiene but is not essential. As
the foreskin progressively becomes more retractable with a child's increasing age,
routine cleansing under the foreskin (retract, cleanse, dry, return) can maintain
appropriate hygiene.

Circumcision has not been shown to significantly affect (either positively or negatively)
sexual performance and satisfaction.

Although circumcision reduces the risk of acquiring HIV and certain STIs (eg, HPV)
through penile-vaginal sex, it does not protect against all STIs, including many of the
most common (eg, gonorrhea, chlamydia).

PENILE INJURY Penile fracture (PF) is a urologic emergency most commonly caused by blunt
trauma (eg, sexual intercourse) to the erect penis. PF results from rupture of the
fibrous tunica albuginea that envelopes the corpus cavernosum. At the moment of
rupture, patients often hear a snapping sound and experience sudden-onset pain.
Afterward, as blood exits the engorged corpus cavernosum, patients experience rapid
penile detumescence and formation of a penile shaft hematoma (eg, swelling,
ecchymosis).

Diagnosis of PF is usually clinical, and urgent surgical repair of the tunica albuginea
is the mainstay of treatment. However, because concomitant urethral injury occurs in
approximately 20% of PF cases (due to the close proximity of the corpus
spongiosum), the following findings warrant investigation with retrograde
urethrography (or urethroscopy) prior to surgery:

• Blood at the meatus


• Hematuria
• Dysuria
• Urinary retention
If urethral injury is confirmed by urethrography, then the urethra should also undergo
repair at the time of surgery.

……….

Medical management of PF (eg, cold compresses, pressure dressings, penile splinting)


was previously routine, but it has overwhelmingly been replaced by surgery due to
the much higher rate of long-term complications (eg, ED, painful erections) with
nonoperative management.

CT scan is recommended in the evaluation of patients with blunt genitourinary trauma


who have suspected renal injury, which would be unlikely with PF.
In some cases of urethral injury (typically posterior urethral injuries associated with
pelvic fracture), a suprapubic catheter is required to decompress the bladder until the
urethra can be repaired. However, PF causes isolated anterior urethral injury that is
more commonly managed with urgent urethral repair and intraoperative urethral
catheter placement (for bladder drainage as well as postoperative protection of the
repair).

The first priorities of burn management are stabilization (eg, ABCs) and
resuscitation (eg, IV fluids). Extensive burns can rapidly lead to hypovolemic shock due
to a large release of proinflammatory mediators that increase vascular permeability
and lead to third spacing of fluid. For this reason, patients with major burns (ie,
>20% total body surface area [TBSA]) receive aggressive fluid resuscitation, which is
usually begun based on a formula (eg, Parkland formula) and then titrated to
maintain adequate urine output (goal: ≥0.5 mL/kg/hr), a marker of organ
perfusion. For accurate monitoring of urine output in these patients, a urethral
catheter (eg, Foley catheter) is required.

In patients who have genital burns and other burns covering a large TBSA,
a catheter should be placed ASAP; delay can lead to edema that obstructs
visualization and catheterization of the urethra. In addition to
facilitating measurement of urine output, the urinary catheter also ensures urinary
continence so that urine does not soil burned areas or dressings.

Following urethral catherization, treatment of this patient's burns involves copious


irrigation and gentle gauze debridement of affected areas. This is followed by
application of topical antimicrobial agents (eg, topical antibiotic) and nonstick
dressings.

………

Cool, not cold, saline-soaked gauze is sometimes applied to burned tissues within
the first few hours after injury to decrease pain. However, cold application (eg, ice) is
discouraged because it can further injure (eg, frostbite) the burned tissues and/or
cause systemic hypothermia.
Inhalational injury is unlikely in this patient who has none of these signs and who
spilled hot oil directly onto his trunk, perineum, and legs.

Sharp surgical debridement and skin grafting will likely be required for this patient's
full-thickness burns. However, these surgical procedures are not performed until after
approx. 72 hours, when burns have fully demarcated; partial-thickness burns at
presentation may progress to become full-thickness over that time.

PEYRONIES A dorsal penile plaque and


DISEASE pain/curvature with erection
likely indicate Peyronie
disease (PD), a common
condition (affecting ~5% of
men) that arises due to
repetitive blunt trauma to
the penis during sexual
intercourse with
subsequent aberrant
wound healing. It is
characterized by the
formation of fibrous
plaques (due to TGF-1
upregulation) in the tunica albuginea, which reduce tissue elasticity and expansion
during erections.

Manifestations generally include penile pain, curvature, and dorsal nodules


or plaques. Distortion of the penis during erection may make sexual intercourse
difficult or impossible, leading to body image and relationship issues. Most cases are
diagnosed based on clinical findings, but ultrasound is sometimes necessary. In many
individuals, the pain and deformity resolve spontaneously over 1-2 years. However,
those with active or progressive PD often require NSAIDs for pain; pentoxifylline to
reduce fibrosis; and/or intralesional injections of collagenase. Surgery may be
indicated in refractory cases.

……..

Genital warts (condyloma acuminata) typically present with clusters of small pink or
skin-colored papules. Although plaques can occasionally occur, they are usually
painless and do not result in erectile curvature.

Bowen disease (cutaneous SCC) may develop on the penile shaft but typically
presents as a painless, erythematous plaque. It may progress to penile cancer, which
is most common in older men (age >60) and usually presents as a painless
ulcer/nodule with inguinal adenopathy.
Systemic sclerosis can cause skin induration and fibrosis, but it primarily affects
the hands, fingers, and face. Vascular manifestations such as Raynaud phenomenon
and telangiectasias are often prominent.
Primary syphilis is associated with a painless genital ulcer. Tertiary syphilis may
cause mucocutaneous plaques (gummas), but the lesions are typically painless and
often ulcerate.

CRYPTORCHIDIS
Cryptorchidism
M
• Prematurity
• SGA
Risk factors • LBW (<2.5 kg [5.5 lb])
• Genetic disorders

• Empty, hypoplastic, poorly rugated scrotum or hemiscrotum


Clinical features • ± Inguinal fullness

Treatment • Orchiopexy before age 1 year

• Inguinal hernia
• Testicular torsion
Complications • Subfertility
• Testicular cancer

Cryptorchidism, or an undescended testis, is due to failure of in utero testicular


descent from the abdomen into the scrotum. Risk factors include prematurity, small for
gestational age (SGA), and low birth weight (LBW; <2.5 kg). At birth, boys with
cryptorchidism typically have an empty, poorly
rugated scrotum. The testis may be palpable in the
inguinal canal, or undetectable.

Because the cryptorchid testis often descends


spontaneously in the first few months of
life, monitoring for testicular descent is
appropriate for this patient with unilateral
cryptorchidism. However, testes that have not
descended by age 6 months are unlikely to
descend and require surgical intervention. Orchiopexy (surgical pinning of the testis) is
performed, ideally before age 1, to reduce complications associated with
cryptorchidism such as testicular torsion, infertility, and testicular malignancy.

……………..

Bilateral nonpalpable testes are concerning for a DSD, such as CAH. Workup
includes karyotyping to distinguish a virilized genotypic female from an undervirilized
genotypic male. In addition, electrolyte monitoring is required to evaluate for salt-
wasting CAH, which can result in a potentially fatal adrenal crisis. In contrast, this
patient has unilateral cryptorchidism with a palpable testis in the inguinal canal.
Immediate surgical correction is required for testicular torsion, which occurs with
increased frequency in cryptorchid testes. Torsion of an undescended testis presents
with inguinal pain and swelling (intracanalicular testis) or an acute abdomen
(intraabdominal testis). In contrast, asymptomatic unilateral cryptorchidism, is not an
emergency, and routine monitoring is appropriate.
Increased local testosterone is required for testicular descent. Systemic testosterone
therapy is not recommended because it does not sufficiently increase the local
testosterone level.

PROSTATITIS
Chronic bacterial prostatitis

• Young & middle-aged men


Epidemiology
• ↑ Risk with DM, smoking, urinary tract procedure

• Coliforms enter from urethra via intraprostatic reflux


Pathogens
• E coli causes >75% of cases

• Recurrent UTIs (with the same organism)


• +/- Prostatic tenderness & swelling (often absent)
Presentation
• Pain with ejaculation
• History of antibiotic treatment → transient improvement

• Pyuria and bacteriuria on urinalysis


Diagnosis
• Bacteria in prostatic fluid > urine

Treatment • Fluoroquinolones (eg, ciprofloxacin) for 6 weeks

Recurrent urinary symptoms (eg, urgency, frequency, dysuria), prostatic tenderness,


and pyuria/bacteriuria suggest chronic bacterial prostatitis (CBP). Most cases
arise when coliform organisms contaminate the urethra and subsequently enter the
prostate via intraprostatic reflux of urine. E coli is the leading pathogen, causing >75%
of cases; other enteric organisms such as Enterococcusfaecalis, K pneumoniae,
P mirabilis, and P aeruginosa are also common. Risk is greatest in those with DM,
tobacco abuse, and recent urinary tract procedure.

CBP manifestations are often subtle, but patients generally have:


• Symptoms of recurrent UTIs (eg, dysuria, frequency, urgency, perineal
tenderness, pyuria, bacteriuria) that transiently resolve with a short course of
antibiotics

• An enlarged, swollen, and tender prostate

The diagnosis is generally made presumptively, but confirmation requires prostatic


massage and examination of prostatic fluid (levels of bacteria in prostatic fluid are
greater than those in urine prior to prostatic massage). Prolonged antibiotic
therapy (4-6 weeks) is necessary to prevent recurrence; fluoroquinolones are
considered first-line therapy because they achieve high concentration in prostatic
tissue and provide good coverage for enteric pathogens.

………….

Chlamydia trachomatis generally causes nongonococcal urethritis with painful urination


and mucoid urethral discharge. It is an infrequent cause of CBP and cannot be
identified on urinalysis gram-stain because the cell wall lacks peptidoglycan.

TB primarily causes pulmonary infection; the prostate is not a frequent site of


bacterial spread.

Gram-positive skin flora such as S aureus primarily cause urinary infections in the
setting of bacteremia or indwelling urinary hardware (eg, long-standing Foley
catheter). S aureus is a less common cause of CBP.

Ureaplasma urealyticum is often a urinary commensal organism but may occasionally


cause CBP. However, Ureaplasma is not associated with bacteriuria on urinalysis
because the pathogen lacks a cell wall.

Fever, dysuria, leukocytosis, and tender, swollen prostate indicate acute bacterial
prostatitis (ABP). In ABP, prostatic swelling can sometimes impinge the b and cause
difficulty voiding or acute urinary retention (with renal insufficiency). A suprapubic
catheter is generally required for decompression because passage of urethral catheters
can lead to sepsis (dislodging of bacteria from infected prostate) or prostatic rupture.
Most cases of ABP are caused by colonic pathogens (eg, E coli, Proteus species) that
have contaminated the urethra and subsequently enter the prostate via intraprostatic
urinary reflux. Urine culture helps define the underlying organism, but prolonged
treatment (~6 weeks) with a fluoroquinolone (eg, levofloxacin) or TMP-SMX is
generally required to ensure eradication. These agents have excellent coverage against
enteric pathogens and achieve high prostatic tissue levels.
……….

Clindamycin is effective against a number of GP organisms (eg, S aureus) and


anaerobes (eg, Bacteroides) but these pathogens are less common causes of ABP.
BENIGN UTIs (eg, PN) are
PROSTATIC uncommon in
HYPERPLASIA men due to the
(BPH) distance between the
anus and urethral
opening, the length
of the male urethra,
and the antibacterial
substances in
prostatic fluid.
However, risk is
elevated with BPH
because periurethral
prostatic
enlargement
impinges the urethra and prevents complete evacuation of the bladder; this allows
fecal flora that have contaminated the urinary tract to proliferate rather than be
evacuated with urination. Bacterial ascent to the kidneys can lead to unilateral (or less
commonly bilateral) pyelonephritis.
As BPH usually presents with manifestations of gradual urethral impingement,
including:

• Voiding symptoms (slow stream, split stream, hesitancy, straining)


• Incomplete bladder emptying (eg, elevated postvoid residual volumes)
with bladder storage symptoms (eg, frequency, nocturia, urgency).

This patient's pulmonary crackles on examination are likely due to pulmonary fibrosis
rather than prostatic or infectious issues.

……….
Detrusor overactivity can cause urinary frequency, urgency, nocturia, and
incontinence. However, voiding symptoms (eg, decreased force of stream, dribbling)
and recurrent UTIs are atypical because outlet obstruction does not occur.

Selective IgA deficiency can be associated with respiratory tract (eg, sinusitis,
pneumonia) and gastrointestinal (eg, giardiasis) infections but the risk of urinary tract
infection is not generally increased.
BPH usually affects men age
>50. Patients typically
develop LUTS (eg, increased
urinary frequency,
hesitation, dribbling,
nocturia) and a feeling of
incomplete voiding. Rectal
examination shows a
smooth and symmetrically
enlarged prostate without
nodules. However, this
patient also has gross and
microscopic hematuria. Causes of microscopic hematuria include renal (eg, RCC, IgA
nephropathy), ureteral (eg, stricture, stone), bladder (eg, cancer, cystitis),
and prostate/urethral (eg, BPH, prostate cancer, urethritis) abnormalities.
Cystoscopy is recommended for all patients with unexplained gross hematuria (e.g.,
w/ normal UA) or with microscopic hematuria and other risk factors for bladder
cancer. Risk factors for bladder cancer include cigarette smoking, certain occupational
exposures (eg, painters, metal workers), chronic cystitis, iatrogenic causes (eg,
cyclophosphamide). and pelvic radiation exposure. Cigarette smoking is the most
important risk factor for bladder cancer. While smoking cessation decreases this risk,
patients still have a higher risk than nonsmokers even up to 20 years after smoking
cessation. Although BPH can cause hematuria, this patient should first be
evaluated with cystoscopy to exclude bladder cancer. If negative, he can receive
BPH treatment with alpha adrenergic antagonists (eg, terazosin, tamsulosin) with or
without 5-αR inhibitors (eg, finasteride).

……….
Although prostate cancer can present with obstructive voiding symptoms and
hematuria, rectal examination usually shows an asymmetrically enlarged prostate that
can be firm or have nodules. In a patient with a normal PSA and benign features on
prostate exam, biopsy is not needed.

Urine culture is generally recommended to rule out infections (eg, cystitis) as the
cause of microscopic hematuria. Cystitis usually presents with acute onset of
hematuria, LUTS, and abnormal urinalysis (eg, pyuria, RBCs).

Maximal urinary flow rate testing can also diagnose BPH. Flow rates >15 mL/sec
usually exclude significant bladder outlet obstruction. This is considered an optional
test for most patients.

PROSTATE
CANCER Comparison of benign prostatic hyperplasia & prostate cancer

BPH Prostate cancer


Risk • Age >40, African American & family histo
• Age >50
factors ry

Affected • Central portion (transitional • Usually peripheral zone of prostate but


part zone) can be anywhere

• Symmetrically enlarged & • Asymmetrically enlarged, nodules & firm


Examina
smooth prostate prostate
tion • Can have elevated PSA • Markedly elevated PSA

BPH = benign prostatic hyperplasia; PSA = prostate-specific antigen.


Prostate adenocarcinoma is the second MC malignancy in men worldwide. Although
most cases are asymptomatic, prostate cancer can present with systemic symptoms
(eg, fatigue, weight loss), bone pain (due to metastases), and/or lLUTS. Prostatic
nodules or asymmetry on DRE, inguinal LAD, and significant elevation of PSA support
the diagnosis but are not present in all cases. Confirmation generally requires
transrectal biopsy.

The greatest risk factor for prostate cancer is advanced age. Men age <40 rarely have
prostate cancer, but prevalence steadily increases as age advances; 30%-80% of men
age >70 have histologic evidence of prostate cancer. Other risk factors for prostate
cancer are less potent, including:

• Ethnicity: Black men have the greatest risk of prostate cancer and tend to
have more aggressive tumors and present at a later stage of disease. Risk is
thought to be increased in this population due to genetic predisposition.
Although Asian men are at less overall risk for prostate cancer than black, white,
or Hispanic men, risk increases when they live in or move to Western countries
(likely due to dietary changes).

• Diet: Diets high in animal fat (eg, alpha-linoleic acid) and low in fruits and
vegetables cause a mild increase in risk for prostate cancer. Vegetarian and
fruit-heavy diets do not increase risk.

……………

Alcohol intake does not seem to increase the overall risk of prostate cancer.
However, patients who consume excessive alcohol and develop prostate cancer may
have more aggressive tumors.

Most studies indicate minimal or no elevation in the risk of prostate cancer in


patients with underlying BPH. This is supported by the fact that BPH typically affects
the central/transitional zone of the prostate, but prostate cancer primarily occurs in
the periphery of the gland. Treatment of BPH with alpha-1-adrenergic antagonists
(eg, tamsulosin) does not seem to have a strong effect on the development of
prostate cancer. In contrast, 5-alpha reductase inhibitors (eg, finasteride,
dutasteride), which block peripheral conversion of testosterone to DHT (the more
active form), reduce risk.

VARICOCELE
Varicocele

• Soft scrotal mass ("bag of worms")


o ↓ In supine position
Clinical
o ↑ With standing/Valsalva maneuvers
presentation • Subfertility
• Testicular atrophy

• Retrograde venous flow


Ultrasound
• Tortuous, anechoic tubules adjacent to testis
findings • Dilation of pampiniform plexus veins

• Gonadal vein ligation (boys & young men with testicular atrophy)
Treatment • Scrotal support & NSAIDs (older men who do not desire additional
children)

NSAIDs = nonsteroidal anti-inflammatory drugs.

Painless scrotal masses are most commonly due to


a hydrocele, inguinal hernia, or varicocele and less
commonly caused by malignancy. This patient has a
soft, irregular mass ("bag of worms") directly
above the testis that increases in size with
Valsalva and does not transilluminate. These
findings are consistent with a varicocele, a
tortuous dilation of the pampiniform
plexus surrounding the spermatic cord and
testis. Varicoceles typically present in adolescent
and young adult males and are more common on
the left side. They are often asymptomatic but may
cause a dull ache after prolonged standing, as in
this patient.
Clinical examination can help differentiate between
varicocele and inguinal hernia, which can also
present as a painless, soft scrotal mass that does
not transilluminate. Like varicoceles, indirect
inguinal hernias (caused by a patent processus
vaginalis) also increase in size with Valsalva
maneuver. However, unlike varicoceles, hernias
are often reducible with manipulation; in
addition, palpation reveals a soft-tissue mass, unlike the characteristic "bag of worms"
texture of a varicocele. A scrotal ultrasound can be useful if the diagnosis remains
unclear after clinical examination.
……….

A spermatocele is a fluid-filled cyst of the head of


the epididymis that transilluminates. It presents as
a painless mass at the superiorpole of the testis. A
spermatocele does not change in size with
positioning or Valsalva maneuvers.

Hydroceles are peritoneal fluid collections between


the parietal and visceral layers of the tunica
vaginalis. Although communicating hydroceles can
change size with positioning, the fluid within a
hydrocele is easily transilluminated.

Widening of the femoral ring can lead to


a femoral hernia. Femoral hernias occur most
commonly in older women, do not protrude
into the scrotum, and present with
discomfort at the upper thigh, groin, or
pelvis.

A patient has a nontender, clustered scrotal mass superior to the left testis that is
consistent with a varicocele, a tortuous dilation of the pampiniform plexus of
spermatic veins. Varicoceles are very common and occur in up to 20% of
males. The left testis (which frequently hangs lower than the right) and hemiscrotum
are often affected due to drainage of the left spermatic (gonadal) vein into the left
renal vein, which is subject to compression between the SMA and the aorta. The
increased pressure and retrograde blood flow results in venous dilation and the classic
"bag of worms" appearance that typically decreases with recumbency. The diagnosis
is typically made clinically but can be confirmed with US.

Although most patients are asymptomatic, varicoceles are associated with decreased
fertility in males, likely due to mildly increased scrotal temperatures. This can result in
testicular atrophy, reduced sperm production, and impaired spermatic
motility. Patients who desire fertility but have abnormal semen parameters or evidence
of testicular atrophy are candidates for surgical venous ligation or embolization,
which is associated with improved pregnancy rates. Older patients who do not desire
fertility may be conservatively managed with NSAIDs and scrotal support.

………..

The vas deferens is responsible for the transport of sperm from the testes to the
ejaculatory ducts; its absence is common in patients with CF. However, due to the lack
of transportation, the semen analysis would be expected to show a complete
deficiency of sperm. In addition, most cases of CF are diagnosed at birth, and patients
are typically symptomatic (eg, recurrent pulmonary infections).

Ejaculatory duct obstruction (obstructive azoospermia) results in very low or absent


sperm counts with normal sperm motility.

Although hydroceles may occasionally cause pain or irritation of the scrotal skin, they
are not associated with decreased fertility.

Inguinal hernias can extend into the scrotum and may reduce with recumbency.

Varicocele usually occurs between age 15-25. Bilateral varicoceles are common;
however, unilateral right varicoceles are relatively rare and should raise suspicion for
thrombosis or malignant compression (IVC compression) → abdominal CT

The diagnosis of varicocele can be confirmed on ultrasound, which shows dilation of


the pampiniform plexus and retrograde venous flow. Initial interventions for scrotal
discomfort due to a varicocele include scrotal support and simple analgesics (eg,
NSAIDs).
Varicoceles are associated with increased risk for infertility and testicular atrophy,
which may be due to slightly increased scrotal temperature. For older men who do
not have concerns about fertility, no intervention may be needed. However, younger
men should be followed for signs of testicular atrophy or changes in semen
analysis; abnormal findings generally warrant surgical intervention. When
intervention is indicated, surgical venous ligation can improve fertility.

………

Chronic epididymitis is usually a noninfectious inflammatory process, and is believed


in most cases to be related to retrograde flow of urine into the epididymis. It carries an
independent risk of infertility, but it is not a complication of varicocele.

Intestinal strangulation is less likely to reduce in supine position.

On rare occasions, large hydroceles can cause irritation and ulceration in the
overlying skin. In addition, congenital hydroceles are associated with an increased risk
for testicular torsion due to inadequate fixation of the lower pole of the testis to the
tunica vaginalis.
Cancer is not a notable complication of varicocele.

RENAL VEIN
Renal vein thrombosis (RVT)
THROMBOSIS
(RVT) • Hypercoagulability
o Nephrotic syndrome
o Malignancy (particularly renal)
Risk factors o OCP
• Volume depletion (infants)
• Trauma

• Hematuria
• Flank or abdominal pain
Clinical features
• Elevated LDH, ± AKI
• Enlarged kidney on imaging

• CTA or MRA
Diagnosis
• Renal venography

• Anticoagulation
Treatment • Thrombolysis/thrombectomy (if AKI present)

AKI = acute kidney injury; LDH = lactate dehydrogenase; OCP = oral contraceptive pills.

RVT results in markedly increased glomerular pressure, leading to renovascular


congestion, capsular stretch (manifesting as flank pain), and hematuria. RVTs most
commonly occur in the setting of nephrotic syndrome (due to urinary loss of
anticoagulant proteins and alteration of hemostatic balance) but can also occur
with acquired hypercoagulability associated with malignancy or trauma.
Laboratory analysis typically demonstrates hematuria (with an otherwise bland
urinalysis) and elevated LDH levels (possibly related to necrosis in the setting of
thrombosis); AKI may occur but is more typical in the setting of bilateral RVTs. Renal
Duplex US and abdominal CT scan have a low SN and may demonstrate only an
enlarged kidney, although a Doppler ultrasound may show a reversal of diastolic
flow.

The diagnosis is confirmed by CTA, MRA, or renal venography.

Management depends on the presence of AKI, which, if present, requires urgent


clot removal (eg, thrombolysis, thrombectomy); otherwise, anticoagulationalone is
appropriate.

………..

In adults, membranous nephropathy, a common cause of nephrotic syndrome, is also


an important risk factor for acute RVT. However, this patient does not have
proteinuria, making nephrotic syndrome unlikely. In addition, membranous
nephropathy alone would not explain the patient's enlarged right kidney or gross
hematuria (microscopic hematuria can be seen).
Nephrolithiasis can cause flank pain and hematuria; however, it would not explain the
enlarged right kidney without hydronephrosis, and the pain of nephrolithiasis is
often paroxysmal rather than dull and constant.
Pyelonephritis presents with dysuria, fevers, hematuria, and flank pain. The absence of
fevers, irritative urinary tract symptoms (eg, dysuria), and urinary leukocytes or nitrites
make this diagnosis less likely.

There is significant overlap in the presentation of renal infarction and acute RVT;
however, renal infarction MCly occurs due to cardioembolic disease (eg, a-fib),
resulting in incomplete infarction and a wedge-shaped area of ischemia on imaging.
In addition, patients frequently report abdominal pain in addition to flank pain.
An acute increase in BP due to renin release is also common.

EPIDIDYMITIS
Acute epididymitis

• Age <35: sexually transmitted (chlamydia, gonorrhea)


Etiology
• Age >35: bladder outlet obstruction (coliform bacteria)

• Unilateral, posterior testicular pain


• Epididymal edema
Manifestations
• Pain improved with testicular elevation
• Dysuria, frequency (with coliform infection)

• NAAT for chlamydia & gonorrhea


Diagnosis • Urinalysis/culture

NAAT = nucleic acid amplification test.

A patient with posterior testicular pain and swelling likely has acute epididymitis.
Most cases arise when pathogens from the urethra travel in a retrograde fashion
through the ejaculatory duct to the ductus deferens and epididymis. The likely
underlying pathogen differs based on the age of the patient:
• Age <35 – most cases are caused by STIs, particularly N gonorrhoeae or C
trachomatis. Patients often have minimal urinary symptoms (eg, dysuria,
frequency, urethral discharge) and mild or no pyuria.

• Age >35 – most cases are associated with bladder outlet obstruction (BOO; eg,
BPH) and are caused by coliform bacteria (eg, E coli). Patients generally have
urinary symptoms and significant pyuria on urinalysis.

Epididymitis is often suspected when physical examination reveals posterior testicular


swelling and tenderness that improves with elevation of the testes (Prehn sign).
Confirmation requires UA/UCx and NAAT for N gonorrhoeae and C trachomatis.
Treatment with ceftriaxone plus doxycycline (for N gonorrhoeae or C trachomatis) or
levofloxacin (for enteric pathogens) is generally curative. NSAIDs and testicular
elevation may provide symptomatic relief but must be accompanied by antibiotic
therapy.
……….

Neither varicocele nor hydrocele generally presents with pyuria or acute pain.

TESTICULAR
Testicular torsion
TORSION
Epidemiology • Most common in adolescents

• Testicular, inguinal, abdominal pain


• N, V
Clinical
• Horizontal testicular lie with elevated testicle
features • Absent cremasteric reflex
• Swollen, erythematous scrotum

Imaging • No blood flow on scrotal ultrasound with Doppler

• Surgical detorsion & fixation with exploration of the contralateral side


Management • Manual detorsion (if immediate surgery is not available)

Abrupt-onset scrotal pain, testicular swelling (e.g., enlarged right testicle),


and heterogeneous testicular echotexture with a small hydrocele are concerning
for testicular torsion. This condition occurs due to insufficient fixation of the testis to
the tunica vaginalis, which can lead to
testicular hypermobility, twisting of
the spermatic cord, ischemia, and necrosis.
Although testicular torsion occurs MCly
in adolescent boys (due to rapid testicular
growth during puberty), it can present in
any age group. Patients typically develop
acute-onset, severe scrotal pain, often
after physical activity or mild testicular
trauma; associated N and V are common.
Boys classically awaken at night due to
pain, and some patients report prior episodes that resolved without intervention
(intermittent torsion).
Physical examination classically reveals a profoundly tender, enlarged, high-riding
testicle associated with marked scrotal swelling and erythema. The cremasteric reflex
(elevation of the ipsilateral testicle when the inner thigh is stroked) is absent.
The diagnosis is often made clinically. However, Doppler ultrasound of the scrotum
can be used for confirmation or in case of equivocal findings. It may demonstrate
twisting of the spermatic cord or reduced blood flow, and a reactive hydrocele may be
present. Heterogeneous echotexture, as seen in this patient, is a late finding
indicating testicular necrosis, which develops after >12 hours of ischemia and can
result in nonviability. Therefore, if testicular torsion is suspected, urgent urologic
evaluation is indicated.
………..

Inguinal hernias: pain is typically localized to the groin or abdomen rather than the
scrotum, the testicles remain normal sized, and ultrasound demonstrates loops of
bowel within the scrotum.

Testicular neoplasms typically present subacutely as a painless, slow-growing testicular


mass. Acute pain and swelling are more consistent with testicular torsion.

Hydrocele is a collection of peritoneal fluid within the scrotum that forms due to
a patent processus vaginalis; reactive hydrocele can occur in the setting of testicular
trauma. While hydroceles cause scrotal swelling, they do not cause testicular
enlargement and typically cause mild, dull pain rather than the acute, severe
pain noted in this patient.

An adolescent male has recurrent abdominal pain and nausea after strenuous
physical activity; examination reveals a normal abdomen but a tender, erythematous
scrotum. This presentation suggests testicular torsion, which occurs due to insufficient
fixation of the testis to the tunica vaginalis, allowing testicular hypermobility. Exercise,
mild trauma, or movement during sleep can induce twisting of the testis about the
spermatic cord, leading to obstruction of blood flow and testicular ischemia.
Testicular torsion occurs most commonly in adolescents due to enlargement of the
testes during puberty. Classically, patients experience severe, acute-onset scrotal
pain that does not resolve with elevation of the testicle (negative Prehn sign);
however, some patients report inguinal or lower abdominal pain as their primary
symptom. Nausea and vomiting are common. Prior episodes that resolve without
intervention can indicate intermittent torsion.

Physical examination classically reveals a tender, high-riding testicle; a horizontal lie of


the testis may be visible. The scrotum is typically erythematous and markedly
swollen. The cremasteric reflex (stroking the inner thigh causing testicular elevation) is
characteristically absent. The diagnosis is often made clinically, but a Doppler
ultrasound can be used for confirmation.

Testicular torsion is a surgical emergency and should prompt urgent urologic


evaluation for detorsion because loss of testicular viability can occur within a matter of
hours.

…………………..

Acute epididymitis can cause similar symptoms in a sexually active patient; however,
pain (which initially affects the posterior testis, where the epididymis is located) is
unlikely to occur intermittently and is typically improved with elevation of the
testis (positive Prehn sign). Dysuria and urethral discharge may also be present.

Incarcerated inguinal hernia can cause scrotal pain and swelling, but patients typically
have a history of a mass that increases with elevated intraabdominal pressure (eg,
bearing down). In addition, an incarcerated hernia would not usually occur
intermittently and resolve without intervention.

Infected hydrocele is very rare but could present with lower abdominal or scrotal
pain; however, infection would more likely cause progressive pain and fevers as
opposed to the intermittent, recurrent symptoms this patient has experienced. In
addition, hydroceles transilluminate when light is shined behind the scrotum.

Testicular hematoma and testicular rupture can cause significant pain, scrotal swelling,
and erythema. However, these typically occur in the setting of significant trauma (eg,
baseball to the scrotum), which was not reported in this case.

Acute-onset scrotal pain and edema occurring after trauma has a broad differential
diagnosis (eg, scrotal contusions, hematoma, testicular rupture). However, testicular
torsion is an underrecognized diagnosis that can occur after testicular trauma and
should be considered in any patient with significant scrotal pain and swelling. Although
the diagnosis of testicular torsion is often made clinically, a Doppler ultrasound of the
scrotum is the imaging of choice to evaluate scrotal contents and can confirm torsion
in those without a clear diagnosis or evaluate other causes of scrotal
pain. Ultrasound findings of reduced or absent blood flow or twisting of the spermatic
cord are suggestive of torsion.
Testicular torsion occurs due to insufficient attachment of the testicle to the tunica
vaginalis, allowing for testicular hypermobility; mild trauma or exercise can induce
twisting of the testis on the spermatic cord, leading to testicular ischemia.
Spontaneous torsion can also occur. Other classic findings of torsion include absent
cremasteric reflex (elevation of the testicle with stroking of the inner thigh) and scrotal
pain unrelieved by elevation of the testicle (negative Prehn sign), although this test
cannot reliably rule out torsion. Testicular torsion is a surgical emergency; up to
80% of patients with untreated torsion develop a nonviable testicle within 12 hours.

……..

CTAP is indicated to evaluate significant trauma (eg, GSW) or for Fournier gangrene,
which causes severe scrotal pain and edema but is typically associated with fever,
erythema, and crepitus on examination.
Torsion of the appendix testis (an appendage of tissue located on the upper
testis) also causes acute-onset testicular pain, but examination often demonstrates
a localized, tender mass on the testis, which may have a blue discoloration. It can be
distinguished from testicular torsion with Doppler ultrasound demonstrating normal
or increased blood flow to the affected testis. Management is supportive and
includes pain management and scrotal elevation.

Surgical evacuation of a hematoma can be considered in patients with evidence of a


rapidly expanding testicular hematoma that may cause compartment syndrome.
However, testicular hematomas are visible on ultrasound and typically occur after
blunt trauma. Mild ecchymosis after minimal trauma makes a rapidly expanding
hematoma unlikely.

TESTICULAR
Testicular cancer
CANCER
• Age 15-35
Epidemiology
• Risk factors: family history, cryptorchidism

• Germ cell tumors (95%): seminomatous or nonseminomatous (embryonal


Types carcinoma, yolk sac, choriocarcinoma, teratoma, mixed)
• Sex cord–stromal tumors: Sertoli cell, Leydig cell

• Unilateral, painless testicular mass


Manifestations
• Dull ache in lower abdomen

• Examination: firm, ovoid mass


Diagnosis • Elevated tumor markers (AFP, β-hCG, LDH)
• Scrotal ultrasound

AFP = alpha-fetoprotein; LDH = lactate dehydrogenase.

Solid, hard, nontender testicular mass raises strong suspicion for testicular cancer,
the MC solid-organ malignancy in men age 15-35. Most cases present with a
unilateral testicular nodule that is generally painless but may be associated with a dull
ache in the lower abdomen or perineum. Subsequent bimanual examination of the
testes usually reveals a hard, ovoid mass within the tunica albuginea.

Prompt diagnostic workup and treatment is required to provide the best chance for
cure. This generally includes:

• Scrotal ultrasound, which usually reveals a solid, hypoechoic lesion


(seminoma) or a lesion with cystic areas and calcifications (nonseminomatous
germ cell tumor [NSGCT]).

• Serum tumor markers such as β-hCG, alpha-fetoprotein, and lactate


dehydrogenase, which are often elevated (particularly with NSGCTs).

• Radical inguinal orchiectomy, which is performed to confirm the


diagnosis histologically and provide definitive treatment.

……….

Disseminated prostate cancer is often treated with androgen deprivation therapy.

Epididymo-orchitis requires antibiotics and is generally associated with painful swelling


of the posterior testes.
Malignant testicular neoplasms

Semin • Retain features of spermatogenesis


Germ oma • β-hCG, AFP usually negative
cell
(GCT) Nons • ≥1 partially differentiated cells: yolk sac, embryonal carcinoma, teratoma,
(95%) emino and/or choriocarcinoma
ma • β-hCG, AFP usually positive

• Often produces excessive estrogen (gynecomastia) or testosterone


Leydi
(acne)
g
Stroma • Can cause precocious puberty
l
(5%) • Rare
Sertoli • Occasionally associated with excessive estrogen secretion (eg,
gynecomastia)

AFP = alpha-fetoprotein.
A patient with a testicular mass, gynecomastia, and
elevated estrogen levels likely has a Leydig cell
tumor, the MC type of testicular sex cord stromal
tumor. These tumors arise from supporting cells of
the testes such as Leydig, Sertoli, and granulosa
cells; they account for approximately 5% of
testicular tumors (GCT account for ~95%), arise in a
wide range of ages, and have no clearly defined
risk factors.
Leydig cells are the primary source of testicular
testosterone but are also capable of generating
estrogen. Therefore, Leydig cell tumors often
present with endocrine manifestations due to
excessive estrogen (eg, gynecomastia, loss of
libido, erectile dysfunction) or testosterone (eg,
acne, hirsutism). Examination frequently reveals a
testicular mass, which is typically confirmed by
bilateral scrotal US. In contrast to many GCTs,
Leydig cells do not generally produce serum tumor markers such as β-hCG or AFP.
However, the generation of estrogen or testosterone often leads to FSH and
LH suppression.
Testicular cancer

• Age 15-35
Epidemiology • Risk factors: family history, cryptorchidism

• Unilateral, painless testicular nodule or swelling


Manifestations • Dull lower abdominal ache
• Metastatic symptoms (eg, dyspnea, neck mass, low back pain)

• Examination: firm, ovoid mass or unilateral swelling


• Scrotal ultrasound
Diagnosis
• Tumor markers (AFP, β-hCG)
• Staging imaging (CT scan, chest x-ray)

• Radical orchiectomy
Treatment • Chemotherapy
• Cure rate ~95%

AFP = alpha-fetoprotein.
A young patient with 2 months of cough has evidence of pulmonary nodules and
retroperitoneal LAD, raising concern for a neoplastic process. Malignancies are rare in
young men; most are caused by testicular cancer, lymphoma, or leukemia.

Patients with testicular cancer usually seek medical care after incidentally finding a
painless testicular mass; however, a minority present with symptoms of metastatic
disease. Because testicular cancer spreads primarily through regional lymphatics,
the retroperitoneal lymph nodes are often initially affected. Bulky retroperitoneal
lymphadenopathy may compress adjacent structures (nerve roots, psoas muscle) and
lead to lumbar back pain. Solid organ metastatic disease to the lungs and liver are
also common. Lung metastases (nodules) may result in cough or dyspnea.

This patient requires a testicular examination to evaluate for a mass. If a mass is


present, scrotal ultrasound and tumor markers should be performed without delay.

………..

Leukemia may be diagnosed with bone marrow biopsy

CRC may spread to regional lymph nodes and the lungs; however, this malignancy is
uncommon in individuals age <40 in the absence of hereditary CRC syndromes
(eg, FAP, Lynch syndrome) or UC. In addition, most patients with metastatic colorectal
cancer have liver lesions (due to portal venous drainage).

Esophageal, gastric, and duodenal tumors are quite uncommon in young individuals.

NECROTOZING Rapid-onset skin infection of the lower abdomen, scrotum, and perineum with crepitus
SOFT TISSUE and significant systemic manifestations (eg, HoTN, high fever, leukocytosis) likely
INFECTION indicates Fournier gangrene, a life-threatening necrotizing fasciitis that quickly
progresses to sepsis and death without intervention. Most cases arise in the setting of
cutaneous breakdowns in the perianal or genital region, which allow a portal of entry
for polymicrobial colonic or urogenital organisms. The infection then spreads along
the subcutaneous fat via fascial planes and results in microthrombi of cutaneous
vessels, leading to subsequent gangrene of the skin. Risk is greatest in those with
poorly controlled DM and obesity.

Manifestations generally arise rapidly and include:

• Swelling, tenderness, and crepitus in the perineum, scrotum, and/or lower


abdomen

• Severe systemic symptoms such as high fever and HoTN

• Significant but nonspecific laboratory abnormalities such as leukocytosis,


acidemia, renal insufficiency, and coagulopathy

Broad-spectrum antibiotics and IV fluids should be administered with urgency, but


definitive diagnosis and treatment requires early surgical exploration and
debridement. Although early surgical intervention improves outcomes and survival
rates, >20% of patients with Fournier gangrene die during hospitalization.
……….
Imaging delays surgical intervention and is generally not recommended when the
presentation is highly suspicious for Fournier gangrene (eg, skin crepitus, rapid onset
of symptoms, hypotension).

Hyperbaric oxygen therapy increases oxygen tissue delivery and may help reduce the
ischemia seen in Fournier gangrene. However, surgery is required first in order to
remove nonviable tissue and prevent the propagation of the infection.
Glucocorticoids are sometimes given to patients in refractory septic shock (eg, SBP
<90 mm Hg for >1 hour despite IV fluids and vasopressor agents) as they speed
shock resolution (although effects on mortality are limited).

///////////////////RENAL, URINARY SYSTEM, & ELECTROLYTES///////////////////


BLUNT A patient who sustained a direct blow to the flank (eg, handrail impact) now has flank
ABDOMINAL (ie, left midback) pain, ecchymosis, and tenderness to palpation. This presentation is
TRAUMA (BAT) concerning for renal injury and should prompt imaging of the kidneys via CT scan.

Blunt trauma can injure the kidneys by either a direct blow, or by rapid compression
of the kidneys against the paravertebral muscles. Clinical findings frequently associated
with renal injury include the following:

• Flank pain/tenderness
• Flank ecchymosis, which may represent retroperitoneal hematoma from renal
bleeding
• Hematuria, either gross or microscopic (eg, >3 RBCs/hpf on urinalysis)
Hematuria significantly increases suspicion for renal injury; however, lack of hematuria
does not exclude the need for further workup. Hematuria is absent in up to one-
third of renal injuries, even when they are severe (eg, avulsion of the ureteropelvic
junction [UPJ]).

CTAP with contrast (typically multiphase: noncontrast, arterial, corticomedullary,


excretory) is used to diagnose renal trauma and should be obtained in patients
with concerning clinical findings (eg, flank tenderness/ ecchymosis) and/or
mechanism of injury (eg, direct blow to the flank). Once diagnosed, most blunt renal
injuries can be managed nonoperatively.
……..

This patient has tachycardia and flank ecchymosis, which may indicate renal injury with
bleeding. Management of such an injury may eventually include admission for serial
hematocrits (eg, for certain small bleeds expected to tamponade) or surgical
intervention (eg, diagnostic laparoscopy). However, imaging is necessary first for
visualizing and grading the renal injury, determining the extent of ongoing bleeding,
and informing the management strategy.

Retrograde cystourethrography can be used to evaluate for bladder or urethral injury.


These injuries typically cause gross hematuria, difficulty urinating, suprapubic pain
(bladder injury), or blood at the urethral meatus (urethral injury).

A patient with blunt lower abdominal trauma incurred during a night of heavy
drinking (eg, likely full bladder) now has inability to void, abdominal distension with
ascites on examination (eg, dullness to percussion, positive fluid wave),
and elevated BUN and creatinine. This presentation is most concerning
for intraperitoneal bladder rupture.

Blunt trauma can cause a full bladder to rupture at its weakest point, the dome. As a
result, urine is diverted from the urinary tract into the peritoneal cavity (eg, inability
to void). Although rupture is often painful, it may be masked by acute intoxication,
as in this patient, or distracting injuries (eg, polytrauma). Following rupture,
peritonitis often does not develop acutely because urine is typically sterile.
Therefore, patients may not seek medical attention until after urine has accumulated
within the peritoneal cavity, at which point the abdominal distension from urinary
ascites and the increased BUN and creatinine from peritoneal reabsorption may be
present. Retrograde cystography can confirm the diagnosis.

……..

Severe cirrhosis (eg, alcoholic liver disease: symptoms of ascites typically develop
over the course of weeks to months versus the 1 day in this patient.

***Rhabdomyolysis, which may occur with both short-term alcohol intoxication (eg,
immobilization) and long-term alcohol abuse (eg, electrolyte disorders), can cause
AKI with elevated BUN and creatinine. However, dark urine is typical, and ascites is
not expected.

Although significant splenic bleeding can cause abdominal distension, it is typically


accompanied by signs of hemorrhagic shock (eg, tachycardia, HoTN). This
patient's mild anemia is likely attributable to chronic alcohol use.

RENAL TRAUMA A patient with blunt trauma (eg, snowmobile accident) now has left mid-back pain,
costovertebral area (CVA) tenderness, and microscopic hematuria (ie, >3 RBCs/hpf on
urinalysis). This presentation is concerning for renal injury and should prompt
imaging of the kidneys via CT scan.

The kidneys are the MCly injured organ of the genitourinary system, typically from
blunt trauma (eg, direct blow, compression against paravertebral muscles). Clinical
findings vary depending on the degree of injury. Renal contusion can cause flank (ie,
left mid-back) pain, CVA tenderness, and/or hematuria (gross or microscopic). More
severe injuries, such as renal laceration or renal vessel injury, can result in
retroperitoneal bleeding with flank ecchymosis and/or hemodynamic instability.
CTAP with contrast (typically multiphase: noncontrast, arterial, corticomedullary,
excretory) is used to diagnose renal trauma and should be obtained in patients
with concerning clinical findings (eg, CVA tenderness) or mechanisms of
injury. Hematuria is an additional indication; however, the absence of hematuria
does not exclude the need for CT scan because hematuria is absent in up to one-third
of renal injuries, including severe injuries.

…………

Reassurance and analgesics are only appropriate once renal injury has been excluded.
Retrograde cystourethrogram (RCUG) is used to evaluate for bladder or urethral
injuries, which can cause hematuria. However, these injuries typically cause gross,
rather than microscopic, hematuria and are accompanied by difficulty urinating,
suprapubic pain (bladder injury), or blood at the urethral meatus (urethral injury).
Given this patient's mid-back pain and CVA tenderness, renal injury is the more likely
cause of hematuria, and a CT scan should be performed first. If the CT scan is
negative, RCUG may be indicated, especially prior to urinary catheter insertion
because catheterization can worsen urethral injuries.

Selective renal angiography and embolization may be used to evaluate and treat
some renal injuries with active bleeding. However, this procedure is typically only
undertaken after visualizing the extent of renal injury via CT scan.

BLADDER A patient with suprapubic fullness and tenderness and gross hematuria (ie, bloody
TRAUMA urine) in the setting of a pelvic fracture likely has a bladder injury. Given the absence
of intraperitoneal free fluid (eg, urine) on FAST, the extraperitoneal bladder (eg,
anterior bladder wall) is most likely affected.
Extraperitoneal bladder injuries are almost always associated with a pelvic fracture
(eg, left pubic ramus fracture), and sometimes, a bony fragment can directly puncture
and rupture the bladder. Other clinical findings concerning for bladder injury include
the following:

• Gross hematuria
• Suprapubic pain/tenderness
• Difficulty voiding

Urine leaked from extraperitoneal bladder rupture stays contained within adjacent
tissues (eg, retropubic space). Therefore, FAST is negative for intraperitoneal free fluid
and pain typically remains localized to the pelvis (eg, suprapubic tenderness).

In contrast, intraperitoneal bladder (eg, dome) rupture spills urine into the peritoneal
space, causing a positive FAST and, if undiagnosed, abdominal distention (eg, urinary
ascites) and/or chemical peritonitis. Retrograde cystography, in which the bladder is
passively filled (eg, via Foley catheter) with water-soluble contrast and imaged (eg, CT
scan, fluoroscopy), can confirm the diagnosis.
………….

Ureteral injury may present with hematuria. However, it is overwhelmingly caused by


iatrogenic trauma (eg, pelvic surgery) or penetrating trauma rather than blunt
trauma. In addition, flank/abdominal pain (depending on the location of the injury) is
more common than suprapubic fullness and tenderness.

Renal laceration may present with hematuria, but suprapubic fullness and tenderness
are unlikely. Patients with renal laceration typically have flank pain and/or hemorrhage
into the retroperitoneal space (eg, flank ecchymosis) instead.

Urethral injury occurs in approximately 10% of pelvic fractures in male patients.


However, this patient lacks findings suggestive of urethral injury (eg, blood at the
urethral meatus, high-riding prostate). In addition, the ability to pass a Foley
catheter into the bladder without resistance makes urethral injury unlikely.

A patient with BAT has suprapubic tenderness, hematuria, and intraperitoneal free fluid
(eg, urine) on Focused Assessment with Sonography for Trauma (FAST). This
presentation is most concerning for intraperitoneal bladder rupture and should be
evaluated with cystography.

Blunt lower abdominal trauma can abruptly increase intravesical pressure and
rupture the bladder at its weakest part (ie, dome). Peritonitis often does not develop
acutely because urine is typically sterile. Instead, the MC clinical findings of bladder
rupture are:

• Hematuria (~90%).
• Suprapubic tenderness.
• Difficulty voiding.
• Associated pelvic fracture (eg, widening of the pubic symphysis).

Retrograde cystography, in which the bladder is passively filled (eg, via Foley
catheter) with water-soluble contrast and then imaged (eg, CT scan), can confirm the
diagnosis. Surgical repair is typically required for intraperitoneal rupture because
continued efflux of urine through the defect prevents spontaneous healing.

………..
In the triage of BAT patients, diagnostic peritoneal lavage may be used to evaluate
hemodynamically unstable patients for intraabdominal hemorrhage (which would
necessitate immediate laparotomy). This patient is hemodynamically stable and can
tolerate further testing (eg, CT scan with cystography) to determine the source of his
intraperitoneal free fluid (eg, urine).
Prior to widespread CT scan availability, intravenous pyelography (IVP) was used to
detect kidney stones and renal/urothelial neoplasms. In IVP, intravenous contrast is
followed in an anterograde manner through the kidneys, ureters, and bladder via serial
x-rays. Although IVP can possibly detect contrast extravasation from a ruptured
bladder, the slow, anterograde filling of the bladder with contrast is significantly less
sensitive than cystography's rapid, retrograde instillation of contrast.

Renal US may be used to evaluate for renal injury, which can occur from blunt
abdominal trauma (due to compression against the paravertebral muscles) and result in
hematuria. However, renal injury more typically causes retroperitoneal (vs
intraperitoneal) free fluid and flank (vs suprapubic) pain.

Transrectal ultrasonography is commonly used to evaluate the prostate following


abnormal DRE. It is typically performed electively in the setting of suspected prostate
cancer and does not evaluate the bladder.

URETHRAL A patient has evidence of a pelvic


INJURY fracture (eg, adducted and
internally rotated right lower
extremity, perineal bruising),
accompanied by blood at the
urethral meatus and a high-riding
prostate on digital rectal
examination. This presentation is
most concerning for posterior
urethral injury (PUI) and should
prompt retrograde urethrography.

Because of the length and


positioning of the male urethra,
men with pelvic fractures are at
significant risk for PUI. Abrupt upward shifting of the bladder and prostate can lead
to urethral tearing, MCly at the bulbomembranous junction (transition point
between the anterior and posterior urethra). Findings consistent with PUI
include blood at the urethral meatus, inability to void (due to urethral
discontinuity), perineal bruising, and a high-riding prostate on DRE.
In patients with suspected urethral injury, retrograde urethrography should be
performed. This diagnostic test involves x-ray of the urethral tract following injection of
radiopaque contrast through the urethral meatus. Extravasation of contrast from the
urethra is diagnostic of urethral injury. Urethrography should precede any attempts at
urethral (eg, Foley) catheterization because it can worsen the injury, potentially
converting a partial urethral tear into a complete urethral laceration.

…………

In all cases of urethral injury, urethrography or urethroscopy is recommended prior


to surgery to both confirm the diagnosis and aid operative planning. After
imaging, anterior urethral injuries (eg, penile fracture, straddle injury) are
typically repaired urgently (eg, within 24 hours), whereas most PUIs are treated with
temporary urinary diversion via suprapubic catheter, followed by delayed repair.

Retrograde cystography images (eg, via CT or fluoroscopy) the bladder after it has
been passively filled with radiopaque contrast via Foley catheter. Bladder rupture, as in
PUI, is commonly associated with pelvic fracture and could also be present in this
patient. However, urethral injury should be ruled out prior to any attempts at
urethral catheterization for cystography in this patient with signs of likely urethral
injury.

Ultrasound is appropriate for evaluating for testicular pathology (eg, torsion,


rupture). Although this patient has scrotal ecchymosis, which may be seen in testicular
injury, the lack of testicular tenderness suggests that the ecchymosis is more likely
due to a nontesticular (eg, urethral) injury.

RENAL ARTERY
Fibromuscular dysplasia (FMD)
STENOSIS (RAS)
• 90% women (in adults)
• Internal carotid artery (ICA) stenosis
o Recurrent headache
o Pulsatile tinnitus
Clinical
o Transient ischemic attack (TIA)
presentation o Stroke
• Renal artery stenosis (RAS)
o Secondary HTN
o Flank pain

Physical • Subauricular systolic bruit


examination • Abdominal bruit
• Imaging preferred (eg, duplex US, CTA, MRA)
Diagnosis • Catheter-based arteriography

• Antihypertensives (ACE inhibitors or ARBs 1st line)


Treatment • PTA
• Surgery (if PTA unsuccessful)

ARB = angiotensin II receptor blocker; CTA = CT angiography; MRA = MR angiography; PTA =


percutaneous transluminal angioplasty; US = ultrasonography.

FMD is a systemic noninflammatory disease that typically affects the renal and
internal carotid arteries (ICAs) and leads to arterial stenosis, aneurysm, or dissection.
Less commonly, the vertebral, iliac, or mesenteric arteries can be affected.

In adults, approximately 90% of FMD cases occur in women. Recurrent headache


caused by carotid artery stenosis or aneurysm is the MC presenting symptom.
Pulsatile tinnitus, neck pain, or flank pain are also common, and symptoms of TIA (eg,
focal weakness, vision loss) may occur. HTN results from renal artery stenosis
(RAS) leading to secondary hyperaldosteronism. An abdominal bruit may be
present. In young patients, a subauricular systolic bruit is highly suggestive of FMD
as carotid atherosclerosis should not be present. Even in older patients, such a bruit
suggests FMD as the subauricular location indicates involvement of the internal
carotid artery, which, in contrast with the carotid bulb, is not commonly affected by
atherosclerosis.
Diagnosis of FMD is typically made by vascular imaging (eg, duplex ultrasonography,
CTA or MRA). For patients with HTN, treatment involves antihypertensive medication
(eg, ACE inhibitor) and definitive management of RAS with either percutaneous
transluminal angioplasty (PTA) or surgery.

………

Aortic coarctation presents with upper extremity hypertension, headaches, and lower
extremity claudication.
Renal atherosclerosis is extremely unlikely in this young woman in the absence of a
familial hypercholesterolemia.

A patient with a history of renal transplant has severe, persistent HTN. In association
with the acute kidney injury (AKI) that developed after initiation of lisinopril, this
presentation suggests renovascular hypertension due to transplant RAS. Although
most cases of RAS occur in elderly men with diffuse atherosclerotic disease, it can also
occur in patients with a transplanted kidney and is commonly associated with
operative abnormalities (eg, trauma during organ procurement, abnormal suture
placement), viral infection (CMV, BK virus), and atherosclerosis of the donor
artery. Transplant RAS typically occurs in the first 2 years after transplantation.
Similar to RAS due to other causes, transplant RAS typically manifests with resistant
hypertension. A decline in renal function after the addition of ACE inhibitors or
ARBs is highly suggestive of the diagnosis. Other findings that suggest RAS include
a lateralizing abdominal bruit and recurrent flash pulmonary edema. The diagnosis is
made with renal vascular imaging (eg, renal Doppler USG).
Management of transplant RAS usually includes angioplasty (e.g., stent placement).

…………
Kidney biopsy is indicated to evaluate for AKI due to acute allograft rejection, which
is somewhat less likely in the absence of graft tenderness or fever. In addition,
biopsy is invasive, and noninvasive methods of assessment (eg, imaging) should be
performed prior to biopsy.
***Because transplant RAS is the equivalent of bilateral RAS in a non-transplant
patient, the initiation of an ACE inhibitor is likely to trigger AKI. Increased lisinopril
dosing would likely further worsen this patient's renal function.

High-dose prednisone is a common cause of HTN; however, low doses are not
typically associated with marked HTN.
Toxicity to calcineurin inhibitors (eg, tacrolimus) can cause HTN and AKI; however,
such adverse effects are more common with elevated plasma drug levels. In addition,
the timing of this patient's AKI following initiation of lisinopril makes RAS more
likely than an adverse effect of immunosuppression.

A patient has headaches and recurrent epistaxis, which likely result from her
severe, uncontrolled HTN. In the setting of the unilateral abdominal bruit and renal
size discrepancy, this presentation suggests renal artery stenosis (RAS). Most cases of
RAS occur in older males with diffuse atherosclerosis; however, young women can
develop RAS due to FMD. A lateralizing systolic-diastolic abdominal bruit is a highly
specific examination finding.

Decreased renal perfusion in the poststenotic kidney results in atrophy of the affected
kidney. Low perfusion pressures lead to increased renin secretion by the JG cells of
the affected kidney and activation of the RAAS. This results in secondary
hyperaldosteronism. Although this increases blood flow to the stenotic kidney and
improves affected kidney's GFR, it is maladaptive and results in sodium retention,
vasoconstriction, and HTN.

In contrast, the unaffected kidney is exposed to high systemic pressures, leading


to suppression of local renin secretion and pressure natriuresis.

Although measurement of renal vein renin levels can help diagnose RAS, it is neither
sensitive nor specific and requires an invasive procedure to obtain
measurements. Less invasive approaches (eg, renal Doppler US) are used as first-line
diagnostic techniques.

Clinical clues to renovascular disease


• Resistant HTN (uncontrolled despite 3-drug regimen)
• Malignant HTN (with end-organ damage)
HTN-related
• Onset of severe HTN (>180/120 mm Hg) after age 55
symptoms • Severe HTN with diffuse atherosclerosis
• Recurrent flash pulmonary edema with severe HTN

Physical examination
• Asymmetric renal size (>1.5 cm)
• Abdominal bruit
Supportive Laboratory results
evidence • Unexplained rise in serum creatinine (>30%) after starting ACEIs or
ARBs
Imaging results
• Unexplained atrophic kidney

ARBs = angiotensin II receptor blockers; HTN = hypertension.

An older patient with diffuse atherosclerosis (coronary intervention, carotid


intervention) has recurrent flash pulmonary edema (eg, pulmonary crackles,
distended neck veins, no lower extremity edema) despite a normal EF. In association
with severe HTN, this presentation suggests renal artery stenosis (RAS).

Renal ischemia (caused by limited blood supply to the poststenotic kidney) induces the
activation of the RAAS, resulting in sodium and water retention, generalized
vasoconstriction, and HTN. Laboratory abnormalities reflect secondary
hyperaldosteronism (ie,
hypokalemia, elevated serum
bicarbonate). Unlike unilateral RAS,
in which the unaffected kidney can
typically compensate and the
creatinine remains relatively
normal, bilateral RAS can result in
CKD. Urinalysis is typically bland.
The diagnosis is confirmed with
renal imaging (eg, Duplex US, CTA,
MRA).

Management includes BP control,


aggressive treatment of
atherosclerotic disease (eg, statins,
smoking cessation), and ACE
inhibitors (with close monitoring of
renal function) or revascularization.

……….

Cushing syndrome: pulmonary edema is unexpected.


Bland urinary sediment suggests against nephritic/nephrotic syndrome, and
noninvasive diagnostic tests should be performed first before bx.

Pheochromocytoma: Recurrent pulmonary edema is unexpected.

A urine light chain assay tests for multiple myeloma, which causes CKD but is
associated with hypercalcemia, bone pain, and anemia; in addition, a paraprotein gap
(serum protein − serum albumin >4) is expected.

VON HIPPEL
Von Hippel-Lindau (vHL) disease
LINDAU (VHL)
DISEASE • Germline mutation in the VHL tumor suppressor gene on
Etiology chromosome 3

• Cerebellar & retinal hemangioblastomas


Manifestations • Pheochromocytoma
• RCC (clear cell subtype)

• Surveillance screening for associated malignancies


Management • Resection of identified tumors

A patient has evidence of von Hippel-Lindau (VHL) disease, an AD disorder that


results in benign and malignant multiorgan tumors. The MC tumors associated with
VHL are hemangioblastomas of the CNS (particularly in the cerebellum, with a classic
cystic nodular radiographic appearance) and retina. Although the tumors are
noninvasive, symptoms occur due to hemorrhage or compression of surrounding
tissues. Bleeding from retinal tumors can lead to retinal exudates and impaired
vision or, if untreated, retinal detachment with vision loss. Acute hemorrhage in a
brain tumor can result in herniation and death.
Malignant manifestations include RCC, which occurs in upward of 70% of patients by
age 60. RCCs are of the clear cell subtype and typically bilateral. They are often
preceded by the formation of multiple renal cysts, which are considered
a premalignant condition. Pheochromocytomas also occur with increased frequency
and are characterized by paroxysms of hypertension, tachycardia, and diaphoresis. Less
common manifestations include vascular tumors of the endolymphatic sac in the ear
and pancreatic disease (eg, cysts, neuroendocrine tumors).
Genetic testing for germline VHL mutations is key to diagnosis.

Management requires frequent tumor surveillance beginning in infancy or early


childhood.

……………….

ADPKD also manifests as CNS and renal disease. However, the CNS disease is largely
due to aneurysms (which would not appear as cystic nodules on imaging) rather than
tumors; the renal disease is characterized by progressive parenchymal obliteration by
cysts, which commonly leads to flank pain and hematuria; and the condition is not
associated with retinal capillary hemangioblastomas.

Patients with ARPKD are typically diagnosed in infancy or childhood. Although the
disease is associated with multiple renal cysts, patients characteristically develop ESRD
in early childhood, and hepatobiliary involvement is common (eg, hepatic fibrosis,
liver cysts). It is not associated with hemangioblastomas.

Li-Fraumeni syndrome is an AD disorder associated with alterations of the tumor


protein p53 gene. It usually results in early onset of malignancies such as sarcomas,
breast cancer, and adrenal carcinomas. CNS disease is characterized by gliomas and
medulloblastomas, not hemangiomas.

NF-2: Cataracts and retinal hamartomas are common ophthalmic manifestations.

HEMATURIA Gross (ie, visible or


///////////////// macroscopic) hematuria
***** can be classified based on
the stage of voiding at
which bleeding
predominates:

• Initial hematuria is
characterized by blood
at the beginning of the
voiding cycle and often
reflects a urethral
source.
• Total hematuria is
characterized by blood
during the entire
voiding cycle and can
reflect bleeding from
anywhere in the
urinary tract (eg,
bladder, kidneys).
• Terminal hematuria is characterized by blood at the end of voiding cycle and
often suggests bleeding from the prostate, bladder neck or trigone, or posterior
urethra.
Urothelial (bladder) cancer is
associated with blood vessel
formation and often presents with
painless, total hematuria; however,
terminal hematuria can also be seen if
the cancer originates from the bladder
neck or trigone.

A patient with several risk factors for


urothelial cancer (age >40, male sex,
smoking) has terminal hematuria with clots (which suggest a nonglomerular —and
usually lower urinary tract—source of bleeding). Therefore, he should undergo
cystoscopy to evaluate for bladder cancer and other sources of terminal hematuria.
Delays in diagnosis of bladder cancer are associated with poor prognosis.

…….

Glomerular diseases can cause nephritic syndrome with microscopic or gross


hematuria. Patients can also present with total hematuria. However, clots are unusual
in glomerular disease, and urinalysis frequently shows RBC casts and may show
proteinuria.

Urethritis or urethral injury (eg, Foley catheterization) typically manifests as initial


hematuria.

All urinary tract infections (pyelonephritis, cystitis, urethritis) may present with
microscopic or gross hematuria.

POLYCYSTIC ADPKD: Affected patients begin developing asx kidney cysts early in life. By age 30-
KIDNEY DISEASE 40, the cystic changes have often progressed to the point that disease manifestations
(PKD) become clinically apparent. The MC early disease manifestations include flank pain,
hematuria, and HTN.

Hypertension typically precedes any clinically detectable renal dysfunction and its
progression correlates with the cystic structural derangement of the kidneys. It is
believed the hypertension results from cyst expansion leading to localized renal
ischemia and consequent increased renin release. The increased activation of the
renin-angiotensin-aldosterone system (RAAS) in effect leads to secondary
hyperaldosteronism. Due to their inhibition of RAAS, ACE inhibitors (eg, lisinopril) are
the drugs of choice for treating hypertension in ADPKD.

Other complications of ADPKD include nephrolithiasis (MCly uric acid stones due in
part to a tendency toward acidic urine in ADPKD) and cerebral aneurysms. A hx of
sudden death of a patient's father at a young age suggests rupture of a cerebral
aneurysm in the setting of ADPKD.
………………..
Urine concentrating ability is reduced in ADPKD due to cystic destruction of nephron
distal tubules and impairment of vasopressin (V2) Rs. In effect, a mild nephrogenic
diabetes insipidus (NDI) is created, leading to upregulated and increased production of
vasopressin. (↑ ADH)

Renal blood flow (↓ RBF) decreases as cystic structural derangement of the kidneys
progresses and damages the renal vasculature. In response, production of
prostaglandins increases (↑ PGs) to attempt to dilate renal afferent arterioles and
restore RBF.

Early in the course of ADPKD, erythropoietin levels may be increased (↑EPO), resulting
in mild polycythemia; however, the increase is not a major cause of
HTN. As ADPKD progresses, EPO ↓ as structural derangement progresses and CKD
develops.

Autosomal dominant polycystic kidney disease (ADPKD)

• Most patients asymptomatic until age 30-40


• Flank pain, hematuria
Clinical
• Hypertension
presentation • Palpable abdominal masses (usually bilateral)
• Chronic kidney disease (CKD)

• Cerebral aneurysms
• Hepatic & pancreatic cysts
Extrarenal
• Mitral valve prolapse, aortic regurgitation
features • Colonic diverticulosis
• Ventral & inguinal hernias

Diagnosis • Ultrasonography showing multiple renal cysts

• Aggressive control of risk factors for CV & CKD


Management • ACE inhibitors preferred for hypertension
• Hemodialysis, renal transplant for ESRD

CV = cardiovascular; ESRD = end-stage renal disease.

Recurrent flank pain and hematuria are common in patients with ADPKD; these
symptoms often result from cyst rupture that can be triggered by activities involving
bending and exertion (eg, yard work). HTN is a typical early disease manifestation that
results from localized renal ischemia with increased renin secretion. As the disease
advances, patients experience progressive renal dysfunction (eg, elevated cr) that
eventually leads to ESRD.

Early in the disease process, patients also develop a urinary concentrating defect
(evidenced by low specific gravity [↓SG] on urinalysis), likely due to renal tubular
destruction. In effect, a mild NDI is created with resulting increased circulating
vasopressin (ADH) levels. The high vasopressin levels may encourage renal cyst
growth, and vasopressin-2 receptor antagonists (eg, tolvaptan) have been shown to
slow ADPKD progression in some patients.

Other treatments for ADPKD are mostly supportive and involve minimizing risk factors
for worsening of CKD.

………..
GN can occur due to rheumatologic disease (eg, SLE) or vasculitis (eg, polyangiitis
with granulomatosis) and may affect young patients. Although hematuria and HTN
are a common presentation, RBC casts and protein are expected on urinalysis.

Rhabdomyolysis results from excessive skeletal muscle breakdown following injury or


intense exertion. It causes myoglobinuria, which appears grossly similar to
hematuria and is mistakenly detected as blood on urinalysis. However, it is not true
hematuria and no red blood cells are present on urine microscopy.

UTI often causes hematuria; however, leukocyte esterase, nitrites, and/or bacteria are
also expected on urine studies.

URINARY
RETENTION

A patient presents with acute abdominal pain, oliguria (<250 mL urine in 12 hours),
increased BUN, and increased serum creatinine in the postoperative setting. Acute
kidney injury (AKI) with oliguria can be due to pre-renal, intrinsic renal disease, or post-
renal causes. Postoperative urinary retention (PUR) is a common complication of
surgery and anesthesia. A precipitating event (eg, bladder distention during GA,
epidural anesthesia use) can lead to inefficient detrusor muscle activity and acute
urinary retention. The risk of PUR also increases with advancing age, high fluid intake
during surgery, and concomitant use of other medications (eg, opiates,
anticholinergics).

Postoperative oliguria (< 0.5 mL/kg/hr) requires immediate assessment with initial
portable bladder scan (if available) to assess bladder volume. Patients with significant
urinary retention and likely distal obstruction require Foley catheterization to restore
normal urine output and resolve or prevent hydronephrosis, tubular atrophy, and renal
injury. If catheterization does not relieve the patient's oliguria or if there is no
significant urinary retention, the patient's AKI may be due to other etiologies (ie,
intrinsic, pre-renal). This patient recently had surgery, and the oliguria and abdominal
discomfort suggest urinary retention. If the portable bladder scan is inconclusive (e.g.,
due to obesity), bladder catheterization is the appropriate next step.

……..

CTA of the abdomen is useful for diagnosing renovascular disease due to


atherosclerosis (unilateral or bilateral RAS). Progressive renal vascular disease can cause
CKD but is an unlikely cause of ARF. Moreover, the use of IV contrast for CTA is
contraindicated in ARF.

Intravenous fluids are critical for treating pre-renal AKI due to hypovolemia. However,
administering fluids without first placing a bladder catheter in patients with
suspected post-renal obstruction would further increase the strain on the kidneys and
bladder and potentially worsen symptoms.
Patients with HF can have volume overload but low cardiac output (ie, cardiorenal
syndrome). Renal perfusion in these patients can be improved with IV loop diuretics.
They should not be used in those with oliguria due to suspected bladder outlet
obstruction (BOO).
Urine sediment analysis is helpful for differentiating between pre-renal and intrinsic
renal disease. Muddy brown casts are present in ATN; urine eosinophils suggest AIN.
UTIs and/or PN would be suggested by a positive urine culture.

An elderly male patient with agitation, tachycardia, and lower abdominal


(suprapubic) tenderness 2 days following surgical repair of a hip fracture likely
has acute urinary retention (AUR). The major risk factors for development of AUR
include:

• Male sex (AUR rarely occurs in women)


• Advanced age (~33% of men age >80 will develop AUR)
• History of benign prostatic hyperplasia
• History of neurologic disease (eg, mild cognitive impairment !!!)
• Surgery (especially abdominal surgery, pelvic surgery, and joint arthroplasty)
It is likely that this patient has also recently taken medications eg, anesthetics, opioids,
anticholinergics) that are common precipitants of AUR.

In a patient with suspected AUR who is unable to void, the diagnosis is confirmed
by bladder ultrasound demonstrating >300 mL of urine. Treatment is with insertion
of a Foley catheter, and urinalysis should be collected to rule out UTI (a potential
cause of AUR).

Obesity, abdominal ascites, or tissue edema may render bladder US inaccurate; in


such cases, Foley catheter insertion can be both diagnostic and therapeutic.

………….
A quiet environment may help improve delirium, which is a common cause
of agitation and confusion in hospitalized elderly patients. However, AUR commonly
causes confusion and agitation and should be suspected first in this patient with
numerous risk factors. If possible, the use of haloperidol and other typical
antipsychotics should be avoided in elderly patients.

A patient's abdominal pain and suprapubic fullness are consistent with amitriptyline-
induced urinary retention. Amitriptyline is a TCA with anticholinergic properties. The
bladder empties under muscarinic control with both detrusor muscle contraction and
internalurethral sphincter relaxation. Anticholinergic agents can cause urinary retention
by reducing detrusor contraction and preventing urethral sphincter relaxation. In
addition, elderly male patients often have some degree of underlying BPH and are at
risk for urinary retention. Both the anticholinergic properties of amitriptyline and
pressure from urinary retention could have caused this patient's constipation as well.

This patient's clinical presentation is sufficient to initiate prompt urinary


catheterization for suspected urinary retention. Bedside ultrasound or bladder scan
(if available) can also help in diagnosis but should not delay urinary catheterization.

Urinary catheterization can document a postvoid residual bladder volume (>50 mL


is considered diagnostic for urinary retention) and provides symptomatic relief by
draining urine from the distended bladder. This patient should also discontinue
amitriptyline therapy.

…………..

Upright AXR is not as reliable for evaluating urinary retention as it may not show a
distended bladder (unless obstructed by a bladder stone). AXR are more useful for
diagnosing ileus or SBO. Amitriptyline may cause ileus, but these patients typically
develop N, V, hypoactive bowel sounds, distended abdomen, diffuse mild
abdominal pain, and abdominal imaging showing dilated bowel loops without
air/fluid levels. This patient's normal bowel sounds, infraumbilical fullness, and lack of
wet diapers for 2 days make urinary retention more likely than ileus.
Perforation usually has absent bowel sounds. Absence of guarding/rigidity or GI
symptoms (eg, vomiting) makes this less likely.

URINARY TRACT
Urethral stricture
OBSTRUCTION
• Male > female
• Urethral trauma (eg, catheterization)
Etiology • Urethritis
• Radiotherapy (RTX)

• Weak or spraying stream


Symptoms • Incomplete emptying
• Irritative voiding (eg, dysuria, frequency)

• Acute urine retention


• Recurrent UTIs
• Bladder stones
Complications • Hydronephrosis
• Renal insufficiency
• Fistulas

• Postvoid residual, uroflowmetry


Diagnosis • Urethrography
• Cystourethroscopy

• Dilation
Management • Urethroplasty

A patient has decreased urinary flow and a sensation of incomplete emptying. In a


young man, this is consistent with a urethral stricture. Urethral stricture is a common
disorder characterized by fibrotic narrowing of the urethra and occurs most frequently
in the bulbar portion. Strictures are commonly idiopathic, but secondary causes
include urethral trauma (eg, pelvic fracture, iatrogenic instrumentation), infection (eg,
sexually transmitted urethritis), and radiotherapy. In some cases, the inciting cause is
temporally remote and may not be immediately remembered by the patient.
Urethral strictures can lead to acute urine retention, recurrent UTI, bladder stones,
or, rarely, hydronephrosis and renal insufficiency. Patients typically have obstructive-
type voiding symptoms (eg, weak urine stream, urinary spraying), although dysuria can
occur. The diagnosis is suggested by an elevated postvoid residual
volume. Urethrography (eg, VCUG) or cystourethroscopy can confirm the diagnosis.
In addition to voiding problems, complications can include bladder stones,
hydronephrosis, and fistulas.
Mild strictures may be observed cautiously, but significant strictures typically require
correction with either urethral dilation or surgical urethroplasty.

………
Abnormal detrusor overactivity can cause similar symptoms but is more likely to
cause urinary frequency and urgency, nocturia, or incontinence (storage-type
symptoms related to bladder filling and urine storage). Obstructive symptoms (ie,
decreased force of stream) and increased postvoid residual volume are more consistent
with stricture.
BPH is rare at age <40.

Although a stone in the lower ureter (eg, VUJ) can cause symptoms of irritative
voiding (eg, urgency, frequency), patients frequently have hematuria and severe, acute
flank pain. Postvoid residual volume is normal.

Urethral diverticulum is a rare condition characterized by lateral extension of urethral


mucosa into the surrounding tissues. Usually seen in women, it often presents as
a painful mass in the anterior vaginal wall. Other symptoms include hematuria,
incontinence, and recurrent infection.

HYPONATREMIA A patient has symptomatic hyponatremia, most likely due to the use of hypotonic
///////////////// intravenous fluids (ie, 0.45% saline) and to postoperative SIADH release (which is
***** stimulated by pain, nausea, or stress). Low serum tonicity results in the influx of water
into brain cells, leading to swelling and cerebral edema. Symptoms reflect elevated
ICP: mild/moderate symptoms include nausea, malaise, headache, and confusion,
whereas severe symptoms include seizure, coma, and respiratory arrest.
Adaptations to normalize brain volume (eg, extrusion of osmolytes from brain cells)
typically take ~48 hours.

• Acute hyponatremia (present for <48 hr) is poorly tolerated, and patients are
at high risk of brain herniation. Therefore, patients with serum sodium of
<130 mEq/L with any symptoms of elevated ICP should be treated
with hypertonic 3% saline (HTS) boluses to rapidly correct serum sodium.
Because neural adaptations have not occurred, patients are at relatively low risk
of osmotic demyelination syndrome (ODS).

• Chronic hyponatremia (present for ≥48 hr) is typically better tolerated;


therefore, HTS is often reserved for those with severe hyponatremia (<120
mEq/ L), severe symptoms (eg, seizure), or concurrent intracranial pathology
(eg, masses, hemorrhagic stroke [ICH]).

The goal of hypertonic saline infusion is to raise serum sodium levels by 4-6
mEq/L over a period of hours, which markedly reduces the risk of herniation. The
maximum rate of correction is 8 mEq/L in 24 hours to prevent ODS.

………..
Acute adrenal insufficiency may occur in the postoperative setting and requires rapid
administration of hydrocortisone or DXM. It causes nausea, weakness, confusion, and
hyponatremia, but most patients also have hyperkalemia and severe HoTN.
Vasopressin receptor antagonists (eg, tolvaptan) may be used for patients with
chronic hyponatremia due to SIADH; however, they have slower onset than HTS and
are not indicated in the treatment of acute, symptomatic patients.
Patients with symptomatic hyponatremia require rapid correction of sodium levels
to prevent herniation; this cannot be accomplished with isotonic solutions (eg, 0.9%
saline, lactated Ringer), which slowly raise serum sodium. Additionally, the use of
isotonic fluids in patients with SIADH will likely worsen hyponatremia.

URINARY TRACT A patient who recently had a UTI now has several days of unilateral flank pain, weight
INFECTION (UTI) loss, fever, and leukocytosis, with no UTI symptoms or bacteriuria. This presentation
is concerning for a renal abscess. Renal and perinephric abscesses can form in the
setting of urologic infection (eg, pyelonephritis, UTI) or through hematogenous
spread (eg, bacteremia, endocarditis). Necrosis of infected tissue progresses over time
to cavity formation; therefore, manifestations typically develop insidiously (average
~12 days of symptoms) in patients with a history of a predisposing infection
in the prior 1-2 months.

Risk factors include anatomic abnormalities (eg, VUR, neurogenic bladder),


nephrolithiasis, uncontrolled DM, or tobacco use.
Systemic symptoms (eg, fever, fatigue, diaphoresis, weight loss) are common. Because
of the localized nature of the process, unilateral flank pain/tenderness and a palpable
mass may be present, but UTI symptoms (eg, dysuria, urinary urgency) are often
absent.

Although urinalysis results often include


pyuria, bacteriuria, and proteinuria,
findings may be normal if the abscess is
not in contact with the collecting ducts.
Other laboratory abnormalities include
leukocytosis and elevated inflammatory
markers (eg, ESR, CRP); AKI may occur.
The diagnosis is confirmed
with CT scan or US.

Management includes antibiotic therapy


and percutaneous drainage.

…………

AIN can cause AKI and fever but is typically associated with rash, pyuria, and urine
eosinophilia with WBC casts. Weight loss and flank pain would be unexpected.
Acute papillary necrosis can cause AKI, fever, and flank pain; however, it typically
causes hematuria and is most common in patients with analgesic overuse or sickle
cell anemia. It does not usually cause weight loss.
RCC can cause weight loss, fever, and anemia, but marked flank tenderness is
unexpected. Hematuria is also common.

Renal tuberculosis (due to hematogenous spread of miliary tuberculosis) can result in


abscess formation or GN; pyuria, hematuria, urinary casts, and LUTS are common. In
addition, tuberculosis typically occurs in patients with risk factors (eg, incarceration,
employment in high-risk health care).

RENAL CALCULI A patient has acute flank pain


and nausea due
to ureterolithiasis, which is
confirmed on CT scan.
The majority of patients with
uncomplicated kidney stones
can be managed
conservatively.

• Most stones ≤5 mm in
diameter pass
spontaneously;
increased
oral fluid intake (>2-
2.5 L/day) is
recommended to
ensure adequate flow
of dilute urine. Alpha
blockers (eg, tamsulosin) can facilitate passage, especially for larger stones (6-
10 mm).
• Stones ≥10 mm are unlikely to pass spontaneously and may warrant
(outpatient) urologic evaluation to consider additional interventions (eg,
lithotripsy).
• Regardless of stone size, urologic consultation and hospitalization is
generally appropriate for patients with signs of urosepsis (eg, fever, AMS); AKI;
complete obstruction; or refractory pain, N, or V.

A patient has acute flank pain and nausea due to a 4-mm stone. There are no signs of
urosepsis, renal function is normal, he is able to take oral fluids and medications, and
his pain is well-controlled. The stone will likely pass spontaneously, and the patient can
be discharged home with instructions to increase fluid intake and to strain the
urine; straining urine for stone composition analysis is not by itself an indication for
hospitalization.
…………….

Prolonged intravenous (forced) hydration is not proven superior to oral hydration


for stone passage and is not needed for low-risk patients such as this one. NSAIDs—
which, in patients with nephrolithiasis, are as effective as opiates and associated with
fewer side effects (eg, nausea)—would be appropriate in this patient with normal
renal function.

Struvite (magnesium ammonium phosphate) stones

• Recurrent URTI
Risk factors
• Urease-producing organisms (eg, Klebsiella, Proteus)

• Hydrolysis of urea to yield ammonia:


Urea → 2 NH3 + CO2
Pathogenesis NH3 + H2O → NH4+ + OH−
• Increased urine pH
• Precipitation of magnesium ammonium phosphate salts

• Large staghorn calculi


Clinical
• Fever, mild flank pain due to infection
features
• Obstruction of collecting system & atrophy of renal parenchyma

This patient has a large, irregular kidney stone associated with fever and a history
of recurrent UTIs. This presentation suggests a magnesium ammonium phosphate
(struvite) stone. Struvite stones are caused by urease-producing organisms
(eg, Proteus, Klebsiella). Hydrolysis of urea yields ammonia, which alkalinizes the urine
and facilitates the precipitation of struvite crystals.
Because of the large quantities of urea excreted in urine, these stones can grow very
rapidly and fill the renal calyces (staghorn calculi). The large size of these calculi
prevents them from passing into the ureter, so patients typically have symptoms
related to associated recurrent infections rather than acute renal colic.

If left untreated, the stones can seed further infections, and chronic infection and
obstruction can lead to loss of kidney function. Eradication of staghorn calculi is
difficult; antibiotics alone do not eliminate the source of the infection because bacteria
grow within the stone matrix. Stone removal (eg, percutaneous nephrolithotomy) is
typically recommended, and nephrectomy (after DMSA scan) may be required for
patients with a chronically infected, nonfunctioning kidney.

…………

**HPTH is associated with an increased risk for calcium stones; although PTH
increases renal reabsorption of calcium, the increased release of calcium from bone
leads to an increased filtered calcium load and net calcium excretion. In contrast,
HoPTH decreases renal calcium reabsorption, but because of low overall calcium
levels, kidney stones are rare.

Increased production of uric acid (eg, myeloproliferative disorders [MPD]) can


increase the risk of uric acid stones, which form in acidic urine (impaired uric acid
reabsorption is VERY RARE). Uric acid stones are typically small, relatively
uncommon, and less likely to be seen in the context of recurrent urinary tract infection.
If intestinal concentrations of calcium are low (eg, low-calcium diet, fat
malabsorption), absorption of free oxalate is increased; the increased oxalate load is
filtered by the kidneys and can cause calcium oxalate stones. Although a small oxalate
stone may serve as a nidus for the formation of a struvite stone, struvite formation is
related primarily to urinary ammonia rather than to calcium and oxalate.
Initial management includes hydration, pain medication (NSAIDs [eg, ketorolac,
diclofenac] are generally preferred over opioids), and antiemetics (eg,
metoclopramide). Patients should be counseled to strain the urine to confirm stone
passage and recover the stone for qualitative analysis.

Patients with signs of urosepsis (eg, fever, tachycardia); anuria; AKI; or refractory
pain require urgent urologic consultation for possible percutaneous nephrostomy
or ureteral stent insertion.

In addition, those with large stones that are unlikely to pass spontaneously (≥10 mm),
or those whose stones do not pass within 4-5 weeks, also warrant urologic evaluation
(eg, in the outpatient setting) to consider more aggressive interventions (eg,
lithotripsy, ureteroscopic intervention).

………
Hematuria (gross or microscopic) is present in most patients with symptomatic stones.
Ureteral dilation is also seen in >80% of those with symptomatic stones and would be
expected given that ureteral obstruction is the source of much of the pain in these
patients. However, in the absence of complete obstruction (eg, anuria) and marked
ureteronephrosis or hydronephrosis, acute interventional management is not
required.
Acute renal colic is often associated with severe pain and vomiting, but these
symptoms usually abate with symptomatic treatment. Although urologic intervention
is warranted for refractory symptoms, this patient has only had 2 episodes of
vomiting and her symptoms are likely to resolve with spontaneous stone passage..

Most stones ≤5 mm in diameter pass spontaneously with conservative management


and do not require additional intervention. Alpha blockers (eg, tamsulosin) can be
used to facilitate passage, especially for intermediate-sized stones (6-10
mm). Urologic evaluation is recommended for stones ≥10 mm.

Uric acid kidney stones

• Increased uric acid excretion: Gout, myeloproliferative disorders


• Increased urine concentration: Hot, arid climates; dehydration
Risk factors
• Low urine pH: Chronic diarrhea (GI bicarbonate loss), metabolic
syndrome/diabetesmellitus

• Acidic urine favors formation of uric acid (insoluble) over urate


Pathophysiology (soluble)
• Supersaturation of urine with uric acid precipitates crystal formation
• Radiolucent stones (not visible on x-ray)
Clinical
• Uric acid crystals on urine microscopy
characteristics • Urine pH usually <5.5

Treatment • Alkalinization of urine (potassium citrate)

GI = gastrointestinal.
This patient has colicky flank pain, hematuria, and hydronephrosis, consistent with
acute ureterolithiasis. In light of the acidic urine and history of gout, this patient's
symptoms are most likely due to uric acid stones. Excess urinary excretion of uric
acid in the setting of low urine pH can cause supersaturation of urine and the
formation of uric acid crystals. The stones are radiolucent and typically not visible on
x-ray but can be seen on CT scan.

US is useful in detecting large stones but less sensitive for ureteral or small stones,
although, depending on stone location and extent of possible obstruction, ureteral
dilation or hydronephrosis may be seen. Patients often report passage of small
crystals ("sand") in the urine, but this is not a specific symptom.
Risk factors for uric acid stones include gout, obesity, DM/metabolic syndrome,
chronic diarrhea (due to intestinal bicarbonate loss and compensatory acidification
of the urine), and increased systemic uric acid production (eg, myeloproliferative
disorders [MPD], hemolytic anemia).
Alkalinization of the urine with potassium citrate or potassium bicarbonate
effectively dissolves the stones, and interventional efforts to clear the stones (eg,
ureteroscopic extraction) are usually unnecessary.
……………..
Prostatitis:

1. Acute bacterial prosatitis (ABP): bacteria on initial void and midstream urine
2. Chronic bacterial prostatitis (CBP): bacteruria after prostate massage for 1 min
3. Chronic pelvic pain (non-infectious prostatitis): non-infectious
4. Microscopic prostatitis
…………….

Hydrolysis of urea yields ammonia, which alkalinizes the urine.

… Failure to adequately absorb bile salts in states of fat malabsorption also cause
decreased bile salt reabsorption in the small intestine. Excess bile salts may damage
the colonic mucosa and contribute to increased oxalate absorption.
………….
Bile salt recycling and fatty acid absorption are decreased in Crohn disease.

Uncomplicated stones <1 cm can be managed conservatively with hydration,


analgesics, and alpha blockers. Alpha receptors are found on the distal ureter, base
of the detrusor, bladder neck, and urethra. Sympathetic activation stimulates alpha
receptors to maintain high muscular tone for normal urinary continence. Reflex
ureteral spasm secondary to stone impaction causes the typical waxing and waning
pain seen in ureteral colic. Tamsulosin is an alpha 1 antagonist that relaxes ureteral
muscle and decreases intraureteral pressure. This facilitates stone passage and reduces
the need for analgesics.

……..

Cholinergic receptors are found on the dome of the bladder, contraction of which
facilitates voiding. These receptors are not known to be present on the ureter.
Bethanechol is a cholinergic agent used to treat urinary retention or atonic bladder.
Oxybutynin is an anticholinergic agent that inhibits cholinergic input during bladder
filling. It helps improve bladder capacity and is used to treat overactive bladder
(detrusor instability).

Finasteride is used as a second-line agent in the management of BPH.


Imipramine is an antidepressant used to treat childhood enuresis. It has
anticholinergic effects and promotes urinary retention.

Phenazopyridine is an analgesic for urinary tract mucosa frequently used to treat the
dysuria of cystitis (eg, after infection or instrumentation). It may worsen stone
formation (mechanism unknown).

PRERENAL
Prerenal acute kidney injury (AKI)
AZOTEMIA
• Decreased renal perfusion
o True volume depletion
o Decreased EABV (eg, heart failure, cirrhosis)
Etiology o Displacement of intravascular fluid (eg, sepsis, pancreatitis)
o Renal artery stenosis
o Afferent arteriole vasoconstriction (eg, NSAIDs)

• Increase in serum creatinine (eg, 50% from baseline)


• Decreased urine output
Clinical
• Blood urea nitrogen/creatinine ratio >20:1
features • Fractional excretion of sodium <1%
• Unremarkable ("bland") urine sediment

Treatment • Restoration of renal perfusion

EABV = effective arterial blood volume; NSAIDs = nonsteroidal anti-inflammatory drugs.

This patient with elevated serum creatinine and no underlying renal disease likely
has prerenal acute kidney injury (AKI) due to intravascular volume depletion.
Preoperative infection and intraoperative blood loss predispose patients to volume
depletion, and volume depletion is further suggested by tachycardia. There is no
intrinsic damage to the kidneys in prerenal AKI, but decreased renal perfusion leads to
a decreased GFR and increased serum creatinine. The kidneys increase tubular sodium
and water resorption, resulting in markedly increased passive resorption of urea.
Therefore, a BUN/Cr ratio >20:1 (43:1 in this patient) is highly suggestive of a prerenal
state. Oliguria (<500 mL of urine/24 hours) is often present, as is unremarkable urine
sediment (absence of casts, cells, or protein).

Although HF can lead to prerenal AKI in the setting of hypervolemia (due to decreased
effective arterial blood volume [EABV]), most prerenal AKI is due to a volume-
depleted state. In the absence of clear signs of volume overload (eg, elevated JVP, lung
crackles), patients with suspected prerenal AKI should be treated with IV isotonic
fluid (eg, normal saline) to restore renal perfusion.
…………

Low-dose DA acts primarily as a DA-1 R agonist, resulting in renal arteriole


dilation and increased RBF. However, fluid bolus is the best treatment in a patient with
evidence of hypovolemia.
Intravenous urography (IVU) visualizes radiopaque contrast filling the urinary tract and
is useful in diagnosing ureteral injury. Iatrogenic ureteral injury can occur following
abdominal surgery; however, unilateral injury does not result in AKI, and bilateral
injury (a potential cause of postrenal AKI) is extremely unlikely.

RENAL CELL
Renal cell carcinoma (RCC)
CARCINOMA
(RCC) • Age >50
Epidemiology
• Risk ↑ with smoking, obesity, chemical exposure

• Hematuria: microscopic or gross ± clots


• Abdominal/flank mass
Manifestations • Flank pain
• Left-sided varicocele that does not decompress
• Paraneoplastic syndrome (eg, EPO production, ↑ calcium)

• Abdominal CT
Diagnosis
• Partial/complete nephrectomy (for histology)

EPO = erythropoietin.

Unintentional weight loss, smoking history, and hard flank mass raise strong suspicion
for RCC, a primary malignant neoplasm of the kidney. Most cases arise in patients age
>50; risk is greatest in former or current smokers and those with obesity, HTN, and/or
occupational exposure to toxic compounds (eg, asbestos).

Although RCC is classically associated with the triad of flank pain, hematuria, and a
palpable abdominal mass, <10% of patients have all 3 of these symptoms. RCC
is sometimes discovered incidentally on abdominal imaging, but patients often have
≥1 the following:

• Unintentional weight loss


• An abdominal or flank mass that is firm, nontender, and moves with respiration
• Hematuria (when the tumor invades the renal collecting system), which may
lead to iron deficiency anemia (IDA)
• Intermittent fever
• Paraneoplastic syndromes (eg, ectopic EPO, HrCa)

Patients with suspected RCC usually require abdominal CT with and without contrast
for further evaluation followed by staging imaging (eg, CT chest). Nephrectomy or
partial nephrectomy provides a tissue diagnosis.
…………
Although acute diverticulitis is common in older adults, it usually causes significant
lower abdominal pain and (sometimes) a tender abdominal mass and leukocytosis.
Acute appendicitis typically arises in younger (not older) adults and usually causes
RLQ abdominal pain, anorexia, vomiting, and leukocytosis. This patient with 6 weeks of
symptoms, no abdominal pain, and no leukocytosis is unlikely to have acute
diverticulitis or appendicitis.

NHL often causes B symptoms (eg, intermittent fever, night sweats, weight loss), but it
usually (>70%) also causes painless peripheral LAD and can cause HSM.
Furthermore, it rarely arises in the kidney.

A patient age >60 with PKD would likely have a more significant creatinine elevation.

Differential diagnosis of polycythemia

Primary (↓ EPO) Secondary (normal/↑ EPO)

• Polycythemia vera (JAK2 mutation) • Hypoxemia


• EPO receptor mutations o Cardiopulmonary disease
o Obstructive sleep apnea
o High altitude
• EPO-producing tumors (renal, hepatic)
• Congenital (high-affinity hemoglobin)
• Following renal transplantation
• Androgen supplementation

EPO = erythropoietin.
A patient with an acute ischemic stroke has polycythemia, defined as a hematocrit
level >49% in men or >48% in women. Hematocrit is the main determinant of blood
viscosity; therefore, significant elevations in hematocrit can result in cerebral
microcirculation sludging and potentially life-threatening ischemic events.
Polycythemia is generally classified as follows:

• Primary polycythemia is largely caused by malignant transformation of


erythrocyte progenitor cells, which results in unregulated erythrocyte
production (eg, PCV). Erythropoietin (EPO), the hormone that stimulates red
blood cell production, will be low or absent because elevated red blood cell
mass exerts a negative feedback effect on EPO-producing cells in the renal
cortex.

• Secondary polycythemia is typically due to elevated circulating EPO levels.


Most cases are caused by conditions associated with chronic hypoxia
(eg, cardiopulmonary disease, OSA), which stimulates EPO secretion, or by EPO-
producing tumors (eg, RCC).

Elevated EPO level indicates secondary polycythemia. Given the patient's normal BMI,
pulse oximetry, and CXR, chronic hypoxia is unlikely. Therefore, exploration for
an EPO-secreting tumor should be undertaken. Because the kidney is the primary site
of EPO production, RCC, a primary malignant neoplasm of the kidney, is most likely
(and may explain her several weeks of generalized weakness); abdominal CT scan is
the first test of choice for diagnosis. CT would also evaluate for HCC, which
sometimes causes EPO production (although it does not always lead to
polycythemia, possibly due to inhibited erythropoiesis).
………..
Primary polycythemia, which is associated with low (not high) EPO levels, requires
BMB/aspiration with JAK2 mutation testing (to evaluate for polycythemia
vera). Secondary polycythemia does not usually require bone marrow evaluation.

BLADDER Voiding symptoms (eg, dysuria, frequency), hematuria, and acute right-sided flank
CANCER pain with hydronephrosis raise strong suspicion for bladder cancer. In the US, most
cases arise from the bladder surface epithelium (urothelium) due to chronic exposure
to chemical carcinogens in the environment or in cigarette smoke. Older adults are
most often affected and typically have ≥1 of the following:

• Hematuria: tumor growth drives the formation of new blood vessels that may
bleed, leading to gross or microscopic hematuria; it is classically painless and
present throughout voiding

• Hydronephrosis: bladder outlet or ureter obstruction by the tumor leads to


hydronephrosis, which can be associated with flank pain and ARF (eg, elevated
creatinine).

• Voiding symptoms: bladder tumors can reduce bladder capacity, cause


detrusor hyperactivity, or result in bladder neck/urethra obstruction, leading
to voiding manifestations such as nocturia, frequency, urgency, and dysuria

Adults age >40 who have painless hematuria require prompt investigation for bladder
cancer when no evidence of infection (eg, dysuria, pyuria, bacteriuria), GN (eg, RBC
casts, dysmorphic RBCs), or nephrolith are present. The gold-standard initial test is
urinary cystoscopy, which allows direct visualization of the bladder wall and biopsy of
suspicious masses. Abdominal CT is then generally necessary for staging.

……….
Older men with BPH often have hematuria and chronic voiding symptoms and can
sometimes develop acute urinary tract obstruction, which requires decompression
with a Foley catheter. However, prostatic urinary obstruction usually causes bilateral
(not unilateral) hydronephrosis because it is below the level of the bladder. Unilateral
hydronephrosis is usually decompressed with ureteral stent (not Foley catheter).
Patients with BPH who have recurrent urinary obstruction or significant LUTS
sometimes require TURP.

Hematuria and acute urinary retention are rare in prostate cancer because most
tumors form in the periphery of the prostate, not in the periurethral zone.

Bladder cancer is the MC malignancy of the urinary tract. Bleeding into the urinary
tract is common and often results in painless hematuria that lasts throughout
micturition (tumors in the bladder neck occasionally present with only terminal
hematuria).
The workup of gross hematuria begins with urinalysis to confirm the diagnosis (>3
RBCs/hpf) and to evaluate for common underlying causes, such as UTI (eg, pyuria,
bacteriuria) and GN (eg, RBC casts, dysmorphic RBCs).

Patients who have no clear cause of hematuria and are age >40 require urgent
evaluation for urinary tract malignancy; the first tests of choice are generally
cystoscopy (to visualize the bladder wall and biopsy/resect suspicious lesions) and CT
abdomen (to visualize the kidney and to evaluate for metastases). Although urine
cytology is often performed as part of the evaluation for hematuria, it has low
sensitivity (<35%) and cannot be relied on to rule out bladder or kidney cancer.
……….
Abdominal US is not routinely used in the evaluation of hematuria because it
frequently misses small bladder masses and cannot evaluate tumor depth, extension
to surrounding tissues, or nodal spread.
Painless hematuria in an adult age >40 must be evaluated urgently with cystoscopy
because delays in diagnosis of bladder or kidney cancer are associated with poor
prognosis.

Most cases arise in adults age >40 who have chronic exposure to chemical carcinogens
such as cigarette smoke or industrial chemicals. Common manifestations include:
• Hematuria: Tumors are fed by new, friable blood vessels that often bleed into
the urinary tract. Hematuria tends to occur throughout micturition and may
be gross (visible) or microscopic.

• Voiding symptoms: Tumors often protrude into the bladder and reduce
bladder volume or cause detrusor overactivity, leading to subacute/chronic
voiding symptoms such as dysuria, frequency, and urgency. Although bladder
cancer is often associated with painless hematuria, some patients with
bladder cancer have dysuria as part of their voiding symptoms.

• Suprapubic pain: This usually indicates a more advanced tumor that has
penetrated the muscle and invaded the surrounding soft tissue or nerves.

Urinalysis is generally the first test of choice to confirm hematuria (≥3 RBCs/hpf)
and to rule out other common causes of hematuria such as infection (eg, significant
pyuria, bacteriuria, leukocyte esterase) or GN (eg, RBCcasts, dysmorphic RBCs).
Cystoscopy is then required if no clear cause is evident.

………..
Although bladder infections frequently cause voiding symptoms, suprapubic pain, and
hematuria, urinalysis usually shows significant pyuria (eg, >10 WBCs/hpf). In addition,
most cases arise acutely, not over 4 weeks.

Chronic bacterial prostatitis often causes recurrent symptoms of UTI (eg, dysuria,
frequency, urgency) that improve with short courses of antibiotic therapy but then
recur. However, urinalysis often shows pyuria and bacteriuria, hematuria is unlikely,
and prostate examination frequently reveals a boggy, tender prostate.

/////////////////////////CARDIOVASCULAR SYSTEM//////////////////////////
CARDIAC
TAMPONADE

- Increased diastolic pressure in the chambers


- S/s of cardiogenic shock
- Characteristically, there is elevation and equalization of intracardiac diastolic
pressures (right atrial, right ventricular, and PCWP suggestive of LA pressure)
- Cardiac tamponade can rapidly lead to cardiac arrest if left untreated. Urgent
echocardiography confirms the diagnosis and can identify moderate or large
pericardial effusion, right atrial diastolic collapse, right ventricular diastolic collapse
(highly specific for tamponade), and bowing of the interventricular septum into the
left ventricle during inspiration. Immediate percutaneous or surgical (pericardial
window) drainage.
- Increased pericardial pressure causes compression of cardiac chambers and limits
diastolic filling of the right-sided chambers. This causes a decrease in preload and
reduces cardiac output, resulting in hypotension and syncope.
- Subacute tamponade typically develops from chronic processes that cause the slow
accumulation of pericardial fluid (eg, malignancy, renal failure) → classic globular
cardiac silhouette on x-ray.
- Acute tamponade (e.g., BCI) → small amount of pericardial fluid (eg, 100-200 mL)
can dramatically increase intrapericardial pressure without causing cardiomegaly on
x-ray (typically seen with >200 mL). Therefore, acute tamponade is associated with
normal cardiac contours. A small, left-sided pleural effusion might be seen due to
blood leaking from the pericardium (eg, through a small tear) or a concomitant
injury (eg, lacerated lung parenchyma).
- Acute MI due to failure of a bypass graft can lead to a similar presentation with
cardiogenic shock and may necessitate coronary angiography for diagnosis or
dobutamine infusion for inotropic LV support. However, acute heart failure due
to myocardial infarction does not typically cause equalization of intracardiac
diastolic pressures and also is less likely in the absence of significant ECG changes.
- Aortic rupture is often immediately fatal. In rare instances, aortic rupture with
bleeding into the left pleural space that has been contained (eg, by a hematoma)
can present with left-sided pleural effusion and hypotension. However, it would
cause hypovolemic (rather than obstructive) hypotension with flat (rather than
distended) neck veins.
- Traumatic bronchial rupture (air leaking from the tracheobronchial tree into the
pleural space) can cause hypotension and jugular venous distension. However, this
typically occurs with massive extrapulmonary air leakage, which would be
associated with tension pneumothorax, pneumomediastinum, and subcutaneous
emphysema (not seen on this patient's x-ray).
- Esophageal rupture can allow efflux of gastrointestinal contents into the pleural
space (causing pleural effusion) and mediastinum (causing mediastinitis, a systemic
inflammatory response, and vasodilatory hypotension). However, flat (rather than
distended) neck veins would be expected due to peripheral vasodilation.
- Lung contusion with bleeding sufficient to cause hypotension (from hypovolemia)
would be associated with flat (rather than distended) neck veins.
- CTPA
is used
to
diagnose
acute PE;
however,
low or
normal
(rather
than
elevated) PCWP is expected in acute PE due to impaired blood flow through the
pulmonary circulation to the left atrium.
VENOUS - Venous insufficiency is the most common cause of lower extremity edema; it affects
INSUFFICIENCY approximately 2% of the population at large.
- Failure of venous valves allows blood to pool in dependent areas such as the legs
resulting in an increase in capillary hydrostatic pressure. This increased pressure
favors increased filtration of fluid out of the capillaries into the interstitial tissue.
This process causes a decrease in intravascular volume, which stimulates the
kidneys to retain water and salt ultimately leading to further progression of edema.
- Lymphatic obstruction is an uncommon cause of edema. It may result from
malignant obstruction of lymph nodes, lymph node resection, trauma and filariasis.
It classically affects the dorsa of the feet and causes marked thickening and rigidity
of the skin.
ABDOMINAL
AORTIC
ANEURISM (AAA)

- These patients often have few symptoms until the AAA markedly expands or
ruptures, leading to abdominal, back (or costovertebral angle tenderness), or
flank pain that may be accompanied by a pulsatile abdominal mass. Rupture can
occur into the retroperitoneum or freely into the peritoneal cavity, the latter often
leading to the rapid development of hemodynamic instability and shock.
- The evaluation and management of a suspected symptomatic AAA depends on
both the hemodynamic stability of the patient and prior knowledge of the presence
of AAA. Any symptomatic patient with a known history of AAA and hemodynamic
instability is considered to have rupture and should be emergently transferred to
the operating room for repair. However, in patients with hemodynamic instability
and signs and symptoms consistent with AAA but without a known history, a
focused bedside ultrasound should be performed to confirm the diagnosis (may be
completed in the operating room). For symptomatic patients with or without a
known history of AAA who are hemodynamically stable, an urgent abdominal CT is
most appropriate.
- For patients who undergo emergent repair without prior imaging, options include
open surgical repair or endovascular aneurysm repair. Laparoscopy is only
occasionally considered in stable patients undergoing elective repair.
- Femoral artery aneurism: A pulsatile groin mass below the inguinal ligament is
characteristic; anterior thigh pain is due to the compression of the femoral nerve
that runs lateral to the artery. Femoral artery aneurysm is the second most
common peripheral artery aneurysm after popliteal aneurysm. It may be associated
with an abdominal aortic aneurysm.

- Less common presentations include distal ischemia due to embolization of the


thrombus or atherosclerotic debris from the aneurysm and aortocaval fistula,
leading to venous congestion and high-output heart failure.
- AAA rupture usually occurs posteriorly into the retroperitoneum, which can delay
the onset of hemodynamic instability; however, rupture may occur anteriorly with
direct leakage into the peritoneum and rapid onset of hemodynamic instability and
shock. Physical examination may reveal a pulsatile abdominal mass at the
umbilicus as well as flank or umbilical hematomas. However, findings are
sometimes subtle or nonspecific. Delayed or missed diagnosis is common because
ruptured AAA often mimics other abdominal pathologies (eg, renal colic, acute
pancreatitis) and approximately 75% of presenting patients have no known history
of AAA.
- Pain is the most common initial manifestation and can vary according to aneurysm
location. Proximal aneurysms tend to present with upper abdominal, flank, or back
pain, whereas distal lesions present with lower abdominal or groin pain.
- Other clues to the diagnosis of AAA include a pulsatile abdominal mass (present in
slightly over half of patients) and prevertebral aortic calcification on plain x-ray,
consistent with extensive atherosclerosis. In symptomatic patients who are
hemodynamically stable, the diagnosis should be confirmed with abdominal CT.
Hemodynamically unstable patients require emergency surgical repair with
confirmation obtained by rapid bedside ultrasound if necessary.
- Infections such as vertebral osteomyelitis or epidural abscess can present with back
pain, and blood cultures may aid in diagnosis. However, these infections generally
cause elevations in the erythrocyte sedimentation rate, fever may be present, and
movement usually exacerbates the pain. Vertebral osteomyelitis would also likely
cause point tenderness on palpation of the spine.
- Acute renal infarction presents with abdominal or flank pain accompanied by
hypertension (rather than hypotension) due to increased renin secretion. Nausea,
vomiting, and fever are also common.
- Left lower quadrant pain, nausea, and vomiting are commonly present in acute
diverticulitis. When perforation occurs, patients usually have signs of peritonitis
(eg, rebound, guarding).
FLAIL CHEST
- Multiple x-ray views are
usually required to see both
locations (on the same rib).
The paradoxical motion
impairs generation of
negative intrathoracic
pressure during inspiration.
As a result, TV decreases and
work of breathing increases.
- In addition, the extreme
blunt force required to create
flail chest typically injures the
underlying lung. The resulting pulmonary contusion (eg, seen on x-ray as
infiltrates underlying this patient's rib fractures) decreases oxygen diffusion (due
to alveolar hemorrhage and edema). As a result, patients must breathe harder to
maintain oxygenation. The combination of increased work of breathing and
decreased oxygenation causes many patients to fatigue and develop respiratory
failure, requiring mechanical ventilation.
- These pts are at risk for ARDS from both trauma and pulmonary contusion.
However, ARDS typically takes 6-72 hours (or longer) to evolve after the inciting
event (while the inflammatory response builds) and causes more diffuse bilateral
infiltrates on x-ray.
- Blunt cardiac injury can cause myocardial dysfunction leading to cardiogenic
pulmonary edema and hypoxia. However, concomitant HOTN (from acute pump
failure) is expected in addition to more symmetric bilateral alveolar infiltrates.
- Diaphragmatic rupture → intrathoracic NG tube
- Fat embolism can cause hypoxemia but typically occurs after long bone or pelvic
fracture and does not manifest until 24-72 hours after the inciting event.
EXTRAGONADAL - A large anterior mediastinal mass with elevated β-hCG and AFP is consistent with a
GERM CELL nonseminomatous GCT (NsGCT). Primary mediastinal germ cell tumors occur
TUMORS (EG- predominantly in young male patients and are locally invasive. β-hCG is typically
GCTs) elevated in both SGCT and NSGCT, but nonseminomatous germ cell tumors
typically also produce AFP, which is not produced by seminomas.
- The diagnosis can be confirmed with biopsy. Testicular ultrasound should be
performed to exclude a small primary tumor as management and prognosis differ
between primary mediastinal and metastatic germ cell tumors. Almost all germ cell
tumors in the anterior mediastinum are primary rather than metastatic.
- Benign teratomas typically include tissue from all 3 germ layers and may present
as mediastinal tumors. However, they do not produce tumor markers.
- Bronchogenic cysts are uncommon causes of anterior mediastinal masses. They
are congenital and arise due to abnormal development of the foregut, although
symptoms may not develop until late childhood or early adulthood. They appear
as fluid-filled or air/fluid-filled cystic structures on imaging.
- Thymoma → Pemphigus, MG, or Pure Red cell aplasia.
AORTIC
DISSECTION

- Acute ascending aortic


dissection is a surgical
emergency. Aortic
dissections are commonly
classified based on
ascending aorta
involvement. Those that
involve the ascending
aorta are known as type A
dissections (60%). Those
originating in the
descending aorta are
known as type B
dissections. Dissections
can propagate proximally or distally; therefore, a type B dissection may extend to
involve the ascending portion, becoming a retrograde type A dissection.
- Pleural effusion may occur due to either direct extension causing hemothorax or
from an inflammatory reaction to blood irritating the pleural lining.
- Pulse differential blood pressure in the upper extremities is an important clinical
sign but is present in only 20%-30% of patients; its absence should not be used to
exclude aortic dissection.
- D-dimer elevation is common (sensitive but not specific) due to the presence of
fibrin degradation products, and acute anemia can result from extravasation of
blood into the chest or abdomen.
- The MC symptom of acute aortic dissection is sudden-onset chest or back pain
that is typically severe, sharp, and tearing. Complications of type A dissections,
including syncope, stroke, MI, or heart failure, depend on the vessels and
structures involved (eg, subclavian or carotid arteries, coronary artery ostia, aortic
root). Additional signs may include asymmetry of upper extremity blood
pressures, HOTN, and aortic regurgitation. D-dimer elevation (SN but not SP)
is frequently seen due to the presence of fibrin degradation products. ECG findings
depend on underlying comorbidities (eg, HTN) and on whether the coronary ostia
have been compromised. Plain radiographs may demonstrate a widened cardiac
silhouette; CTA confirms the diagnosis, often revealing an intimal flap and double
lumen.
- Type A dissections have a very high mortality rate in the absence of immediate
surgical intervention. Intraoperative TEE is frequently performed to identify
further abnormalities that may require intervention including aortic root/valve
damage or tamponade.
- Sudden, transient increases in blood pressure (eg, cocaine use, heavy lifting, cough)
can precipitate acute aortic dissection.
- Early identification of aortic dissection is essential because therapies for similarly
presenting conditions (eg, aspirin and heparin for myocardial infarction;
thrombolytics for pulmonary embolism or ischemic stroke) worsen outcomes and
are contraindicated. In this patient, the normal troponin, the lack of acute ECG
changes, and the presence of imaging findings consistent with dissection make
myocardial infarction and pulmonary embolism unlikely.
- Type B dissections that do not result in malperfusion to thoracic or abdominal
organs (eg, mesenteric ischemia) may be managed medically with pain and blood
pressure control in the ICU. Type B dissections complicated by organ malperfusion
benefit from urgent surgical or endovascular repair.
- As type A aortic dissections are surgical emergencies with mortality rates of 1%-2%
per hour following symptom onset, confirming or excluding the diagnosis is critical:
1. CTA is the initial study of choice in hemodynamically stable patients with no
evidence of renal dysfunction. It can reveal an intimal flap separating the
true and false lumens in the aorta.
2. MRA is more time consuming and requires the administration of
gadolinium-containing contrast agents for contrast enhancement; it should
be avoided in patients with moderate to severe kidney disease due to the
risk of NSF.
3. TEE (not TTE) has excellent sensitivity and specificity and is the preferred
diagnostic study in patients with hemodynamic instability or renal
insufficiency; a transthoracic echocardiogram may not visualize parts of the
aorta well.
- Emergency pericardiocentesis is indicated in patients with pericardial effusion and
cardiac tamponade (eg, Beck's triad with hypotension, dilated neck veins, and
muffled heart sounds) with hemodynamic instability or cardiogenic shock.
- Acute necrotizing pancreatitis
commonly presents with epigastric pain
radiating to the back, often with an
associated pleural effusion.
- Presentation with chest pain,
dyspnea, sinus tachycardia on ECG,
elevated D-dimer (again nonspecific),
and pleural effusion can be typical of
pulmonary embolism. However, the
chest pain is usually pleuritic; epigastric
pain, significant hypertension, and
anemia are not common and make
cocaine-induced aortic dissection a
more likely diagnosis.
- Turner syndrome: short stature is
nearly universal. Other frequent
musculoskeletal abnormalities include
micrognathia, cubitus valgus, short and
webbed neck, broad chest with widely
spaced nipples, and scoliosis. Renal
disease and ovarian disorders (eg, decreased fertility) are also common.
- Cardiovascular manifestations of Turner syndrome include BiAV, Ao root dilation,
Ao Coa, and HTN; these changes increase the risk for aortic dissection. The
hemodynamic changes of pregnancy (eg, increased blood volume) place additional
strain on the aorta and can further increase the risk of dissection. Acute dissection
typically presents with sharp/tearing chest or back pain that can radiate to the neck
and abdomen.
- Most patients with Turner syndrome have infertility, and pregnancy is generally
achieved through in vitro fertilization (IVF). Although rare, spontaneous pregnancy
can occur in up to 5% of patients. However, because of the risk of aortic dissection
and the associated mortality, the pursuit of IVF in patients with Turner syndrome
requires careful consideration and, in many cases, (eg, cardiac anomalies) may be
contraindicated.
- Patients are at an increased risk for MI during pregnancy compared to similar age-
matched controls. However, myocardial infarction remains rare in women of child-
bearing age, and the risk is not increased with Turner syndrome. The absence of
ECG changes further reduces the likelihood of this diagnosis, and the presentation
of severe, constant chest pain associated with neck pain and tingling is more
consistent with Ao dissection.
- MR and HF can occur due to flail mitral leaflet, which is associated with mitral valve
prolapse, endocarditis, and papillary muscle rupture. Turner syndrome is not
associated with increased risk for these conditions.
- Pregnancy, especially through IVF, is associated with increased risk of venous
thromboembolism. The risk is greatest in the later stages of pregnancy, and Turner
syndrome does not further increase the risk. In addition, the absence of dyspnea,
pleuritic chest pain, or lower extremity swelling reduces this possibility.
ASPIRATION
SYNDROME

- Pneumonitits: The symptoms (and infiltrates) usually resolve rapidly (24-48 hours)
with supportive management.
- By contrast, aspiration pneumonia, which usually occurs in individuals with impaired
level of consciousness (eg, following overdose, seizure, or anesthetic use), is caused
by inhalation of pathogenic bacteria that colonize oropharyngeal secretions. A
typical presentation would be that of an institutionalized elderly patient who has
dysphagia and develops fever and cough days (not hours) following an (often
unwitnessed) aspiration event.
- Negative pressure pulmonary edema occurs when a patient has upper airway
obstruction (eg, laryngospasm during extubation) that results in large negative
intrathoracic pressure (due to inspiration against the obstruction), causing
noncardiogenic pulmonary edema. It is more common in young men or after head
and neck surgery.
- VAP is a lung infection that typically develops after 2 days of intubation and
frequently presents with fever.
PANCOAST
TUMORS

- MCly squamous cell carcinoma or adenocarcinoma


- Tumors located in the SPS often present with shoulder pain as the initial symptom
due to invasion of the brachial plexus or adjacent structures.
- The pain may also radiate up to the head and neck or down the ipsilateral arm in
the ulnar nerve distribution, and weakness and atrophy of the medial hand muscles
may occur.
MITRAL
STENOSIS (MS)

A patient's progressive exertional dyspnea and early diastolic sound (ie, opening
snap) followed by a rumbling diastolic murmur are consistent with mitral
stenosis (MS). MS usually occurs in the setting of underlying rheumatic heart
disease; in the US it is most frequently seen in patients who emigrated from Latin
America, Africa, or Asia. Progressive dyspnea on exertion is the MC presenting
symptom.
The hemodynamic alterations that occur with MS result from restricted diastolic filling
of the left ventricle. As the severity of the stenosis worsens, there is increased left
atrial pressure that leads to left atrial dilation. Consequent increased pulmonary
artery (PA) pressures (systolic and diastolic) then occur via 2 mechanisms:

• The passive (or postcapillary) component of pulmonary hypertension is most


prominent and results from transmission of elevated pressure
backward from the LA to the pulmonary veins, pulmonary capillaries, and
pulmonary arteries. Clinically, this process is typically evidenced by pulmonary
edema and its associated symptoms (eg, orthopnea, PND, hemoptysis), as well
as peripheral edema.
• The reactive (or precapillary) component of pulmonary hypertension is only
sometimes present and occurs independent of left atrial pressure. In this
process, MS triggers endothelin-mediated pulmonary arteriolar
vasoconstriction and pathologic vascular remodeling through a poorly
understood mechanism.
Left ventricular (LV) diastolic pressure (LVEDP) is generally unaffected in MS as the left
ventricle is downstream of the pathologic stenosis. In severe MS, LVEDP may be
lower than normal.
………….

****Increased LV diastolic and PA systolic and diastolic pressures occur in pulmonary


hypertension that is secondary to LV failure (Group 2 PHTN). MS is a unique cause of
Group 2 PHTN in that LV diastolic pressure remains normal.

Classification of pulmonary hypertension

• Primary change in pulmonary arteries


Pulmonary arterial o Hereditary (eg, BMPR2 mutation)
hypertension o Connective tissue disease (eg, RA, SS)
(Group 1) o HIV infection
• Treatment targeted at endothelial dysfunction

• Secondary to another disease process


Pulmonary o Left-sided heart failure
hypertension o Chronic lung disease/hypoxia
(Groups 2-5) o Chronic pulmonary thromboembolism
• Treatment aimed at underlying disease

RA = rheumatoid arthritis; SS = systemic sclerosis.


Reduced pulmonary arterial compliance, as occurs with age, increases PA systolic
pressure and has little effect on PA and LV diastolic pressures.
Increased LVEDP with normal PA pressures occurs in early LV failure, before the
elevated pressures are transmitted back to the pulmonary circulation and clinical
decompensation occurs.

Normal PA pressures are not expected in a patient with symptomatic evidence of


advanced MS (eg, severe dyspnea on exertion, peripheral edema).

ACUTE MITRAL
Hemodynamic changes in mitral regurgitation
REGURGITATION
(AMR) Compensate Decompensate
Acute MR
chronic MR chronic MR

Preload ↑↑ ↑ ↑

Afterload ↓ No change ↑

Contractile function No change No change ↓

Ejection fraction (EF) ↑↑ ↑ ↓

Forward stroke volume ↓ No change ↓


MR = mitral regurgitation.

A patient meets modified Duke criteria for infective endocarditis (ie, positive blood
culture likely representing S aureus, echocardiographic valvular vegetation), likely from
an infected AV fistula. He now has acute-onset dyspnea and pulmonary edema and
a holosystolic murmur concerning for acute mitral regurgitation (MR), which can
develop as a complication of endocarditis (eg, chordae tendinae rupture).

Infective endocarditis – modified Duke criteria

Major criteria
• Blood culture positive for typical microorganism
(eg, S aureus, Enterococcus, viridans streptococci)
• Echocardiogram showing valvular vegetation
Minor criteria
• Predisposing cardiac lesion
• IVDU
Diagnostic criteria
• Temperature >38 C (100.4 F)
for IE • Embolic phenomena
• Immunologic phenomena (eg, glomerulonephritis)
• Positive blood culture not meeting above criteria
Definite IE
• 2 major OR 1 major + 3 minor criteria
Possible IE
• 1 major + 1 minor OR 3 minor criteria

• Fever (>90%)
• Heart murmur (85%)
• Petechiae (≤50%)
Clinical findings • Subungual splinter hemorrhages (<50%)
(frequency) • Osler nodes, Janeway lesions (<50%)
• Neurologic phenomena (embolic) (≤40%)
• Splenomegaly (≤30%)
• Roth spots (retinal hemorrhage) (<5%)

IE = infective endocarditis.

In acute MR, there is sudden-onset large-volume backflow of blood from the LV to the
LA. Because of the large volume (and rapid equilibration of LV and LA pressures), the
associated murmur is often low-grade (eg, 2/6) or absent, unlike in chronic MR. The
clinical features primarily reflect the heart chambers' lack of time to adapt to the
hemodynamic changes.

• The left atrium has inadequate time to stretch to accommodate the increased
blood volume and remains normal in size. The consequent increase in LA
pressure is transmitted backward to the pulmonary circulation, resulting
in pulmonary edema (eg, bibasilar crackles).
• The left ventricle receives increased blood volume from the LA with each
diastole (representing the additional
regurgitant volume from the previous
systole); this results in ↑ LVEDP but
normal LV size
• The ejected blood can flow either
forward through the aortic valve or
backward through the relatively low-
resistance pathway to the LA (ie, total
resistance [LV afterload] is reduced).
Although absolute volume of forward
blood flow (CO) is decreased,
total SV (forward and backward) is
increased, resulting in a normal or hyperdynamic LV EF (>70%).
In contrast to acute MR, the slower development of chronic MR allows time for
compensatory atrial stretch to accommodate the regurgitant blood volume at a lower
filling pressure, preventing pulmonary edema. In addition, the volume-overloaded LV
undergoes eccentric hypertrophy to compensate and temporarily maintain cardiac
output. However, overwhelming wall stress eventually leads to LV contractile
dysfunction and decompensated HF.
CHRONIC MITRAL
VALVE
REGURGITATION
(CMR)

Primary mitral regurgitation (MR) is defined as that caused by an intrinsic defect of the
mitral valve apparatus (eg, leaflets, chordae tendineae) and is differentiated from
secondary MR, which results from other cardiac disease (eg, myocardial ischemia,
DCM). A primary defect resulting from myxomatous degeneration of the mitral valve
(manifesting on echocardiography as mitral valve cusps bulging into the left atrium
during systole).

Chronic severe MR of primary etiology is best treated with surgical repair, and the
timing of surgery should precede the development of significant left ventricular (LV)
dysfunction. An important consideration is that measured LV ejection fraction (LVEF)
overestimates LV function in severe MR because regurgitant flow accounts for a
large amount of the stroke volume. Therefore, LVEF 30%-60% is generally an
indication for surgical repair or replacement in patients with primary chronic severe
MR, regardless of symptoms. Patients with LVEF <30% likely have life-limiting,
permanent systolic dysfunction, and surgery should be considered only when there is
a high likelihood of successful valve REPAIR (rather than replacement). Preemptive
surgery can also be considered in asymptomatic patients with LVEF >60% who are
excellent candidates for successful valve repair.

Treatment with ARBs (eg, losartan) and/or Bbs (eg, metoprolol) can be beneficial for
chronic severe MR that is secondary to DCM (ie, the MR is due to LV dilation and the
MV itself is normal). However, pharmacologic therapy is of limited benefit in patients
with chronic severe MR of primary etiology; timely surgical repair is most effective.

Referral to an exercise program or repeat echocardiography in 6 months can be


appropriate for patients with primary chronic severe MR and LVEF >60%. However,
even patients with LVEF >60% may be considered for surgery if they have a high
likelihood of successful and durable mitral valve repair.
VENTILATOR- VAP is defined as pneumonia
ASSOCIATED that develops ≥48 hours after
PNEUMONIA endotracheal intubation. It is
(VAP) generally caused by
aspiration of microorganisms
from the oropharynx or
stomach into the pulmonary
parenchyma. Although ETTs
have inflatable cuffs that seal the upper airway to prevent passage of secretions into
the lungs, aspiration can still occur due to leakage around the cuff because of supine
positioning, pooling of secretions in the subglottal area, or excessive ETT movement
(eg, during patient transport).

The following is recommended for intubated patients to reduce the risk of VAP:
1. Head of bed elevation at 30-45 degrees to reduce retrograde movement of
gastric secretions
2. Continuous or intermittent suction of subglottic secretions to prevent pooling
above the endotracheal cuff
3. Minimization of patient transport to prevent movement of the endotracheal
tube
4. Limited use of gastric acid inhibitors (eg, proton pump inhibitor, histamine-2
receptor blockers, antacids) to reduce the burden of microorganisms in gastric
secretions

Although enteral nutrition through a nasogastric tube can increase the risk of
aspiration (particularly when patients are supine), it should be used when possible
because total parenteral nutrition, which is administered through a central line, is
associated with a high rate of complications (eg, bloodstream infection due to the
central line).

Excessive sedation increases the risk of VAP because it prolongs the number of days on
mechanical ventilation and suppresses gag reflex. Daily sedation breaks help lower the
risk of VAP due to the reduction of time on mechanical ventilation.

Elevated gastric pH allows microorganisms to grow in gastric secretions, which


increases the risk of VAP. Therefore, gastric acid suppression (eg, via proton pump
inhibitor) should not be routinely used for stress ulcer prophylaxis. Acid suppression is
generally reserved for patients at high risk for gastrointestinal bleeding (eg,
thrombocytopenia, a recent gastrointestinal bleed).
Tight glycemic control is not recommended in the intensive care unit setting due to an
elevated risk of hypoglycemia. In general, blood glucose levels between 140 and 180
mg/dL are recommended. Although poor glycemic control increases the risk of
infection, rates of VAP are not strongly linked to moderate elevations in blood glucose.

VAP is usually suspected when an intubated patient develops several of the following:
1. New pulmonary infiltrates
2. Increased respiratory secretions
3. Signs of worsened respiratory status, such as worsening oxygenation, lower
tidal volumes, and increased inspiratory pressure
4. Systemic signs of
infection, such as fever,
leukocytosis, and
tachycardia

However, no constellation of
manifestations conclusively
proves VAP (eg, intubated
patients often have pulmonary
infiltrates on x-ray in the
absence of VAP). Therefore,
noninvasive (eg, endotracheal
aspiration) or invasive (eg,
bronchoalveolar lavage)
sampling of the lower
respiratory tract is required for
confirmation; moderate or
heavy growth of ≥1
microorganism is generally
diagnostic.
ARDS is often seen in patients with severe trauma and frequently causes increased
oxygen requirements and pulmonary infiltrates. However, most cases occur within 72
hours of the inciting event; in addition, fever and increased secretions would be
atypical.
ATRIAL FLUTTER

This patient's ECG demonstrates saw-


toothed flutter waves consistent
with atrial flutter. The atrial rate is
typically approximately 300/min, and
the ventricular response can be
constant or variable.
This patient has a variable ventricular
response rate that creates
an irregular rhythm, whereas a
constant ventricular response rate
(eg, a constant 3:1 ventricular
response at a rate of 100/min) can
create a regular-appearing rhythm.

Atrial flutter usually results from a


large reentrant circuit involving the
cavotricuspid isthmus of the right
atrium. Risk factors are similar to
those for a-fib and include CAD, HTN, and HF. The arrhythmia is also frequently seen
following cardiac surgery. Atrial flutter carries a similar risk of arterial
thromboembolism to a-fib; therefore, in a patient with multiple CHA2DS2-VASc risk
factors should be started on chronic anticoagulation (eg, rivaroxaban, apixaban)
therapy.
CHA2DS2-VASc score

Risk criteria Points Total score Annual stroke risk

C Congestive heart failure 1 0 0.2%

H Hypertension 1 1 0.6%

A2 Age ≥75 2 2 2.2%

D Diabetes mellitus 1 3 3.2%

S2 Stroke or TIA 2 4 4.8%

V Vascular disease (eg, prior MI) 1 5 7.2%

A Age 65-74 1 6 9.7%

Sc Sex category female 1 7 11.2%

Maximum score 9 8 10.8%

9 12.2%

MI = myocardial infarction; TIA = transient ischemic attack.

Although not needed in this patient (whose pulse is ~60-70/min), management also
involves rate control, which can sometimes be difficult to accomplish with
medications alone (eg, metoprolol, verapamil). Definitive management often
requires radiofrequency ablation (RFA) to disrupt the reentrant circuit, or electric
cardioversion is also sometimes an option. These interventions increase the risk of a
thromboembolic event and should be preceded by several weeks of anticoagulation
therapy.

………
AV nodal slow pathway ablation is used to treat AVNRT, a type of SVT. ECG would
show a regular rhythm with rapid rate and no P waves.

Exercise stress testing is used to evaluate for CAD in patients with suggestive
symptoms (eg, chest pain on exertion), but it is not indicated in this patient.

Flecainide is a class IC antiarrhythmic that is sometimes used to treat a-fib. Such


rhythm-control agents are generally not used for atrial flutter because of poor
efficacy and the risk of precipitating RVR.

Routine follow-up in 6 months is not appropriate because atrial flutter significantly


increases the risk of embolic stroke in this patient with multiple CHA2DS2-VASc risk
factors.

PARAPNEUMONI
C EFFUSION

- Bacterial pneumonia often causes a pleural effusion. Typically, the effusion is


small, sterile, free-flowing, and resolves with antibiotics (uncomplicated). However,
if bacteria persistently invade the pleural space, a complicated parapneumonic
effusion or empyema may develop. Patients with these pleural space infections
tend to have continued symptoms (fever, pleuritic pain) despite adequate
antibiotics. Chest x-ray often shows loculation (walled-off pleural fluid).
Thoracentesis will show an exudative effusion characterized by:
1. Low glucose (<60 mg/dL) due to consumption (high metabolic activity) by
activated neutrophils and bacteria
2. Low pH (<7.2) due to anaerobic utilization of glucose by neutrophils and
bacteria
3. High protein due to increased microvascular permeability and cellular
destruction
- Empyemas are differentiated from complicated parapneumonic effusions by the
presence of gross pus or bacteria on Gram stain. Most complicated
parapneumonic effusions and all empyemas require drainage (eg, chest tube) in
addition to antibiotics.
- Transudative effusions are marked by low protein (eg, <3 g/dL). Glucose and pH
are usually normal. Parapneumonic effusions are exudative (not transudative).
COMPARTMENT It is most common following traumatic injuries (eg, long-bone fracture) or prolonged
SYNDROME compression of an extremity but may also
occur following reperfusion of an acutely
ischemic limb due to interstitial edema and
possibly intracellular swelling.
- The diagnosis of CS can be
confirmed by measuring compartment
pressures (eg, needle manometry); a delta
pressure (diastolic blood pressure −
compartment pressure) ≤30 mm Hg is
strongly suggestive of CS. Definitive
management includes urgent fasciotomy,
although some patients may be managed conservatively with close observation if
compartment pressures are falling and delta pressures are rising.
PULMONARY http://www.uworld.com/media/L13067.png
EFFUSION (PE)
- Although prophylactic-dose
anticoagulation may reduce
the risk of PE, it does not
eliminate the risk
completely, especially in
high-risk patient
populations.
- CT pulmonary angiography
is the initial diagnostic test
of choice due to high
diagnostic accuracy.
Ventilation-perfusion scan is
an alternative option in
patients unable to receive
intravenous contrast (eg,
due to renal dysfunction or
allergy).
- Therapeutic-dose (as
opposed to prophylactic
dose) low molecular weight heparin (eg, enoxaparin, dalteparin) is often the
preferred treatment for acute PE in cancer patients due to a possible increased risk
of malignancy-related bleeding with direct oral anticoagulants (eg, rivaroxaban).
- Postoperative atelectasis is common, often resulting from shallow breathing in the
setting of anesthesia and postoperative pain. Positive-pressure ventilation is
sometimes used to help open collapsed alveoli and improve atelectasis; however,
it would not address this patient's likely probability of PE. In addition, atelectasis
does not typically cause chest pain.
AIR EMBOLISM - venous air embolism (VAE) results from the introduction of air into the venous
circulation; potential causes include trauma, certain surgeries (eg, neurosurgical,
otolaryngological), central venous catheters, and pulmonary barotrauma.
- A small VAE often causes
minimal sequelae,
traveling to the
pulmonary capillaries
where it can diffuse into
the alveoli without
consequence. However,
a large VAE (eg, >50 mL)
can lodge in the right
ventricle to cause right
ventricular outflow tract
obstruction or lodge within the
pulmonary arterioles to
obstruct pulmonary blood
flow. Affected patients usually
develop sudden-onset
dyspnea and respiratory
distress, often accompanied by
obstructive shock (with
hypotension and tachycardia)
that can lead to cardiac arrest
(eg, pulseless electrical
activity). As with thrombotic
pulmonary embolism,
hypoxemia can result from
dead space ventilation and ventilation-perfusion mismatching due to obstructed
pulmonary blood flow.
- Patients with suspected VAE should immediately be placed in the left lateral
decubitus or left lateral Trendelenburg (left lateral decubitus with head down)
position, which traps the VAE on the lateral wall of the right ventricle, preventing
RVOT obstruction and further embolization of air into the pulmonary circulation.
High-flow oxygen (or, in severe cases, hyperbaric oxygen) is also important as it
shrinks the air embolus by encouraging the absorption of nitrogen gas into the
blood.
………………………………………………………………….
- Air embolism can also affect the arterial circulation, most commonly due to trauma
or neurosurgical procedures. Manifestations of arterial air embolism include
stroke and myocardial infarction (due to cerebral or coronary microarterial
occlusion, respectively), the latter of which can lead to rapid cardiac arrest.
- ARDS involves the development of diffuse pulmonary edema; patients usually
experience relatively gradual (eg, over 1-2 hr) decompensation. VAE can
sometimes precipitate ARDS by triggering endothelial damage and inflammation;
however, this patient's normal breath sounds (absence of crackles) and abrupt
decompensation are not consistent with ARDS.
- Cardiac tamponade is a potential cause of obstructive shock, but it would be
unlikely to cause sudden ventilation-perfusion mismatching and hypoxemia with
clear lungs.
- Tension pneumothorax can cause obstructive shock by cutting off venous return
to the right atrium; however, unilaterally absent breath sounds are expected.
- Upper airway obstruction (eg, due to an expanding hematoma with tracheal
compression) would cause hypoxemia and respiratory distress; however, stridor is
also expected.
- Prone positioning reduces atelectasis in the posterior lung (where most of the lung
mass is located) and is sometimes used to improve oxygenation in patients with
severe acute respiratory distress syndrome and persistent hypoxemia despite
optimal mechanical ventilation settings.
- Right lateral decubitus positioning is likely to encourage movement of air into the
RVOT and may worsen the clinical manifestations of VAE.
- Semirecumbent positioning may lower the risk of ventilator-acquired pneumonia
in intubated patients on mechanical ventilation.
- Patients with suspected arterial (rather than venous) air embolism should be
placed in supine positioning, as it helps prevent the embolism from traveling to
the brain and causing a stroke.
LOCAL VASCULAR
COMPLICATIONS
AFTER CARDIAC
CATHETERIZATIO
N

- Most patients are initially asymptomatic but gradually develop mild swelling and
localized tenderness. As arterial pressure exceeds venous pressure throughout the
cardiac cycle, a continuous bruit with a palpable thrill is typically present. In
addition, distal pulses may be diminished in the affected extremity. An untreated
AVF can progressively enlarge and lead to limb edema (due to venous
hypertension), limb ischemia (due to redirection of arterial blood flow), and high-
output heart failure (due to blood returning to the right atrium without passing
through peripheral resistance).
- The diagnosis is typically confirmed by duplex ultrasound. Management of small
AVFs involves observation (sometimes resulting in spontaneous closure) or
ultrasound-guided compression. Large AVFs typically require surgical repair.
- Femoral artery pseudoaneurysm develops when an inadequately sealed arterial
puncture site leads to a hematoma contained within periarterial connective tissue.
Ongoing communication with the arterial lumen creates a tender, pulsatile mass
with an audible systolic (rather than a continuous) bruit.
SEPTIC SHOCK - Crystalloid should be given as intravenous boluses (500-1000 mL) to improve
systolic blood pressure >90 mm Hg, at which perfusion is considered adequate.
This is continued with close monitoring for evidence of volume overload (eg,
pulmonary edema, hypoxia) or until fluid no longer improves the blood pressure.
If the patient fails to respond or develops evidence of volume overload without
improvement in blood pressure, then vasopressors (eg, dopamine) should be
started to improve perfusion.
- Poor urine output → he first should be given a fluid challenge through boluses to
see if urine output improves.
- The use of bicarbonate is not clearly indicated unless there is a severe acute
acidosis (pH <7.2).
- Generally, even in patients with septic shock, the goal of transfusion is to keep the
hemoglobin >7 g/dL unless they have not responded to crystalloid and
vasopressors.
- Acute pancreatitis can progress to systemic inflammatory response syndrome and
lead to distributive shock. Pulmonary artery catheterization (PAC) reveals high CI
and low PCWP.
- Hypotension due to adrenal crisis is caused by distributive shock and
intravascular volume depletion. PAC reveals normal or high CI and normal or low
PCWP.
- Obstructive shock: PCWP is usually normal or low.
- Hypovolemic shock: CI (cardiac index) and PCWP are typically low.
SHOCK

- Hypovolemic shock: The left ventricle, decreased in size due to low filling volume,
also compensates by increasing ejection fraction.

- Despite the compensatory mechanisms in hypovolemic shock, cardiac output


cannot be sustained and circulatory collapse (and death) occurs in the absence of
aggressive volume resuscitation and control of volume loss.
- CVP provides an estimate of cardiac preload.
1. The CVP value can help differentiate between different types of shock:
2. A low CVP (low preload) typically reflects hypovolemic or distributive
shock.
- An elevated CVP (high preload) typically reflects cardiogenic or obstructive shock.
- Diastolic collapse (failure of ventricular filling) due to elevated right ventricular
filling pressure is characteristic of cardiac tamponade.
- A dilated left ventricle with apical hypokinesis and engorgement of the inferior
vena cava are characteristics of cardiogenic shock.
- Right ventricular dilation and hypokinesis can occur due to massive pulmonary
embolism. If present, should be accompanied by jugular venous distension.
RIB FRACTURE

- Left shoulder pain (with normal neuromuscular examination) is concerning for


referred pain from the left diaphragm. Because of the intimate association
between the diaphragm and splenic capsule, a contained splenic bleed can
present with referred left shoulder pain prior to the onset of more significant
abdominal symptoms.
- Suspicion of intraabdominal organ injury should prompt CT scan of the abdomen,
preferably with IV contrast to better visualize solid organ injury and potentially
detect contrast "blush" (extravasation) at the site of bleeding.
- CTA of the chest can identify injuries to the intrathoracic vasculature. Injuries to
major vessels (eg, aorta, subclavian vessels) are more common with fractures of
ribs 1-3.
MECHANICAL
COMPLICATIONS
OF MYOCARDIAL
INFARCTION (MI)

- IV wall rupture is pathologically similar to ventricular free wall rupture and both
have a peak incidence around 5 days after infarction. However, interventricular
wall rupture causes a ventricular septal defect and not pericardial tamponade.
Patients typically present with sudden onset of hypotension, congestive heart
failure (predominantly right heart failure), and a loud holosystolic murmur heard
best at the lower left sternal border.
- Acute peri-infarct pericarditis can occur within 1-3 days after an MI and can
present with a pericardial friction rub with or without chest pain. It is usually self-
limited and resolves with supportive care. Dressler syndrome is post-MI
pericarditis occurring weeks to months after the MI.
ATELECTASIS - This patient likely has atelectasis of
the left lung due to a left mainstem
bronchial mucus plug. Airway
obstruction (eg, mucus, tumor, foreign
body) creates distal air trapping in the
alveoli. Patients typically present with
respiratory distress (eg, dyspnea,
tachypnea, tachycardia) and hypoxemia,
and lung examination reveals dullness to
percussion and absence of breath sounds
in the affected lung area. Chest x-ray
demonstrates opacification of the
affected lung area with mediastinal
shifting toward the side of opacification.
In addition, rib spacing becomes
narrower in the affected hemithorax.
Surgery (under anesthesia) and smoking increase the risk of mucus plugging. Chest
physiotherapy is often useful in preventing mucus plugging and can be used to
treat relatively minor atelectasis due to mucus plugging. Large-volume atelectasis
typically requires bronchoscopy to remove the mucus plug.
- Diaphragmatic perforation is usually caused by blunt abdominal trauma and
typically presents with respiratory distress or signs of small bowel obstruction (eg,
vomiting, colicky pain).
- An endobronchial tumor can lead to obstructive atelectasis; however, lack of
cough or hemoptysis and his acute presentation following surgery make bronchial
mucus plugging more likely.
- A large pleural effusion can cause large opacification on chest x-ray. However,
because the effusion is occupying space, the mediastinum will be shifted away
from (rather than toward) the side of effusion.
-
- Multilobar pneumonia can cause opacification of an entire hemithorax on chest x-
ray. However, the mediastinum does not typically shift in either direction, and
fever and cough would be expected.
HYPOXEMIA

- Fluid from pulmonary edema fills the alveoli and facilitates alveolar collapse.
Alveolar ventilation (V) is zero, which results in an extreme ventilation/perfusion
(V/Q) mismatch (right-to-left intrapulmonary shunting), causing hypoxemia.
Because diffuse pulmonary edema prevents air from reaching the alveoli
throughout much of the lungs (eg, >50%), an increase in the fraction of inspired
O2 (FiO2) does not correct the hypoxemia.
- Increased alveolar-arterial (A-a) gradient is another characteristic of pulmonary
edema, as with any process causing V/Q mismatch
- In contrast to hypoxemia due to diffuse pulmonary edema, hypoxemia due to
localized intrapulmonary shunting (eg, lobar pneumonia, atelectasis) typically does
correct with increased FiO2 because only a portion of the lungs is affected and the
normally ventilated alveoli compensate via increased O2 transfer.
- Large-scale dead-space ventilation (eg, massive pulmonary embolism) is the other
major cause of hypoxemia that does not correct with supplemental O2. Although
alveolar ventilation is normal, there is no perfusion of large portions of the lung
(extreme V/Q mismatch), resulting in minimal gas exchange.
PNEUMOTHORAX
- Primary spontaneous
pneumothorax (PSP): Other
risk factors include smoking,
Marfan syndrome, and
thoracic endometriosis.
- Radiographically,
pneumothoraces are
categorized as small (≤2 cm)
or large. Small
pneumothoraces in clinically
stable patients, such as this
patient, can be managed with
observation and supplemental oxygen (regardless of oxygen saturation), which
enhances the speed of resorption. Stable patients with large pneumothoraces
should undergo decompression with a large-bore needle (eg, 14- to 18-gauge)
inserted in the second or third intercostal space in the midclavicular line or at the
fifth intercostal space in the mid or anterior axillary. Patients who are
hemodynamically unstable should undergo emergent placement of a tube
thoracostomy. If tube thoracostomy is not available or will be delayed, urgent
needle decompression can be performed.
- Chest CT scan is not necessary unless diagnostic uncertainty exists after a plain
radiograph or if there is concern for tube thoracostomy placement or a loculated
pneumothorax.
- If the lung fails to adequately re-expand (eg, <90% expansion) and a persistent air
leak is present, placement of a one-way valve, creation of a blood patch, or VATS
can be used. For patients with recurrence, VATS pleurodesis or chemical (eg,
tetracycline derivative, talc) pleurodesis can be performed.
- When TP is suspected,
decompression (eg,
needle thoracostomy)
should be performed
immediately to
prevent cardiovascular
collapse. Needle
thoracostomy can be
performed quickly and
should precede
intubation. This is an
important exception to
the typical order of
establishing the airway
first (ie, airway,
breathing, circulation)
but is necessary
because positive-
pressure ventilation
(eg, intubation and
mechanical ventilation) rapidly increases accumulated air and intrathoracic
pressure, exacerbating TP and causing cardiovascular collapse. Following needle
decompression, tube thoracostomy is required for definitive pneumothorax
management.

HYDROTHORAX - Pleural effusions occur in almost half of patients who undergo CABG surgery.
Most are small, asx, and benign effusions that usually develop on the left side and
are the result of the body's reaction to surgical incision of the pleura, disruption of
mediastinal lymphatic channels, cardiac cooling, or postoperative pericarditis.
Post–cardiac surgery effusions that meet the following criteria can be managed
conservatively with observation only:
1. Small to moderate in size and not enlarging
2. Early onset (postoperative day 1 or 2)
3. Not associated with respiratory symptoms

- Effusions that do not meet the above criteria should be actively investigated and
treated because they may be due to serious post–cardiac surgery complications,
including hemothorax, central line erosion into the pleural space, mediastinitis,
chylothorax, and MI with heart failure.
- The initial management of large, progressively expansive, or symptomatic pleural
effusions should involve diagnostic thoracentesis and fluid analysis. Chest tube
placement may be necessary for large effusions or some effusions of infectious
etiology; tube placement is also sometimes used in the management of
hemothorax or pneumothorax.
HEMOTHORAX - In the setting of trauma (eg, unrestrained passenger in a motor vehicle collision),
hypovolemic shock is attributed to hemorrhage until proven otherwise. A source of
bleeding should be sought, with special attention directed toward compartments
that can conceal massive, traumatic hemorrhage: the chest, abdomen/pelvis
(including retroperitoneum), and thigh.
- Hemothorax may result from injuries to larger intrathoracic structures (eg, aorta,
hilar vessels) and/or smaller structures (eg, lung parenchyma, internal mammary or
intercostal blood vessels). Each hemithorax can hold up to 40% of the circulating
blood volume, and massive hemothorax (>1,500 mL) can present with signs of
hemorrhagic shock and tracheal deviation from mass effect. Tube thoracostomy
should be performed, with immediate bloody drainage of >20 mL/kg (~1,500 mL)
being an indication for emergent surgical thoracotomy.
- Lung atelectasis (with collapsed alveoli) and lung contusion (with resulting alveolar
edema) can cause tachypnea (to compensate for ventilation-perfusion mismatch)
and tachycardia (to compensate for hypoxemia). In addition, both may cause
decreased breath sounds and dullness to percussion.
ARTERIOVENOUS -Hemodialysis access is
(AV) FISTULA commonly obtained through
a surgically created
arteriovenous fistula. The
fistula forms an enlarged
vein, which serves as an
access point and facilitates
adequate blood flow to and
from the hemodialysis
machine.
- Marked changes in these hemodynamic parameters can lead to high-output heart
failure; therefore, care must be taken to ensure that a surgically created
arteriovenous fistula is not too large.
FAT EMBOLISM - FES: The condition
SYNDROME (FES) can occur 24-72
hours following
fracture or surgical
manipulation of
bones that contain
abundant marrow
(eg, femur, pelvis).
The marrow,
consisting of fat and
hematopoietic cells,
embolizes into the
venous circulation
and can cause
physical obstruction in the pulmonary capillaries. Some of the emboli may pass
through the pulmonary circulation to cause microvascular occlusion in the systemic
circulation (eg, brain, dermal capillaries). The circulating fat globules likely also
induce a systemic inflammatory response that further contributes to microvascular
dysfunction.
- Patients with FES classically have the triad of respiratory distress (eg, tachypnea,
hypoxemia), neurologic dysfunction (eg, confusion), and a petechial rash; however,
the rash is present in less than half of cases. Thrombocytopenia may occur due
to platelet adherence and aggregation to circulating fat globules. Patients typically
develop pulmonary edema (scattered ground glass opacities; mimicking ARDS)
after 24-48 hours, evidenced in this patient by bilateral ground-glass opacities on
chest CT. No pulmonary arterial filling defects are seen because the emboli
obstruct the pulmonary capillaries and are too small to be detected on CT.
- Aspiration pneumonitis → fever, leukocytosis
- DRESS: seen with certain antiepileptic medications (eg, lamotrigine, CBZ); however,
levetiracetam is not a common culprit.
Patients may have respiratory distress
and confusion due to systemic organ
involvement, but a macular rash is
also expected as it is the most
common manifestation of DRESS.
- Pulmonary contusion results from
chest trauma and involves localized
pulmonary edema at the site of injury.
The edema can take ~24 hours from
the time of injury to develop;
however, it is not expected to occur in a diffuse, scattered pattern.
CORONARY Sternal dehiscence: a complication of cardiac surgery that occurs when the 2
ARTERY BYPASS approximated edges of the bony sternum separate, typically due to loosening or
GRAFT (CABG) fracture of the suture wire. It can occur with or without the involvement of superficial
COMPLICATIONS soft tissues (ie, superficial dehiscence) or concurrent deep wound infection (ie,
mediastinitis). Patients may have mild pain or sensation of chest wall instability and
"clicking" with chest movement. The diagnosis can be made radiographically (eg,
displaced sternal wires) or clinically; palpable rocking or clicking of the sternum
confirms the diagnosis.

Risk factors are related to impaired wound healing. Obesity, macromastia (large
breasts), and the sequelae of chronic obstructive pulmonary disease (eg, barrel chest
deformity, chronic cough) increase tensile forces on the wound closure, predisposing
patients to sternal separation. Internal thoracic artery harvesting can result in sternal
ischemia, whereas patients with diabetes or who smoke are at risk for poor wound
healing and infection.

Sternal dehiscence is a surgical emergency and requires urgent surgical exploration,


debridement, and sternal fixation to prevent cardiac damage from loose wire or bone
fragments. Patients without evidence of underlying infection can undergo rewiring or
sternal plate placement to stabilize the sternum.

Sternal dehiscence is a surgical emergency and clinical observation without surgical


intervention risks increases in patient morbidity and mortality.

Compressive sternal dressings and negative pressure wound therapy are frequently
used in patients with sternal wound failure to protect the wound and speed its healing.
However, this patient has palpable rocking of the sternum consistent with sternal
dehiscence and therefore requires surgical intervention and sternal fixation to prevent
cardiac injury.

Blood and wound cultures and broad-spectrum intravenous antibiotics are indicated if
there is concern for infection; however, empiric oral antibiotics should be avoided due
to the high mortality risk associated with deep sternal wound infection. In addition,
this patient does not have signs of infection (ie, fever, chills, erythema on examination),
and fixation of the mobile sternum is more appropriate.
----------------------------------------------
Wound dehiscence after CABG is a relatively common complication, and management
is determined by the extent of tissue involvement.
1. Soft tissue dehiscence occurs when only the superficial tissues (eg, skin,
muscle) separate. There are no signs of sternal instability or systemic illness;
local wound care or debridement followed by primary closure is indicated.
2. Sternal dehiscence occurs with separation of the edges of the approximated
sternum and may occur with or without soft tissue dehiscence. Sternal
instability and nonunion is characterized by "clicking" or "rocking" on sternal
palpation. It is a surgical emergency and requires sternal rewiring to prevent
cardiac trauma.

A high-mortality complication of dehiscence is deep tissue infection (mediastinitis),


due to either contiguous spread of superficial infection or intraoperative deep tissue
contamination. Although it classically presents with systemic symptoms (eg, fever,
tachycardia), chest pain, chest wall edema/crepitus, and purulent wound discharge,
atypical presentations can occur; therefore, any patient with significant sternal wound
drainage should be evaluated with chest and sternal imaging (eg, mediastinal fluid
collections or pneumomediastinum on CT scan). Management includes emergency
surgical debridement, tissue cultures, and empiric intravenous antibiotics.
- Patients with uncontrolled diabetes are more likely to develop postoperative
wound complications
- Topical and oral antibiotics do not have a role in the initial management of
postoperative wound infection due to the risks of underlying deep tissue infection.
Broad-spectrum intravenous antibiotics should be initiated if mediastinitis is
suspected, but in this hemodynamically stable patient with no signs of sternal
instability, imaging and cultures should be pursued first to confirm the diagnosis
and guide management.
……………………………………………………..
- Acute mediastinitis, a possible complication of cardiac surgery that is usually due to
intraoperative wound contamination. Mediastinitis can complicate up to 5% of
sternotomies. Patients typically present post-operatively (usually within 14 d) with
fever, tachycardia, chest pain, leukocytosis, and sternal wound drainage or purulent
discharge.
- CXR usually shows a widened mediastinum in non-postoperative mediastinitis,
but this can also be seen in postoperative mediastinitis after cardiac surgery. The
diagnosis is usually clinically made and confirmed during surgery when pus is
noted in the mediastinum. Postoperative mediastinitis requires drainage,
surgical debridement with immediate closure, and prolonged antibiotic
therapy. Antibiotics alone do not appropriately treat mediastinitis. Acute
mediastinitis has a mortality rate of 10%–50%, even with appropriate treatment.
- A-fib commonly (up to 15%–40%) occurs within a few days after CABG and is
usually self-limited, with resolution in <24 hours. Rate control with beta-
blockers or amiodarone is best. Anticoagulation and/or cardioversion is
reserved for patients with a-fib lasting >24 hours after CABG. Of the patient
requires immediate surgery anticoagulation should not be given.
- Postpericardiotomy syndrome can present with fever, leukocytosis, tachycardia, and
chest pain. However, it is usually autoimmune and occurs a few weeks following a
procedure with a pericardium incision. NSAIDs or steroids treat the inflammation,
and pericardial puncture is indicated if tamponade occurs.
Post–cardiac injury (Dressler) syndrome (PCIS)

• MI
Risk
• Cardiac surgery or trauma
factors
• PCI

• Latent period of several weeks to months


Clinical • Pleuritic chest pain, fever, leukocytosis
features • CXR: pleural effusion ± enlarged cardiac silhouette
• Echocardiography: pericardial effusion

• NSAIDs + colchicine
Treatment
• Corticosteroids in refractory disease

• Usually self-limited disease course


Prognosis
• May cause chronic/recurrent disease leading to constrictive pericarditis

This patient with pleuritic chest pain occurring several weeks after coronary artery
bypass graft (CABG) surgery most likely has acute pericarditis due to post–cardiac
injury (Dressler) syndrome (PCIS). This condition results from immune-complex
deposition in the pericardium and pleura that can occur following any event or
intervention (eg, MI, CABG surgery, percutaneous coronary intervention [PCI]) that
facilitates immune system exposure to cardiac antigens. PCIS is characterized by
a latent period of several weeks to months from the time of cardiac injury to
presentation of symptoms.

PCIS presents like other types of acute pericarditis; in addition to pleuritic chest pain,
patients may have fever, leukocytosis, and elevated inflammatory markers (eg, ESR,
CRP). ECG findings are usually nonspecific, but may demonstrate the classic widespread
ST-segment elevation associated with acute pericarditis. Pericardial effusion is usually
present and should be evaluated with echocardiography.

The diagnosis is made based on a consistent clinical picture. Treatment of PCIS is


usually with NSAIDs plus colchicine; corticosteroids are reserved for cases refractory
to initial therapy. Colchicine monotherapy can also be effective in preventing PCIS
when administered following cardiac surgery. The course of PCIS is typically self-
limited; however, a small percentage of patients can experience a chronic course with
multiple disease recurrences that can lead to constrictive pericarditis.

………
Bacterial mediastinitis, a serious complication of CABG surgery, usually presents with
fever, chest pain, and incisional purulence, dehiscence, or crepitus within 2 weeks of
surgery. It is unlikely in a patient with a normal wound appearance 6 weeks after
cardiac surgery.
Hemopericardium can result from ventricular free wall rupture following myocardial
infarction. It typically leads to cardiac tamponade, which is unlikely in the absence of
hypotension.

Venous graft failure following CABG surgery typically presents with a recurrence of
symptoms of stable angina (eg, exertional pressure-like chest pain).

Viral infection is considered the MCC of acute pericarditis, but PCIS is more likely in
this patient who had CABG surgery 6 weeks ago.
EHLER-DANLOS
SYNDROME (EDS)

- Although manifestations vary depending on the genetic mutation, most cases are
marked by ≥1 of the following:
1. Joint hypermobility/laxity, which may result in multiple joint dislocations
2. Skin hyperextensibility, which allows skin to stretch ≥4 cm
3. Tissue fragility, which leads to easy bruising and spontaneous organ rupture
4. Poor wound healing, which leads to the formation of wide, atrophic scars
("cigarette paper–like")
- Confirmation of EDS generally requires referral to a clinical geneticist
BLUNT
THORACIC
AORTIC INJURY
(BTAI)

- Patients who undergo rapid deceleration (eg, motor vehicle collision, fall >3 m [10
ft]) are at risk for blunt thoracic aortic injury (BTAI). Complete aortic rupture (ie,
full-thickness tearing through intimal, medial, and adventitial layers) typically results
in rapid exsanguination and death prior to hospital arrival.
- Although external signs of blunt chest trauma (eg, bruising, steering wheel imprint)
are concerning for BTAI, there are no clinical findings specific for aortic injury.
Chest x-ray findings suggestive of BTAI include a widened mediastinum, abnormal
(eg, enlarged) aortic contour, and/or left-sided effusion due to hemothorax (which
explains this patient's diminished breath sounds on the left). Concomitant injuries
(eg, rib fractures) may be present. Diagnosis of BTAI can be confirmed via CT
angiography for hemodynamically stable patients or transesophageal
echocardiography (likely in the operating room) for unstable patients. Although
hemodynamic support (eg, fluids, blood products) is indicated in hypotensive
patients, systolic blood pressure is generally kept at <100 mm Hg to prevent injury
extension and rebleeding while awaiting emergent operative repair.
- Rapid deceleration exerts stretching, shearing, and torsional forces capable of
rupturing the aorta. The aortic isthmus—the transition zone between the relatively
mobile ascending aorta/arch and the fixed descending aorta—is particularly
vulnerable to these forces and the most common site of BTAI.
-
- Complete aortic rupture (ie, tear of the intima, media, and adventitia) typically
results in rapid exsanguination and death. This patient who survived the fall likely
has an incomplete rupture (ie, tear of the intima ± media), which may result in:
• Creation of a secondary, false lumen similar to aortic dissection
• Creation of an obstructive intimal flap or intramural hematoma that impedes
distal blood flow (pseudocoarctation), resulting in proximal hypertension
and distal hypotension (eg, upper extremity hypertension with diminished
femoral pulses)
• Expansion of the adventitia under high-flow pressure, causing
compression/stretching of surrounding structures such as the left
recurrent laryngeal nerve (eg, hoarse voice)
- CTA of the chest is highly sensitive and specific for thoracic aortic injury and is
readily available. TEE can also evaluate the thoracic aorta but requires an
experienced echocardiographer. TTE does not adequately visualize the thoracic
aorta, which progressively becomes a more posterior anatomical structure (closer
to the esophagus) as it arches and transitions into the descending aorta.
Incomplete rupture should prompt emergent surgical evaluation.
- Bronchial rupture → severe pneumothorax, pneumomediastinum, subcutaneous
emphysema (palpable crepitus), and/or audible crunch on cardiac auscultation (ie,
Hamman sign, from pneumomediastinum).
- Myocardial contusion can cause hypotension and tachycardia but is classically
accompanied by arrhythmia or new bundle branch block
- Chest x-ray findings of esophageal rupture may include widened mediastinum and
pleural effusion (from accumulation of leaked esophageal contents). However,
sudden-onset shock shortly after the insult makes BTAI much more likely.
Mediastinitis from untreated esophageal rupture can result in septic shock but
typically develops over hours to days (rather than minutes).
- Although esophageal rupture can occur with blunt chest trauma and cause severe
chest and back pain, it does not cause differential blood flow between the upper
and lower extremities.
- Flexible bronchoscopy visualizes the proximal airway lumen and can diagnose
tracheobronchial injury
- ……………………………………………………………………
- This patient with a fall >3 m [10 ft] followed by rapid deceleration (landing on
pavement) is at risk for blunt thoracic aortic injury (BTAI) because the abrupt
change in velocity exerts a combination of stretching, shearing, and torsional forces
capable of rupturing the aorta. Patient presentation depends on the degree of
aortic rupture; those with an incomplete rupture (as with this patient) may be
normotensive to hypertensive (due to sympathetic response) and may initially
appear stable even though they have a life-threatening injury.
- All patients with blunt chest trauma require a chest x-ray after primary trauma
survey. Although patients with BTAI may have a normal chest x-ray, findings
concerning for BTAI include:
1. Widened mediastinum or abnormal aortic contour (eg, enlarged aortic knob)
from either an intimal tear (creating a secondary, false lumen similar to aortic
dissection) or incomplete rupture of the intima and media (where the adventitia
expands under high-flow pressure)
2. Left-sided effusion (ie, hemothorax) from aortic bleeding into the thorax
- Diagnosis can be confirmed via CT angiography for hemodynamically stable
patients or transesophageal echocardiography (likely in the operating room) for
unstable, hypotensive patients.
- Diaphragmatic rupture may cause a mass-effect mediastinal shift (but not
widening).
- Left ventricular aneurysm appears as a prominence or bulge along the left heart
border, not as a widened mediastinum.
- Myocardial contusion has no specific chest x-ray findings, although sternal fracture
should raise suspicion. It is typically diagnosed by ECG (showing arrhythmia or new
bundle branch block) or echocardiography (showing wall motion abnormality or
decreased contractility).
- Pulmonary contusion causes parenchymal hemorrhage and edema that manifests
as irregular, nonlobular opacities.
- Tracheobronchial disruption may cause mediastinal widening due to rapid efflux of
air from the injured tracheobronchial tree; however, air would be visualized as
pneumomediastinum. In addition, pneumothorax and/or subcutaneous
emphysema would likely be present.

HEMOPTYSIS

- In pts from TB-endemic area (e.g., Mexico, india, china…) and typical radiographic
abnormalities (eg, patchy or nodular opacity, multiple nodules, cavity) involving the
apical-posterior segments of the upper lobes. If the pt is stable → Respiratory
Isolation PLUS Sputum analysis. Respiratory isolation should be continued until the
diagnosis of tuberculosis can be confirmed or refuted by additional testing (eg,
acid-fast bacilli smear and culture).
- Bronchoscopy should be performed to directly visualize and control the site of
bleeding in patients with massive hemoptysis (>600 mL/day or 100 mL/hr). This
patient is stable, and sputum analysis would be the next appropriate step.
- Endotracheal
intubation should be
performed to protect
the airway in patients
with hemoptysis who
are hemodynamically
unstable or have
poor gas exchange,
severe dyspnea, or
massive hemoptysis.
- The greatest danger
in massive
hemoptysis is not
exsanguination but
asphyxiation due to
the airway flooding
with blood. Initial
management
involves establishing
an adequate patent
airway, maintaining
adequate ventilation
and gas exchange,
and ensuring
hemodynamic stability. The patient should be placed with the bleeding lung in the
dependent position (lateral position) to avoid blood collection in the airways of the
opposite lung.
- Bronchoscopy is the initial procedure of choice in such patients as it can localize the
bleeding site, provide suctioning ability to improve visualization, and include other
therapeutic interventions (eg, balloon tamponade, electrocautery).
- High-resolution CT scan of the chest can be used in patients with mild-to-moderate
hemoptysis to better visualize the underlying lung pathology.
- FFP should be given to patients with known or suspected coagulopathy (eg, INR
>1.5) as the cause of hemoptysis.
- Pulmonary arteriography should be performed when there is persistent bleeding
and the initial bronchoscopy is unable to localize the source. It can precisely
identify the bleeding vessels and be used for therapeutic embolization to stop the
bleeding.
- Urgent thoracotomy and surgical intervention are reserved for patients with
unilateral bleeding who continue to bleed despite initial bronchoscopy and/or
pulmonary artery embolization.
ATELECTASIS - Atelectasis is one of the most common postoperative pulmonary complications and
is particularly common after abdominal and thoracoabdominal surgery. Following
such surgeries, pain and changes in lung compliance can cause impaired cough and
shallow breathing. Shallow inhalations limit recruitment of alveoli at the lung
bases, and weak cough predisposes to small-airway mucus plugging. This results in
hypoxia (low pO2), which stimulates an increase in respiratory rate, causing low
pCO2.
- Atelectasis can be asx or manifest as increased work of breathing. It is most likely
to occur after the second postoperative night to the fifth postoperative night.
Radiologic findings demonstrate loss of lung volume due to the collapse of lung
tissue. Adequate pain control, deep-breathing exercises, directed coughing, early
mobilization, and incentive spirometry decrease the incidence of postoperative
atelectasis.
- General anesthesia impairs laryngeal defenses, predisposing to aspiration of gastric
contents. Residual effects of general anesthesia can also cause hypoventilation.
Both the aspiration and residual anesthetic effects usually appear within the first
few hours after surgery, often before the patient leaves the perioperative unit.
Signs and symptoms of aspiration generally include sudden-onset dyspnea, cough,
fever, and rhonchi/crackles on examination.
- Bronchospasm is common in the postoperative period, particularly in patients with
underlying chronic obstructive pulmonary disease or asthma. Wheezing and
dyspnea would be expected in addition to hypoxemia.
- Diaphragmatic paralysis may occur as a consequence of phrenic nerve injury from
thoracic surgery, cervical manipulation, or tumor compression but is less likely
during abdominal surgery. Unilateral diaphragmatic paralysis is often
asymptomatic. Bilateral diaphragmatic paralysis causes hypoxemia; rapid, shallow
breathing; orthopnea; and even respiratory failure.
RETROPERITONE - Retroperitoneal hematoma due to bleeding from the arterial access site (with
AL HEMATOMA retroperitoneal extension): Local vascular complications at the catheter insertion
site - bleeding, hematoma (localized or with retroperitoneal extension), arterial
dissection, acute thrombosis, pseudoaneurysm, or AVF formation - are the MC
complications of cardiac catheterization. Most hemorrhage or hematoma
formation occurs within 12 hours of catheterization.
- Patients with hematoma present with localized discomfort and/or swelling of the
soft tissue. If the arterial puncture site is above the inguinal ligament, the
hematoma can extend into the retroperitoneal space, even with minimal visible
localized hematoma, and present with sudden hemodynamic instability and
ipsilateral flank or back pain. Diagnosis is confirmed with non-contrast CT scan
of abdomen and pelvis or abdominal ultrasound. Treatment is usually
supportive, with:
1. intensive monitoring,
2. bed rest, and
3. IV fluids or blood transfusion.
Surgical repair of hematomas or retroperitoneal hemorrhage is rarely required.
- Since local hematoma formation is not uncommon, patients should be advised to
avoid strenuous activity or lifting heavy objects for 1-week post
catheterization. Radial artery approach leads to fewer local vascular
complications, which has led to an increasing number of cardiac catheterizations
being performed via this approach.
CENTRAL - A CVC is commonly used for the administration of critical care medications (eg,
VENOUS pressors, hypertonic saline) and in the setting of difficult vascular access or need for
CATHETER (CVC) long-term medication (eg, chemotherapy). The preferred points of central venous
access are the internal jugular vein (typically by ultrasound guidance) or the
subclavian vein (typically by anatomic landmark guidance). CVC complications
occur in up to 15% of cases, and many are due to inappropriate catheter
placement.

- The ideal placement of a CVC tip is in the lower superior vena cava. Tip placement
in smaller veins (eg, subclavian, jugular, azygous) predisposes to venous
perforation. In addition, inappropriately placed catheter tips may cause lung
puncture, leading to pneumothorax, or myocardial perforation, leading to
pericardial tamponade. Arterial (eg, subclavian) puncture is also a risk, particularly
in the absence of ultrasound guidance.

- Confirmatory chest x-ray may be omitted in the setting of an uncomplicated (eg,


first needle insertion, no resistance to catheter advancement) ultrasound-guided
CVC placement. Otherwise, a portable chest x-ray should be performed
immediately following CVC placement to facilitate timely recognition of a
misplaced catheter tip, identify possible injury (eg, pneumothorax), and prevent
exacerbation of injury (eg, infusion of fluid into the pericardial space). Visualization
of the catheter tip just proximal to the angle between the trachea and right
mainstem bronchus confirms appropriate placement.

- In this patient, catheter tip location should be confirmed prior to the administration
of medications (eg, antibiotics) or fluids.

- Bedside echocardiography may determine the position of a CVC tip and detect a
pneumothorax but only if performed by an experienced operator. Chest x-ray is
the standard of care as it is more practical and reliable.

- A CVC may trigger cardiac arrhythmias if inserted too far into the right atrium.
Patients in critical care settings are typically on continuous telemetry that will detect
cardiac arrhythmia. In the absence of continuous telemetry, patients should be
monitored for hemodynamic instability (eg, tachycardia, hypotension) during and
after catheter placement, but routine ECG is not indicated.
ACUTE
RESPIRATORY
FAILURE AFTER
TRANSFUSION

- TRALI is a rare but life-threatening complication of blood product transfusion


marked by the massive release of cytokines, ROS, and other inflammatory
mediators from neutrophils in the pulmonary vasculature in response to transfused
blood components. Manifestations begin within a few hours of transfusion
initiation and include acute, inflammatory pulmonary edema with hypoxia,
tachypnea, and bilateral pulmonary infiltrates. Fever, tachycardia, and
hypotension (like shock) also commonly occur. Treatment includes immediate
cessation of the transfusion and ventilatory support. Some patients recover
within 24-48 hours, but up to 50% die.
- Anaphylactic transfusion reaction most often occurs in those with IgA deficiency
due to the reaction of anti-IgA antibodies with transfused blood. Respiratory
distress and hypotension are common, but patients also have hives, wheeziness,
stridor, and/or angioedema.
- Transfusion-transmitted bacterial infection is associated with high fever, rigors,
tachycardia, and hypotension.
DIAPHRAGMATIC
INJURY

- A patient experienced blunt thoracoabdominal trauma (MVA, anterior chest wall


bruises) resulting in hemopneumothorax that required chest tube placement. He
now has continued shortness of breath despite appropriate chest tube drainage. A
repeat chest x-ray shows that the chest tubes (shown in green) are in place;
however, the nasogastric tube (shown in blue), correctly positioned along the
greater curvature of the stomach, is located abnormally in the left hemithorax. This
presentation is consistent with diaphragmatic hernia.
- Blunt thoracoabdominal trauma causes diaphragmatic rupture by a sudden
increase in pressure that creates large radial tears or avulsion from side wall
attachments. The diaphragmatic defect allows migration of intra-abdominal
contents into the chest (ie, diaphragmatic hernia) with resulting symptoms due to
lung compression (eg, respiratory distress) or bowel obstruction (eg, nausea,
vomiting, abdominal pain). Most patients with diaphragmatic rupture require
immediate treatment, often with associated injuries. However, some patients with
smaller ruptures can have a delayed presentation (as seen in this case) when
defects enlarge over time.
- Diagnosis is made with imaging. Chest x-ray may show abdominal organs within
the chest (with or without mediastinal deviation) but is typically nondiagnostic
(elevation of the hemidiaphragm may be the only abnormal finding). CT scans of
the chest and abdomen are usually required to confirm the diagnosis. Patients
require surgical repair because diaphragmatic hernia can potentially result in
intestinal strangulation and death.
-
CARDIAC
MYXOMA

- MC primary cardiac neoplasm


- Fragments of a left atrial myxoma can embolize into the systemic circulation,
leading to stroke or other acute ischemia (eg, limb, mesenteric).
- Left atrial myxomas frequently obstruct blood flow from the left atrium to the left
ventricle, leading to a murmur mimicking that of mitral stenosis (ie, middiastolic
rumble at the apex). The obstruction can also lead to decreased cardiac output
that manifests as dyspnea, lightheadedness, or syncope. Because the mass is
typically mobile, obstructive symptoms may be transient and influenced by position
(ie, mitral obstruction is exacerbated by upright posture but is alleviated by lying
down); tumor movement occasionally causes a characteristic "tumor plop" sound at
the end of diastole on auscultation. In addition, some myxomas can produce
cytokines (eg, IL-6) that lead to systemic inflammation (evidenced by an elevated
erythrocyte sedimentation rate) and constitutional symptoms (eg, fever, weight
loss).
- The diagnosis is typically established by echocardiography. Prompt surgical
resection is recommended to minimize the risk of embolization and sudden cardiac
death (due to impaired cardiac output).
…………………………………….
- Although infective endocarditis can cause fever, weight loss, and systemic
embolization, echocardiography typically shows valvular vegetations rather than an
intracardiac mass in the atrium (as seen in this patient). In addition, infective
endocarditis typically causes a regurgitant rather than stenotic murmur
- Myxomatous valve degeneration → Midsystolic click rather than middiastolic
rumble
ACUTE LIMB Acute arterial occlusion can result
ISCHEMIA (ALI) from 1 of 3 major causes:
embolus from a cardiac or
arterial source, arterial
thrombosis, or iatrogenic or
direct blunt trauma to the artery.
It occurs mostly due to
thromboembolic occlusion from
LA thrombus due to A-fib in a
patient with preexisting
peripheral vascular disease
(multiple risk factors, intermittent
claudication, diminished pulses).

LE>UE. Potential cardiac sources of emboli include left atrial thrombus due to atrial
fibrillation, left ventricular thrombus following an anterior myocardial infarction,
infective endocarditis (ie, septic emboli), and thrombus from prosthetic valves.

Patients with suspected acute arterial occlusion leading to an immediately-threatened


limb (sensory loss, rest pain, muscle weakness) should be immediately started on
anticoagulation while further diagnostic procedures are performed. IV Heparin
prevents further thrombus propagation and thrombosis in the distal arterial and
venous circulation.
…….
- Arterial Doppler study or duplex ultrasonography can identify the presence and
location of acute arterial occlusion; however, the clinical suspicion for acute embolic
occlusion in this patient is so high that anticoagulation initiation should not be
delayed while obtaining additional confirmatory testing.
- Arterial thrombosis usually develops at the site of an atherosclerotic plaque in
patients with preexisting peripheral vascular disease. Due to the presence of
collateral circulation in these patients, limb ischemia manifestations are typically
less severe than they are in patients with an acute embolus. In addition, pulses are
often diminished in both extremities in patients with peripheral vascular disease.
- DVT: dull and aching in contrast to the severe, sudden-onset pain seen with acute
arterial occlusion.
A patient likely has peripheral artery disease (PAD) given his risk factors (eg, smoking,
hypertension, hyperlipidemia), symptoms (eg, rest pain that improved with dangling),
and clinical findings (eg, shiny, hairless legs). In patients with PAD, atherosclerosis of
peripheral arteries (eg, popliteal artery) may be complicated by plaque disruption.
This can lead to thrombosis and acute-on-chronic limb ischemia (eg, mottling,
coolness, prolonged capillary refill, paresis).
In contrast to patients without PAD who develop ALI (eg, due to arterial embolus
from a-fib) and typically have abrupt onset of the classic 6 Ps of ischemia, those with
existing PAD who develop acute-on-chronic limb ischemia (eg, due to thrombosis
from plaque disruption) often lack these classic features or develop them more
slowly (eg, over 1 day). This is because preexisting collateral circulation, which
formed in response to progressive accumulation of atherosclerosis in the primary
arterial circulation (eg, popliteal artery), can supply some perfusion to the affected
distal limb, as in a patient with prolonged but not absent capillary refill. Even with
collateral circulation, distal limb perfusion is still typically inadequate; therefore,
most patients with acute-on-chronic limb ischemia require emergency
intervention (eg, IV heparin infusion, thrombolysis, thrombectomy).

………..

Thromboangiitis obliterans, a tobacco-associated vasculitis, can lead to extremity


ischemia, pain, and discoloration. However, symptoms typically begin distally (eg,
digital ischemia) before progressing proximally. In addition, patients are typically
age <45 and often have normal distal pulses.

EXTREMITY The initial evaluation and management of patients with severe extremity injury includes
VASCULAR hemorrhage control, radiography of skeletal injuries, and evaluation of the
TRAUMA neurovascular bundle. Vascular assessment should focus on identifying hard signs of
injury, including:
1. Observed pulsatile bleeding
2. Presence of a bruit or thrill over the injury
3. Expanding hematoma
4. Signs of distal ischemia (eg, absent pulses, cool extremities)
- If the area of damage is unclear, arteriography can be performed intraoperatively to
clarify anatomy.
- If hard signs are absent, further evaluation for soft signs (eg, history of hemorrhage,
diminished pulses, bony injury, neurologic abnormality) should occur. If present,
these signs indicate the need for additional testing such as the injured extremity
index (similar to ankle-brachial index), which involves comparing the systolic
occlusion pressure distally in an injured extremity to the occlusion pressure at a
proximal site in an uninjured extremity. If the index is abnormal (<0.9), patients
should be considered for CT scan or conventional arteriography and surgery in
conjunction with management of other injuries (eg, bony damage)
- Compression dressing should
be considered during out-of-
hospital management, and
prophylactic antibiotics are
important in cases of open
fractures. However, neither is
definitive or appropriate for
hospitalized patients with clear
evidence of vascular injury and
hemodynamic instability.
- Doppler USG can be an
acceptable alternative to
angiography in selected
settings if performed by
experienced providers.
However, when hard signs of
vascular injury are present, it
would not be the next step.
ANESTHESIA Laparoscopy → CO2 is introduced into the abdomen via a needle or port, with
continuous monitoring of intraabdominal pressure. Pressure → stimulate peritoneal
stretch receptors → increased vagal tone. Close monitoring is required as patients may
develop severe bradycardia, atrioventricular block, and sometimes asystole. The
increased intraabdominal pressure can also cause a mechanical increase in systemic
vascular resistance leading to elevations in blood pressure.
- CO2 gas embolization is a rare complication of CO2 insufflation, usually resulting
from inadvertent insertion of the insufflation needle directly into an artery, vein, or
organ (eg, liver). Affected patients typically develop end-organ infarction (arterial
embolism) or hypotension and obstructive shock (venous embolism). Severe
bradycardia and atrioventricular block are not expected.
- Agents used for general anesthesia (eg, propofol) commonly have a cardiac
suppressive effect; therefore, inadequate anesthesia is expected to cause
tachycardia rather than bradycardia.

- Systemic absorption of intraabdominal CO2 can occur during laparoscopic


procedures, leading to an increase in blood PaCO2. This causes peripheral
vasodilation with a reflexive increase in heart rate.
MECHANICAL
VALVE

rosthetic valve dysfunction (PVD), which most commonly occurs in the following forms:

Paravalvular leak (regurgitation around the valve): more commonly occurs with
mechanical (rather than
bioprosthetic) valves and
results from dehiscence of the
valve from the aortic or mitral
annulus, often due to annular
degeneration or underlying
infective endocarditis.
Transvalvular regurgitation
(regurgitation through the
valve): more commonly affects
bioprosthetic (rather than
mechanical) valves and can
result from cusp degeneration
or occasionally valvular thrombus that impairs valve closure. Patients are often initially
asymptomatic but can develop severe heart failure; those with significant regurgitation
generally have a poor prognosis.
PVD can also involve valvular obstruction (stenosis), which typically results from
valvular thrombus or cusp malfunction (ie, failed opening) and presents with a
characteristic stenotic, rather than a regurgitant, murmur.

- The best initial evaluation for PVD is echocardiography, which allows visualization
of the valve and surrounding anatomy. Depending on the cause and extent of
dysfunction, further studies and possible surgical intervention may be indicated.
- PVD can cause mechanical damage to red blood cells (as they are passing through
the malfunctioning valve) and lead to macroangiopathic hemolytic anemia.
Complete blood count and peripheral blood smear (revealing schistocytes) are used
for diagnosis. However, significant anemia is unlikely in this asymptomatic patient;
in addition, mild hemolysis may be seen normally with a prosthetic valve, also
making a peripheral smear less helpful.
TRICUSPID - Severe tricuspid regurgitation (TR) due to an adverse effect of permanent
REGURGITATION pacemaker: The RV lead of a transvenous implantable pacemaker or cardioverter-
(TR) defibrillator passes through the SVC into the RA and then through the TV to
terminate in the endocardium of the RV. Damage to the tricuspid valve leaflets or
inadequate leaflet coaptation can occur, leading to severe TR in 10%-20% of
patients.

- Chronic, severe TR typically presents


with right-sided HF. Patients can
have distended jugular veins,
pulsatile and tender hepatomegaly,
abdominal distension with ascites,
and lower extremity edema.
Cardiac examination typically reveals
RV heave and a holosystolic
murmur best heard at LSB; the
murmur intensifies with maneuvers
that increase RV preload (eg, deep
inspiration, leg raise). The diagnosis
of TR is confirmed by echo.
- Mitral chordal rupture is not
associated with pacemaker placement
as the leads do not cross the mitral
valve (even in a biventricular
pacemaker). Spontaneous rupture
can occur due to myxomatous degeneration in patients with mitral valve prolapse;
the typical presentation is with prominent pulmonary edema and a holosystolic
murmur heard at the cardiac apex.
- Pulmonic valve stenosis MCly occurs as a congenital defect and is often
asymptomatic; however, some patients develop right-sided heart failure. Physical
examination typically reveals a systolic ejection murmur best heard at the left upper
sternal border; as with TR, the murmur intensifies with increases in right ventricular
preload.
- VSD → palpable thrill and a harsh holosystolic murmur with maximal intensity at
the left midsternal border.
AORTIC
STENOSIS (AS)
///////////////////
****

- Severe aortic stenosis (AS) is diagnosed based on aortic jet velocity or mean
transvalvular pressure gradient; in most cases the valve area is ≤1 cm2, but valve
area is not considered for diagnosis. Physical examination findings that suggest
severe AS include a late-peaking, crescendo-decrescendo systolic murmur best
heard at the right upper sternal border and diminished and delayed pulses (pulsus
parvus et tardus). Patients with severe AS often have symptoms of angina,
syncope/presyncope, or heart failure (eg, dyspnea).
- Surgical aortic valve replacement (AVR) should be considered in patients with
severe AS and ≥1 of the following criteria:
1. Presence of symptoms attributable to AS: Patients with symptomatic,
severe AS have a relatively high risk of sudden cardiac death.
2. LVEF <50%, regardless of symptoms: A depressed LVEF is often due to
excessive afterload created by the stenotic valve, and it frequently normalizes
with AVR.
3. Undergoing other cardiac surgery (eg, CABG): The valve can be repaired
concomitantly.
- The definition of severe AS encompasses many patients who are asymptomatic
because it was designed to identify nearly all patients who may benefit from AVR
(high SN). Some of these patients are truly asymptomatic. Others lack symptoms
only because of a sedentary lifestyle; when subjected to exertion (eg, stress testing)
they have typical severe AS symptoms.
- Serial echocardiography is appropriate in patients with severe AS who are truly
asymptomatic and have normal LVEF
- Severe AS is a progressive disease that requires either prompt AVR or close
monitoring
- Severe AS creates a tenuous hemodynamic situation because adequate preload is
required to maintain cardiac output and adequate diastolic pressure is required to
maintain coronary artery perfusion. This limits the use of medical therapy because
diuretics (eg, furosemide) reduce preload and vasodilators (eg, amlodipine,
hydralazine, ACE inhibitors) reduce diastolic pressure, often precipitating clinical
decompensation. Cardiac exercise programs are also of limited benefit. AVR is the
only therapy proven to improve survival in patients with severe AS.
BLUNT CARDIAC
INJURY (BCI)

- BCI can induce myocardial edema, hemorrhage, and necrosis capable of causing a
spectrum of structural, ischemic, and electrical complications. The electrical
disturbances are frequently subtle and asymptomatic, but they require further
monitoring and evaluation because they may be a sign of more severe cardiac
sequelae.
- Patients with findings concerning for BCI require 24-48 hours of continuous cardiac
monitoring because most life-threatening arrythmias occur during that time. In
patients with significant findings (eg, arrythmias, hypotension, heart failure),
echocardiography is performed to evaluate for myocardial dysfunction and/or
structural (eg, valvular) injuries. TEE is superior to TTE but is also more invasive;
therefore, in this nonintubated, hemodynamically stable patient, transthoracic
echocardiography is the best next step.
- Blunt cardiac injury (BCI) due to blunt chest trauma can lead to cardiogenic shock
(with acute right heart dysfunction causing elevated CVP and left heart dysfunction
causing refractory hypotension [also caused by tamponade]) or to obstructive
shock (eg, cardiac tamponade). Other manifestations of BCI can include
arrhythmia, acute coronary syndrome, and valve or wall rupture.
- Coronary angiography is appropriate for suspected acute coronary syndrome,
which can be caused by traumatic coronary dissection or thrombosis.
- CTPA can diagnose pulmonary embolism, a possible complication of trauma due to
the inflammatory prothrombotic state.
- Esophagography with water-soluble contrast is the best initial study for suspected
esophageal rupture. Although spillage of gastrointestinal contents from
esophageal rupture can cause a systemic inflammatory response that includes
tachycardia, the systemic inflammatory response typically causes fever, which is not
seen in this patient.
- Open fractures have increased infection risk compared with closed fractures, and
the standard of care involves immediate (<3 hr postinjury) intravenous prophylactic
antibiotics and urgent (<6 hr) surgical irrigation and debridement. Although open
fractures can cause bacteremia, local infection (eg, soft tissue infection,
osteomyelitis) is much more common, and peripheral blood cultures are not
typically drawn, especially in the absence of fever.
- Thyroid function tests can diagnose thyroid storm, which may be incited by trauma.
Clinical manifestations of thyroid storm include tachycardia; however, hyperpyrexia
and hypertension are also typically present.
PREIPHERAL
ARTERY/VASCUL
AR DISEASE (PAD
/PVD)

- symptoms consistent with intermittent claudication. The next step in management


should be to obtain physiologic testing with ABI to confirm the presence of PAD as
a cause of his symptoms. ABI is an inexpensive and noninvasive vascular test used
to confirm the diagnosis of PAD in such patients. It is defined as the ratio of resting
systolic blood pressure at the ankle to the systolic brachial blood pressure. An ABI
of <0.90 is considered diagnostic of occlusive PAD with a 90% sensitivity and
95% specificity in symptomatic patients.
- Arterial ultrasound of the lower extremities is another noninvasive modality for
diagnosing PAD. Arterial duplex ultrasound can be used to localize the site and
severity of vascular obstruction. However, it is less SN and SP than ABI for the
initial diagnosis of PAD. It is generally performed in symptomatic patients with
abnormal ABI who are being considered for interventional procedures.
LERICHE - Characterized by the triad of:
SYNDROME 1. Bilateral hip, thigh, and buttock claudication
2. Absent or diminished femoral pulses: from the groin distally, often with
symmetric atrophy of the bilateral lower extremities due to chronic ischemia
3. Impotence: almost always present in men with this condition; in the absence of
impotence, an alternate diagnosis should be sought
- Men with a predisposition for atherosclerosis, such as smokers, are at the greatest
risk of this condition. Because impotence is not uncommon in this age group and
hip and thigh pain with walking may also be attributed to osteoarthritis, the
diagnosis of aortoiliac occlusion can be missed if a thorough vascular examination
is not performed.
- Snoring may be primary (ie, not associated with underlying pathology) or may be a
symptom of obstructive sleep apnea.
- Temporal arteritis is associated with new-onset headache, temporal tenderness and
pulselessness, older age, and an increased ESR. Jaw claudication and amaurosis
fugax may also occur.
- Anorexia may be a sign of an underlying malignancy, mesenteric ischemia, or bowel
disorder.
- Ankle swelling may result from venous insufficiency, renal insufficiency, right heart
failure, or hepatic disease
/////////////////////////////PULMONARY & CRITICAL CARE/////////////////////////////
RIB FRACTURE Flail chest occurs when fracture of ≥3 contiguous ribs in ≥2 locations creates an
isolated chest wall segment (flail segment) that moves paradoxically(opposite) to the
rest of the rib cage during respiration. Typically, the extreme trauma to the chest wall
that creates the flail segment also contuses the underlying lung (eg, causing
diminished anterior breath sounds beneath this patient's anterior flail segment).

Flail chest negatively impacts respiration and oxygenation in multiple ways, including
the following:

• Impaired generation of negative intrathoracic pressure during inspiration


and increased dead space during expiration cause ineffective ventilation.
• Pulmonary contusion (with alveolar hemorrhage and edema) in the underlying
lung impedes oxygen diffusion.
• Fracture-related pain causes respiratory splinting and atelectasis.

Flail chest often results in respiratory failure requiring mechanical positive pressure
ventilation, which, due to positive pressure, can force the flail segment to move
outward with the rest of the rib cage during inspiration.
………
Fat embolism, which is most commonly associated with long bone fracture (especially
femur fracture), can occlude pulmonary capillaries, leading to hypoxia. However, this is
typically associated with neurologic deficits and does not manifest until 24-72 hours
after the initial insult.

Myocardial contusion from blunt thoracic trauma can lead to myocardial dysfunction
and cardiogenic shock. The resulting left heart failure can lead to pulmonary edema
and hypoxia; however, it typically also causes hypotension from forward pump failure.

Even in the absence of concomitant injuries, rib fractures can cause sufficient pain to
result in shallow breathing (ie, small tidal volumes) and atelectasis. Without adequate
pain control, splinting and atelectasis increase the risk of pneumonia, a frequent
complication of rib fracture.
After significant associated injuries have been ruled out or treated, the mainstay of rib
fracture management is pain control to ensure appropriate ventilation and adequate
cough. Oral NSAIDs (eg, ibuprofen, ketorolac) and opioids are commonly used.
Opioids can cause central respiratory depression, but this disadvantage is typically
outweighed by the benefits of adequate pain control. For patients who require
hospitalization (eg, for more extensive rib fractures), IV analgesics and regional
anesthetic techniques (eg, epidural infusion, intercostal nerve blocks) may be added.
In addition to pain control, patients should be taught the importance of pulmonary
toilet and the use of incentive spirometry.

………..

Continuous positive airway pressure provides gentle pressure to prevent repetitive


upper airway collapse/obstruction (eg, OSA); it would not provide enough pressure to
reexpand large segments of collapsed lung.

Percussive chest physiotherapy can loosen thick secretions in the lungs to facilitate
removal by coughing. Although this patient would benefit from pain control to ensure
adequate cough, there is no evidence of thick secretions that require percussive
physiotherapy; in addition, this technique likely would be extremely painful in the
setting of rib fracture.

Rib fractures typically heal without surgery, especially nondisplaced fractures. Surgical
fixation is typically indicated only for severe cases (eg, flail chest with failure to wean
from the ventilator, significant chest wall deformity).

PULMONARY
Pulmonary contusion
CONTUSION
Clinical • Present <24 hours after blunt thoracic trauma
features • Tachypnea, tachycardia, hypoxia

• Rales or decreased breath sounds


Diagnosis • CT scan (most sensitive) or CXR with patchy, alveolar infiltrate not restricted by
anatomical borders

• Pain control
Managem
• Pulmonary hygiene (eg, incentive spirometry, chest PT)
ent
• Supplemental oxygen & ventilatory support

CXR = chest x-ray; PT = physiotherapy.

Despite being restrained during the high-speed motor vehicle collision, patients may
sustain blunt thoracic trauma with chest wall (and facial) bruising, likely from airbag
impact. Blunt force to the chest wall can injure the underlying lung, resulting
in alveolar hemorrhage and edema. Subsequent resuscitative fluid administration can
exacerbate the alveolar edema, leading to progressive dyspnea, tachypnea,
and hypoxemia. On CT scan, the alveolar edema can appear as ground-glass opacities
in the lung adjacent to the affected chest wall (eg, anterior, peripheral lung); this
pattern and distribution is classic for pulmonary contusion.
Pulmonary contusion is common (25%-35% of cases) following blunt thoracic trauma.
Because clinically significant alveolar edema may take up to 24 hours to accumulate,
pulmonary contusion may not be apparent immediately following an injury, and initial
chest x-ray is often negative. CT scan of the chest is more sensitive and can identify the
classic irregular, nonlobular (ie, not restricted by anatomic landmarks) infiltrates.
Management involves pain control, pulmonary hygiene, and respiratory support.

………

Although bronchial rupture can cause progressive shortness of breath and hypoxemia,
evidence of air leakage from the tracheobronchial tree (eg, pneumothorax,
pneumomediastinum) is typically present on CT scan.

Tachypnea and hypoxemia in the setting of femur fracture are concerning for fat
embolism. However, patients with fat embolism classically have accompanying
neurological abnormalities and petechial rash, as well as a latency period of 12-
72 hours after the initial injury.

ACUTE Return to consciousness after anesthesia (emergence) typically occurs within 15


RESPIRATORY minutes of extubation; at a minimum, patients should be responsive with intact
FAILURE protective (eg, gag) reflexes within 30-60 minutes of the last administration of an
anesthetic or adjuvant agent (eg, opiate, muscle relaxant). Delayed emergence occurs
when a patient fails to regain consciousness within the expected window. The etiology
is typically multifactorial but generally occurs due to 1 of 3 major causes:

• Drug effect: Preoperative drug ingestion (eg, opiates, BZs, illicit drugs,
anticholinergic drugs, antihistamines) may potentiate anesthetic effects.
Prolonged anesthesia duration or higher medication doses may also delay
emergence.

• Metabolic disorder: Common etiologies include hyper- or hypoglycemia,


hyper- or hypothermia, hyponatremia, and liver disease.

• Neurologic disorder: Intraoperative stroke, seizure (or postictal state), or


elevation of intracranial pressure can cause prolonged alterations in mental
status.

A patient's arterial blood gas demonstrates acute hypercarbic and hypoxic respiratory
failure (low pH, elevated pCO2, low pO2) due to hypoventilation. In association with
the worsening bradypnea and drop in pulse (which were likely initially elevated due to
agitation postextubation), this suggests that the patient's delayed emergence is due to
a prolonged medication effect, possibly potentiated by preoperative drug or alcohol
ingestion in this patient with a history of polysubstance abuse. Management of acute
respiratory failure includes ventilatory support (eg, bag and mask, reintubation);
reversal agents (eg, naloxone) may also be indicated.
……….
Acute cardiogenic pulmonary edema can occur in cardiogenic shock, which may be
precipitated by surgery. However, patients with cardiogenic shock are typically elderly,
have multiple cardiac risk factors, and exhibit elevated jugular venous pressure,
hypotension, tachycardia, and tachypnea.

Malignant hyperthermia is a hypermetabolic state that occurs in response to volatile


anesthetics (eg, isoflurane, halothane) or succinylcholine. Due to increased
CO2 production, patients have a respiratory and metabolic acidosis; however, malignant
hyperthermia causes hyperventilation rather than hypoventilation. In addition, patients
typically have muscle rigidity, hyperthermia, and tachycardia.
Postextubation laryngeal edema could cause hypercarbic and hypoxic respiratory
failure; however, patients would be expected to be stridorous, with tachypnea (rather
than bradypnea) and tachycardia.

A patient with a severe COPD exacerbation (eg, wheezing, dyspnea, cough) requiring
mechanical ventilation developed abrupt-onset hypoxemia associated with elevated
peak and plateau pressures (measured on the ventilator) and decreased right-sided
breath sounds. This presentation suggests a pneumothorax, or the presence of air
within the pleural space.
Pneumothoraxes may occur
spontaneously or secondary to
underlying lung disease, trauma, or
iatrogenic injury. Positive pressure
ventilation (eg, mechanical
ventilation) poses the risk
of pulmonary barotrauma, leading
to alveolar rupture and
pneumothorax formation. Patients
with COPD are at higher risk due to
preexisting pulmonary
hyperinflation and the presence of
bullae or blebs, which can rupture.

Patients with a large pneumothorax


often demonstrate abrupt-
onset tachycardia, tachypnea, hypoxemia, and decreased or absent breath
sounds on the affected side. The collapsed lung, receiving a relatively stable volume of
air from the ventilator, results in increased peak pressure (a measure of increased
resistance to air flow from the ventilator) and increased plateau pressure (a measure
of the lung's decreased ability to expand [compliance]). Management of
pneumothorax involves placement of a chest tube, which allows reexpansion of the
lung.

………
ARDS typically takes multiple hours to days to manifest and is not associated with
asymmetric breath sounds.

A large pulmonary embolism can cause acute hypoxemia but would not cause
elevated ventilatory pressures or decreased unilateral breath sounds.
Right mainstem bronchus intubation would cause hypoxemia and elevated peak and
plateau pressures due to the limited volume of the ventilated right lung but would
cause decreased breath sounds in the left, unventilated lung (rather than the right
lung, as in this patient).

High PaCO2 and low PaO2 levels are suggestive of alveolar hypoventilation, although an
elevated PaCO2 alone, in the range of 50-80 mm Hg, is sufficient to make the
diagnosis. Causes of alveolar hypoventilation and respiratory acidosis include the
following:

• Pulmonary/thoracic diseases: Chronic obstructive pulmonary disease,


obstructive sleep apnea, obesity hypoventilation, scoliosis
• Neuromuscular diseases: MG, LEMS, GBS
• Drug-induced hypoventilation: Anesthetics, narcotics, sedatives
• Primary central nervous system dysfunction: Brainstem lesion, infection, stroke

In addition, the A-a gradient (PAO2 – PaO2) can help determine the specific cause of
hypoxemia. The A-a gradient is a measure of oxygen transfer from the alveoli to the
blood. The alveolar oxygen tension can be calculated using the following equation:

PAO2 = (FiO2 x [Patm – PH2O]) – (PaCO2/R)

Then, calculate the A-a gradient:


A-a gradient = PAO2 – PaO2

A normal A-a gradient is <15. Values increase with age, but an A-a gradient >30 is
considered elevated regardless of age.

The A-a gradient is normal in patients with reduced inspired oxygen tension and
hypoventilation.

Recent subdiaphragmatic surgery, narcotic pain medications, and obesity


make the patient at risk for alveolar hypoventilation. The resultant respiratory acidosis
confirms the hypoventilation (CO2 retention).

………….

Pulmonary embolism, atelectasis, pleural effusion, and pulmonary edema cause V/Q
mismatch. In V/Q mismatch, the A-a gradient is elevated. In all these conditions, there
is a decrease in PaCO2 (respiratory alkalosis) due to compensatory tachypnea.
DIAPHRAGMATIC This patient has a diaphragmatic
HERNIA rupture with herniation of
abdominal contents into the
thoracic cavity. Diaphragmatic
rupture can occur after blunt
thoracoabdominal trauma (eg,
motor vehicle collision) due to a
sudden and unequal increase in
thoracoabdominal pressure,
resulting in tears or avulsion. The
left diaphragm is more prone to
injury than the right due to
congenital weakness in the
diaphragm's left posterolateral
region and the liver's protective
effects on the right side. Some
patients (especially children) with
traumatic diaphragmatic injury
may initially have no symptoms and can present months to years later after
progressive expansion of the diaphragmatic defect.

Due to the mass effect of the abdominal organs in the thorax, symptoms may be
related to lung compression (eg, chest discomfort, dyspnea), and CXR may
show bowel loops within the thoracic cavity and mediastinal shift. Delayed
diagnosis is also associated with increased risk of hernia formation and bowel
strangulation, which can be fatal. Although this patient's x-ray findings suggest
diaphragmatic injury, CT scan of the chest and abdomen is performed because it is
more sensitive and is the definitive diagnostic modality. Surgical repair is indicated
after the diagnosis is confirmed, and CT scan aids in planning of operative approach
(eg, thoracotomy vs laparotomy).

AIRWAY Ludwig angina is a rapidly progressive cellulitis of


EMERGENCY the submandibular and sublingual spaces. Most cases arise
from contiguous (rather than lymphatic) spread of
polymicrobial dental infections in the mandibular molars. This
contiguous spread results in bilateral edema of the
submandibular and sublingual spaces. As the submandibular
area becomes tender and indurated, the floor of the mouth
becomes elevated and displaces the tongue posteriorly, which
may lead to acute airway obstruction. Additional clinical
features of Ludwig angina include a neck that is often described
as "woody" or "brawny" but has no associated LAD.
As with other lesions that threaten the airway, patients with examination findings
concerning for impending airway obstruction (eg, drooling, inability to lay
flat, tripod positioning) should have their airway secured, REGARDLESS of current
oxygen saturation. This is because oxygen saturation is often maintained until very
rapid decompensation (within minutes). Therefore, these patients require continued
assessment and, if needed, intervention to secure the airway (eg, nasotracheal
intubation, awake tracheostomy).
………..

 Infection within the


retropharyngeal space can
present with dysphagia,
hoarseness, and SOB; however,
this is due to edema in the
posterior pharyngeal wall rather
than displacement of the tongue.
It is most likely to extend
through the alar fascia into
the "danger space", which can
then progress into the posterior
mediastinum and result in acute
necrotizing mediastinitis.
Carotid artery rupture occurs
when the vasa vasorum
supplying the carotid is injured; it
is most often a complication of head and neck cancer (eg, tumor invasion, tissue
removal) and is not typically associated with deep neck space infection.

Infections originating in the maxillary space, not the mandibular space, can spread to
the infratemporal space and orbit and progress to cavernous sinus thrombosis.
AOM can spread throughout the temporal bone and cause facial nerve palsy due to
inflammation of the nerve as it travels through the tympanic and mastoid portions of
the temporal bone. Submandibular space infections do not typically cause facial nerve
palsy.

Deep neck space infections are prevented from spread into the epidural space by the
vertebral column, posterior longitudinal ligament (PLL), and dura.
PNEUMOTHORAX This patient with chronic obstructive pulmonary disease (COPD) and acute-onset
shortness of breath, hypoxia, and unilaterally decreased breath sounds likely has
a secondary spontaneous pneumothorax
(SSP). SSP occurs in patients with known
lung disease (eg, COPD, cystic fibrosis).
Other common manifestations include chest
pain and hyper-resonance on
percussion. Cigarette smoking markedly
increases the risk of pneumothorax. Chronic
destruction of alveolar sacs leads to the
formation of large alveolar blebs (red
arrow), which can eventually rupture and
leak air into the pleural space.

Diagnosis of pneumothorax is made by chest x-ray, which demonstrates a visceral


pleural line beyond which no pulmonary markings are apparent. Tracheal deviation
can occur but is rare (more commonly seen in patients with primary spontaneous
pneumothorax). Management depends on the size of the lesion and the clinical status
of the patient, and ranges from observation with supplemental oxygen to emergency
tube thoracostomy.

………

Although rapid bronchial occlusion resulting in extensive atelectasis can result in


similar symptoms (eg, chest pain, dyspnea, hypoxia), this patient does not have risk
factors for obstructive atelectasis such as a history of foreign body aspiration,
malignancy, or severe pneumonia (mucus plug).
Patients with COPD are at increased risk for pneumonia due to chronic inflammation
and chronic use of corticosteroids. However, patients with pneumonia typically present
with fever and productive cough in addition to chest pain and breathlessness.

Pneumothorax

Spontaneous pneumothorax Tension pneumothorax

• Primary: no preceding event or lung


• Life-threatening
Associated disease; often thin, young men
• Often due to trauma or
features • Secondary: underlying lung disease
mechanical ventilation
(eg, COPD, CF)

Same as spontaneous plus:


• Chest pain, dyspnea
Signs & • Hemodynamic instability
• ↓Breath sounds, ↓chest movement
symptoms • Tracheal deviation away from
• Hyperresonant to percussion
affected side

Same as spontaneous plus:


Imaging • Visceral pleural line
• Contralateral mediastinal shift
• Absent lung markings beyond pleural • Ipsilateral hemidiaphragm
edge flattening

• Small (≤2 cm): observation & oxygen


Manageme administration • Urgent needle decompression
nt • Large & stable: needle aspiration or or chest tube placement
chest tube

CF = cystic fibrosis; COPD = chronic obstructive pulmonary disease.

Tension pneumothorax (TP) is a complication of subclavian central venous catheter


placement. TP is a life-threatening condition caused by air within the pleural space
that displaces mediastinal structures and compromises cardiopulmonary function. It
develops when injured tissue forms a one-way valve allowing air to enter the pleural
space but preventing it from escaping naturally. TP is characterized by rapid-onset
severe shortness of breath, tachycardia, tachypnea, hypotension, and distension of the
neck veins due to SVC compression. Positive-pressure ventilation can worsen TP by
increasing intrathoracic pressures and intensifying the one-way valve effect. As the
pleural cavity fills with air, increased pressure is required to initiate inspiratory flow.

TP is a clinical diagnosis, and


decompression should be initiated
immediately with needle
thoracostomy (or, if available, direct
emergency tube thoracostomy)
in hemodynamically
unstable patients. Classically, the
second intercostal space in the
midclavicular line has been used, but
this fails in up to 50% of patients due to
chest wall anatomy. Consequently, the
fifth intercostal space in the
midaxillary line is an excellent alternate site. Needle thoracostomy should be
followed by an emergency tube thoracostomy.

………..

In TP, an x-ray will show deviation of the trachea and mediastinum away from the
affected side and increased lucency on the affected side of the chest. If there is
diagnostic uncertainty or the patient is hemodynamically stable, radiographic
confirmation should be considered. Otherwise, treatment of this emergent condition
should not be delayed by waiting for radiographic studies.

OBSTRUCTIVE Hemodynamic measurements in shock


SHOCK
Hypovolemic Cardiogenic Obstructive Septic
Parameter shock shock shock shock
CVP (right-
↓ ↑ ↑ ↓
sided preload)

PCWP (left-
↓ ↑ ↓* ↓
sided preload)

Cardiac index
↓ ↓ ↓ ↑
(LV output)

SVR (afterload) ↑ ↑ ↑ ↓

SvO2 ↓ ↓ ↓ ↑
CVP = central venous pressure; LV = left ventricular; PCWP = pulmonary capillary wedge
pressure; SvO2 = mixed venous oxygen saturation; SVR = systemic vascular resistance.
*In tamponade, left-sided preload is decreased, but measured PCWP is paradoxically increased due to
external compression by pericardial fluid.

The expected hemodynamic parameters in obstructive shock include elevated central


venous pressure (↑ CVP) and low cardiac output (↓ CO); however, pulmonary capillary
wedge pressure (PCWP), an estimate of left atrial pressure and representation of left-
sided preload, can vary depending on the location of the obstruction.

• Prepulmonary obstructive shock (due to massive PE or tension


pneumothorax) is MC and results from obstruction involving the RA, RV, or PA.
Because blood is unable to pass from the right side of the heart to the left
side, PCWP is low or normal. (PCWP ↓ or N)
• Postpulmonary obstructive shock typically involves obstruction within the left
side of the heart or aorta; PCWP is elevated, making the hemodynamic
parameters the same as those expected in cardiogenic shock. (PCWP ↑)

Of note, cardiac tamponade causes prepulmonary obstructive shock as blood is


unable to fill the compressed right-sided heart chambers. Despite reduced left-sided
preload, the expected PCWP is paradoxically elevated due to external compression
of the LA, a rare exception when left-sided preload and PCWP are discordant.
…………
Aortic dissection is a potential cause of postpulmonary obstructive shock because
the dissecting aortic wall can create a false aortic lumen that obstructs blood flow in
the true aortic lumen. Severe aortic stenosis can also cause postpulmonary
obstructive shock. Elevated PCWP is expected with these postpulmonary
obstructions. Aortic dissection may also cause hemorrhagic (hypovolemic) shock;
however, low CVP is expected.
Low CVP and low PCWP are expected in septic shock.

Left ventricular failure (LVF) can result from acute MI and leads to cardiogenic
shock characterized by elevated CVP and PCWP. RVMI and failure will demonstrate
elevated CVP and low or normal PCWP, the same findings expected in
prepulmonary obstructive shock.
PULMONARY Postoperative massive pulmonary embolism (PE) complicated by cardiogenic shock.
EMBOLISM (PE) Massive PE is defined as PE complicated by hypotension and/or acute right heart
strain. Although dyspnea and pleuritic chest pain are common symptoms of segmental
PE, syncope tends to occur only in massive PE. JVD on physical examination and
RBBB on ECG are signs of acute right heart strain. The right heart strain progresses
rapidly to right ventricular dysfunction, decreased return to the left side of the heart,
decreased cardiac output, left heart pump
failure, and resultant bradycardia. This results
in cardiogenic shock and causes central
nervous system effects, such as the dilated
pupils and unresponsive mental status.
Survival following massive PE is poor, with
death often occurring within an hour of the
onset of symptoms. If time permits, massive PE
can be confirmed with CT pulmonary
angiography. Although echocardiography has
poor sensitivity for segmental PE, massive PE often has visible echocardiographic
abnormalities that allow for rapid bedside diagnosis. In conjunction with respiratory
and hemodynamic support, fibrinolysis is indicated in the treatment of massive PE.
However, hemorrhagic complications are frequent, and surgery within the preceding
10 days is a relative contraindication to fibrinolytic therapy.

…………

Confusion and slurred speech always raise concern for an acute ischemic
stroke. However, new-onset jugular venous distension and RBBB are indicative of right
heart strain, which is commonly seen in association with PE. Moreover, blood pressure
is typically elevated with ischemic stroke.
Classic cardiovascular changes in the setting of sepsis include hypotension, decreased
peripheral vascular resistance, increased cardiac output, and decreased pulmonary
capillary wedge pressure. Fever and warm extremities might also be seen.

A dramatic postoperative
presentation with sudden-
onset dyspnea,
nonproductive cough,
tachycardia, and mild
hypoxia is highly
suggestive of acute PE.
Early, effective
anticoagulation decreases
the mortality risk of acute
PE and should be
considered in patients
without absolute
contraindications (eg,
hemorrhagic stroke,
massive gastrointestinal
bleed).

PUD is not an absolute


contraindication to
anticoagulation, although
it increases risk of bleeding
with anticoagulation. This
patient's modified Wells
score is 6 (absence of a
more likely diagnosis = 3 points, tachycardia = 1.5 points, immobilization due to recent
knee surgery = 1.5 points), which correlates to PE being likely (modified Wells score
>4). Given that she is clinically stable (normotensive, mild hypoxemia) with no evidence
of distress, the diagnosis of PE can be confirmed with CT angiography (CTA). If CTA
confirms PE, clinical judgment can dictate whether anticoagulation is initiated or other
options are pursued (eg, inferior vena cava filter placement) based on the estimated
risk of bleeding from the peptic ulcer.

Modified Wells criteria for pretest probability of PE

• Clinical signs of DVT


+3 points
• Alternate diagnosis less likely than PE

• Previous PE or DVT
+1.5 points • Heart rate >100
• Recent surgery or immobilization
• Hemoptysis
+1 point
• Cancer

≤4 = PE unlikely
Total score
>4 = PE likely

DVT = deep venous thrombosis; PE = pulmonary embolism.


………..

Compression ultrasonography is the preferred initial test for diagnosis of deep venous
thrombosis (DVT). This patient has no clinical manifestations of DVT. In addition, a
negative ultrasound result does not rule out PE.

Bedside transthoracic echocardiogram (TTE) may show acute PE findings of increased


right ventricular size and right ventricular dysfunction. However, these findings are
often only seen in massive PE accompanied by syncope, shock, and hemodynamic
instability. Therefore, TTE is generally reserved for clinically decompensating patients
in whom CTA cannot be obtained.

HYPOVOLEMIA A patient with evidence of hypovolemia (eg, flat neck veins), abdominal trauma (eg,
bruising, distension), and poor organ perfusion (eg, altered mental status) is likely
in hypovolemic shock due to internal hemorrhage. Severe hypovolemia lowers central
venous pressure, leading to decreased venous return to the right atrium and
therefore decreased CO. Positive pressure mechanical ventilation causes an acute
increase in intrathoracic pressure, which, in a severely hypovolemic patient with low
central venous pressure, can collapse venous capacitance vessels (eg, inferior vena
cava) and cut off venous return. This sudden loss of RV preload can cause acute
circulatory failure and cardiac arrest. In addition, sedatives used prior to intubation
cause relaxation of venous capacitance vessels and can also contribute to decreased
venous return.

………..
Decreased systemic vascular resistance occurs in distributive shock (eg, sepsis,
anaphylaxis, CNS injury) due to peripheral vasodilation. Recent trauma with evidence of
abdominal injury and absence of neurologic signs (eg, focal weakness) make
hypovolemic shock much more likely.

Vasovagal syncope results from a sudden increase in parasympathetic activity, leading


to bradycardia, peripheral vasodilation, and a temporary decrease in CO. Cardiac arrest
does not occur. Patients with hypovolemic shock have a marked increase in
sympathetic activity, leading to tachycardia and peripheral vasoconstriction.

Acute PE can cause cardiac arrest due to a rapid increase in pulmonary vascular
resistance, leading to acute right-sided heart failure and loss of left ventricular preload.
JVD would be expected. In addition, it would be unusual for an acute pulmonary
embolism to occur at exactly the time of mechanical ventilation.

Reduced myocardial contraction occurs in myocardial infarction and can lead to


cardiogenic shock. When severe, loss of CO due to cardiogenic shock can cause
cardiac arrest. However, jugular venous distension would be expected.

PULMONARY
Pulmonary contusion
CONTUSION
Clinical • Present <24 hours after blunt thoracic trauma
features • Tachypnea, tachycardia, hypoxia

• Rales or decreased breath sounds


Diagnosis • CT scan (most sensitive) or CXR with patchy, alveolar infiltrate not restricted by
anatomical borders

• Pain control
Managem
• Pulmonary hygiene (eg, incentive spirometry, chest PT)
ent
• Supplemental oxygen & ventilatory support

CXR = chest x-ray; PT = physiotherapy.


A patient with blunt thoracic trauma (eg, right chest wall bruising) has progressively
worsening tachypnea, hypoxemia, and ipsilateral irregular infiltrates on CXR,
concerning for pulmonary contusion. Pulmonary contusion represents lung
parenchymal bruising (due to transmitted kinetic energy) with resulting alveolar
hemorrhage and edema. It is frequently associated with rib fractures but may occur in
their absence, especially in children (whose more elastic chest walls can transmit more
kinetic energy without
fracturing).

Manifestations of pulmonary
contusion include tachypnea,
hypoxemia, and (often)
decreased breath sounds
(due to alveolar infiltrates).
Because clinically significant
alveolar edema may take hours
to accumulate, the initial chest
x-ray is often negative. CT scan
(more sensitive) or repeat chest
x-ray (as in this patient) may be
necessary to reveal
the patchy, irregular alveolar infiltrates characteristic of pulmonary
contusion. Because these infiltrates reflect alveolar damage in the lung tissue adjacent
to the injury site (eg, right peripheral lung in this patient), they are typically nonlobular
(not restricted by anatomic landmarks). Management is supportive and involves pain
control, pulmonary hygiene, and respiratory support.

……..

ARDS can cause increasing dyspnea, hypoxemia, and alveolar infiltrates following
trauma; however, the alveolar infiltrates are typically bilateral and do not manifest until
24-48 hours following the traumatic event.

Chest wall pain following trauma can lead to shallow breathing and atelectasis (lung
tissue collapse), with resulting tachypnea and hypoxemia. However, atelectasis typically
manifests on x-ray with bilateral, linear densities in the dependent, collapsed portions
of the lungs.

Fat embolism can


cause hypoxemia but
typically occurs after
long bone or pelvic
fracture and is
accompanied by
petechial rash and
neurologic changes.

 Pneumothorax

THERMAL BURN The initial management of burn injuries is identical to the management of all trauma
patients: airway, breathing and circulation must always be secured first. Burn patients
are at high risk for respiratory compromise because the supraglottic airway, which
efficiently exchanges heat with inhaled air, is very susceptible to direct thermal injury
and acute obstruction by edema and blistering. (In contrast, the subglottic airway is
protected from injury by reflexive closure of the vocal cords upon exposure to
extremely hot air.)

Clinical indicators of thermal and smoke inhalation injury include:

• Burns on the face


• Singeing of the eyebrows
• Oropharyngeal inflammation
• Blistering or carbon deposits
• Carbonaceous sputum
• Stridor
• CO-Hb level >10%
• History of confinement in a burning building.
All burn victims should be treated initially with high-flow oxygen via a non-
rebreather mask, although caregivers should maintain a low threshold for
intubation in any patient with physical evidence of thermal damage to the upper
airway. A key reason for early intubation is that progressive airway edema may
preclude intubation later in the patient's clinical course, potentially necessitating an
emergent surgical airway.

………..

High-dose corticosteroids are not used in the acute treatment of burn wounds. The
diabetogenic and immunosuppressive effects of systemic steroids make them
contraindicated in severely burned patients, who are already at high risk for metabolic
derangement and immunosuppression as a result of their injuries.

Prophylactic antibiotics are not indicated in the acute management of burn patients.
While burn patients are at high risk for infection, especially
by Pseudomonas aeruginosa, treatment in the acute setting should focus on
restoration of airway, breathing and circulation.
N-acetylcysteine (NAC) is the antidote for acetaminophen-induced hepatotoxicity. It is
also used as a mucolytic agent in cystic fibrosis. There are case reports to suggest that
this agent may also be of value in treating carbon monoxide poisoning, however this
intervention would always come second to airway protection by intubation.

PENETRATING Hypovolemic shock (ie, severe HoTN, tachycardia, flat neck veins) in the setting
THORACIC of penetrating chest trauma (ie, stab to chest) is attributed to hemorrhage until
TRAUMA proven otherwise; trauma primary survey should seek to identify sources of bleeding.

A small external wound can conceal extensive internal injury. Central penetrating
wounds inside "the box"—delineated superiorly by the clavicles, inferiorly by the
costal margins, and laterally by the nipples—are particularly dangerous because of
the underlying heart and mediastinal structures. However, even lateral wounds (eg, 3-
cm lateral wound) can cause significant injury, including massive hemothorax (>1,500
mL) from laceration of underlying lung parenchyma or intercostal vessels.

Although tube thoracostomy is often sufficient to manage hemothorax, some patients


(up to 15%) require emergent thoracotomy for extreme bleeding, including those
with:

• Initial bloody output >1,500 mL.


• Persistent hemorrhage: >200 mL/hr for >2 hours, or continuous need for
blood transfusion to maintain hemodynamic stability.

…………..
Clamping the chest tube is not recommended; blocking the outflow of blood does
not staunch the bleeding (the capacity of the hemithorax is too great), and
accumulated intrathoracic blood negatively impacts respiratory mechanics (eg,
restricted lung expansion).

A contrast-enhanced CT scan can assist in identifying a bleeding source (via


visualization of contrast extravasation); however, it is only appropriate for
hemodynamically stable patients.

Although acute tamponade can cause severe hypotension and tachycardia, it


classically causes elevated JVP with distended (rather than flat) neck veins.

Vasopressors are appropriate for patients with severe peripheral vasodilatation due to
distributive (eg, septic, neurogenic) shock. Patients with shock due to blood loss (ie,
hemorrhagic shock) should receive volume resuscitation with blood transfusion while
awaiting definitive management of the hemorrhage (eg, surgical intervention).
Vasopressor administration without appropriate resuscitation can worsen tissue
perfusion.

BLUNT A patient with isolated blunt chest trauma (eg, blow to the chest on the table) now
THORACIC has symptomatic HoTN (lightheadedness), tachycardia, and flat jugular veins,
TRAUMA concerning for hypovolemic shock. Because each half of the chest can hold up to
40% of the circulating blood volume, large intrathoracic hemorrhage (eg,
hemothorax) can lead to acute
hemodynamic instability.
Hemothorax (eg, diminished breath sounds,
dullness to percussion) may result from
injuries to large (eg, aorta, hilar vessels) or
small intrathoracic structures (eg, intercostal
blood vessels, lung parenchyma). Pleuritic
chest pain and exquisite chest wall
tenderness to palpation make the most
likely source is fractured
ribs with intercostal vessel injury.

Hemothorax is treated with tube


thoracostomy, which is sufficient to resolve
many cases, although if immediate chest
tube output is >1,500 mL of blood, emergent surgical thoracotomy is indicated.

…….

Blunt chest trauma can transmit stretching, shearing, and torsional forces capable of
tearing the aorta, which (if the tear is full-thickness) may cause massive hemothorax
and hemorrhagic shock. However, this typically occurs after a more severe mechanism
of rapid deceleration (eg, fall >3 m [10 ft], motor vehicle collision), and the aortic
isthmus (rather than the ascending aorta) is the most commonly affected site.
Ascending aortic involvement would likely have additional findings such as ACS,
cardiac tamponade, or FNDs (from cerebrovascular ischemia).

Unilateral diaphragmatic dysfunction due to phrenic nerve injury or traumatic


diaphragmatic rupture can present with tachypnea, shallow breathing, and unilateral
diminished breath sounds (due to decreased lung expansion). However, neither injury
typically results in severe hypotension or tachycardia (ie, shock).

A patient with blunt thoracic trauma (eg, chest run over by a tractor) has evidence of
extensive extrapulmonary air, with both a tension pneumothorax (absent breath
sounds, tracheal deviation, hypotension) and crepitus (crackling of the neck and chest
skin). Even after appropriate treatment of the tension pneumothorax with tube
thoracostomy, there is a persistent large air leak. This suggests continuous, rapid
extrapulmonary air accumulation from a tracheobronchial injury that allows (due to
the larger diameter of the proximal airway) a large quantity of air to escape with each
breath.

Tracheobronchial injury should be considered in any patient with thoracic trauma and
extensive extrapulmonary air. Bronchoscopy, which can directly visualize the lumen of
the tracheobronchial tree, is the definitive test for diagnosis and can be easily
performed through the endotracheal tube in intubated patients. HRCT scan can
diagnose major injuries but may miss small tears.

Most patients require operative repair.

……….

In some mechanically ventilated patients with hypoxemia, increasing PEEP can help
keep alveoli open at the end of the expiratory phase and improve oxygen diffusion.
This patient is no longer hypoxemic; in addition, for conditions affecting the large
airways (eg, tracheobronchial injury) rather than the alveoli, changes in PEEP (eg, 5 cm
H2O) would not have a significant impact.

Insertion of an additional chest tube is indicated if the existing one is inadequate (eg,
small diameter limiting the ability to adequately drain fluid). This patient's chest tube is
functioning well (ie, draining large amounts of air), so the source of the large air leak
(ie, tracheobronchial injury) should be investigated.

Talc pleurodesis obliterates the pleural space to prevent recurrence of frequent


pleural effusion (eg, malignant effusion) or pneumothorax (eg, COPD). It is not used
for primary treatment. In addition, pleurodesis requires physical contact between the
visceral and parietal pleurae, which is likely not present in this patient with a large air
leak due to tracheobronchial injury.

PNEUMONECTO Indications for lung resection include malignancy, severe COPD, and persistent
MY infection despite medical treatment. An important consideration prior to lung
resection is whether the patient has enough pulmonary reserve to maintain adequate
lung function following surgery. FEV1 and DLCO, obtained by preoperative pulmonary
function testing, are the best predictors of postoperative outcomes following lung
resection surgery.

Based on preoperative FEV1 and DLCO measurements, the percentage of planned


lung resection (eg, approx.. 50% in unilateral pneumonectomy) is used to estimate
postop FEV1 and DLCO. Patients with an estimated postoperative FEV1 or DLCO of
<40% are at elevated risk of postoperative morbidity.
………..

End-tidal CO2 (ETCO2) measurement (capnography) is indicative of aerobic


metabolism and can be used to monitor correct endotracheal tube placement or to
measure the effectiveness of CPR.

Maximal inspiratory pressure (or negative inspiratory force) is a measurement of


inspiratory muscle strength. It is often used to monitor patients with hypoventilation
due to neuromuscular disease (eg, MG, GBS) and help assess the need for
mechanical ventilation.
Measurement of oxygen saturation during exercise is used to evaluate the need for
home supplemental oxygen therapy in patients with COPD.

The ratio of RV to TLC increases in patients with COPD due to air trapping and lung
hyperinflation; a high ratio correlates with poorer overall outcomes in patients with
COPD, but it is not predictive of postoperative outcomes following lung resection.

BRONCHOGENIC The diagnosis of mediastinal tumors is based on CXR and CT scans. A bronchogenic
CYST cyst may be seen on the AP chest x-ray. The diagnosis is best made with a CT
scan. Bronchogenic cysts are located in the middle mediastinum and are benign
entities. Other middle mediastinal masses include: tracheal tumors, pericardial
cysts, lymphoma, LN enlargement, and aortic aneurysms of the arch.

…………..

Thymoma is usually found in the anterior mediastinum. The diagnosis is suspected


when an anterior mediastinal mass presents in a young male or female. About 20% of
patients with myasthenia gravis have a thymoma. Other anterior mediastinal masses
include: retrosternal thyroid, teratoma, and lymphoma. If the mass is large, patients
may complain of chest heaviness or discomfort. Hoarseness, Horner's syndrome, and
facial and upper extremity edema may occur when the tumors invade locally.
All neurogenic tumors are located in the posterior mediastinum. These include:
meningocele, enteric cysts, lymphomas, diaphragmatic hernias, esophageal tumors,
and aortic aneurysms. MRI is the best modality to evaluate posterior
mediastinal masses.
Esophageal leiomyomas are located in the posterior mediastinum. Esophageal
leiomyomas are submucosal and usually asymptomatic. They only produce symptoms
when > 5 cm. Only large tumors should be removed.
LUNG NODULE
Solitary pulmonary nodules

• Large size*
• Advanced patient age
• Female sex
Factors increasing
• Active or previous smoking
malignant probability • Family or personal history of lung cancer
• Upper lobe location
• Spiculated radiographic appearance

*Size >2 cm independently correlates with >50% malignant probability.

A solitary pulmonary nodule (SPN) is defined by the following features:

• Rounded opacity
• ≤3 cm in diameter (>3 cm is considered a "mass")
• Surrounded by pulmonary parenchyma
• No associated lymph node enlargement

The size of an SPN strongly correlates with the chances of it being malignant. Nodules
<0.6 cm are unlikely to be malignant and generally do not require follow-up;
however, nodules >0.8 cm require additional management or surveillance.

In addition to size, other factors that influence the probability of an SPN being
malignant include patient age, sex, smoking history, family history, location of the
nodule in the lung, and radiographic appearance of the nodule (eg, regular
versus irregular borders). Nodules >0.8 cm that are intermediate or high probability
for malignancy (ie, ≥5% risk) based on these factors require tissue diagnosis
with biopsy or surgical excision. A relatively large SPN with irregular borders in a 65-
year-old patient with a significant smoking history has high malignant probability and
should be biopsied or surgically excised.

For patients in whom the need for tissue diagnosis is unclear, positron emission
tomography (PET) scan can be helpful. Nodules demonstrating high metabolic activity
on PET scan are more likely malignant than benign and warrant biopsy or
surgical excision.

………….

Infectious granulomas (eg, tuberculosis, histoplasmosis, atypical mycobacteria) are the


most common cause of benign pulmonary nodules; however, the nodules are typically
multiple rather than solitary and, in the absence of active (symptomatic) disease (eg,
fever, cough, weight loss), are relatively small (ie, <1 cm).

Surveillance with repeat CT scan in 3 months is appropriate for nodules >0.8 cm that
are low probability for malignancy (eg, 1 cm nodule in a 50-year-old man with no
smoking history).
LUNG CANCER Most cases of SCC arise in the central portion of the lung, therefore manifesting as
a hilar mass (due to squamous metaplasia → dysplasia → carcinoma-in-situ within the
central tracheobronchial tree). Patients generally present with cough, dyspnea,
and/or hemoptysis due to erosion of the airway mucosa. Smoking is the single
greatest risk factor.
SCC is associated with hypercalcemia of malignancy (mnemonic: sCa++mous) due to
the release of parathyroid hormone–related protein (PTHrP), which binds to the PTH
receptor in bones and in the distal tubule of the kidney and increases calcium
reabsorption. Common manifestations of hypercalcemia include anorexia/
constipation, renal dysfunction (eg, nephrogenic diabetes insipidus [NDI;
polydipsia]), and musculoskeletal pain.

Hypercalcemia of malignancy is generally due to either osteolytic bone metastases or


the release of PTHrP. Differentiation can be made with whole-body bone scan and
PTHrP testing. This patient's normal whole-body bone scan (no metastatic bone
lesions) indicates that the source of his hypercalcemia is PTHrP.

………..

Most cases of adenocarcinoma of the lung are associated with peripheral lung
lesions. Although hypercalcemia of malignancy can occur, it is usually due to osteolytic
bone metastases (not PTHrP release). The presence of a central lesion and normal bone
scan make adenocarcinoma less likely.

LUNG ABSCESS
Lung abscess

• Aspiration pneumonia (most common)


o Dysphagia, substance abuse
Epidemiology
• Gingival disease usually present
• Oropharyngeal anaerobes

• Indolent symptoms
Manifestations • Fever, night sweats, weight loss
• Cough with putrid sputum

• Cavitary infiltrates with air-fluid levels


Diagnosis
• Cultures rarely useful

Treatment • Clindamycin

A patient lost consciousness due to alcohol intoxication 3 weeks ago and now has
fever, leukocytosis, and a cavitary pulmonary infiltrate with air-fluid levels,
suggesting aspiration pneumonia with lung abscess.

Lung abscesses typically arise due to the aspiration of oropharyngeal anaerobic


bacteria. Although healthy individuals routinely aspirate upper airway microbes, illness
is uncommon due to small inoculum size and mechanical/immunologic clearance.
Patients with lung abscess usually have an underlying medical condition that results in
large inocula (eg, dysphagia) or episodes of impaired consciousness (eg,
drug/alcohol abuse, hepatic failure).

Anaerobic bacteria are usually less virulent than typical community acquired
pneumonia pathogens (eg, Streptococcus pneumoniae); therefore, symptoms of lung
abscess are often indolent. Manifestations include fever, night sweats, weight loss, and
cough with putrid sputum. Physical examination usually shows evidence of gingival
disease and poor dentition. Hyponatremia may be present due to infection-induced
antidiuretic hormone secretion. Imaging typically reveals cavitary infiltrates with air-
fluid levels in the superior lower lobe or posterior upper lobe of the lung (sections that
are dependent with recumbency).

Anaerobic bacteria are difficult and costly to isolate; as such, the diagnosis is made
largely on clinical grounds. Treatment with antimicrobial medication
(eg, clindamycin) is usually curative.

……….
Pulmonary aspergillosis is seen primarily in immunocompromised patients and may
manifest with the classic triad of fever, pleuritic pain, and hemoptysis. Imaging typically
reveals nodules with surrounding ground-glass infiltrate (halo sign).
Squamous cell carcinoma of the lung commonly manifests with a cavitary lung lesion,
weight loss, and hyponatremia; however, fever, leukocytosis, and a short duration of
symptoms would be uncommon.

Patients at risk for aspiration may inhale gastric contents, resulting in chemical
pneumonitis. Manifestations typically arise within 2 hours (not a week) and are
associated with significant dyspnea and minimal fever.

A patient with a history of homelessness and alcohol abuse is at risk for pulmonary
tuberculosis. Manifestations may include the indolent onset of fever, weight loss, and
cough; however, putrid sputum and an infiltrate with air-fluid levels are more likely due
to lung abscess.

A long-standing cough productive of yellow sputum, systemic symptoms (eg, weight


loss, malaise, night sweats), leukocytosis, and a cavitary lesion in the right lower lobe
raises strong suspicion for lung abscess. Most cases are caused by the aspiration of
oropharyngeal anaerobic bacteria into dependent portions of the lung during
periods of impaired consciousness (eg, alcohol/drug abuse, seizure disorder)
or swallowing dysfunction.
Parkinson disease is frequently associated with significant swallowing dysfunction due
to loss of dopaminergic neurons that control the striated muscles in the oral cavity,
pharynx, and esophagus. Coughing during
eating (due to microaspiration of food
particles) is often an important clue; as
dysphagia worsens, patients are at very high
risk for aspiration pneumonia with or without
lung abscess.

Lung abscess usually causes indolent


fever and cough productive of foul-smelling
sputum. Systemic manifestations such as night
sweats, weight loss, and malaise are often
present, and laboratory evaluation frequently
reveals leukocytosis and signs of chronic
inflammation (eg, anemia of chronic disease
[ACD], thrombocytosis). The diagnosis is
generally confirmed when chest imaging shows
a cavitary infiltrate in a dependent portion of the lung (eg, superior segment of a
lower lobe, posterior segment of an upper lobe).

……………

Histoplasma capsulatum is endemic to the Ohio and Mississippi River valleys. It is a


common cause of CAP in these regions and frequently presents with productive
cough and subacute systemic symptoms. However, CXR usually reveals a lobar
infiltrate and hilar/mediastinal LAD; cavitation very rarely occurs.
GPA is a necrotizing vasculitis that frequently presents in older adults with subacute
systemic manifestations and dyspnea from trachea or lung involvement. However,
productive cough is atypical, and CXR usually reveals nodular infiltrates, not
cavitary disease.
Lung cancer can cause weight loss, malaise, cough, anemia, thrombocytosis, and a
cavitary infiltrate. However, the presence of fever, leukocytosis, and malaise in the
setting of swallowing dysfunction make a lung abscess far more likely.

TB reactivation can lead to a cavitary lung lesion, subacute systemic symptoms, and
leukocytosis. However, this patient with a cavitary lesion in the lower lobe in the setting
of swallowing dysfunction is far more likely to have aspiration pneumonia with
subsequent lung abscess.
VTE often presents with pleuritic chest pain, tachycardia, and dyspnea. Although lobar
infiltrates can be seen when pulmonary infarction occurs, cavitary infiltrates and
chronic productive cough are atypical.

EMPYEMA
Parapneumonic effusions
Uncomplicated Complicated

Sterile exudate in pleural Bacterial invasion of pleural


Etiology space space

• pH ≥7.2 • pH <7.2
Pleural fluid analysis • Glucose ≥60 mg/dL • Glucose <60 mg/dL
• WBC ≤50,000/mm3 • WBC >50,000/mm3

Pleural fluid Gram stain


Negative Negative*
& culture

Treatment Antibiotics Antibiotics + drainage

*Gram stain & culture are typically false negative due to low bacterial count. Both are typically
positive in empyema.

Approximately 40% of pneumonias are associated with pleural effusions. Most are
small, free-flowing, sterile, and resolve with antibiotics (ie, uncomplicated
parapneumonic effusion). However, if bacteria persistently cross from the infected
pulmonary parenchyma into the pleural space, a complicated parapneumonic effusion
or an empyema can develop. Both empyemas and complicated parapneumonic
effusions are marked by large, (often) loculated effusions and typical thoracentesis
abnormalities (low pH, low glucose); unlike complicated parapneumonic
effusions, empyemas also have frank pus or bacteria (by Gram stain) in the pleural
space.

This patient likely has pneumonia (fever,


productive cough), and the presence
of pleuritic chest pain and a large left-sided
pleural effusion suggests the pneumonia is
likely complicated by an empyema. Risk for
pneumonia complications such as empyema is
greater in patients who are
immunocompromised (eg, HIV, post-
transplant). All empyemas (and many
complicated parapneumonic effusions) require
prolonged antibiotics (2-4 weeks) and
drainage (usually with a chest tube).

………
Acute pericarditis presents with sharp, anterior
chest pain. Viral myocarditis can cause chest pain and could be associated with upper
respiratory symptoms.
Bronchopleural fistulas often occur with malignancy or trauma, or after surgical
procedures involving the bronchi (eg, pneumonectomy). They can cause empyemas,
but there is usually radiographic evidence of air in the pleural space as well.
 A lung abscess is marked by a cavity with air fluid level
on chest x-ray. Aspiration pneumonia commonly causes
lung abscesses and is frequently associated with foul-
smelling sputum and symptoms that develop over days
or weeks.

 Pneumocystis jiroveci pneumonia is


common in patients with HIV with a CD4
count <200/mm3. Symptoms (eg, shortness of
breath, nonproductive cough, fever) typically
arise over several weeks. CXR can be normal
but often reveals diffuse bilateral interstitial or
alveolar infiltrates, not a large, unilateral
effusion.

 Pneumothorax
FOREIGN BODY
Foreign body aspiration
ASPIRATION
• Sudden-onset cough, dyspnea
Clinical
• Cyanosis
features • ± History of choking episode

Examination • Wheezing &/or stridor


findings • Focal area of diminished breath sounds

• Hyperinflation of affected side


• Mediastinal shift toward unaffected side
X-ray findings • Atelectasis if obstruction is complete
• ± Foreign body

Management • Rigid bronchoscopy

Wheezing, a common chief complaint in children, is primarily due to asthma or a viral


illness. However, sudden-onset, unilateral wheezing that is unresponsive to
albuterol raises suspicion for foreign body (FB) aspiration.
FB aspiration most commonly occurs in children age 1-3 due to oral exploratory
behavior and a relatively narrow airway diameter. Following aspiration, patients
classically develop abrupt-onset respiratory distress (eg, cough, dyspnea,
hypoxia). The aspiration event
may be unwitnessed, and a
history of choking is not always
present. In most cases, the object
partially obstructs one of the
bronchi, resulting in a prolonged
expiratory phase due to air
trapped distal to the obstruction.
Wheezing and decreased breath
sounds on the affected side are
characteristic,
and hyperresonance to
percussion can occur over the
hyperexpanded lung.

Although x-ray may be normal, findings suggestive of FB aspiration include unilateral


lung hyperinflation with mediastinal shift toward the unaffected side. Atelectasis
may be present in cases of complete bronchial obstruction. Most aspirated objects are
radiolucent and therefore not identified on x-ray.

Management includes immediate rigid bronchoscopy to confirm the diagnosis and


remove the object.
……….
Pneumonia can present with cough and respiratory distress but also typically causes
fever, crackles, and a lobar consolidation on x-ray.

ATELECTASIS Postoperative atelectasis: Thoracic or abdominal pain following surgery can restrict
chest expansion and lead to low tidal volume (TV) respiration with basilar alveolar
collapse. Pharyngeal secretions, airway tissue edema, and residual anesthetic effects
can also contribute. Postoperative atelectasis most commonly manifests on day
2 following surgery and may be seen up to postoperative day 5.

Causes of hypoxemia

Corrects with
A-a
Examples supplemental
gradient
O2 ?

Reduced PiO2 High altitude Normal Yes

Hypoventilation CNS depression, morbid obesity Normal Yes

Diffusion limitation Emphysema, ILD Increased Yes

V/Q mismatch* Small PE, lobar pneumonia Increased Yes

Large intrapulmonary
Diffuse pulmonary edema Increased No
shunt

Large dead-space Massive PE, right-to-left


Increased No
ventilation intracardiac shunt

*Caused by localized dead-space ventilation and/or intrapulmonary shunting.


A-a gradient = alveolar-arterial oxygen gradient; ILD = interstitial lung disease; PE = pulmonary
embolism; PiO2 = partial pressure of inspired oxygen; V/Q = ventilation/perfusion ratio.

Atelectasis causes hypoxemia (ie, low PaO2) due to localized intrapulmonary shunting,
leading to ventilation-perfusion (V/Q) mismatch. This triggers an increased respiratory
rate (RR), which more than compensates for the reduction in tidal volume (TV). Because
minute ventilation = TV × RR, there is an increase in minute ventilation
(ie, hyperventilation), which leads to decreased PaCO2 and consequently increased
pH (primary respiratory alkalosis). In patients without underlying chronic lung disease,
any cause of significant ventilation-perfusion mismatch (eg, pulmonary embolism,
pneumonia) usually presents similarly with hypoxemia and primary respiratory alkalosis.

Postoperative pulmonary complications

• Atelectasis, bronchospasm, pneumonia


Complications
• Prolonged ventilator requirement

• Age >50, active smoking


Risk factors • Underlying heart failure or obstructive lung disease
• Emergency surgery or surgery duration >3 hr
• Smoking cessation >4-8 weeks prior to surgery
Perioperative
• Symptomatic control of underlying lung disease
prevention
• Pain control, deep-breathing exercises, incentive spirometry

A patient developed postoperative atelectasis likely from a combination of retained


airway secretions and postoperative abdominal pain. He likely would have benefitted
from postoperative deep-breathing exercises and incentive spirometry, which
encourage high-tidal-volume respiration and help keep alveoli open. In patients with
underlying lung disease (eg, COPD), these interventions are often
pursued preoperatively as well. Adequate postoperative pain control is also
beneficial to minimize pain-induced restriction of chest expansion during respiration.

Once atelectasis develops, it can be treated with CPAP to help open collapsed alveoli.
Patients with abundant secretions may benefit from the addition of chest
physiotherapy and suctioning. In some cases, atelectasis due to large mucus
plugging may require mucus plug removal via bronchoscopy.

……….
Perioperative broad-spectrum antibiotics are not useful for the prevention of
pneumonia, atelectasis, or other perioperative pulmonary complications, even in
patients with asthma or COPD. Antibiotics should be used only if there is clear evidence
of infection prior to urgent or emergency surgery. Elective procedures should be
postponed until the infection has been adequately treated.
Preoperative systemic glucocorticoids and albuterol have not been shown to prevent
surgical complications, even in patients with asthma or COPD. Patients with poorly
controlled asthma or COPD should be symptomatically optimized prior to
elective surgery.

Cigarette smoking is associated with an increased risk of perioperative pulmonary


complications. Smoking cessation has been shown to reduce the risk of both
pulmonary and overall surgical complications. However, the benefit is present only
when cessation occurs >4-8 weeks prior to surgery, perhaps because the airway
inflammation and increased mucus production caused by smoking require several
weeks to improve.

////////////////////////GASTROINTESTINAL & NUTRITION///////////////////////


COLONIC - Ischemic colitis: Fever,
ISCHEMIA nausea, ↑ lactic acid, and/or
leukocytosis. Older patients with
atherosclerotic vascular disease are
at particularly high risk.
Contributing factors may include
loss of collateral circulation,
manipulation of vessels with
surgical instruments, prolonged
aortic clamping, and impaired
blood flow through the inferior
mesenteric artery.
- CT imaging in IC can show edema and air (pneumatosis) in the bowel wall.
Colonoscopy shows segments of cyanotic mucosa and hemorrhagic ulcerations,
with a sharp transition from
affected to unaffected
mucosa. Unless the patient
has perforation or bowel
gangrene, most cases are
managed conservatively
with intravenous fluids,
bowel rest, and antibiotics.
The MCly involved
segments of the colon
include the splenic flexure
at the "watershed" line
between the territory of the
SMA and IMA and the
rectosigmoid junction at the
watershed between the
sigmoid artery and
superior rectal artery.
- Abdominal CT scan with IV
contrast may show thickened bowel
wall, although it may demonstrate only
nonspecific findings. CT should be
performed urgently to identify patients
with ischemic colitis who need
immediate surgical intervention (eg,
extensive bowel damage,
perforation). If colon resection is not
needed, patients are treated with IV
fluids and antibiotics and
endoscopic evaluation
(colonoscopy) is performed to confirm the diagnosis. Typical findings include pale
mucosa with petechial bleeding, bluish hemorrhagic nodules, or cyanotic
mucosa with hemorrhage w/ rectal sparing.
- Small bowel (mesenteric) ischemia is usually due to atheroembolic (eg,
endovascular procedures) or thromboembolic (eg, A-fib) events rather than acute
HoTN. Pain is typically severe but is poorly localized and out of proportion to
examination findings; hematochezia may occur but is a late finding. CT would show
focal or segmental bowel wall thickening and mesenteric stranding.
- C. diff colitits: nonbloody D., typical colonoscopy findings include erythema, edema,
and ulceration of the bowel wall.
- Chronic radiation proctopathy often causes bloody stools, but usually presents
for the first time within the first year. Colonoscopy would show pale mucosa with
ulcers, strictures, telangiectasias, and focal hemorrhage.
PERFORATION - Can be caused by postoperative adhesions leading to bowel obstruction (eg,
anorexia, x-ray findings of dilated small-bowel loops with air-fluid levels)
complicated by increased intraluminal pressure.
- Plain radiographs often confirm the diagnosis (sensitivity ~50%-70%).
- Free air on x-ray and clinical signs of peritonitis should prompt emergent surgical
exploration to prevent deterioration and possible death.
- In patients in whom perforation is suspected and plain radiographs are negative, CT
scan of the abdomen with IV contrast can help detect smaller amounts of free air or
free fluid. In this patient with peritonitis and an x-ray confirming perforation,
operative intervention should be immediately initiated (if operating availability is
delayed, it would be reasonable to perform a CT scan to try to visualize the site of
perforation)
…………………………..
- Barium enema and upper gastrointestinal series use contrast to outline the lumen
of the gastrointestinal tract for diagnosis of functional or structural abnormalities
(eg, obstructing mass). However, barium contrast is contraindicated if perforation
is suspected because it incites a severe inflammatory response if leaked into the
peritoneal cavity. In addition, emergent surgical exploration should not be delayed
for further imaging because of the patient's peritonitis and radiographic
confirmation of perforation.
- Risk factors for chronic mesenteric ischemia (which can be evaluated by contrast
angiography) include, hyperlipidemia, smoking history…etc, but this type of
ischemia typically causes recurrent episodes of postprandial pain, which are not
seen in this patient. Pneumoperitoneum is not expected except in severe cases
with bowel necrosis, in which case surgical intervention is the correct next step.
- NG tube placement may be helpful for decompressing the patient's SBO.
………………………………………….
- Full-thickness erosion of a peptic ulcer through the stomach or duodenal wall
releases both air and caustic (ie, pH ~1-2) gastric secretions/contents into the
peritoneal cavity. This quickly results in chemical peritonitis (eg, marked abdominal
tenderness with guarding) and an early systemic inflammatory response (eg, fever,
tachycardia) that can progress to sepsis and shock if left untreated.
- Patients with free perforation causing gross spillage of gastrointestinal contents (vs
microperforation with little to no spillage) can often note the precise time the
perforation occurred. Sudden, severe pain is typical. Demonstration of
intraperitoneal free air (eg, on upright x-ray) confirms the diagnosis and should
prompt immediate surgical consultation, since many patients require surgical
intervention and delay increases mortality (which is up to 30%, even with
treatment).
- Immediately following diagnosis, the following should be initiated:
• Surgical consultation
• Intravenous proton pump inhibitors and broad-spectrum antibiotics
• Fluid resuscitation and nasogastric suction
- Patients require emergent surgical exploration (laparotomy or laparoscopy)
because delay to surgery has been associated with increased mortality (up to 30%).
A minority of patients (eg, minimal/localized symptoms, stable vital signs) may
receive a trial of medical management with surgical backup available.
- If plain x-rays are negative and perforation is suspected, CT scan can help detect
smaller amounts of free air or fluid.
- CT scan with oral contrast is rarely used for life-threatening conditions (eg,
perforated peptic ulcer) because oral contrast use delays disposition by 45-60
minutes.
- Upper endoscopy should be performed after the perforation has healed to evaluate
for cancer and/or Helicobacter pylori infection. Endoscopy is contraindicated in the
setting of acute perforation because it can worsen the injury.
…………………………….
- Acute cholecystitis can cause epigastric pain, fever, nausea/vomiting, and focal
peritonitis. However, pain onset is not typically as sudden, and peritoneal
inflammation is usually isolated to the right upper quadrant. Gallbladder
perforation can cause upper
abdominal pain but typically
only after several days of
symptoms (vs 2 hours in this
patient) and would not
cause pneumoperitoneum
because the gallbladder
does not contain significant
air. This patient likely has
asymptomatic gallstones,
which are common in the
general population. Asx gallstones are common in the general population.
- Pancreatitis does not cause subdiaphragmatic free air. Abdominal x-ray is often
normal but may show gallstones or paralytic ileus due to surrounding inflammation.
- A patient developed abdominal pain and absent bowel sounds following
a traumatic injury. X-ray demonstrates gastric dilation and gas-filled loops of both
the small and the large intestines on abdominal x-ray,
suggesting paralytic (adynamic) ileus. Ileus is most commonly a complication of
abdominal surgery but can also be seen in other conditions such
as retroperitoneal/abdominal hemorrhage, intraabdominal inflammation (eg,
pancreatitis), intestinal ischemia, and electrolyte abnormalities. Contributors to
the pathophysiology of ileus include irritation and temporary paralysis of the
abdominal sympathetic and parasympathetic nervous system, local release of
inflammatory mediators, and
opioid analgesic use.

Signs and symptoms of ileus


include nausea,
vomiting, abdominal
distension, failure to pass flatus
or stool (obstipation), and
hypoactive or absent bowel
sounds. The diagnosis is
clinical, but abdominal x-rays
(classically revealing
uniformly dilated, gas-filled
loops of bowel with no transition point) can be helpful
in confirmation. Management is conservative and
includes bowel rest, supportive care, and treatment of
secondary causes.
← The MC sites of volvulus in adults are the cecum/proximal ascending colon and
sigmoid colon. Abdominal x-rays frequently reveal a transition point at the cecum
or sigmoid colon. Patients are usually much older than this patient and have
a more insidious onset of symptoms. Neither cecal nor sigmoid volvulus is a
common complication of trauma.
- In complete small bowel obstruction, the bowel
distal to the obstruction is not distended. In
addition, hyperactive "tinkling" bowel sounds are
typically present on physical
examination. Peristaltic waves on the abdominal
wall can also be observed.
BLUNT A patient with blunt chest trauma (high-speed MVA, anterior chest wall bruises)
THORACIC initially has a tension pneumothorax (absent breath sounds, hypotension) and
TRAUMA subcutaneous air (crepitus). Despite appropriate tube thoracostomy, repeat CXR
shows rapid pneumothorax reaccumulation (leading to declining oxygen saturation)
and increased subcutaneous emphysema. This suggests a tracheobronchial injury in
which a large quantity of air escapes with each breath. The rapidity of extrapulmonary
air accumulation in this patient is consistent with proximal (eg, bronchial)
airway rupture because the larger diameter of the more proximal airway leads to more
air leakage.

Tracheobronchial injury should be considered in trauma patients with extensive


extrapulmonary air. Classic findings are rapid, large air leak into the chest-tube
drainage system and persistent pneumothorax/pneumomediastinum despite tube
thoracostomy.

Bronchoscopy is the definitive test for diagnosis; HRCT scan can diagnose major
injuries but may miss small tears.

Most patients require operative repair.

………..
Aspiration pneumonitis (eg, aspiration of inflammatory gastric acid during post-
collision unconsciousness) and trauma can lead to acute respiratory distress
syndrome (ARDS). Both aspiration pneumonitis and ARDS may cause progressive
desaturation and increased oxygen requirement; however, infiltrates (rather than
extensive extrapulmonary air) would be expected on CXR.

Esophageal rupture may allow efflux of esophageal air into surrounding tissues,
potentially resulting in pneumomediastinum and pneumothorax. However, the
quantity and rapidity of leaked air in esophageal rupture is significantly less than
with bronchial rupture. Therefore, rapid reaccumulation of pleural air in the presence
of a chest tube makes esophageal rupture unlikely.

Fat embolism: typically takes 24-72 hours to manifest following the traumatic insult.
PENETRATING A patient with gunshot wounds (GSWs) to the left thoracoabdominal region has
THORACIC shock (ie, severe HoTN and tachycardia) despite initial crystalloid resuscitation. Based
TRAUMA on the location of his wounds
(left sixth and seventh
intercostal spaces), both
intrathoracic and
intraabdominal causes of
shock should be investigated
because the diaphragm and
intraabdominal contents
can rise as high as
approximately the fourth
thoracic dermatome
(ie, nipple level) on
expiration.
In this patient, there is no
evidence of cardiac
tamponade (eg, no pericardial
effusion) or hemothorax (eg, equal breath sounds, no pleural effusion), and the
irregular opacities revealed on chest x-ray likely represent a pulmonary contusion
that does not require immediate intervention. Given the absence of an intrathoracic
source of shock, intraabdominal organ (eg, spleen) injury due to penetrating
abdominal trauma (PAT) should be suspected. For patients with PAT, immediate
exploratory laparotomy is indicated for:

• Hemodynamic instability (SBP <90 mm Hg).


• Peritonitis (eg, rigidity, rebound tenderness).
• Evisceration (ie, externally exposed intestines).
• Impalement (i.e., penetrating object still in situ).

Because this patient already has an indication for laparotomy, the abdominal portion
of the FAST examination is not necessary as it does not change management.
However, it is sometimes performed in practice (eg, immediately following the
cardiac portion) because a positive result (ie, intraperitoneal free fluid) further
confirms the need for laparotomy.
…………

CT scan of the abdomen (commonly used, noninvasive) and DPL (rarely used, invasive)
can help detect intraabdominal injury and determine the need for laparotomy in
hemodynamically STABLE patients with PAT. Plain radiographs of the abdomen may
help determine the trajectory and resting location of a bullet. However, this patient
requires immediate laparotomy rather than additional imaging or testing because of
hemodynamic instability.
Local wound exploration to determine whether violation of the peritoneum has
occurred is inappropriate in a patient with an indication for immediate ExLap. In
addition, this technique is rarely performed in patients with GSW (vs abdominal stab
wounds) because tissue destruction makes following the wound tract difficult.

BLUNT Hepatic subcapsular hematoma and—most important—intraperitoneal free air.


ABDOMINAL
TRAUMA (BAT) Perforation may occur
acutely (eg, during
rapid compression of
the GI tract that
causes high
intraluminal pressure)
or in a delayed fashion
(eg, several hours to
days following
trauma), as in this
patient. Delayed
perforation can be due
to the following:
1. Bowel contusion (eg, thickened [edematous] proximal small bowel on initial CT
scan) progressing to full-thickness injury
2. Injured mesenteric vasculature (eg, mesenteric hematoma on initial CT scan)
progressing to ischemia and necrosis
Many patients with bowel contusion or mesenteric hematoma do not need further
intervention. However, an extended period of observation should be considered
due to the risk of progression to perforation

Triage of patients with BAT without frank peritonitis (eg, abdominal rigidity, rebound
tenderness) includes bedside FAST to assess for intraperitoneal free fluid, a marker of
intraabdominal organ injury.

For polytrauma patients, a trauma survey can quickly identify serious injuries and triage
them for possible emergency surgery. When BAT is present, intraabdominal injury and
hemorrhage must be investigated. The initial approach is similar in both
hemodynamically stable (HDS) (eg, systolic blood pressure >90 mm
Hg) and hemodynamically unstable (HDUS) patients:

• Peritonitis (eg, rigidity, rebound tenderness) is an indication for immediate


surgery (not seen in this patient).
• In the absence of peritonitis, FAST is typically the first step in abdominal
imaging. This noninvasive, ultrasound-based examination can be rapidly
performed at the bedside to detect significant intraperitoneal free
fluid (eg, blood, urine [bladder rupture]).

A positive FAST strongly suggests intraabdominal injury. In HDS patients, such as this
one, there is time to pursue more definitive imaging (eg, CT scan) to visualize the site
and extent of injury. In contrast, HDUS patients with a positive FAST require
immediate laparotomy.

A negative FAST does not rule out intraabdominal organ injury because injuries that do
not produce large amounts of free fluid (eg, bowel perforation, diaphragmatic rupture)
can be missed. Therefore, if FAST is negative or equivocal, further investigations (eg,
CT scan) may be required depending on the patient's hemodynamic status and the
clinician's level of suspicion for intraabdominal injury.

HDS patients with polytrauma often undergo a CT of the head and chest following a
trauma survey. However, in a patient with diffuse abdominal tenderness (ie, higher
likelihood of intraabdominal injury) and no emergency indications for CT of the head
(eg, low Glasgow Coma Scale score) or chest (eg, widened mediastinum concerning for
thoracic aortic injury), FAST should precede these studies.

DPL has been largely reduced to evaluating for intraabdominal hemorrhage in


hemodynamically unstable patients with questionable bedside ultrasound results.

Angiography should not delay surgical intervention once perforation is confirmed.

Upper GI series: this study is time-consuming (eg, time to drink contrast, time for
contrast to pass through the GI system) and unnecessary because the diagnosis of
perforation has already been established by CT scan.
…………………………………………………………………..
- A child has epigastric discomfort, N, and worsening V approximately 1 day after
BAT (eg, balance beam impact). This
presentation is concerning for duodenal
hematoma (DH) and is best evaluated
with abdominal CT scan
- Rapid compression of the
duodenum against the vertebral column
during BAT can injure blood vessels
between the submucosal and the
muscularis layers, causing bleeding and
hematoma formation. As the
hematoma progressively expands over
the subsequent 24-48 hours, partial or
complete (inability to tolerate PO intake, progressively worsening
emesis) obstruction of the duodenal lumen can develop. As a result, patients
typically have a delayed presentation with suggestive symptoms (eg, epigastric
pain, bilious emesis) and radiographic findings (eg, gastric dilation with scant distal
gas). Children typically have less protective abdominal walls (eg, thinner
musculature, less adipose tissue) and are therefore more vulnerable to developing
DHs from BAT.
- CT scan of the abdomen can confirm the diagnosis and visualize the extent of the
hematoma, which typically appears as a homogenous density (ie, coagulated
blood) within the C-shaped duodenum. Once confirmed, DHs typically are
managed with:
• Bowel rest, Antiemetics
• NGT decompression, and
• Parenteral nutrition
• Followed with serial CT scans or duodenal USG.
Most DHs resolve within a few weeks with nonoperative management. But
percutaneous drainage or surgery may be considered for DHs that persist beyond
a few weeks.
- Pancreatic injury may result from BAT rapidly compressing the pancreas against the
vertebral column. Resulting pancreatic leakage may form a pseudocyst, with the
possible symptoms of emesis and epigastric tenderness. However, pseudocysts
typically develop over weeks rather than 2 days.
- The role of DPL has been largely reduced to evaluating for intraabdominal
hemorrhage in:
• Hemodynamically unstable patients WITH:
Questionable bedside ultrasound results
OR
When FAST scan is not available
- Exploratory laparotomy is indicated when patients with BAT have peritonitis (eg,
rigidity, rebound tenderness) or hemodynamic instability (eg, systolic blood
pressure <90 mm Hg) with evidence of intraabdominal hemorrhage (eg,
intraperitoneal free fluid).

- Gallbladder USG is commonly used to evaluate for suspected cholecystitis, which


can cause epigastric discomfort and emesis but typically also causes fever and RUQ
tenderness. Isolated gallbladder injury from BAT is extremely rare
……………….
Splenic injury: Management depends on the patient's hemodynamic status following
resuscitation.
In patients who are hemodynamically stable enough to undergo further imaging, CT of
the abdomen and pelvis with IV contrast is ideal for visualizing the extent of injury
and diagnosing ongoing bleeding (eg, contrast extravasation). Often, blunt splenic
injury can be managed nonoperatively (eg, observation with serial hemoglobin
measurement, embolization). However, continued hemodynamic instability (eg,
despite initial resuscitation) or frank peritonitis are indications for exploratory
laparotomy and potential splenectomy.

- Urinary bladder rupture from BAT can cause intraperitoneal free fluid (ie, urine) on
FAST. However, shock is uncommon, and difficulty voiding and/or gross hematuria
are typically present.
- Rapid compression of the upper abdomen against the vertebral column can crush
the duodenum or pancreas, potentially causing duodenal hematoma or pancreatic
transection. However, duodenal hematoma does not cause intraperitoneal free
fluid because bleeding remains confined within the duodenal wall. Likewise,
pancreatic transection does not cause an acute effusion of intraperitoneal fluid;
even with leakage of pancreatic juice (ie, ductal injury), formation of a large,
ultrasound-detectable peripancreatic fluid collection (eg, pseudocyst) typically
takes days to weeks.
A hemodynamically unstable patient with blunt chest and abdominal trauma and no
life-threatening intrathoracic injuries has free intraperitoneal fluid on FAST
examination. This warrants ExLap to control likely intraabdominal hemorrhage.

During the trauma survey of hemodynamically unstable (eg, SBP <90 mm Hg)
patients, the source(s) of instability should be investigated quickly. This includes ruling
out rapidly fatal intrathoracic conditions (eg, tension pneumothorax, cardiac
tamponade) and identifying major sources of hemorrhage. This patient's bilateral
breath sounds and lack of pericardial effusion make tension pneumothorax and
tamponade unlikely, respectively. However, his distended abdomen with diffuse
tenderness is concerning for intraabdominal hemorrhage due to blunt abdominal
trauma (BAT).

For patients with BAT (regardless of hemodynamic status), frank peritonitis (eg,
rigidity) warrants immediate laparotomy. In the absence of peritonitis, FAST
examination should be performed to identify intraperitoneal free fluid (eg, blood,
urine, spilled enteric contents).
In hemodynamically unstable patients, a positive FAST confirms significant
intraabdominal injury and therefore warrants emergent laparotomy. If FAST is
negative or inconclusive, additional investigations should be considered:

• CT scan can often visualize the site and severity of intraabdominal injury but
requires improved stability following resuscitation due to the risk of rapid
deterioration in the CT scanner.
• Bedside DPL may be considered in patients who remain too unstable for CT
scan despite resuscitation.

……………

Contrast angiography (with embolization) is sometimes used for BAT patients with
intraabdominal hemorrhage due to splenic or hepatic laceration. However, this is only
appropriate for hemodynamically stable patients with isolated, lower-grade injuries
(typically identified by CT scan).

Vasopressor therapy is CONTRAINDICATED in the setting of isolated hemorrhagic


shock because peripheral vascular tone is already increased (ie, vasoconstriction) to
compensate for decreased blood volume. Continued volume resuscitation (eg,
crystalloid, blood products) is more appropriate.

This patient with blunt chest/abdominal trauma has possible evidence of hemorrhage
(eg, tachycardia), along with left upper quadrant (LUQ) pain and left shoulder pain
that is likely referred (from blood irritating the phrenic nerve). This presentation is
most concerning for splenic injury, one of the MC intraabdominal complications
of blunt abdominal trauma (BAT).

Patients with BAT who do not have frank peritonitis (an indication for immediate
laparotomy) should undergo Focused Assessment with Sonography for Trauma
(FAST). A positive FAST strongly suggests intraabdominal injury; however, a negative
FAST does not rule it out because it can miss early bleeding or injuries (eg, bowel
perforation) that do not produce detectable (eg, >200 mL) amounts of free fluid.

For hemodynamically stable (eg, systolic blood pressure >90 mm Hg) patients with
a negative FAST:

• High suspicion of intraabdominal injury, as in this patient with tachycardia,


LUQ pain, and referred shoulder pain, should prompt contrast-enhanced CT
scan of the abdomen and pelvis. IV contrast "blush" (extravasation) can often
identify active bleeding.
• Low suspicion of intraabdominal injury (eg, normal vital signs, nontender
abdomen) can be managed with serial physical examinations, which are
performed to rule out occult injury progression.

If operative intervention is required, every effort should be made to repair rather


than remove the spleen, especially in children. If removal is unavoidable,
postoperative immunization against encapsulated bacteria is mandatory.

………….

Splenic injury is often associated with left lower rib fractures. Although dedicated
radiographs of the ribs are indicated (eg, costal margin tenderness), they are a lower
priority than CT scan for ruling out active bleeding with likely splenic injury (eg, LUQ
pain, referred left shoulder pain) and early shock (eg, tachycardia). Shoulder
radiographs are also lower priority, especially because this patient has normal
shoulder ROM and his pain is likely referred (eg, blood irritating left phrenic nerve).
(Diagnostic peritoneal lavage (DPL) may be considered for a negative/inconclusive
FAST in hemodynamically unstable patients whose condition may clinically
deteriorate during CT scan. Immediate exploratory laparotomy is indicated for
peritonitis (regardless of hemodynamic status) or for a positive FAST in an
hemodynamically unstable patient.

A patient with a negative abdominal CT scan following BAT (eg, handlebar impact) now
has persistent upper abdominal discomfort and N accompanied by a low-grade
fever and a large upper abdominal fluid collection. This presentation is most
concerning for pancreatic duct injury, with resulting leakage of inflammatory
pancreatic juice leading to accumulation of peripancreatic fluid.
Pancreatic injuries may result from BAT that rapidly compresses the fixed,
retroperitoneal pancreas against the vertebral column. Pancreatic injury is often
difficult to diagnose immediately following trauma because physical examination is
nonspecific, initial serum amylase and lipase levels are unreliable markers, and
abdominal CT scan has variable (~50%-80%) SN. Therefore, the development of
the following symptoms and signs in patients with recent BAT should raise suspicion
for undiagnosed pancreatic injury:

• Persistent abdominal discomfort/tenderness


• Persistent N/V
• Increasing amylase over serial measurements
• Peripancreatic fluid collection (due to pancreatic duct injury)

Ductal injury may require cholangiopancreatography for diagnosis.

Some (eg, low-grade) pancreatic injuries can be managed nonoperatively, but most
ductal injuries require surgical intervention.

……….
Abdominal aortic injuries due to BAT are rare (<1%). Many patients with traumatic
aortic injury die from exsanguination before reaching the hospital. Those who survive
(because bleeding has been initially contained within the retroperitoneum) would
be extremely unlikely to have a large amount of free fluid on FAST without
accompanying significant hemodynamic instability (eg, SBP <90 mm Hg).

Diaphragmatic rupture, which can occur after BAT due to a sudden increase in
intraabdominal pressure, may result in herniation of abdominal organs into the thoracic
cavity. This may trap the stomach or small bowel and cause obstructive symptoms (eg,
N, V). However, it does not typically cause significant intraabdominal free fluid and is
unlikely to cause a low-grade fever.

The liver and spleen are the most commonly injured solid organs in blunt abdominal
trauma. Although both hepatic laceration and splenic rupture can generate
intraabdominal free fluid (ie, blood), injuries severe enough to cause a large amount
of intraabdominal bleeding (eg, large amount of free fluid) are nearly always
detected on INITIAL abdominal CT scan.
PENETRATING A patient has penetrating
ABDOMINAL abdominal trauma (PAT) (ie,
TRAUMA (PAT) stab wound) to the LUQ with
possible injury to underlying
organs (eg, stomach, small
bowel, spleen). In addition to
tachycardia (ie, possible
early hemorrhagic shock), the
patient has peritonitis
(rigidity, diffuse tenderness).
This presentation is highly
concerning for
intraabdominal injury with
ongoing hemorrhage and
warrants immediate ExLap.

For patients with PAT, any of the following is an indication for immediate ExLap:

• Evidence of ongoing hemorrhage: hemodynamic instability (SBP <90 mm Hg)


or frank blood in the NG tube or rectum
• Peritonitis (eg, rigidity, rebound tenderness)
• Evisceration (ie, externally exposed intestines)
• Impalement (ie, penetrating object still in situ)

In contrast, patients without an indication for immediate laparotomy should undergo


further evaluation to determine whether peritoneal penetration occurred (eg, CT
scan, local wound exploration for anterior stab wounds) and/or whether
intraabdominal injuries are present (eg, FAST, CT scan). The presence of either
typically warrants surgical exploration.

…………
DPL is not commonly performed, but it can help rule out intraabdominal injuries in
patients with PAT who have no indication (eg, peritonitis) for immediate laparotomy.

In patients with isolated anterior stab wounds, an experienced surgeon may perform
local wound exploration under local anesthetic to look for violation of the
peritoneum (eg, visualization of intraabdominal contents), which usually prompts
definitive surgical exploration. However, peritonitis implies peritoneal penetration with
intraabdominal injury and warrants immediate surgical intervention.

In some patients, a stab wound may violate the peritoneum but miss important
intraabdominal organs. When this is suspected (eg, small wound with otherwise
benign abdominal examination, normal vital signs, nonconcerning imaging), serial
abdominal examinations may be performed to rule out progression of an occult
injury.
NUTRITION - Due to a hypercatabolic state, critically ill patients (ie, patients in the ICU) have
increased energy expenditure and protein requirements. This is especially true
for a patient who also has an open abdomen resulting in loss of protein-rich
peritoneal fluid. In such patients, early nutritional support can prevent
malnourishment and improve overall outcomes.
- When initiated early (ie, ≤48 hours), enteral (ie, via the intestinal route) nutrition
(EN) has multiple clinical benefits for critically ill patients, including reduction in
infections (eg, pneumonia) and possible reduction in mortality. EN may also
help maintain gut integrity, preventing atrophy of gut-associated and MALT. EN
formulas can be tailored to provide different amounts of carbohydrates (simple and
complex), protein, fatty acids, vitamins, minerals, and electrolytes. They are typically
delivered via a feeding tube to the stomach or, in patients with a high risk of
aspiration, the postpyloric duodenum.
- An open abdomen is not a contraindication to EN. For patients with an open
abdomen, EN is typically safe (eg, does not cause ileus or exacerbate bowel
edema) and beneficial (eg, earlier abdominal closure, lower fistula rate).
- 50% dextrose is typically administered in small doses (eg, 10 mL) for reversal of
acute hypoglycemia in unresponsive patients. 5% dextrose in normal saline is a
commonly used isotonic maintenance fluid. Although the administration of
dextrose would provide a small number of calories, these solutions lack additional
nutrients (eg, protein). In addition, they would not provide the benefits (eg,
maintenance of gut integrity) associated with early enteral feeding.
- TPN provides a complete but more elemental nutrient mixture and is typically
administered via central venous catheter (vs peripheral venous catheter) because
the high osmotic load is damaging to peripheral veins. TPN is typically used
only for patients with contraindications to enteral nutrition (eg, intestinal
discontinuity, prolonged ileus) because early initiation may increase the risk of
infection (eg, central line–associated bloodstream infection) and lead to
prolonged ICU and hospital stays.
…………………………………………..
- Refeeding syndrome: The development of hypophosphatemia, hypokalemia,
weakness, and arrhythmias after the initiation of tube feeding suggests refeeding
syndrome, which occurs after the reintroduction of nutrition in patients with
chronic malnourishment (eg, anorexia nervosa, malignancy).
- Starvation results in the depletion of phosphate, although serum levels may
remain normal due to transcellular shifts. The reintroduction of carbohydrates (ie,
tube feeding) leads to increased insulin secretion. This stimulates the cellular
uptake of electrolytes (ie, PO4, K, Mg) and increases phosphate utilization
during glycolysis (eg, formation of ATP, 2-3 DPG). This eventually leads to
phosphate depletion and failure of cellular energy metabolism, as well as massive
fluid and electrolyte shifts.
- Manifestations of refeeding syndrome include muscle weakness (which can
include the muscles of respiration), arrhythmias, CHF, rhabdomyolysis, and
neurologic dysfunction (eg, paresthesia, seizure). The disorder is characterized by
hypophosphatemia; other common laboratory abnormalities include
hypomagnesemia, hypokalemia, and LFT abnormalities. Management includes
close laboratory monitoring and aggressive repletion of electrolytes.
………………………………….
- Hypermetabolic syndrome typically occurs in the setting of severe burns, sepsis,
or hyperthyroidism, not chronic malnutrition, which suppresses metabolism. It
typically results in ongoing protein catabolism and weight loss despite
adequate nutrition; severe electrolyte derangements and muscle weakness would
be unexpected early in the clinical course.
- Paraneoplastic syndrome can cause metabolic disturbances but typically results in
hyponatremia (SIADH), hypercalcemia (humoral hypercalcemia of malignancy), or
hypoglycemia.
Approximately 1 day after sustaining blunt abdominal trauma (BAT), a patient has
fever, diffuse abdominal tenderness with epigastric guarding, and retroperitoneal
free air on imaging. This delayed presentation is concerning for a perforated
viscus within the retroperitoneum, such as a duodenal tear.

When a hollow viscus (eg, duodenum, ascending or descending colon) perforates


within the retroperitoneum, gastrointestinal spillage may be
initially sequestered away from the intraperitoneal space (which is separated from the
retroperitoneum by the posterior peritoneal lining). This may delay the development of
classic symptoms and signs of perforation (eg, fever, diffuse abdominal pain). In
addition, back or flank pain (eg, right flank pain) may be present due to
retroperitoneal inflammation.

Aspects of duodenal anatomy that increase the duodenum's susceptibility


to perforation include:

• Its location anterior to the vertebral column: compression against the vertebral
column during BAT can rapidly increase intraluminal pressure, causing rupture.
• Its multiple attachments (eg, hepatoduodenal ligament, ligament of Treitz):
abrupt force against the duodenum can cause duodenal tearing at fixed points
of attachment rather than stretching.

Duodenal perforation is typically treated with surgical repair.

……….
The stomach and transverse colon
Retroperitoneal abdominal organs
are also susceptible to rupture from
• Suprarenal (adrenal) glands rapid change in intraluminal pressure
• Aorta & IVC during BAT. However, both are
• Duodenum* (except 1st part)
intraperitoneal (vs retroperitoneal)
• Pancreas* (head & body)
• Ureters
structures. Therefore, intraperitoneal
• Colon* (ascending & descending) free air (eg, free air under the
• Kidneys diaphragm on CXR) would be
• Esophagus expected on imaging (vs
• Rectum (mid-distal) retroperitoneal free air seen in this
patient).
Mnemonic: SAD PUCKER
BAT can injure the pancreas or
*Secondarily retroperitoneal (developed
intraperitoneal & migrated retroperitoneal). kidney, causing spillage of pancreatic
enzymes or urine into the
retroperitoneal space. Although both renal pelvis laceration and traumatic
pancreatitis could result in a delayed presentation of abdominal and right flank
pain, they would typically cause free fluid rather than free air on imaging.
SOLIC LIVER
MASSES

- Hepatic adenoma: painful hepatic mass. Elevated ALP and GGT levels suggest
biliary compression or obstruction. MC in young and middle-age women. Risk
factors include OCPs, anabolic androgen use and pregnancy.
- Ultrasound most commonly shows well-demarcated, hyperechoic lesions, and
contrast-enhanced CT can show early peripheral enhancement. Needle biopsy is
not recommended for suspected hepatic adenoma due to the risk of bleeding, and
surgical excision is preferred. Possible long-term complications include progressive
growth, rupture, and malignant transformation.
- Hepatic adenoma can clinically resemble focal nodular hyperplasia (FNH), a
common mass lesion in young women caused by hyperperfusion from anomalous
arteries. However, FNH is not associated with oral contraceptives, and imaging will
show evidence of increased arterial flow and sometimes a central scar.
- HCC is characterized by a rapidly enlarging liver mass, often with satellite lesions.
- Hydatid cysts are caused by Echinococcus tapeworm infections. In the United
States, most cases are seen in immigrants, although occasional cases may be seen
in individuals from the Southwest who have been exposed to sheep and dogs. It
causes cystic, not solid, liver lesions.
- Most patients with metastatic liver disease will have a known extrahepatic primary
malignancy (eg, colon, pancreas). Although the liver lesions may occasionally be
found prior to the primary tumor, most patients are older, have systemic symptoms
(eg, weight loss), and often have multiple liver lesions.
- Acute or chronic liver injury can lead to nodular regeneration. This is usually
multifocal and occurs most commonly in the setting of cirrhosis.
HEPATIC
ADENOMA

- Often discovered incidentally


- Can cause life-threatening complications such as malignant transformation and
rupture. Rupture should be suspected in those with sudden-onset, severe right
upper quadrant pain who have signs of hemorrhagic shock; low-grade fever and
mild leukocytosis also sometimes occur due to peritoneal irritation from blood in
the abdominal cavity. US generally reveals a single, solid liver lesion (usually in the
right hepatic lobe) and free fluid (blood) in the abdomen. Urgent circulatory
support and surgical intervention are required to prevent death.
- Hepatic abscess is most common in the setting of bowel perforation or biliary
infection. Although patients often have fever and progressive right upper quadrant
pain
- US: a lesion with internal debris or septation (not a solid mass).
- Hepatic hemangioma is the most common benign liver lesion. It is generally
asymptomatic, but larger lesions can sometimes cause right upper quadrant pain.
Spontaneous rupture of this vascular lesion is very rare; therefore, a solid liver
lesion with surrounding free fluid is unlikely.
- HCC: Most cases present with progressive, subacute right upper quadrant
abdominal pain or decompensated cirrhosis.
- Hydatid cysts can occasionally rupture into the abdomen, leading to sudden-onset,
right upper quadrant pain, fever, and leukocytosis. However, ultrasound would
reveal a smooth, round cyst in the liver, not a solid mass.
HYDATID CYST Echinococcus granulosus is a dog tapeworm endemic to rural, developing
countries. Prevalence is particularly high in areas of sheep farming (sheep are
intermediate hosts), as dogs are often fed sheep viscera. Humans acquire the infection
incidentally after ingesting food or water contaminated by dog feces. Eggs hatch in
the small intestine, penetrate the intestinal wall, and travel to the liver (or, less
commonly, lung) where >1 hydatid cysts form.

Many patients remain asymptomatic for years and develop clinical illness only due to
large cyst size (>10 cm) or rupture. Manifestations include right upper quadrant pain,
nausea, vomiting, and hepatomegaly. Rupture may result in fever. Ultrasound has
excellent sensitivity (~95%) and typically shows a large, smooth hepatic cyst often
with daughter cysts (internal septations). Positive serology for E granulosus IgG
corroborates the diagnosis
(sensitivity ~95%) and usually
obviates the need for percutaneous
biopsy. Small cysts (<5 cm) are
typically treated with albendazole;
larger or complex cysts usually also
require percutaneous therapy or
surgery.
………

Eating contaminated pork may


result in a Taenia solium GI
tapeworm infection. Humans can
transmit tapeworm eggs to others,
causing
cysticercosis. Cysticercosis usually
affects the brain (seizures) or
cerebral ventricular system
(intracranial hypertension); liver
cysts are uncommon.

Entamoeba histolytica is often asx but may cause dysentery/colitis and liver
abscess. Amebic abscess usually manifests with fever, RUQ pain, and a smooth, cystic,
subcapsular mass. An afebrile pt that has a cystic lesion with daughter cysts makes E
granulosus more likely.
Undercooked fish may transmit liver flukes (eg, Clonorchis sinensis) or fish tapeworm
(eg, Diphyllobothrium latum). Liver flukes typically cause biliary disease (not hepatic
cysts). Fish tapeworm rarely causes symptoms but may cause megaloblastic anemia
due to vitamin B12 deficiency.

Prolonged and excessive alcohol intake increases the risk of HCC, which may cause
weight loss, abdominal pain, and complications related to rupture or tissue
compression. Imaging typically reveals a dense, irregular mass (not a cystic, smooth
mass).

Entamoeba
Entamoeba histolytica
histolytica
• Developing nations (travel/residence)
Risk factors • Contaminated food/water
• Fecal-oral, sexual transmission (rare)

• 90% of patients asymptomatic


• Colitis (diarrhea, bloody stool with mucus, abdominal pain)
Manifestations
• Liver abscess (RUQ pain, fever)
o Complications: rupture to pleura/peritoneum

• Stool ova & parasites, stool antigen testing (colitis)


Diagnosis
• E histolytica serology (liver abscess)

Treatment • Metronidazole & intraluminal antibiotic (eg, paromomycin)

RUQ = right upper quadrant.

Entamoeba histolytica is a protozoan common in developing countries and is


transmitted through consumption of contaminated food and water. Approximately
10% of patients have clinical manifestations of colitis or extraintestinal (liver, pleura,
brain) disease. Amebic liver abscessdevelops when E histolytica spreads from the
colonic mucosa to the liver via the portal vein.

Symptoms are subacute and include RUQ pain and fever, sometimes with recent or
concurrent bloody diarrhea (colitis). The pain is often dull and can be referred to the
right chest wall or shoulder. Systemic symptoms (eg, anorexia, weight loss, cough) can
occur, and hepatomegaly and elevations in leukocyte count, alkaline phosphatase, and
transaminases are common. As with other protozoal infections (eg,
giardiasis), peripheral eosinophilia is not a frequent finding (in contrast to many
helminthic infections, which can stimulate IL-5 production). A characteristic imaging
finding is a single, subcapsular, low-density lesion in the right lobe of the liver (due
to greater portal blood supply compared with the left). Diagnosis is made
with serology; needle aspiration is generally unnecessary.

………

Alcoholic hepatitis usually develops following years of heavy use.


Gonococcal perihepatitis (Fitz-Hugh–Curtis syndrome) occurs in ~10% of women with
pelvic inflammatory disease and is marked by RUQ abdominal pain, fever, pleurisy, and
mild elevations in aminotransferases.
Polymicrobial pyogenic (bacterial) abscesses share similar clinical and radiographic
appearances with amebic abscesses. However, pyogenic abscesses are associated with
jaundice and are more common in patients with diabetes or underlying hepatobiliary
or pancreatic disease. This patient's recent travel to an endemic E histolytica region
also makes amebic abscess more likely.

E granulosis can cause a hydatid liver cyst with symptoms of RUQ pain, nausea,
vomiting, and hepatomegaly.

ESOPHAGEAL
PERFORATION

- Esophageal instrumentation e.g., TEE, endoscope. Patients may develop severe


chest pain (and/or back pain due to the esophagus's posterior anatomical location)
and systemic findings (eg, fever) within hours (not days) of the procedure. A
minority have crepitus (subcutaneous emphysema) or crunching heard on
auscultation (Hamman sign). Chest x-ray may reveal findings consistent with
air/fluid leakage into the mediastinum (eg, pneumomediastinum, widened
mediastinum) or thorax (eg, pneumothorax, pleural effusion). Endoscopy is the
MCC of EP, with adjunctive procedures (eg, biopsy, stricture dilation) further
increasing risk. Esophagography is the best initial test. Elevated amylase in the
pleural fluid (due to leaked saliva) could increase suspicion for EP. However,
compared to contrast esophagography, pleural fluid analysis has less specificity for
the diagnosis (eg, pancreatitis-associated effusion also has elevated amylase) and is
unable to localize the tear in preparation for likely surgery.
- Effort rupture of the esophagus (Boerhaave syndrome) can occur with repeated
vomiting, particularly when the patient resists the vomiting reflex. Patients usually
have severe chest pain (and/or back pain given the esophagus's posterior location)
and a systemic inflammatory response (eg, fever, tachycardia). If esophageal
contents leak through the perforation into the pleural space, pleural effusion
results, usually on the left due to intrinsic weakness in the left posterolateral aspect
of the distal intrathoracic esophagus.
- Blunt thoracic trauma can rapidly increase intraesophageal pressure enough to
rupture the esophagus.
- EP may present with severe chest pain (easily missed if attributed to concurrent
chest injuries [eg, rib fractures]) and symptoms/signs related to leaked esophageal
air (eg, pneumothorax, pneumomediastinum, subcutaneous emphysema) or
gastrointestinal fluid (eg, pleural effusion). These manifestations may not be
immediately evident until content (eg, pleural fluid) accumulation increases. If
pleural fluid analysis is performed in the setting of EP, it typically shows low pH and
high salivary amylase and may show food particles. Esophagography or CT scan
with water-soluble oral contrast is the gold standard for confirming EP. Barium
contrast is more SN but can incite a granulomatous inflammatory response; it is
used when water-soluble contrast esophagography is inconclusive. Management
includes emergent surgical consultation, NPO, and IV abx and PPI administration.
Surgical debridement and repair remain the mainstay of treatment, although
select healthy patients (eg, limited leak, minimal symptoms, no signs of sepsis)
may receive a trial of medical management.
- Bronchopleural fistula can occur following chest trauma but is most common
following lung resection. The abnormal flow of inhaled air into the pleural space
would cause persistent air leak into the chest tube drainage system
- Visualization of contrast extravasating from the esophagus into surrounding tissues,
either by esophagography or CT scan with water-soluble contrast (barium is more
inflammatory), confirms the diagnosis. Intravenous, broad-spectrum antibiotics and
proton pump inhibitors should be administered, all oral intake restricted, and
emergent surgical consultation obtained. Most perforations will require operative
debridement and repair.
- Acute pancreatitis can cause a unilateral, left pleural effusion and fever in severe
cases. Associated pain may radiate to the back but typically originates in the
epigastrium, not the chest.
- Aortic dissection: Fever is uncommon.
- Self-induced emesis → risk for aspiration pneumonia
A patient has midline chest pain, fever, and a widened mediastinum in the setting of
protracted vomiting, which are most concerning for esophageal perforation (EP).

Effort rupture of the esophagus (Boerhaave syndrome) can occur with


repeated vomiting, particularly when the patient resists the vomiting reflex. The
perforation allows gastrointestinal contents to leak from the esophagus into the
mediastinum (causing widened mediastinum on CXR) and/or into the pleural space
(causing pleural effusion). The inflammatory gastrointestinal contents often
cause severe chest pain and a systemic inflammatory response (eg, fever,
tachycardia) that can quickly progress to septic shock and death.
Visualization of contrast extravasating from the esophagus into surrounding tissues,
either by esophagography or CT scan with water-soluble contrast (barium is more
inflammatory), confirms the diagnosis. Pleural fluid analysis is NOT required to
confirm EP, but if performed, typically shows low pH and very high amylase (>2500
IU/L from leaked saliva) and may show food particles. Surgical consultation should be
obtained as soon as the diagnosis is made because most perforations require
emergent operative debridement and repair.
……

Acute pancreatitis can cause a unilateral, LEFT-sided, exudative pleural effusion with
a high amylase concentration (due to high serum amylase levels) & high pH.
However, pancreatitis presents more classically as epigastric pain radiating to the
back rather than chest pain and would not cause mediastinal widening on CXR.

***Severe chest pain accompanied by a widened mediastinum and unilateral


pleural effusion (possible hemothorax) is concerning for aortic dissection in a patient
actively using cocaine (a risk factor), as in this patient with injected conjuctivae and
dilated pupils. However, aortic dissection usually has associated findings such as
pulse/BP variation (eg, >20 mm Hg blood pressure difference between the
extremities), and fever is uncommon.

The patient is at risk for aspiration from protracted vomiting. Although fever,
tachypnea, and pleural effusion (possible parapneumonic effusion) are concerning for
aspiration pneumonia, severe chest pain is not typical, and infiltrates would be
expected on chest x-ray. In addition, pleural fluid analysis of a parapneumonic effusion
would not be expected to show elevated amylase.

Severe chest pain in the setting of cocaine use (coronary vasospasm) is concerning for
MI. However, MI is less likely given the lack of ischemic changes on ECG; also, it would
not account for this patient's unilateral pleural effusion or widened mediastinum on
chest x-ray.

GALLSTONE - Gallstone ileus occurs when a gallstone passes through a biliary-enteric fistula into
ILEUS the small bowel. As the stone advances it may cause intermittent "tumbling"
obstruction with diffuse abdominal pain and vomiting until finally lodging in the
ileum, the narrowest section of the bowels, several days later.
- In addition to experiencing colicky pain and vomiting, patients may report
distension and inability to pass flatus or stool and show signs of hypovolemia (eg,
hypotension, tachycardia). Stones can occasionally also lodge in the stomach,
jejunum, or colon. Cholecystitis, which predisposes to biliary-enteric adhesions, is
the most important risk factor, and patients are more commonly elderly women,
which reflects their higher prevalence of gallstone disease.
- Diagnosis can be confirmed by abdominal CT scan, which may reveal gallbladder
wall thickening, pneumobilia, and an obstructing stone. Treatment is surgical and
involves removal of the stone and either simultaneous or delayed cholecystectomy.
………………………………………………
- Acute intestinal ischemia typically results in diminished bowel sounds and
frequently presents with severe abdominal pain despite a relatively normal physical
examination.
- Emphysematous cholecystitis (gallbladder wall infection due to gas-producing
organisms) is more commonly characterized by fever and right upper quadrant pain
and can also cause ileus (decreased or absent bowel sounds). This patient has
hyperactive bowel sounds.
GALLSTONES - In patients on total parenteral nutrition or prolonged fasting, the normal stimulus
for CCK release and gallbladder contraction is absent. This leads to biliary stasis
and promotes the formation of bile sludge and gallstones. Small-bowel (ileal)
resection also contributed to the formation of gallstones in this patient. Decreased
enterohepatic circulation of bile acids results in altered hepatic bile composition,
which becomes supersaturated with cholesterol and promotes gallstone formation.
- Estrogen-induced increase in
cholesterol secretion is the underlying
mechanism for the development of
cholesterol gallstones during pregnancy
and in women taking oral
contraceptives. During pregnancy,
estrogen causes an increase in
cholesterol secretion and progesterone
causes a reduction in bile acid
secretion, causing increased cholesterol
saturation of bile. Progesterone also
slows gallbladder emptying and thus
facilitates the formation of cholesterol
gallstones during pregnancy.
- Decreased enterohepatic recycling of
bile acids, as seen in patients with ileal
Crohn disease or ileal resection, can
predispose to cholesterol gallstones
due to increased concentration of
bilirubin conjugates and total calcium in
the gallbladder.
- Hypertriglyceridemia can predispose to gallstones in some patients.
- Increased red blood cell destruction (hemolytic anemia) increases the risk of
pigmented gallstones
…………………………………….
- Gallstones: Risk factors: HyperTG, obesity… The intra-gallbladder pressure increases
on gallbladder contraction against an obstructed cystic duct (either by stone or
sludge) and causes pain. Subsequent gallbladder relaxation allows the stone to fall
back from the duct with subsequent pain resolution. The pain is in the RUQ or
epigastric region, often constant (rather than colicky), and accompanied by
nausea, vomiting, and right-sided shoulder or subscapular discomfort (referred
pain). Patients may have recurrent symptoms that resolve between episodes.
Features that distinguish biliary colic from cholecystitis are pain resolution
within 4-6 hours and absence of abdominal tenderness, fever, and leukocytosis.
Acute cholecystitis typically presents with severe, unremitting RUQ pain and signs
of systemic illness (eg, fever, tachycardia, leukocytosis). LFTs typically remain
normal, and elevations should raise suspicion for biliary obstruction (eg,
choledocholithiasis, cholangitis). Characteristic ultrasound findings of cholecystitis
include gallbladder wall thickening, edema, and pericholecystic fluid. 85-90% of
gallstones are radiolucent
- Acute cholecystitis: Sudden RUQ pain after a fatty meal, may radiate to scapula,
murphy. Other classical findings include fever, vomiting, and
leukocytosis. Uncomplicated cholecystitis can cause mild elevations in
transaminases, total serum bilirubin from 1-4 mg/dL, and serum amylase without
obvious common bile duct or pancreatic disease. In these cases, the laboratory
elevations are usually due to passage of sludge or pus in the common bile
duct. However, alkaline phosphatase is usually not elevated without associated
cholangitis or choledocholithiasis. Complications: gangrene, rupture, hydrops,
abscess, cholangitis, chronic cholecystitis, porcelain GB. Although infection of the
gallbladder is present in 50-70% of acute cholecystitis cases, the original inciting
event is gallstone impaction in the cystic duct. Therefore, infection would not be
the primary mechanism for this patient's symptoms.
……..
- Epigastric discomfort is accompanied by heartburn secondary to gastric acid
hypersecretion and mucosal irritation. However, pain is usually described as a
burning sensation and does not radiate to the right shoulder. Dyspepsia provoked
by meals is often associated with belching and early satiety. Vomiting may occur
but is uncommon.
- Fat necrosis may accompany acute pancreatitis. Symptoms include nausea,
vomiting, and constant epigastric pain that radiates to the back. Pain usually does
not resolve spontaneously within a few hours.
- In acute cholecystitis, the gallbladder mucosa is inflamed. The cystic duct is
frequently obstructed with gallstones or sludge. Although the pain is similar to
biliary colic pain in location, radiation, and associated symptoms, it usually lasts
longer than 6 hours. Furthermore, patients frequently have fever, leukocytosis, and
tenderness to palpation in the midaxillary line under the right costal margin
(Murphy's sign).
- Pancreatic ca: progressive pain w/ weight loss
- Dyspepsia: burning pain (VS colicky)
…………………………………………..
- Only 20% of
patients with asx
gallstones will
develop
symptoms
within 15 years.
In light of this, no
treatment is
typically required
for such patients.
Exceptions to
this rule include
patients who are
at increased risk
for developing gallbladder carcinoma (eg, porcelain gallbladder).
- Acute cholecystitis: Palpation (or an ultrasound probe) under the costal margin at
the midclavicular line may elicit acute tenderness, especially during inspiration
(Murphy sign).
- Symptoms often subside within a few days with volume resuscitation, antibiotics,
and pain medications. However, early cholecystectomy (within 72 hours) reduces
disease duration, duration of hospitalization, and mortality when compared to
delayed cholecystectomy (>7 days after hospitalization). Laparoscopic
cholecystectomy is the surgical procedure of choice in patients without
contraindications. Early cholecystectomy is also advised for patients with other
complications of gallstones, such as gallstone pancreatitis. CT can also be used to
evaluate the biliary tree but is less sensitive than ultrasound.
- ERCP uses fluoroscopy. It’s indicated for a gallstone in the common biliary duct
(CBD) causing CBD dilation. Normal ALP makes CBD obstruction unlikely
- Lap chole is often performed in patients with biliary colic (intermittent RUQ pain)
or acute cholecystitis. Open cholecystectomy is performed in patients who can’t
tolerate a laparoscopic procedure or who have another intraabdominal process
requiring surgery.
- Porcelain gallbladder: asso. W/ chronic cholecystitis, asx or RUQ pain, might present
as a painless RUQ mass. Asso w/ adenoca in 2-5%, remove esp if symptomatic or
have incomplete mural calcification. It has a bluish color and brittle consistency
- Lithotripsy (laser or mechanical fragmentation) is sometimes used for gallstones
in the bile duct that CANNOT be removed endoscopically.
- Oral ursodeoxycholic acid supplementation reduces symptoms from gallstones
but does not lead to complete dissolution. It is used to treat symptomatic
gallstones in individuals who are poor surgical candidates, but would be of no
benefit for asx stones.
- Gallstone pancreatitis: gallstones are the second MCC of pancreatitis. Some
studies have shown that ALT >150 U/L has a 95% PPV for diagnosing gallstone
pancreatitis.
Once this patient's symptoms and labs have improved, attack of acute
pancreatitis is considered mild and resolved. However, the pt is at increased risk
for recurrent attacks of acute pancreatitis due to gallstones. Early
cholecystectomy is recommended for medically stable patients who recover
from acute pancreatitis and are surgical candidates. Cholecystectomy can
markedly reduce the risk of recurrent gallstone pancreatitis.
…………………………
- Antihypertensives commonly associated with drug-induced pancreatitis include
thiazides and angiotensin-converting enzyme inhibitors.
- ERCP is recommended in patients with gallstone pancreatitis who have:
• Cholangitis,
• Visible CBD dilation/obstruction, or
• Increasing liver enzyme levels.
ERCP allows for cannulation and sphincterotomy in an attempt to relieve the
obstruction.
- Repeat US in 4 weeks can be done in
patients with biliary colic symptoms who do
not have gallstones on initial US.
- Echinococcal cyst: unlikely in a patient
who has never traveled outside the United
States.

← Liver abscess is typically amebic or bacterial


in origin (not a complication of porcelain
gallbladder) and is caused by portal vein
bacteremia, systemic bacteremia, ascending
cholangitis, or trauma. Ultrasound
or CT scan is usually diagnostic.
Acalculous
Acalculous cholecystitis
cholecystitis
• Severe trauma or recent surgery
Risk factors • Prolonged fasting or TPN
• Critical illness (eg, sepsis, ICU)

• Fever, leukocytosis, ↑ LFTs, RUQ pain


Clinical presentation • Jaundice & RUQ mass less common

• Abdominal ultrasound (preferred)


Diagnosis • HIDA or CT scan if needed

• Enteric antibiotic coverage


Treatment • Cholecystostomy for initial drainage
• Cholecystectomy once clinically stable

HIDA = hepatobiliary iminodiacetic acid


Acalculous cholecystitis is most often seen in hospitalized patients who are critically
ill. Common predisposing conditions include recent surgery (particularly
cardiopulmonary, aortic, or abdominal), severe trauma, MOF, burns, extensive burns,
sepsis or shock, prolonged fasting or prolonged TPN, or critical illness requiring
mechanical intubation. These conditions likely cause gallbladder stasis or ischemia
with local inflammation that can lead to gallbladder distension, edema, necrosis, and
secondary bacterial infection.

Acalculous cholecystitis can present with unexplained fever and diffuse or RUQ
pain. Other findings can include jaundice, RUQ mass, leukocytosis, or abnormal LFTs
(normal results do not exclude the diagnosis). Therefore, the diagnosis requires a high
degree of suspicion. Complications include gangrene, perforation, and
emphysematous cholecystitis. Abdominal ultrasound is usually preferred for
dx. Patients with unclear ultrasound findings may need an abdominal CT scan or
cholescintigraphy (HIDA) scans, which are more SN and SP for the diagnosis.
- Most patients affected by this condition have no prior history of gallbladder
disease.
- Can lead to sepsis and death if undetected. The clinical signs of disease (eg,
fever, leukocytosis) are vague, and patients most vulnerable to this condition
are typically non-communicative due to their general medical condition. The
best way to make the diagnosis is a high degree of clinical suspicion and
confirmation with imaging studies. Radiologic signs include gallbladder wall
thickening and distension and the presence of pericholecystic fluid. The
immediate treatment in critically ill patients includes antibiotics followed by
percutaneous cholecystostomy under radiologic guidance. Cholecystectomy
with drainage of any associated abscesses is the definitive therapy once the
patient's medical condition improves (stabilized).
………………………………………………….
- Duodenal perforation usually presents with sudden-onset, diffuse abdominal pain.
The abdomen is rigid on initial examination (becomes distended later) with signs of
peritonitis.
- Mesenteric ischemia usually presents with sudden periumbilical abdominal pain
out of proportion to examination findings. Risk factors include older age, a-fib,
CHF, and atherosclerotic vascular disease. CT of the abdomen typically shows
focal or segmental bowel wall thickening, small-bowel dilation, and
mesenteric stranding.
- CT findings of pancreatitis include parenchymal enhancement with intravenous
contrast (in patients with no pancreatic necrosis), pseudocyst formation, or
peripancreatic fluid collection.
- Acute pancreatitis can occur postoperatively in patients with hypotension (similar
to acalculous cholecystitis). However, acute pancreatitis is less likely due to this
patient's RUQ, rather than epigastric, tenderness and normal amylase levels.
- Subphrenic abscess (rare) can cause fever and abdominal pain. However, it usually
develops due to peritonitis (eg, perforated ulcer, appendicitis, abdominal surgery)
and is unlikely in this patient who did not undergo an abdominal procedure.
EMPHYSEMATOU
S CHOLECYSTITIS

- Predisposing factors include relative immunosuppression (eg, age >50, diabetes


mellitus) and vascular disease (eg, compromised cystic artery blood supply,
atherosclerosis).
- Complications include gangrene and perforation, the latter of which may
transiently relieve pain but subsequently result in peritoneal signs.
- Diagnosis is confirmed with imaging
demonstrating air-fluid levels in the
gallbladder (as gas leaks into it), gas in the
gallbladder wall or surrownding structures,
and occasionally pneumobilia (air within the
hepatobiliary system). However, in patients
with emphysematous cholecystitis, unlike
other forms of acute cholecystitis, US is less
SN due to the modality's poor visualization of
air-filled structures; extensive gas in the gallbladder
wall may be falsely interpreted as failure to identify
the gallbladder due to overlying bowel loops. Pts
present w/ fever, RUQ pain and guarding, RUQ
crepitus…etc. Laboratory findings include mild to
moderate unconjugated hyperbilirubinemia (eg,
from Clostridium-induced hemolysis) and a small
elevation in aminotransferases. Mgmt requires
emergency cholecystectomy, broad-spectrum IV
antibiotic therapy (eg, piperacillin-tazobactam.
Ertapenem) and bile cx. Untreated → gangrenous
cholecystitis and perforation.
- Biliary-enteric fistulas are extremely rare. They cause pneumobilia but would not
be expected to cause intramural gas and are not typically associated with fevers
or sepsis. Large bowel fistulas are characterized by bile acid diarrhea whereas
fistulas with the small bowel are often asx but may cause gallstone ileus, which
presents with symptoms of intermittent bowel obstruction (eg, N, diffuse
abdominal pain) over several days as the gallstone moves through the intestines.
- Acute colangitis → ERCP, if failed → Percutaneous drainage.
POSTCHOLECYST - PCS refers to persistent abdominal pain or dyspepsia (eg, N) that occurs either
ECTOMY postoperatively (early) or months to years (late) after a cholecystectomy. PCS
SYNDROME (PCS) can be due to biliary (eg, retained CBD or cystic duct stone, biliary dyskinesia) or
/////////////////// extra-biliary (eg, pancreatitis, PUD, CAD) causes. Patients usually notice the
**** same pain they had prior to surgery, new pain just after surgery, or the same
pain that never went away.
- Laboratory findings can include elevated ALP, mildly abnormal LFTs, and dilated
CBD on abdominal US. These findings usually suggest CBD stones or biliary SOD.
The next step involves EUS, ERCP, or MRCP for final diagnosis and guiding
therapy. Treatment for PCS is directed at the causative factor.
GILBERT
SYNDROME

A patient with mild scleral icterus following


appendectomy has elevated indirect (unconjugated) bilirubin levels (given his elevated
total bilirubin level of 3.3 mg/dL without direct bilirubin predominance) with normal
LFT results and no evidence of hemolysis (given the normal hemoglobin level). This
presentation is consistent with Gilbert syndrome, the most common inherited
disorder of bilirubin glucuronidation.
Gilbert syndrome is characterized by recurrent episodes of mild jaundice due
to decreased activity of the UDP-glucuronosyltransferase enzyme, responsible
for bilirubin conjugation in the liver. These episodes may be precipitated
by stressors (eg, infection, fasting, vigorous exercise, surgery). Gilbert syndrome is
diagnosed by elevated levels of unconjugated bilirubin on repeat testing with normal
liver function test results and CBC results. No specific treatment is indicated, but it is
important to educate patients about the benign nature of the condition and its
inheritance pattern to prevent unnecessary diagnostic studies.
……..

Acalculous cholecystitis is an inflammatory condition of the gallbladder most


commonly seen in critically ill, hospitalized patients. Laboratory results are nonspecific
but often show a leukocytosis along with hyperbilirubinemia and sometimes mild
elevation in ALP and aminotransferase levels.

Patients with acute viral hepatitis present with anorexia, nausea, and
vomiting. Aminotransferase levels are extremely elevated (eg, >25x upper limits of
normal).

Halogenated inhalational anesthetics (eg, halothane) are associated with severe


hepatotoxicity. Due to this concern, halothane use is not recommended in
adults. There are 2 types of halothane hepatotoxicity: a mild, self-limiting form
characterized by mild elevations in aminotransferase levels, and a severe form that
causes liver necrosis, fever, and jaundice with grossly elevated aminotransferase
levels.

Iatrogenic biliary injury occurs most commonly following laparoscopic cholecystectomy


but would be unlikely following an appendectomy. Manifestations include jaundice,
fever, and epigastric pain.

MALIGNANT
BILIARY
OBSTRUCTION

-
- This patient has predominantly conjugated (direct)
hyperbilirubinemia. Predominantly unconjugated hyperbilirubinemia is due to
increased bilirubin production (eg, hemolysis), decreased bilirubin uptake by the
liver (eg, portosystemic shunt), or abnormal bilirubin conjugation (eg, Gilbert's
syndrome). Predominant conjugated hyperbilirubinemia is due to hepatocellular
injury, decreased bilirubin excretion in bile canaliculi (eg, Dubin-Johnson
syndrome), intrahepatic cholestasis (eg, primary biliary cirrhosis), or extrahepatic
cholestasis from biliary obstruction (eg, malignancy).
Malignant biliary obstruction

• Cholangiocarcinoma
Etiologies • Pancreatic/hepatocellular carcinoma
• Metastasis (eg, colon, gastric)
• Jaundice, pruritus, acholic stools, dark urine
• Weight loss
Manifestations • RUQ pain
• RUQ mass or hepatomegaly
• ↑ Direct bilirubin, ALP, GGT

• Serum tumor markers (CEA, CA-19, AFP)


Diagnosis • Abdominal imaging (ultrasound, CT scan)
• EUS or ERCP for tissue diagnosis if unclear

This patient's conjugated hyperbilirubinemia, elevated ALP, painless jaundice, and


systemic symptoms (fatigue, weight loss) suggest malignant obstruction of the biliary
system. Likely etiologies include pancreatic adenocarcinoma or
cholangiocarcinoma. The next step in this patient's evaluation should be abdominal
imaging (ultrasound or computed tomography), with consideration for endoscopic
retrograde cholangiopancreatography if initial imaging is nondiagnostic.
- Chronic autoimmune hepatitis usually causes elevated aminotransferases and
conjugated hyperbilirubinemia. However, it is usually not associated with
significantly elevated alkaline phosphatase.
- Chronic pancreatitis typically presents with recurrent bouts of abdominal pain
and/or symptoms of fat malabsorption with steatorrhea. Liver function tests are
usually normal in chronic pancreatitis. Recurrent episodes of pancreatitis can lead
to fibrosis, stricture of the intrapancreatic portion of the bile duct, and elevated
bilirubin and alkaline phosphatase. However, this usually develops later in patients
with known chronic pancreatitis.
- Primary biliary cholangitis (PBC) may present with fatigue, itching, jaundice, weight
loss, and elevated alkaline phosphatase levels. However, antimitochondrial
antibodies would be positive.
HEMOBILIA A patient who has melena, right upper quadrant (RUQ) pain, jaundice, anemia, and
hyperbilirubinemia after a recent liver biopsy likely has bleeding into the biliary tract,
known as hemobilia. Hemobilia is a rare cause of upper gastrointestinal
bleeding (UGIB) that usually occurs as a complication of hepatic or biliopancreatic
interventions (eg, liver biopsy, cholecystectomy, ERCP); other etiologies include
hepatobiliary tumors and BAT.

Massive hemobilia results in hemodynamic instability and UGIB that occurs


immediately after the procedure;
Intraductal hematoma formation more commonly results in delayed UGIB until after
dissolution of the clot (usually ~5 days post-procedure).

Patients typically present with RUQ pain, jaundice, and UGIB (eg, melena,
hematemesis). Laboratory findings include direct hyperbilirubinemia from bile duct
obstruction and anemia; leukocytosis can occur due to inflammation. Abdominal
imaging (eg, CT scan, ultrasound) can identify hematomas and intraabdominal free
fluid, and endoscopic evaluation may reveal oozing of blood from the papilla.
Hemobilia is usually self-limited and often managed conservatively (eg, IV fluids,
blood transfusions); however, angiography with embolization or surgery may be
required for persistent bleeding.
………..

Aortoenteric fistula causes massive gastrointestinal bleeding and usually results from
a ruptured AAA or after aortic surgery; it is not a typical complication of liver
biopsy. In addition, hyperbilirubinemia and jaundice are unexpected.

Liver abscesses can form after liver biopsy and present with RUQ pain, leukocytosis,
and possibly hyperbilirubinemia when they compress the biliary tree; however, fevers
are typical, and UGIB would not occur.

A perforated duodenal ulcer can cause abdominal pain and melena, but the pain is
usually generalized and severe (rather than focal RUQ pain) and associated with fever,
abdominal distension, rebound, and guarding. Jaundice and hyperbilirubinemia are
unexpected.
BILIARY ATRESIA
Biliary atresia

Pathogenesis • Extrahepatic bile duct fibrosis

• Asymptomatic at birth
Clinical • Infants age 2-8 weeks:
findings o Jaundice, acholic stools, dark urine
o Hepatomegaly

• Direct hyperbilirubinemia
• Ultrasound:
o Absent/abnormal gallbladder &/or CBD
• Liver biopsy:
Diagnostic
o Intrahepatic bile duct proliferation
evaluation o Portal tract edema
o Fibrosis
• Intraoperative cholangiography (gold standard):
o Biliary obstruction

• Surgical hepatoportoenterostomy (Kasai procedure)


Treatment • Liver transplant

CBD = common bile duct.


Biliary atresia is a neonatal
disorder characterized by
progressive fibrosis and
obliteration of extrahepatic
bile ducts. Conjugated
bilirubin is produced by
hepatocytes but cannot be
transported through the
fibrotic ducts, leading to
cholestasis. Affected infants
usually develop jaundice
and hepatomegaly at age
2-8 weeks. Decreased bile
excretion into the intestines also leads to acholic stools (eg, pale, clay-
colored). Untreated disease can lead to splenomegaly due to progressive cirrhosis
and portal hypertension.

Laboratory evaluation reveals elevated total and direct bilirubin levels. Bile duct
destruction leads to elevated GGT and ALP; aminotransferases (ATs) are usually
normal or mildly elevated. Reticulocyte count is normal due to the absence of
hemolysis.

TAUS may reveal the triangular cord sign (fibrous remnants above the porta hepatis)
or a small or absent gallbladder, findings highly suggestive of biliary atresia.

Definitive diagnosis requires intraoperative cholangiography or liver biopsy.

If biliary atresia is confirmed, hepatoportoenterostomy (Kasai procedure) is


performed to reestablish bile flow to the small intestine. However, most children with
biliary atresia eventually require a liver transplant.

………..

Hemophagocytic lymphohistiocytosis (HLH) is a systemic inflammatory condition


that can present with elevated direct bilirubin and GGT (due to cholestasis) and with
cytopenias and decreased reticulocytes (due to BM failure). In contrast to a patient
with biliary atresia, infants with HLH have a normal gallbladder and are typically febrile
and ill-appearing.

Certain rare, genetic causes of cholestasis, such as progressive familial intrahepatic


cholestasis, cause direct hyperbilirubinemia with paradoxically normal
GGT. Although the presentation is similar to biliary atresia (eg, jaundice, acholic
stools), absence of the gallbladder is highly suggestive of biliary atresia.
ACUTE - This patient with a history of alcohol use and a recent episode of pancreatitis now
PANCREATITIS has progressive abdominal distension, N, and V,
with a CT scan showing a round, well-
circumscribed, encapsulated fluid collection in
the pancreatic bed, consistent with a pancreatic
pseudocyst. Pseudocysts are mature walled-off
pancreatic fluid collections (usually no necrosis
or solid material) surrounded by a thick fibrous
capsule and containing enzyme-rich fluid, tissue,
and debris. They can leak amylase-rich fluid into
the circulation and increase serum amylase.
Complications include spontaneous infection,
duodenal or biliary obstruction,
pseudoaneurysm (due to digestion of adjacent
vessels), pancreatic ascites, and pleural effusion. Abdominal imaging usually
confirms the diagnosis.
- In patients with minimal or no symptoms and without complications (eg,
pseudoaneurysm), expectant management (eg, symptomatic therapy, NPO) is
preferred initially. Endoscopic drainage is typically reserved for those with
significant symptoms (eg, abdominal pain, V), infected pseudocyst, or evidence
of pseudoaneurysm (usually embolized before drainage procedure).
…………………………………………..
- Cholecystectomy is performed (usually following ERCP) in patients who have
recovered from gallstone pancreatitis, which typically presents with elevated LFTs,
amylase, lipase, and (often) serum bilirubin. A patient w/ normal serum chemistry
makes gallstone pancreatitis unlikely.
…………………………………………………..
- Gallstone pancreatitis complicated by acute cholangitis: elevated serum lipase or
amylase, cholestatic LFTs. If the stone remains in the biliary tract, the resultant bile
stasis can allow bacteria to ascend from the duodenum, leading to acute
cholangitis. In addition to cholestatic liver function studies (due to stasis and
obstruction), the presence of Charcot triad (right upper quadrant pain, jaundice,
fevers) suggests the diagnosis, and altered mental status and hypotension (ie,
Reynolds pentad) indicate particularly severe disease. Common bile duct dilation
or choledocholithiasis on a right upper quadrant ultrasound further support the
diagnosis.
- Management includes aggressive IV hydration and antibiotics. ERCP is indicated
to relieve the biliary obstruction, during which a sphincterotomy, stone extraction,
and/or biliary stent placement can be performed. Failure to remove the obstructive
gallstone may result in septic shock and death.
- Although a CT scan can help diagnose pancreatitis, the combination of acute
abdominal pain and elevated serum lipase is sufficient for diagnosis. In addition, a
CT scan is less sensitive than an ultrasound for detecting changes associated
with cholangitis (eg, common bile duct dilation) and may miss gallstones, which
are frequently radiolucent.
- A HIDA scan is used to diagnose cholecystitis in patients with equivocal US findings;
however, in addition to fevers and RUQ pain, cholecystitis results in gallbladder wall
thickening and edema visible on US, whereas biliary dilation suggests cholangitis.
An elective cholecystectomy to prevent further complications of gallstone disease
should be deferred until after the current complications have resolved.

A patient has pancreatitis given her epigastric abdominal pain radiating to the back
and the amylase/lipase levels >3 times the upper limit of normal (ULN). Gallstone
pancreatitis is most likely as she does not use alcohol, takes no medications, has
normal TGs and calcium levels, and has had no recent major illness. Her high
BMI, ALT >150 U/L, and elevated ALP also suggest gallstone pancreatitis.

A RUQ US is advised for all patients with suspected gallstone pancreatitis as it provides
the most accurate information regarding the presence of gallstones. If the US
is nondiagnostic and there is high clinical suspicion for common bile duct disease,
ERCP may be performed to better visualize the biliary tree.

………

CT scan is not required to diagnose pancreatitis in patients with typical abdominal


pain and amylase/lipase levels >3 times ULN. It can be used to confirm the diagnosis
and identify complications (eg, pancreatic hemorrhage or necrosis) in patients without
these classic findings (eg, atypical abdominal symptoms, amylase/lipase elevation <3 x
ULN). However, CT is not as SN as ultrasound for detecting gallstones. Plain
radiographs also have a very low sensitivity for gallstones.
Viruses (eg, hepatitis B, cytomegalovirus, varicella-zoster, HIV) are a less likely
cause of pancreatitis in this patient. Serologic studies can be deferred unless standard
testing does not identify the cause.
Stool elastase is a marker for pancreatic exocrine function. Low levels would be
seen in chronic, rather than acute, pancreatitis.

DIVRTICULOSIS/D - Diverticulosis is characterized by


IVERTICULITIS herniation of the colonic
mucosa/submucosa through the
circular and longitudinal muscle layer
due to elevated intraluminal pressure.
Risk increases with age and is greatest
in those with obesity, poor diet (eg, low
dietary fiber, high red meat/fat), and
tobacco use. Although diverticulosis is usually asymptomatic, approximately 5%-
15% of patients develop diverticular bleeding or diverticulitis.
- In diverticulitis, elevated luminal
pressure or inspissated food particles cause
erosion of the diverticular wall, leading to
micro- or macroperforation and
subsequent colonic wall inflammation. Most
cases arise in the sigmoid colon (the site of
the greatest intraluminal pressure) and
present with the following:
• LLQ pain
• N and V
• Bladder symptoms (eg, urgency,
frequency, dysuria) or sterile
pyuria (eg, positive leukocyte esterase, negative nitrite/bacteria) due to
bladder irritation from adjacent sigmoid colon inflammation
• Alteration in bowel habits (eg, constipation, diarrhea)
- A minority of patients (<20%) develop rebound tenderness/guarding or a tender
left lower quadrant mass. Diagnosis is usually made with imaging (eg, CT scan of
the abdomen); antibiotic therapy is generally curative, but complications (eg,
abscess, bowel obstruction, frank perforation, fistula) can occur and usually
necessitate further therapy.
- Some (10%-15%) have urinary urgency, frequency, or dysuria due to bladder
irritation from an inflamed sigmoid colon. Abdominal CT scan (oral and
intravenous contrast) is the best diagnostic test for diverticulitis,differentiating it
from other diseases (eg, colon cancer, kidney infection). Findings suggestive of
diverticulitis include increased inflammation in pericolic fat, presence of diverticula,
bowel wall thickening, soft tissue masses (eg, phlegmons), and pericolic fluid
collections suggesting abscess.
- Diverticulitis can be classified as uncomplicated (75%) or complicated
(25%). Uncomplicated diverticulitis in stable patients can be managed in the
outpatient setting with bowel rest, PO antibiotics, and observation. However,
hospitalization and IV antibiotics are recommended for patients who are:
1. Elderly,
2. Immunosuppressed,
3. Have high fever or significant leukocytosis, or
4. Have significant comorbidities.
- Complicated diverticulitis refers to diverticulitis associated with:
1. Abscess,
2. Perforation,
3. Obstruction, or
4. Fistula formation.
A fluid collection <3 cm can be treated with IV antibiotics and
observation, with surgery reserved for patients with worsening
symptoms.

However, a fluid collection >3 cm should have CT-guided percutaneous


drainage. If the symptoms are not controlled by the fifth
day, surgical drainage and debridement are recommended. Sigmoid
resection is generally reserved for patients with:
1. Fistulas,
2. Perforation with peritonitis,
3. Obstruction, or
4. Recurrent attacks of diverticulitis.
………………………
- IV antibiotics are typically given for an infected pseudocyst. The absence of
fever and leukocyte count elevation in this patient makes infection less likely.
- Ovarian torsion usually causes pelvic pain, N/V, and (occasionally) fever.
However, the pain is often acute and severe, and most patients have a palpable
adnexal mass.
- Ruptured ovarian cysts most often present with severe, sudden-onset
abdominal pain following sexual intercourse. The vast majority of cases occur in
females of reproductive age.
- SBO usually occurs in individuals who have had a previous abdominal or pelvic
surgery and developed postoperative adhesions or hernia. Although N/V and
low-grade fever can occur, patients typically have paroxysms of periumbilical
crampy abdominal pain (not dull, persistent, LLQ pain) and often cannot pass
flatus. In addition, high-pitched bowel sounds are often present.
………………………………………………..
Diverticulitis s/s eg, vague lower abdominal discomfort, anorexia, constipation.
Patients with free perforation of the gastrointestinal tract in the setting of ongoing
inflammation (eg, diverticulitis) often have a classic pain sequence:
• Moment of perforation: sudden, severe pain (± vomiting, lightheadedness or
syncope)
• After perforation to 2 hours: temporary relief or decreased pain as the
inflamed organ decompresses
• >2 hours after perforation: generalized, constant pain due to peritonitis (±
sepsis/septic shock)
Unlike diverticular microperforation (ie, contained perforation), which usually responds
to medical management (eg, antibiotics, bowel rest), free perforation is a surgical
emergency that requires abdominal washout and resection of the ruptured bowel
segment.
………………………………………….
- Colonic dilatation with loss of haustral markings may be seen in toxic megacolon.
Toxic megacolon can cause fever, abdominal pain, and peritonitis. However,
bloody diarrhea is typically present, as is a history of inflammatory bowel disease or
recent antibiotic therapy (leading to Clostridioides [formerly Clostridium] difficile).
- Dilated small bowel with a transition point is seen in small bowel obstruction. This
condition more commonly causes crampy (vs sudden and severe) pain of the
midabdomen and increased, high-pitched (vs diminished) bowel sounds without
peritonitis, unless perforation has occurred (causing free air on imaging).
- Embolic occlusion of a mesenteric artery causes acute-onset, severe abdominal pain
(acute mesenteric ischemia). Such pain is typically out of proportion to a relatively
benign abdominal examination, whereas this patient has guarding and rebound
tenderness proportionate to his severe pain. In addition, a predisposing risk factor
for embolism (eg, atrial fibrillation) is usually present.
- Barium contrast enema can detect strictures or inflammation in ulcerative colitis or
Crohn disease. It is contraindicated in the setting of diverticulitis until perforation
has been ruled out.
- Sigmoidoscopy or colonoscopy is contraindicated in the setting of acute
diverticulitis as it may cause perforation; colonoscopy can be done to exclude
underlying malignancy following resolution of the diverticulitis.
- Small bowel follow-through is performed by giving oral contrast to the patient and
using fluoroscopy to follow the contrast into the small bowel. The test is usually
indicated for diagnosing small bowel pathology (eg, stricture, obstruction, masses)
and is not as useful for diagnosing diverticulitis in the colon.
- Upright abdominal film can reveal nonspecific findings sometimes associated with
acute diverticulitis (eg, dilated small/large bowel with air-fluid levels due to ileus or
obstruction, soft tissue densities due to abscess). It can also occasionally reveal air
under the diaphragm or in the retroperitoneum (perforation). However, abdominal
CT is more sensitive (~94%) as well as specific (~99%) for diagnosing diverticulitis
and differentiating it from other causes of abdominal pain.

PSOAS ABSCESS
Psoas abscess
• Subacute fever & abdominal/flank pain that may radiate to the groin or
Clinical hip
presentation • Anorexia, weight loss
• Abdominal pain with hip extension (psoas sign)

• CTAP
Diagnosis • Leukocytosis, elevated inflammatory markers
• Blood & abscess cultures

• Drainage
Treatment • Broad-spectrum antibiotics

A patient with a recent history of skin infection who presents with fever and
abdominal pain radiating to the groin likely has a psoas abscess (PA). PA occurs
from either hematologic seeding from a distant infection or from direct extension of
an intraabdominal infection (eg, diverticulitis, vertebral osteomyelitis). Risk factors
include HIV, IVDU, diabetes, and Crohn disease.

PA frequently presents subacutely with fever and lower abdominal or flank pain,
although symptoms (eg, anorexia, weight loss) can be nonspecific. Consequently, PA
should be considered as part of the evaluation for fever of unknown origin. Deep
abdominal palpation is required to elicit tenderness due to the location of the psoas on
the posterior abdominal wall. The "psoas sign," abdominal pain with hip extension,
can often be detected on examination. Laboratory studies commonly show
leukocytosis, thrombocytosis, and elevated inflammatory markers. CT scans are
required to confirm the diagnosis. Drainage is critical, and blood and abscess cultures
should be obtained to guide antibiotic therapy.
………

USG typically has poor SN for PA due to abscess location deep in the pelvis, overlying
bowel gas, and pelvic bones.
AXR are helpful in identifying bowel obstructions, free air, foreign bodies,
and renal calculi, but have much lower sensitivity in identifying PA.

This patient had clear risk factors for PA formation (T1DM, recent furunculosis). In
patients with unexplained PA, colonoscopy may help identify a source for the
abscess.

Retrocecal appendicitis can cause a positive psoas sign, and in the appropriate
setting patients may proceed directly to surgery. However, compared to PA, retrocecal
appendicitis is more likely to cause pain on rectal examination and less likely to
cause RLQ pain. In addition, the patient's subacute presentation and history of recent
soft tissue infection increase the diagnostic uncertainty, making a CT scan the best
next step.
MALIGNANT
BILIARY
OBSTRUCTION

- Cholangiocarcinoma: Risk is greatest in those with fibropolycystic liver disease or


primary sclerosing cholangitis, an inflammatory disorder of the biliary tree that is
almost always (~90%) associated with underlying UC. Elevated CEA and CA 19-9
levels raise strong suspicion.
Patients with cholangiocarcinoma generally come to clinical attention late in the
disease course when the tumor obstructs biliary drainage, which leads to:
• Abdominal pain
• Weight loss
• Hyperbilirubinemia (jaundice, dark urine, pruritus)
• Cholestatic liver enzyme pattern (elevated alkaline phosphatase,
normal/mildly elevated transaminases)
• Hepatomegaly/palpable right upper quadrant mass
CEA, CA 19-9, and alpha-fetoprotein (AFP) can help differentiate
cholangiocarcinoma (↑ CEA/CA 19-9, normal AFP) from hepatocellular carcinoma
(normal CEA and CA-19, sometimes ↑ AFP). Abdominal imaging typically shows
intrahepatic or common bile duct dilation (depending on the level of the lesion)
and biliary mass. Endoscopic ultrasound or endoscopic retrograde
cholangiopancreatography is then required for tissue diagnosis.
- Choledocholithiasis (gallstones in the common bile duct) usually presents with
acute right upper quadrant pain, nausea/vomiting, and a cholestatic liver enzyme
pattern. However, CEA and CA-19 are not typically elevated, and cachexia is
unusual.
- HCC: CEA and CA 19-9 are rarely elevated, but AFP is elevated in ~50% of cases.
- Colon cancer often metastasizes to the liver and is frequently associated with CEA
and/or CA 19-9 elevation. However, metastatic disease to the liver usually causes
multiple liver masses.
- Although pancreatic cancer often increases CA 19-9, imaging generally shows
common bile duct dilation (the obstruction occurs in the distal biliary tree).
HEPATOCELLULA - AFP is produced by the fetal liver and yolk sac, → elevated in primary testicular
R CARCINOMA (yolk sac) or HCC. A patient w/ normal testicular examination, older age, and
(HCC) emigration from endemic areas make HCC most likely.
- HCC Incidence is greatest in Asia (eg, Vietnam), Africa, and the Middle East due
to high rates of underlying HBV/HCV infection. In these regions, viral transmission
often occurs via the maternal-to-child route, leading lifelong viral hepatitis in the
newborn (and increased risk of cirrhosis/HCC later in life).
- Patients with HCC often develop systemic (eg, weight loss, fatigue), abdominal
(eg, right upper quadrant pain, early satiety), or metastatic (eg, bone pain)
manifestations. LFTs may be normal or elevated; ALP is occasionally dramatically
elevated due to osteoblastic bone metastases (can be mixed blastic/lytic mets).
Significant elevation in AFP raises strong suspicion for HCC but is present only in
~50% of cases.
…………………………………………
- Although multiple myeloma and renal cell cancer often spread to the spine, they
usually cause purely osteolytic (not mixed osteolytic/osteoblastic) lesions.
Therefore, alkaline phosphatase is usually normal. AFP elevations are rare.
- Although nonseminomatous germ cell tumors (that contain yolk sac cells) are often
associated with elevated AFP and frequently spread to bone, most testicular
cancers occur in males age 15-35 and are marked by unilateral testicular swelling or
nodules.
A patient's ascites (shifting abdominal dullness), hypoalbuminemia, mildly elevated liver
function tests, thrombocytopenia, and hyperbilirubinemia are consistent with
underlying cirrhosis. Most cases are caused by alcohol abuse, chronic viral hepatitis,
or nonalcoholic fatty liver disease (NAFLD; as is likely in this patient with a history of
DM and obesity).

Patients with cirrhosis are often unaware of the diagnosis until they develop
complications such as variceal bleeding or hepatocellular carcinoma (HCC), a primary
liver tumor. Risk of HCC in patients with cirrhosis is 1%-8% per year. In these patients,
HCC usually presents with decompensated liver failure (eg, ascites, jaundice,
hypoalbuminemia), weight loss, cachexia (eg, temporal wasting), possible
hepatomegaly, and a palpable liver nodule.

AFP, a glycoprotein produced by the fetal liver and yolk sac, is elevated in ~50% of
cases; therefore, AFP can serve as an important diagnostic clue (when elevated)
but cannot be used to rule out the diagnosis. Triple phase arterial contrast CT scan
of the abdomen is diagnostic in most cases.
……….

Focal nodular hyperplasia (FNH) is a benign liver lesion that usually arises in young
women. Although a minority develop a palpable liver nodule, most are asymptomatic;
the lesion does not usually rupture, undergo malignant transformation, cause liver
failure, or result in weight loss/cachexia.
Hepatic adenoma is a benign liver lesion common in young women on oral
contraception. Some lesions undergo malignant transformation (to HCC), which may
result in cachexia, weight loss, and anorexia. However, decompensated liver failure is
uncommon (because the rest of the liver has preserved function).
Hydatid cyst is an Echinococcus tapeworm infection. Most cases are asymptomatic,
but large hydatid cysts can cause hepatomegaly, RUQ pain, or nausea/vomiting.

Pyogenic liver abscess is most common in the setting of biliary infection or bowel
perforation. Although weight loss, anorexia, fatigue,transaminitis, hepatomegaly, and
hyperbilirubinemia commonly occur, almost all patients have fever, abdominal pain,
and leukocytosis.

A patient with alcohol-induced cirrhosis now has abdominal swelling, abdominal


distension, and shifting dullness on percussion, indicating ascites. Although ascites
often develops as a consequence of progressive cirrhosis, new-onset ascites may
indicate acute obstruction of the portal or hepatic veins due to thrombus
or hepatocellular carcinoma (HCC). Therefore, abdominal US is required to evaluate
the abdominal cavity (for free fluid), the portal/hepatic vascular system (for clots), and
the liver (for masses).
Patients with cirrhosis are at extremely high risk for developing HCC (risk is 1%-8% per
annum). New-onset ascites and systemic manifestations (eg, weakness, weight loss,
temporal wasting) should raise suspicion for HCC, as should any sign of liver
decompensation (eg, variceal bleeding). However, patients with HCC are often
asymptomatic. Therefore, in any patient with cirrhosis, SCREENING abdominal US is
recommended every 6 months to evaluate for new-onset HCC. This patient who had
a negative screening ultrasound 18 months ago may have developed HCC in the
interim.

……..
Primary biliary cholangitis (PBC) is associated with antimitochondrial antibodies and
is characterized by fatigue, jaundice, pruritis, and elevated ALP. Patients with PBC
who develop cirrhosis have an increased risk of HCC. However, PBC is rare and occurs
almost exclusively in middle-aged women.
The mainstay of therapy for ascites is diuresis and salt restriction; however, new-onset
ascites requires ultrasound prior to considering therapy.

Patients with congestive heart failure can develop ascites and lower extremity edema
but typically have elevated jugular venous pressure. This patient's history of cirrhosis
and lack of elevated JVP make his ascites most likely due to a hepatic cause;
therefore, abdominal ultrasound, not echocardiography, is recommended.

Lactulose is used to treat hepatic encephalopathy, which is associated


with confusion and asterixis. This patient is alert and oriented; he has no evidence of
encephalopathy.
FOCAL NODULAR - FNH is a benign regenerative liver nodule common in women age 20-50.
HYPERPLASIA - Most cases of FNH are asymptomatic and are discovered during abdominal
(FNH) imaging for other conditions. Lesions tend to be well-circumscribed, solitary, and
<5 cm in size; they characteristically have a central, stellate scar, which surrounds a
large congenital arterial anomaly
that sends arterial branches to
the periphery. Imaging with
triphasic, helical CT scan (which
evaluates the mass during
different phases of vascular
contrast) generally reveals a
hyperdense lesion (ie, filled with
contrast during hepatic arterial
phase) and a central scar.
- Because FNH does not
generally grow over time,
undergo malignant
transformation, or rupture,
treatment is rarely required. Oral contraception is not believed to worsen FNH;
therefore, contraception usually can be continued (in contrast to hepatic adenoma).
- Hepatic hemangioma, the most common benign liver tumor, is often found
incidentally during imaging for other conditions. However, triphasic CT scan will
reveal centripetal enhancement (ie, enhancement moving from periphery to
center) and no central scar.
- Hepatic adenomas: centripetal enhancement is usually seen on triphasic CT scan,
and no central scar would be present.
- HCC enhances w/ contrast.
METASTATIC - GI malignancies, such as
LIVER DISEASE colorectal (MC source;
should be r/u even if no
suggestive s/s) or
pancreatic cancer, are the
most frequent source of
liver metastases as their
venous drainage is through
the portal system directly to
the liver. Lung, breast, and
skin cancers (melanoma)
often also spread to the
liver. The liver is a common
site of metastatic disease
due to its dual blood supply (systemic and portal) and hepatic sinusoidal
fenestrations allowing for easier metastatic deposition.
- Liver cancer. Colorectal cancer is the MC source of liver metastases (as blood from
the colon moves through the portal circulation directly to the liver) and should be
excluded in this patient, even in the absence of specific symptoms (eg, alteration in
bowel habits, gastrointestinal bleeding). Abdominal CT is a useful screening test
but can often miss primary intraluminal tumors. Colonoscopy is the most
appropriate next diagnostic step as it both localizes the tumor and provides a tissue
diagnosis.
Lung and breast cancers also metastasize to the liver, although less commonly
than GI malignancies.
- Liver metastases are often clinically silent unless pressure on the liver capsule or
obstruction of the biliary tree causes pain or jaundice. Results of hepatobiliary
laboratory testing (eg, ALP) may be normal or mildly elevated, even in the setting of
heavy tumor burden. Multiple hepatic nodules are typically seen in metastatic
disease; however, solitary lesions are not uncommon. If a primary tumor is
identified (eg, with colonoscopy), liver biopsy is usually not needed.
- Hepatic angiosarcoma is a rare liver neoplasm more common in older men who
have been exposed to toxins (eg, vinyl chloride gas, inorganic arsenic compounds,
thorium dioxide).
- Primary hepatic tuberculosis is rare due to the low oxygen tension in the liver. Even
with disseminated tuberculosis, a solitary liver mass would be unlikely.
- With pyogenic liver abscesses, aspiration and culture can guide antibiotic
therapy. However, abscesses typically present acutely with fever, right upper
quadrant pain, anorexia, and weight loss. Serologic studies can distinguish
between pyogenic and amebic liver abscesses.
- Liver flukes can be diagnosed with ova and parasite stool tests. However, they
tend to invade the biliary tree rather than the liver parenchyma, and a patient with
no history of recent travel abroad makes this diagnosis unlikely.
ASCITES
/////////////////// ****** Ascites fluid characteristics

• BLOODY: trauma, iatrogenic (e.g., bx, paracentesis), malignancy (tumor eroding


****** blood vesselsHCC is MCC of bloody), TB (rarely)
Color • MILKY: chylous
• TURBID: possible infection
• STRAW COLOR: likely more benign causes

Bloody
• Acitic fluid with RBC > 50,000 mm3
ascites

Neutro
• ≥250/mm3: peritonitis (secondary or spontaneous bacterial)
phils

• ≥2.5 g/dL (high-protein ascites)


Total o CHF, constrictive pericarditis, peritoneal carcinomatosis, TB, Budd-Chiari
protein syndrome, fungal
• <2.5 g/dL (low-protein ascites)
o Cirrhosis, nephrotic syndrome

• ≥1.1 g/dL (indicates portal hypertension)


o Presinusoidal: splenic or portal vein thrombosis, schistosomiasis
o Sinusoidal (hepatic): cirrhosis, alcohol-related liver disease, liver
METASTASIS.
o Postsinusoidal: Cardiac ascites (RHF, cons peri), Budd-Chiari syndrome.
o Pathophysiology:
▪ Cirrhosis (3 main theories):
- Arterial vasodilation hypothesis (This is the most recent theory):
Portal hypertension → vasodilation → reduced SVR and reduced
MAP → Activation of endogenous vasoconstrictors, renal sodium
and water retention, and renal vasoconstriction → hyperdynamic
circulation
- Underfill theory: argues that inappropriate splanchnic
sequestration causes reduced intravascular volume and
subsequent renal sodium and water retention
- Overflow theory: States that the retention occurs without
intravascular volume depletion.
▪ Right-sided heart failure (RHF): backflow of blood obstructing the
venous outflow of the liver (Therefore, especially pulmonary
hypertension has a negative impact on the liver in the long run).
SAAG ▪ Budd-Chiari syndrome: congestion of the portal/hepatic collateral
veins and hypertrophy of the caudate lobe of the liver → compression
of the sinusoids and intrahepatic IVC
• <1.1 g/dL (absence of portal hypertension)
o Hypoalbuminemia
▪ Nephrotic syndrome → primary renal sodium retention.
▪ Severe malnutrition
▪ Protein-losing enteropathy
o Malignancy (e.g., peritoneal carcinomatosis): Various mechanisms of
malignant-related ascites exist:
▪ Peritoneal carcinomatosis (e.g., bladder ca, ovarian ca, and
mesotheliomas) → blockage of lymphatic channels and increased
vascular permeability → accumulation of peritoneal fluid (May occur
in combination with liver metastases [especially with colonic, breast,
pancreatic, and lung cancer] and therefore portal hypertension).
▪ Lymph obstruction (by a lymphoma) → chylous ascites
▪ If an underlying liver condition exists (typically HCC) → loss of
synthetic liver function or portal vein thrombosis
o Infectious (e.g., TB): production of protein-rich fluid from tubercles.
o Pancreatitis: pancreatic fluid in the peritoneal cavity

CHF = congestive heart failure; SAAG = serum-ascites albumin gradient; TB =


tuberculosis.
A patient with chronic alcohol misuse and epigastric pain that worsens postprandially,
suggestive of chronic pancreatitis, has now developed ascites (eg, abdominal
distension, shifting dullness) with high amylase and total protein, most consistent
with pancreatic ascites.
Overview of chronic pancreatitis

• Alcohol use
• Cystic fibrosis (common in children)
Etiology • Ductal obstruction (eg, malignancy, stones)
• Autoimmune

• Chronic epigastric pain with intermittent pain-free intervals


Clinical
• Malabsorption: steatorrhea, weight loss
presentation • Diabetes mellitus

• Amylase/lipase can be normal & nondiagnostic


Laboratory
• CT scan or MRCP can show calcifications, dilated ducts &
results/imaging enlarged pancreas

• Pain management
• Alcohol & smoking cessation
Treatment • Frequent, small meals
• Pancreatic enzyme supplements

MRCP = magnetic resonance cholangiopancreatography.

Chronic pancreatitis MCly occurs in patients with chronic alcohol use disorders. Initial
symptoms include constant epigastric pain that intensifies with meals as well as N and
V; signs of exocrine insufficiency (eg, steatorrhea, fat-soluble vitamin deficiency) and
diabetes mellitus usually develop later. Pancreatic ascites is a rare complication of
chronic pancreatitis that results from damage to the pancreatic duct, leading to
leakage of pancreatic juice into the peritoneal space. Like ascites from other causes,
symptoms include abdominal distension, weight gain, dyspnea, and early
satiety. Examination findings include shifting dullness a fluid wave, features of
malnutrition and/or abdominal wall hernias (e.g., inguinal, umbilical, or incisional).
Ascites in chronic pancreatitis is often identified by imaging (eg, ultrasound, CT scan). A
diagnostic paracentesis should be performed to confirm the etiology. Pancreatic
ascites is typically serosanguinous or straw-colored with analysis showing high
amylase (often >1000 U/L), high total protein (≥2.5 g/dL), and low serum-ascites
albumin gradient (SAAG) (<1.1, indicating the absence of portal hypertension).

………
Diarrhea can occur with portal hypertension due to Impaired venous and lymphatic
drainage of the intestine with decreased GI absorption of nutrients and medication.
Ascites from cirrhosis is straw yellow with normal amylase, low total protein, and high
SAAG. In addition, patients with cirrhosis usually have a low serum albumin due to
hepatic synthetic dysfunction.
Budd-Chiari syndrome (ie, hepatic venous obstruction) can cause ascites but it is
usually straw yellow with normal amylase, high total protein, and high SAAG. Patients
typically have severe RUQ pain with jaundice, hepatic encephalopathy, and possibly
variceal bleeding.

HCC can present with malignant ascites, which is typically bloody (not serosanguinous)
with normal amylase, high total protein, and low SAAG.

Ascites from intestinal perforations have high PMN neutrophil counts (≥250
cells/mm3), low glucose, high LDH, normal amylase, high total protein, and low
SAAG. In addition, perforation typically causes sudden, severe pain associated with
fever and leukocytosis.

Lymphomas can cause lymphatic obstruction and chylous ascites, which is milky with
high triglyceride levels. Normal amylase, high total protein, and low SAAG are
expected.
A patient has new-onset ascites (abdominal distension, shifting dullness) that is
persistently bloody on multiple paracenteses. Bloody ascitesis caused by red blood
cells leaking into the intraperitoneal fluid; ascites appears frankly bloody when the
peritoneal red blood cell count is >50,000 mm3. Most cases occur due to localized
trauma from a paracentesis; however, these bleeds usually resolve without
intervention. Persistently bloody ascites found on multiple paracenteses suggests an
underlying malignancy.

Hepatocellular carcinoma (HCC) is the most common malignancy to present with


bloody ascites due to tumor growth disrupting and eroding nearby blood
vessels. Less frequently, peritoneal metastases from distant primary sites (eg, ovaries,
prostate) can cause bloody ascites. Therefore, patients with bloody ascites should
undergo abdominal imaging, measurement of AFP blood levels (elevated with HCC),
and cytologic analysis of the ascitic fluid to identify the primary tumor.

In hemodynamically stable patients, no specific acute intervention is otherwise


required for nontraumatic bloody ascites; in the rare case of hemodynamic
compromise, angiography with embolization should be considered.

……….
(Nephrotic syndrome can result in ascites in association with diffuse anasarca. It
typically causes non-bloody straw-yellow ascites, and most patients also have severe
lower extremity edema. This patient's trace pitting edema may be related to ascites and
underlying cirrhosis (eg, hypoalbuminemia) but is unlikely to reflect nephrotic
syndrome.

Portal vein thrombosis does not commonly cause ascites because the obstruction
occurs proximal to the hepatic sinusoids. Hepatic vein thrombosis (ie, Budd-Chiari
syndrome) can cause ascites; however, it typically causes acute onset of non-bloody,
straw-yellow ascites in association with fever and abdominal pain.
Spontaneous bacterial peritonitis (SBP) typically causes fever, abdominal pain, and/or
AMS associated with cloudy or turbid ascitic fluid. A formal diagnosis requires an
increase in ascitic fluid polymorphonuclear cells (≥250 cells/mm3).
Abdominal tuberculosis can lead to ascites, but the fluid is generally straw-
colored; bloody ascites is rare. In addition, tuberculous ascites is typically associated
with abdominal pain as well as fever and night sweats, reflecting tuberculosis
reactivation.
TRANSPLANT A patient with a recent liver transplant who now has fevers, malaise, and liver function
REJECTION test (LFT) elevations exhibits histologic findingsconcerning for acute cellular
rejection (ACR).

ACR usually occurs within the first 3 months after transplant and results from the
patient's own immune system targeting the liver allograft. Although some patients
have fevers, malaise, and lethargy, many are COMPLETELY ASYMPTOMATIC.
Therefore, in patients with a recent liver transplant, ACR is often suspected based
upon the development of LFT abnormalities (eg, elevations in ATs, bilirubin, and
ALP). Definitive diagnosis is made by liver biopsy, which classically reveals:

• Mixed inflammatory infiltration of the portal tracts


involving eosinophils, neutrophils, and lymphocytes
• Interlobular bile duct destruction (often referred to as nonsuppurative
cholangitis)
• Endotheliitis (ie, lymphocytic subendothelial invasion of the portal and
hepatic veins), which is the most reliable sign of ACR.

The mainstay of therapy for ACR is increased immunosuppression, which typically


reverses the rejection. Most patients are treated successfully with high-dose
corticosteroids. In steroid-refractory cases, thymoglobulin, sirolimus, and other
immunosuppressants can be used. Very rarely, if a patient fails to respond to any
immunosuppressant, a repeat transplant is required.

…………….
The immunosuppressant mycophenolate is most commonly associated
with cytopenias and GI (eg, pain, V, D) side effects. It is generally discontinued in
patients with active infection due to the risk of neutropenia.

Ursodeoxycholic acid is used in the treatment of primary biliary cholangitis (PBC). The
classic histologic finding in PBC is a florid duct lesion, defined as inflammation limited
to the bile duct.
Calcineurin inhibitors (eg, tacrolimus) are associated with AKI, but significant hepatic
injury is rare. Dose reduction is not indicated and would likely worsen the ACR in this
patient.
Metronidazole is commonly used to treat intraabdominal abscesses or other
infections that can occur in the posttransplant period. Such an infection would present
with fever and, potentially, transaminase elevations; however, leukocytosis is
expected and the infection would not explain this patient's histologic findings.
PANCREATIC
CANCER

A patient has had 4 weeks of constant, progressive epigastric pain that is worse with
eating. Basic laboratory evaluation, US of the biliary tree, and EGD are unrevealing
(mild esophagitis and large gallstones with no gallbladder inflammation/ductal dilation
are unlikely to cause weeks of persistent pain). If the pancreas was not adequately
visualized, the best next step is to perform a CT scan of the abdomen to examine the
pancreas for a mass or inflammation.
Pancreatic cancer is common in those with a history of smoking or alcohol abuse. It
most often presents with epigastric pain that is progressive and constant and often
radiates to the flanks or the back. The pain is typically worse with eating and lying
supine and improved by lying in a curled position. Weight loss, jaundice, and
nausea also commonly occur. Diagnostic work-up begins with CT scan of the
abdomen, which has high sensitivity and is far better than ultrasound for detecting
small (<3 cm) tumors.
The presentation can vary depending on
the tumor's location. Most (60%-70%)
cancers occur in the of the pancreas. As
these tumors expand, they compress the
pancreatic duct and common bile duct
(leading to painless jaundice),
sometimes seen on imaging as
the double duct sign (Lt coronal CT scan).

Subsequent backup of bile leads


to intra- and extrahepatic biliary duct
dilation and a nontender, distended
gallbladder at the right costal margin
(Courvoisier sign). The jaundice can also
lead to pruritus, pale stools, and dark
urine. In contrast, cancers in the body or
tail of the pancreas usually present with abdominal pain but without jaundice.

- Fourth leading cause of cancer-related death in the United States and has a mean
age of 55 at diagnosis. Very high mortality rate as it is frequently diagnosed at
relatively late stages
- Symptoms of pancreatic cancer vary and depend primarily on the location of the
tumor. Most (60%-70%) pancreatic cancers originate in the head of the pancreas.
These cancers are more likely to present with jaundice (common bile duct
obstruction) and
steatorrhea (inability to
secrete fat-digesting
enzymes or blockage in the
main pancreatic duct).
However, jaundice may
appear at a later stage if the
tumor arises from the tail or
body of the pancreas. Most
patients describe epigastric
abdominal pain that is
usually insidious, gnawing,
and worse at night.
However, some patients
may not have pain and
simply present with painless
jaundice. Systemic symptoms such as weight loss and fatigue are common.
Another classic association is migratory thrombophlebitis (Trousseau sign).
- Diagnosis is established with either ultrasound (less expensive) in patients with
jaundice (head tumors; if US is nondiagnostic → CT) or CT scan (highly sensitive) in
patients without jaundice (body and tail tumors).
- Smoking history, weight loss, and insidious back pain with normal neurologic and
x-ray findings raise concern for pancreatic cancer in the tail of the
pancreas. Although tumors in the head of the pancreas are frequently associated
with obstructive jaundice, steatorrhea, and epigastric pain, tumors in the body
or tail of the pancreas may present with progressive, constant back pain that is
worse at night when lying supine.
- The pancreas lies in a retroperitoneal position against the posterior abdominal wall
and is innervated by the splanchnic nerves (from T5-T10). Pancreatic tumors
often irritate visceral afferent fibers that conduct signals via the splanchnic nerve,
leading to referred thoracic back pain. Spinal/neurologic examination and spinal
imaging are generally normal. CT scan of the abdomen, which has a sensitivity
>90% for pancreatic lesions, is required.
- Early satiety, intractable N and V, and weight loss point toward gastric outlet
obstruction (GOO), which results from a mechanical obstruction of the stomach or
duodenum. Physical examination findings include abdominal tenderness;
abdominal distension; and a succussion splash (rocking the patient back and forth
at the hips), suggestive of retained food in the stomach. Hypochloremic
metabolic alkalosis and hypokalemia commonly result from excessive vomiting
and volume contraction. CT findings suggestive of GOO include gastric distension
and air-fluid levels; however, an upper endoscopy is often needed to confirm the
diagnosis.

Although the etiology of GOO is broad (eg, PUD, bezoar, gastric polyps), up to 80%
of cases occur due to malignancy, MCly from pancreatic adenocarcinoma with
gastric or duodenal invasion. The presence of unexplained hyperglycemia (ie,
new-onset diabetes mellitus), particularly in a malnourished, elderly patient with
bitemporal wasting, is highly suggestive of pancreatic adenocarcinoma and occurs
due to islet cell destruction with tumor invasion. Pancreatic malignancy associated
with GOO has a poor prognosis because the tumor is usually too advanced for
surgical resection. Palliative measures include gastro-jejunal bypass and
endoscopic stent placement, which allow patients to tolerate food again.
- Bezoars can cause GOO but are very rare and most patients have an underlying
motility disorder (eg, gastroparesis) or prior gastric surgery.
- Acute gastric volvulus can cause GOO but typically presents acutely with severe
abdominal pain and hematemesis. Chronic volvulus may cause mild abdominal
discomfort and nausea, but intractable nausea, vomiting, and hyperglycemia would
not be expected.
- GOO can result from large gastric or duodenal ulcers; however, proton pump
inhibitors are preventative
- ERCP is used to visualize the biliary tree and pancreatic ducts and allows for tissue
sampling. However, it is invasive and has a lower sensitivity than CT scan for
tumors located in the uncinate process or tail of the pancreas. It is
typically recommended for patients in which other modalities
were inconclusive and can sometimes be used to relieve obstruction (eg, via biliary
stent placement).
- Percutaneous transhepatic cholangiogram (PTC) is an invasive procedure used to
evaluate the biliary tree of patients who have had a less invasive imaging fail. It
can be used for stent placement across malignant obstructions but is not a first-line
diagnostic technique.
- Serum CA 19-9 is a tumor marker that is often elevated in pancreatic cancer, but
sensitivity is related to tumor size and is limited in small cancers. In addition, it has
low specificity and may be elevated in multiple conditions (eg, hepatocellular
cancer, cholangitis). It can be used to monitor disease progression but is less useful
for diagnosis than CT scan.
- Mesenteric angiography can diagnose chronic mesenteric ischemia, which usually
presents with severe pain with eating (intestinal angina) and food
aversion. However, the pain is generally intermittent (NOT progressive). In
addition, the diagnosis is usually made with CTA (which is noninvasive) rather than
mesenteric angiography (an invasive procedure).
- Epidural abscess is associated with insidious back pain, fatigue, and often normal
x-ray findings (early in the course of the
infection). Most cases arise from hematogenous
dissemination of bacteria, which can usually be
identified with blood culture. However, most patients
have focal back pain on examination, and fever and
lower extremity neurologic deficits are common.

← Bone marrow biopsy is required to diagnose


multiple myeloma, which often presents with anemia,
hypercalcemia, bone pain, and renal failure. However,
radiographic studies usually show lytic bone lesions.
Sacroiliac joint imaging can help diagnose ankylosing
spondylitis, an inflammatory joint disease that often
presents with insidious back pain that is worse at
night. However, most cases arise in men age <40,
the pain typically improves with activity, and spinal
mobility (range of motion) is usually impaired.

Upper endoscopy can identify gastric cancer, which is associated with weight loss and
smoking. However, most patients have microcytic anemia (due to tumor bleeding
into the gastric lumen) and mid-epigastric pain. Back pain is uncommon (because
the stomach is intraperitoneal).

…………………………………………………………………….
- Trousseau syndrome is a hypercoagulable disorder that usually presents with
unexplained superficial venous thrombosis at unusual sites (eg, arm, chest area).
The syndrome is usually diagnosed prior to (sometimes months to years before) or
at the same time as an occult visceral malignancy.
Trousseau syndrome is usually associated with cancer involving the pancreas (most
common), lung, prostate, stomach, and colon, and acute leukemias. Factors
associated with higher risk of pancreatic cancer include increasing age and
smoking. The tumor likely releases mucins that react with platelets to form
platelet-rich microthrombi.
- Peripheral septic thrombophlebitis is most often associated with breaks in the skin
due to catheters, venipuncture, or IV injections. Patients often present with fever
and pain with swelling at the infection site. Absence of fever or history of
catheterization or injection makes this less likely.
- Barium swallow is useful for evaluating esophageal abnormalities (eg, dysphagia,
odynophagia) or documenting aspiration.
- Intrahepatic biliary cysts are an uncommon cause of abdominal pain and
jaundice. These cysts tend to occur more frequently in women (up to 50% of cases
in Asian countries) and
children. In addition,
intrahepatic biliary cysts
usually do not cause an
enlarged gallbladder.

- Portal-vein thrombosis (PVT)


is most often associated with
cirrhosis. Acute PVT can
present with gradual or
sudden-onset abdominal
pain, whereas chronic
PVT most often presents as
variceal bleeding.

Older age, weight loss, constant epigastric pain, smoking history, and recent diagnosis
of DM raise strong concern for pancreatic cancer, the 4th MCC of cancer-related death
in the US. Major risk factors include cigarette smoking, chronic pancreatitis,
and disorders of glucose metabolism such as insulin resistance and DM.

Approximately 25% of patients with pancreatic cancer are diagnosed with DM <2 years
prior to discovery of the tumor. In particular, atypical DM (eg, DM presenting in a
thin, older patient) should raise suspicion for pancreatic cancer, particularly when
accompanied by other suggestive findings (eg, pain, weight loss).
It is unclear whether DM promotes carcinogenesis or whether DM occurs as a
consequence of a paraneoplastic syndrome from adrenomedullin secretion, leading
to pancreatic beta cell dysfunction.
Screening for pancreatic cancer is not recommended in adults with new-onset
diabetes because the incidence of DM is much greater than pancreatic cancer.
However, those who have symptoms of pancreatic cancer such as gnawing, constant
epigastric pain and weight loss require further evaluation with CT scan of the
abdomen.

TAUS is used in the initial evaluation of patients with painless jaundice, anorexia, or
weight loss. However, it is not the preferred modality for screening patients who have
abdominal pain without jaundice; it often misses smaller (potentially resectable)
tumors; and it does not delineate tumor extension as well as CT does

………….
Patients with gastric/duodenal ulcers or gastric tumors often have gnawing epigastric
pain that is worse several hours after meals (when the food buffer is cleared).
These conditions are generally diagnosed by EGD followed by H. pylori stool antigen
testing. However, peptic ulcers rarely cause constant epigastric pain or weight loss.
Gastric cancer typically causes anemia (due to tumor bleeding into the stomach) and
is not linked to recent-onset DM.
A nuclear gastric emptying scan can diagnose gastroparesis, which is common in
patients with poorly controlled DM and generally presents with postprandial
bloating, N, and early satiety. Constant abdominal pain would be atypical, and this
patient's DM is controlled with diet.
Pancreatic cancer is
the 4th leading cause of
cancer deaths in the US.
Adenocarcinoma is the
MC tissue type.
Pancreatic cancer
usually occurs after age
45 and is more
common in men and
African American
patients. However, it is
usually diagnosed relatively late in the disease course and has a high mortality rate
(98% in some studies). The major hereditary risk factors for pancreatic cancer include
first-degree relatives with pancreatic cancer, hereditary pancreatitis, and germline
mutations (eg, BRCA1, BRCA2, Peutz-Jeghers syndrome).

Environmental risk factors include cigarette smoking, nonhereditary chronic


pancreatitis, and obesity with low physical activity. Cigarette smoking is the most
consistent reversible risk factor for pancreatic cancer. The risk of cancer increases
significantly with the number of cigarettes consumed, and heavy smokers have nearly
2-3 times higher risk than nonsmokers. Some studies estimate that smoking cessation
could reduce pancreatic cancer deaths by 25%.
…………..

Long-standing diabetes is a risk factor for pancreatic cancer, and conversely, new-
onset diabetes is often a sign of occult pancreatic cancer. The effect of diabetic
treatment on pancreatic cancer risk is variable; insulin and insulin secretagogues (eg,
sulfonylureas) are associated with an increased risk, whereas metformin is associated
with a lower risk. Neither discontinuing metformin nor initiating insulin would be likely
to lower this patient's cancer risk, regardless of improved glycemic control.

Studies have NOT shown a significantly decreased incidence of pancreatic cancer


with alcohol or caffeine reduction. Heavy alcohol use can lead to chronic
pancreatitis, which is associated with a slightly higher risk of pancreatic
cancer. However, this is not as significant as the association between SMOKING and
pancreatic cancer. In addition, this patient is not a heavy alcohol consumer.

PANCREATIC Pancreatic cysts are common incidental findings in the aging population, with an
CYSTS estimated incidence of 40% in individuals age >70. The risk of malignancy in any given
cyst is quite low (~0.01%), and many can be managed conservatively with surveillance
*** imaging.

However, some pancreatic cysts carry a risk for malignant transformation. Because cyst
resection has significant risks of morbidity and mortality, the goal of management is to
identify lesions that are at high risk for malignancy. Several radiographic features are
associated with higher risk for malignancy, including:
• Large size (≥3 cm)
• Solid components or calcifications
• Main pancreatic duct involvement (ie, ductal dilation)
• Thickened or irregular cyst wall

This patient's pancreatic cyst has several high-risk features (ie, large size, main ductal
dilation, thickened walls) and requires further evaluation to rule out malignancy. EUS–
guided biopsy is a minimally invasive approach to tissue sampling; however, surgical
resection (without prior biopsy) may be considered for very high-risk lesions.
…………………………………………..
- Unlike true pancreatic cysts, an uncomplicated pancreatic pseudocyst (eg, rounded,
well-circumscribed fluid collection formed by pancreatic necrosis in pancreatitis)
has a very low risk of malignancy and usually does not require further intervention.
However, symptomatic (eg, abdominal pain) or rapidly enlarging pseudocysts may
require endoscopic drainage.
- Screening for intracranial aneurysms is recommended for patients with ADPKD.
Although ADPKD can be associated with pancreatic cysts, it typically presents with
hematuria, flank pain, HTN, and CKD earlier in life. Additionally, numerous renal
cysts would likely be visible on CT scan.
SMALL BOWEL Mechanical SBO: Vomiting causes HoK and, along with decreased oral intake →
OBSTRUCTION dehydration and orthostasis. SBO is further categorized by anatomic location (ie,
(SBO) proximal versus mid/distal) or simple versus strangulated.

Complete proximal obstructions are characterized by early V, abdominal discomfort,


and abnormal contrast filling on x-ray.

Mid or distal obstructions typically present as colicky abdominal pain, delayed


vomiting, prominent abdominal distension, constipation-obstipation, hyperactive
bowel sounds, and dilated loops of bowel on abdominal x-ray.

Simple obstruction refers to luminal occlusion; strangulation refers to loss of blood


supply to the bowel wall. Patients with strangulated obstructions may have peritoneal
signs (eg, rigidity, rebound) and signs of shock; fever, tachycardia, and leukocytosis are
late findings. This patient most likely has a simple mid or distal SBO.

Adhesions are by far the most common cause of SBO. They may be congenital in
children (eg, Ladd's bands), but typically result from abdominal operations or
inflammatory processes.
……………………………..
- Acute cholecystitis is accompanied by fever, leukocytosis, and occasionally
abnormal liver function tests.
- High alcohol consumption can be associated with acute pancreatitis or alcoholic
hepatitis. Pancreatitis can cause an ileus (decreased bowel sounds) and a tympanic
abdominal examination. However, an aspartate aminotransferase/alanine
aminotransferase ratio >2 is typically seen if the patient is a heavy drinker. In
addition, abdominal pain in pancreatitis is usually epigastric, constant, and radiates
to the back.
- Occasional black or tarry stools (ie, melena) are suggestive of a gastrointestinal
bleed originating above the ligament of Treitz. The most common causes of
melena in a man of the patient's age are peptic ulcer disease, gastritis, esophagitis,
and Mallory-Weiss tear. Peptic ulcer disease is a rare cause of proximal but not
mid-distal SBO. The other etiologies are not associated with bowel obstruction.
- Recent fever and nonbloody diarrhea would be concerning for Crohn disease in a
man this age. Crohn disease can cause SBO; however, Crohn-related SBO usually
results from chronic fibrosis late rather than early in the course of the disease.
Moreover, postoperative SBO is much more common than SBO from an
inflammatory bowel disorder.
- Recent weight loss can predispose patients to superior mesenteric artery syndrome
or be a sign of neoplasm, endocrine disorder, or inflammatory condition. Although
these conditions can cause SBO, they are much less common than postoperative
adhesions, especially in younger patients.
Small-bowel obstruction (SBO)

• Colicky abdominal pain, vomiting


Clinical • Inability to pass flatus or stool if complete (no obstipation if partial)
presentation • Hyperactive → absent bowel sounds
• Distended & tympanitic abdomen

• Dilated loops of bowel with air-fluid levels on plain film or CT scan


Diagnosis • Partial: air in colon
• Complete: transition point (abrupt cutoff), no air in colon

• Ischemia/necrosis (strangulation)
Complications • Bowel perforation

• Bowel rest, NG tube suction, IV fluids, correction metabolic


Management derangement
• Surgical exploration for signs of complications

Patients with SBO typically present with colicky abdominal pain, vomiting, inability to
pass flatus or stool, abdominal distension, and diffuse tenderness. A history of prior
abdominal surgery is an important risk factor due to adhesion development. Increased
pain and hyperactive bowel sounds are sometimes noted during a peristaltic rush;
however, as the SBO progresses, sounds may become diminished and if ischemia
occurs, they may disappear altogether. Depending on the level of obstruction,
abdominal radiographs can reveal dilated loops of bowel with multiple air-fluid
levels. Mild leukocytosis and amylase elevation can also be seen.

Most patients with SBO can be managed initially with conservative measures (eg,
bowel rest, NG tube suction, correction of metabolic derangements). Several findings
indicate a complicated SBO with increased risk of impending ischemia, strangulation,
and necrosis, warranting emergency abdominal exploration. These findings
include changes in the character of the pain, fever, hemodynamic instability
(HoTN, tachycardia), guarding, leukocytosis, and significant metabolic acidosis
(low bicarbonate). Delay in surgery may lead to perforation and significant risk of
mortality.
……….

Antibiotics should not be used in patients with uncomplicated SBO. Patients who have
a complicated SBO with suspected strangulation have increased risk of intestinal
bacterial translocation and sepsis; broad-spectrum antibiotics can reduce infection
risk. However, antibiotics and observation (with serial x-rays) alone would not be
appropriate for a patient with clinical deterioration (eg, worsening pain, fever,
acidosis).

Barium enema is not a useful diagnostic test in patients with suspected SBO. Small-
bowel follow-through can help diagnose partial intestinal obstruction in clinically
stable patients who do not respond completely to initial conservative management.
Acute mesenteric ischemia may present with similar signs and symptoms, including
severe pain, vomiting, distension, decreased bowel sounds, and peritonitis. However,
obstipation and air-fluid levels are more characteristic of SBO. CT angiography is
the preferred imaging choice for acute mesenteric ischemia.

Colonic pseudo-obstruction (obstruction without a mechanical cause) may also


present with pain, distension, and vomiting; however, imaging would instead show
a dilated colon.

A patient with Crohn disease has small bowel obstruction (SBO) due to a fibrotic
stricture. Strictures are a complication of Crohn disease that result from poorly
controlled, severe inflammation. Smoking and young age (<30) at diagnosis are
significant risk factors for uncontrolled inflammation and disease progression
despite medical therapy.

SBO due to fibrotic stricture typically presents with bilious vomiting, severe
abdominal pain, and either partial (ie, ability to pass gas but not stool) or
complete (ie, inability to pass flatus or stool) obstruction. Abdominal examination
commonly reveals distension and high-pitched (tympanic) bowel sounds.

Although medical treatment for Crohn disease (eg, infliximab) can reduce
inflammation and may help prevent fibrotic stricture development it cannot resolve a
stricture once one develops. Depending on the location and length of the
stricture, surgical resection may be required.

…………

Infliximab and other tumor necrosis factor–alpha antagonists (eg, adalimumab) are
commonly used to treat Crohn disease. They are usually well tolerated but can
increase a patient's risk for infection and malignancy. Gastrointestinal distress is not a
typical adverse effect.
Adynamic ileus occurs when small bowel motility is disrupted, leading to intestinal
dilation, obstipation, and bilious emesis. Although it presents similarly to
SBO, adynamic ileus only develops after exposure to an insult that "stuns" the bowel,
such as recent intra-abdominal surgery or high-dose opioids.

Crohn disease only involves the small bowel, and toxic megacolon is exceedingly
unlikely.

Crohn disease is a risk factor for small bowel cancer, which can lead to SBO; however,
one would expect severe weight loss if small bowel cancer were present.

ASCARIASIS A patient with abdominal pain and distension, high-pitched bowel sounds, obstipation,
and imaging demonstrating small bowel dilation with air-fluid levels has a small bowel
*** obstruction (SBO). In a patient with peripheral eosinophilia and recent emigration
from an endemic region (eg, Asia, Africa, South America), this presentation is
concerning for ascariasis. Ascaris lumbricoides is a parasitic roundworm spread via
fecal-oral transmission. Ingested eggs hatch in the colon, and larvae penetrate the
colonic wall to spread hematogenously to the lungs, where they are coughed up,
swallowed, and mature into egg-laying adult worms in the small intestine.

Most patients are asymptomatic, with symptoms generally occurring only in those
with a high worm burden. Pulmonary manifestations (eg, cough, eosinophilic
pneumonitis) are rare but can occur within the first few weeks of an infection.
Intestinal manifestations, which are more common, occur 1-2 months later and are
usually nonspecific (eg, abdominal pain, N/V, anorexia, D). However, adult worms
can obstruct the lumen of the small bowel or hepatobiliary tree, resulting in SBO,
biliary colic, cholangitis, or acute pancreatitis. Laboratory findings include peripheral
eosinophilia (often the FIRST SIGN of ascariasis) and signs of malnutrition (eg,
vitamin deficiency, anemia).

The diagnosis is confirmed with visualization of Ascaris eggs or macroscopic worms


in the stool or respiratory secretions.

Management of an ascariasis SBO is usually conservative and includes NG suction


and fluid/electrolyte repletion, followed by definitive treatment
with albendazole or mebendazole.

………….
Small bowel lymphoma can cause abdominal pain, malabsorption, and
SBO. However, it is very rare and usually associated with unintentional weight
loss. Peripheral eosinophilia is unexpected.

OGILIVIE Acute colonic pseudo-obstruction (Ogilvie syndrome)


SYNDROME
• Major surgery, traumatic injury, severe infection
• Electrolyte derangement (↓ K, ↓ Mg, ↓ Ca)
Etiologies
• Medications (eg, opiates, anticholinergics)
• Neurologic disorders (eg, dementia, stroke)

• Abdominal distension, pain, obstipation, vomiting


Clinical
• Tympanic to percussion, ↓ bowel sounds
findings
• If perforation: guarding, rigidity, rebound tenderness

• X-ray: colonic dilation, normal haustra, nondilated small bowel


Imaging
• CT scan: colonic dilation without anatomic obstruction

• NPO, NG/rectal tube decompression


Management
• Neostigmine if no improvement within 48 hr

A patient presents with abdominal distension and colonic dilation on CT scan is


consistent with acute colonic pseudoobstruction (Ogilvie syndrome). His diuretic use
and recent diarrhea (due to foodborne gastroenteritis) likely caused an electrolyte
imbalance (eg, hypokalemia, hypomagnesemia), which can precipitate Ogilvie
syndrome. The disorder may largely result from autonomic disruption of the
colon. Accordingly, other common causes include severe trauma, severe infection,
or major surgery (increased sympathetic drive relative to
parasympathetic); neurologic disorders (eg, dementia, stroke, MS, Parkinson
disease); and certain medications (eg, anticholinergics).

Patients typically have severe abdominal distension, pain, vomiting, and


OBSTIPATION (inability to pass stool or gas). The abdomen is often tympanic and
exhibits hypoactive bowel sounds on auscultation. Abdominal plain films reveal colonic
dilation (usually predominantly affecting the proximal colon) with normal haustral
markings and a nondilated small bowel. CT scan provides diagnostic confirmation
showing colonic dilation with no anatomic obstruction.

Management involves bowel rest (NPO) and placement of NG and rectal tubes
for colonic decompression. Neostigmine can be given intravenously if symptoms fail
to improve after 48 hours or if the cecal diameter on imaging >12 cm (which is a
strong predictor of impending perforation).

……….

An endoluminal mass can lead to mechanical colonic obstruction; however, symptoms


of obstipation and abdominal distension tend to develop less abruptly than in Ogilvie
syndrome. In addition, the patient's CT scan would likely demonstrate a mass if it were
large enough to cause obstruction.

Colonic hypoperfusion leads to ischemic colitis, which usually presents with


hematochezia and abdominal pain; colonic dilation is not typical.

Entamoeba histolytica infection presents with hematochezia and liver abscesses and
is rarely acquired within the US. UC tends to cause hematochezia, abdominal pain,
and weight loss. Both E histolytica and ulcerative colitis can precipitate toxic
megacolon, but one would expect signs of sepsis (eg, fever, leukocytosis) in addition
to colonic dilation if this were present.

GASTRIC CANCER - Most cases arise in the developing world; incidence is particularly high in Eastern
Asia (eg, South Korea), Eastern Europe, and the Andean part of South America
where diets are rich in salt-preserved foods (damages stomach mucosa) and nitroso
compounds (carcinogenic). Other risk factors include Helicobacter pylori infection,
smoking, alcohol use, and atrophic gastritis.

Patients with gastric cancer usually develop persistent epigastric pain that often
worsens with eating (due to the irritant effects of gastric acid on the tumor) and
weight loss (due to insufficient caloric intake). Proximal stomach tumors may also
cause dysphagia and postprandial nausea and vomiting. The diagnosis is generally
established using EGD to visualize the stomach and obtain biopsy samples of
suspicious lesions.
……………………………………
- Although CEA may be elevated in some cases of gastric cancer, this test has low SN
and cannot be used to rule out or screen for a gastric tumor. CEA is used for
follow-up (not screening or diagnosis) of colorectal cancer
- Mesenteric ischemia, which often causes severe abdominal pain following eating
(ie, "intestinal angina"). However, relief with antacids would be atypical, and most
patients have risk factors (eg, smoking, hyperlipidemia, diabetes mellitus) for
atherosclerotic disease.
- Abdominal CT scan is often normal in early gastric cancer, so it is not used for initial
evaluation. However, abdominal CT scan may be performed later to evaluate for
distant metastases (eg, liver).
- A gastric emptying scan can diagnose gastroparesis, which is most common in
patients with uncontrolled diabetes mellitus. Most cases are marked by nausea,
vomiting, abdominal pain, and bloating following eating.
- EGD is required prior to H pylori stool testing to establish the underlying stomach
pathology.
- For adenocarcinoma of the stomach, treatment options and prognosis are
determined primarily by the disease
stage at the time of diagnosis.
Surgical removal of the affected
tissues is the mainstay of therapy,
therefore early detection of gastric
cancer improves the chances of
successful management. However,
almost 90% of patients with
gastric cancer are diagnosed at
advanced stages (III-IV), at which
point radical resection is very
complicated or impossible. For
this reason, evaluation of the extent
of the cancer is the most important
objective following initial histologic
diagnosis.

- A CT scan of the abdomen and pelvis is commonly employed for disease staging
and is SN for revealing metastases (especially in the liver). Depending on CT
findings, additional staging procedures, such as laparoscopy, EUS, chest CT, or
PET/CT may be necessary. Patients diagnosed with limited-stage disease may be
considered for curative resection, but those with more advanced disease staging
are generally referred for palliative interventions. Laparotomy is eventually
required for curative or palliative management in most patients, but initial CT
imaging is needed to plan the appropriate next steps in evaluation.

- H. pylori infection is a significant risk factor for gastric adenocarcinoma and


lymphoma (MALToma). Eradication is recommended for patients with resectable
disease to reduce the risk of developing a second cancer. Eradication of H pylori
causes remission in some patients with gastric MALT lymphoma, but it is NOT
curative for adenocarcinoma.
- Tumor markers are useless in gastric ca (e.g., CA-125).
- Persistent mid-epigastric abdominal pain, N/V, weight loss, and microcytic
anemia (likely iron deficiency) raises concern for gastric cancer. Risk is greatest in
those from Eastern Asia (eg, China), Eastern Europe, and the Andean portion of
South America due to diets rich in salt-preserved food and nitroso compounds,
which promote gastric injury and carcinogenesis. H. pylori infection, pernicious
anemia, and smoking also increase risk.

Most cases present late in the disease course when tumors have grown large
enough to cause progressive epigastric abdominal pain, weight loss due to early
satiety, and dysphagia or N/V due to proximal stomach obstruction. Because
friable tumor vessels often ooze blood into the gastric lumen, many patients
develop IDA without overt signs of GI bleeding ("slow bleed"). Metastases to
the liver can also cause hepatomegaly, elevated ALP/transaminases, and signs
of liver failure (eg, hypoalbuminemia).

w/u generally begins with EGD to visualize the stomach and to biopsy suspicious
lesions. Staging imaging (eg, CT abdomen) is then usually required to evaluate
for metastatic disease.
------------------------
- Chronic pancreatitis often causes epigastric pain and N/V, but the pain usually
radiates to the back and is worse after eating. Most cases arise in those who
abuse alcohol or have structural abnormalities in the pancreas. Hepatomegaly and
microcytic anemia are rare.
- Chronic viral hepatitis is endemic in Asia and often causes N, weight loss, and
mild transaminitis. However, epigastric pain, emesis, and microcytic anemia
are ATYPICAL.
- Duodenal ulcers often cause epigastric pain and N and V (when they obstruct the
gastric outlet). However, pain typically occurs 2-5 hours after a meal (when acid is
secreted and no food is present to buffer its effects). Furthermore, hepatomegaly
and elevated alkaline phosphatase/transaminases would be atypical.
- Hookworm infections often cause abdominal pain, N/V, and microcytic anemia
due to chronic GI blood loss. However, hepatomegaly and elevated alkaline
phosphatase/transaminases are uncommon. In addition, most cases are associated
with peripheral eosinophilia (total peripheral eosinophils >7% or >500/mm3).
- Pancreatic cancer often presents with subacute epigastric pain that radiates to
the flanks/back, weight loss, and N. Although pancreatic head tumors can
obstruct biliary drainage and lead to elevated transaminases and ALP, most
obstructions also cause hyperbilirubinemia and jaundice.
ESOPHAGEAL
Esophageal cancer
CANCER
• Adenocarcinoma
Subtypes o Distal esophagus, arises from Barrett esophagus
• Squamous cell carcinoma
o Most commonly proximal and mid esophagus

Risk • Uncontrolled GER, obesity, male (adenoC)


factors • Smoking, alcohol use, n-nitroso containing food (SmCC)

• Progressive solid-food dysphagia


Sx • GI bleeding, IDA
• Weight loss, aspiration

• Endoscopy with biopsy


Dx • CT (PET/CT) is used for staging (not initial diagnosis)

May present w/ progressive esophageal dysphagia, characterized by normal initiation of a swallow


followed by a sensation that food "sticks" within the esophagus (vs. oropharyngeal dysphagia,
characterized by difficulty initiating a swallow and nasopharyngeal regurgitation/aspiration). In an
elderly patient with a history of heartburn, this presentation raises the concern for esophageal
adenocarcinoma.

Esophageal cancer should be strongly suspected in patient with age >50, persistent
burning chest pain, significant weight loss, regurgitation, anemia (IDA), hematemesis,
and possible dysphagia (although carefully chewing food before swallowing is
sometimes used as a weight loss technique, it can be a subtle sign of progressive
dysphagia). Because manifestations can be subtle and nonspecific, esophageal cancer
often presents at an advanced stage. Most esophageal malignancies are either
adenocarcinoma or squamous cell carcinoma. Adenocarcinoma usually arises within
an area of Barrett esophagus near the gastroesophageal junction; risk is increased
with smoking and GERD. Squamous cell carcinomacan occur anywhere in the
esophagus; risk is increased with smoking and heavy alcohol consumption.
Definitive diagnosis of esophageal cancer requires esophageal endoscopy with
biopsy.

1. Young, low-risk patients with undetermined esophageal symptoms may start


with barium esophagram,
2. Those who are age >50 as well as those with alarm symptoms (eg, weight loss,
gross or occult bleeding, early satiety) will usually proceed directly to
endoscopy.
Subsequent staging procedures may include CT/PET, and surgery for definitive cure
is advised for patients with limited-stage disease.

A barium esophagram is superior to CT in the diagnosis of esophageal cancer;


however, endoscopy is still the preferred modality.

Nutritional review is advised for patients with weight loss due to behavioral disorders,
swallowing dysfunction, or food intolerance.
PPIs (eg, omeprazole, lansoprazole) are useful for patients with typical symptoms of
GERD who fail other medications. However, patients with alarm symptoms or other
features of malignancy should have a more definitive evaluation first.
PPI trial is appropriate in patients with gastroesophageal reflux disease without red flag
symptoms like dysphagia, weight loss, IDA, or persistent vomiting. An upper
endoscopy is required if any red flags are present or if reflux fails to improve with PPI.

Esophageal manometry can diagnose dysmotility syndromes like achalasia; however, it


cannot rule out cancer. Manometry is indicated if upper endoscopy is unremarkable.

BARIATRIC
Preparation for bariatric surgery
SURGERY/
OBESITY • BMI ≥40 kg/m2
///////////////// Indications • BMI ≥35 kg/m2 with serious comorbidity (eg, T2DM, HTN, OSA)
****** • BMI ≥30 kg/m2 with resistant T2DM or metabolic syndrome

• Review previous attempts at weight loss, diet, exercise habits


Intake
• Review psychiatric history, coping skills, readiness to change
assessment
• Review risk for cardiac (eg, CAD) and pulmonary (eg, OSA) disease

CAD = coronary artery disease; OSA = obstructive sleep apnea; T2DM = type 2 diabetes mellitus.

A patient has obesity and multiple weight-related comorbidities, including T2DM,


HTN, and OSA. Her weight has failed to improve despite appropriate dietary
modifications and exercise. Therefore, medical weight-loss intervention is indicated.
Appropriate options include weight-loss medication and referral for bariatric
surgery.

Weight-loss medication is indicated for patients with a BMI ≥30 kg/m2 (obese) and
those with a BMI of 25-29.9 kg/m2 (overweight) with weight-related complications.
Primary options include orlistat (pancreatic lipase [-]), lorcaserin (5-HT 2C [+]),
naltrexone/bupropion (NDRI), phentermine/topiramate, and liraglutide; the choice
of drug is individualized based on comorbidities, concurrent medications, and side
effect profile. However, sympathomimetic weight-loss medications (eg, phentermine
monotherapy) are usually avoided because although they can induce short-term
weight loss they are associated with extensive regain and minimal long-term
benefit.

Weight-loss (bariatric) surgery is indicated for patients with a BMI ≥40 kg/m2 and
those with a BMI ≥35 kg/m2 and additional weight-related comorbidity. Although
nonsurgical interventions are often attempted first, medication failure is not
required to qualify for weight-loss surgery, and both interventions may be pursued
concurrently, with medication prescribed while the patient is undergoing preoperative
evaluation.

………….
Phentermine is a sympathomimetic amine
Liraglutide is a GLP-1 receptor agonist

…………..

Moderate caloric restriction (eg, 1500-2000 kcal/day) is usually adequate for


producing weight loss in obese patients. A caloric restriction of 800-1500 kcal/day is
generally considered the most aggressive diet that can be sustained. Very-low-
calorie diets (<800 kcal/day) have a high failure rate and usually do not yield greater
long-term success than conventional diets.

**Weight-loss diets with various macronutrient contents (eg, low-fat, low-


carbohydrate, balanced) have been advocated, but the ideal diet appears to be
determined by individual factors and preferences. This patient has already tried a
fairly aggressive diet; changing the macronutrient content may make a small
difference but is unlikely to provide significant, sustained weight loss.

Exercise should be included in any comprehensive program to enhance weight loss,


reduce regain, and improve overall well-being. However, exercise alone does not
typically lead to significant loss and is unlikely to help this patient achieve her goal.

Postbariatric surgery anastomotic leak: Following Roux-en-Y gastric bypass, this


complication can occur due to breakdown of either the gastrojejunal or jejunojejunal
surgical anastomosis, leading to leakage of GI contents into the peritoneum. The leak
usually develops within the first
week after surgery and can include
fever, abdominal pain, tachypnea,
and tachycardia. HR >120/min has
been shown to be the most SN
predictor of postoperative
anastomotic leak.

Patients with a suspected anastomotic


leak should undergo an oral contrast-
enhanced abdominal CT scan or UGI
series, followed by urgent surgical
repair if a leak is demonstrated.
Because an anastomotic leak is
associated with high rates of morbidity
and mortality, urgent surgery is still recommended if the index of suspicion is high
enough, even if imaging is nondiagnostic.
- An EGD can evaluate for a marginal ulcer (ulceration at the gastrojejunal
anastomosis), which usually presents several months to years after surgery with
nausea, abdominal pain, or evidence of bleeding (eg, melena, anemia). However,
EGD is contraindicated if an anastomotic leak is suspected as it can worsen the leak
and cause frank perforation.
- A HIDA scan is used to diagnose cholecystitis by demonstrating cystic duct
obstruction. Although gastric bypass patients are at increased risk for developing
gallstones months to years after surgery, gallstone disease is unlikely in the
immediate postoperative period.
ZENKER - Potential complications include
DIVERTICULUM tracheal compression, ulceration with
bleeding, regurgitation, and pulmonary
aspiration (pneumonia).
- MC in elderly patients, particularly men.
- ZD develops immediately above the
upper esophageal sphincter with
posterior herniation between the fibers
of the cricopharyngeal muscle. Upper
esophageal sphincter dysfunction and
esophageal dysmotility are believed to
cause ZD. A barium esophagram is the
preferred imaging modality to confirm
the diagnosis. Oral contrast administration in patients with a history of aspiration is
associated with a risk of pneumonitis. However, the risk can be reduced by a
number of positioning techniques and is indicated to diagnose ZD unless the
patient has severe swallowing difficulty. Moreover, other options to visualize the
esophagus, including endoscopy, are associated with very serious risks of
complications such as esophageal perforation. Surgical treatment includes excision
and, frequently, cricopharyngeal myotomy.
- Bronchoscopy is generally indicated if there is concern for an obstructing mass that
may be causing recurrent pneumonias in the same anatomical location. Even then,
CT scan of the chest is the
preferred initial study.
- GERD is frequently associated with
ZD but is not causative. It is a
common cause of dysphagia,
often due to esophageal spasm or
stricture.
- External inflammation may cause
traction diverticula (usually in the
mid-esophageal zone), but this is
uncommon.
- Metabolic abnormalities (eg, iron
deficiency) may be associated with
upper-esophageal webs
(Plummer-Vinson syndrome).
These patients may have
dysphagia and stomatitis.
However, this is more common in
women.
This patient's lateral view of a contrast
esophagogram shows a posterior
outpouching of the cervical esophagus
that is connected to the true lumen. This is
consistent with a Zenker
(pharyngoesophageal) diverticulum (ZD).
The diverticular pouch can trap food and
medication. Patients often regurgitate the
undigested food or medications several
hours after eating. This can be more
pronounced when the individual lies
supine.
Abnormal spasm or diminished relaxation of the cricopharyngeal muscles during
swallowing (cricopharyngeal motor dysfunction) is thought to be the
underlying mechanism of ZD formation. Increased intraluminal pressure above the
cricopharyngeus muscle eventually results in herniation of the mucosa through an area
of weakness. Therefore, ZD is classified as
a pulsion (resulting from increased
intraluminal forces), rather than a traction
(resulting from external tugging
forces), pseudodiverticulum.

Treatment of symptomatic patients


involves surgical division of the
cricopharyngeus muscle (cricopharyngeal
myotomy). The diverticular pouch can be
removed or combined with the esophageal
lumen.

………….
Esophageal balloon dilation is used to treat
benign esophageal strictures, which present
with dysphagia limited to solid foods.
However, on swallow study, strictures appear as a smoothly tapering, concentric
narrowing of the esophageal lumen.
Ligamentum arteriosum ligation is used
to treat some patients with vascular ring,
a congenital malformation of
the aortic arch system that encircles the
trachea and/or esophagus and can cause
compressive
symptoms. A contrasted swallow study
would show external compression of the
esophagus rather than a diverticular
pouch.

Oral metoclopramide may be used for


gastroparesis to improve motility but is
rarely used for esophageal disease. Oral
omeprazole is used to treat GERD, which
often coexists with ZD. Medical therapy would not improve the underlying
cricopharyngeal dysfunction or the diverticular pouch.
Swallowing rehabilitation is used to treat patients with oropharyngeal dysphagia
due to neurologic disease, trauma, or surgery. It typically strengthens oropharyngeal
musculature but would not relax the cricopharyngeus muscle.
PYLORIC - Gastric outlet obstruction (GOO) caused by mechanical obstruction, leading to
STRICTURE postprandial pain and V with early satiety. Common causes of GOO include gastric
malignancy, PUD, Crohn disease, strictures (with pyloric stenosis) secondary to
ingestion of caustic agents, and gastric bezoars.
- Physical examination can show an abdominal succussion splash, which is elicited
by placing the stethoscope over the upper abdomen and rocking the patient back
and forth at the hips. Retained gastric material >3 hours after a meal will generate
a splash sound and indicates the presence of a hollow viscus filled with both fluid
and gas.
- Acid ingestion causes fibrosis 6-12 weeks after the resolution of the acute
injury. Upper endoscopy is usually required to confirm the diagnosis, and
treatment is primarily surgical.
……………………….
- Esophageal stricture and dysmotility (e.g., achalasia) tend to present with
dysphagia, which is not this patient's presenting symptom.
- Chronic pancreatitis can lead to inflammation and fibrosis in adjacent structures
(e.g., duodenum, jejunum, and transverse colon) that can rarely lead to obstruction.
However, gastric obstruction is not usually caused by pancreatitis.
- Diabetic gastroparesis tends to occur in patients who have had diabetes for >10
years.
INFLAMMATORY
Classification & management of mild ulcerative colitis
BOWEL DISEASE
(IBD) • <4 watery BM/day
Clinical features • Hematochezia is rare or intermittent

Laboratory • No anemia
findings • Normal ESR & CRP

• 5-Aminosalicylic acid (5-AmSA) agents (eg, mesalamine, sulfasalazine,


Treatment BALSALAZIDE) for both induction and maintenance.

BM = Bowel movements;

A patient has chronic diarrhea, abdominal pain, and shallow ulcerations on


colonoscopy; the most likely explanation is mild ulcerative colitis (UC). UC, a chronic
inflammatory disorder of the colonic mucosa, is often characterized clinically as either
mild or moderate to severe. Mild UC is defined as <4 watery bowel movements a
day with intermittent hematochezia, normal inflammatory markers, and no
anemia. Colonoscopy is needed to confirm the diagnosis and usually
shows inflammation and superficial ulcerations extending from the
anorectum continuously to more proximal regions of the colon.

Initial management for mild UC is with 5-aminosalicylic acid (5-ASA) medications (eg,
mesalamine, sulfasalazine, balsalazide), which are used for both induction and
maintenance therapy. Mesalamine enemas or suppositories are preferred in patients
with UC confined to the rectosigmoid, whereas PO 5-ASA medications are needed
for more extensive disease. Corticosteroids are typically reserved for acute disease
flares or severe chronic disease and can be given topically (eg, hydrocortisone
enema) or systemically (eg, prednisone).

- A total proctocolectomy with ileoanal anastomosis is the preferred surgical


procedure in patients with medically refractory, severe UC.
- TNF-alpha inhibitors (eg, infliximab, adalimumab, golimumab) represent first-line
induction and maintenance therapy for moderate to severe UC. These patients
have >6 bowel movements a day, with frequent hematochezia as well as anemia
and elevated inflammatory markers.
- Metronidazole and paromomycin are used to treat Entamoeba histolytica infection,
which is uncommon in the United States and typically has a subacute presentation
(over several weeks) with diarrhea, hematochezia, weight loss, and possibly liver
abscess development. Colonoscopy is not indicated for diagnosis, but if performed
may show characteristic flask-shaped amebic ulcers.
- EN (CD>>UC) results from a delayed-type hypersensitivity reaction to antigen
exposure. Biopsy of the nodules reveals septal panniculitis without vasculitis. EN
mirrors IBD disease activity, meaning that it worsens during severe IBD flares and
resolves as those flares improve. Other important disease associations, including
sarcoidosis, malignancy (eg, Hodgkin lymphoma), and certain infections (eg,
streptococcal, endemic fungal, viral mononucleosis).
- Migratory superficial thrombophlebitis (eg, Trousseau syndrome) can be seen with
adenocarcinoma of the colon, pancreas, or stomach.
- C jejuni typically causes a self-limited gastroenteritis with diarrhea (bloody or non-
bloody) that resolves within approximately 7 days. Although the organism has
been reported to trigger EN, it more commonly causes reactive arthritis or GBS.
A patient's presentation is
consistent with likely IBD.
Her fever and 2 months of
bloody diarrhea suggest
ulcerative colitis (UC). Patients
with UC typically develop
multiple bloody bowel
movements (sometimes >10-
15/day) with systemic findings
including fever and weight loss.
UC patients usually have
inflammation limited to the
colonic mucosa. However, a
subset develops inflammation
extending to the smooth
muscle layer, leading to muscle
paralysis and colonic dilation.
Toxic megacolon typically
presents with total or
segmental nonobstructive
colonic dilation, severe bloody diarrhea, and systemic findings (eg, fever, tachycardia).
IBD patients are at the highest risk of developing toxic megacolon early in the disease,
within 3 years of diagnosis in most cases. Other causes of toxic megacolon
include ischemic colitis, volvulus, diverticulitis, infections (eg, C. difficile), and
obstructive colon cancer (less common).

Diagnosis is confirmed by plain AXR and >3 of the following: fever >38 C (100.4
F), pulse >120/min, white blood cells >10,500/µL, and anemia.

Other findings can include volume depletion, AMS, HoTN, and electrolyte
abnormalities. Plain films usually reveal dilated right or transverse colon (>6
cm), possible multiple air-fluid levels, and thick haustral markings that do not extend
across the entire lumen (arrows in above image).

Toxic megacolon is a medical emergency that can progress rapidly and result in
colonic perforation.

Treatment includes intravenous fluids, broad-spectrum antibiotics, and bowel rest. IV


corticosteroids are preferred for treating IBD-induced toxic megacolon. Emergency
surgery (subtotal colectomy with end-ileostomy as the procedure of choice) may be
required if the colitis does not resolve.
……….
Toxic megacolon causes:

1. IBD
2. Volvulus
3. Ischemic colitis
4. Infectious (e.g., C. diff)
5. Diverticulitis
6. Obstructive colon tumor (e.g., left colon ca)

……….

Campylobacter, Shigella, and Salmonella infections are rarely complicated by toxic


megacolon.
CMV colitis can cause toxic megacolon more commonly in HIV patients. Negative
HIV test makes this less likely.

This patient has no risk factors (eg, aortoiliac surgery, cardiopulmonary bypass, MI,
hemodialysis) for ischemic colitis. Even if the patient had these factors, her
presentation would be acute rather than after 2 months of symptoms.
Right-sided colon cancer tends to present with anemia, whereas left-sided tumors
present with bowel obstruction. Both types are usually associated with a change in
bowel habits. Left-sided tumors can present with abdominal pain and bowel
obstruction.

Crohn disease (CD) Ulcerative colitis (UC)

• Anywhere mouth to anus (mostly ileum


Involveme & colon) • Rectum (ALWAYS) & colon
nt • Perianal disease with rectal sparing • Continuous lesions
• Skip lesions

Microscop
• Noncaseating granulomas • No granulomas
y

• Transmural inflammation • Mucosal &


Gross
• Linear mucosal ulcerations submucosal inflammation
findings
• Cobblestoning, creeping fat • Pseudopolyps

Clinical • Abdominal pain (varying


• Abdominal pain (often RLQ)
manifestat locations)
• Watery diarrhea (bloody if colitis)
ions • Bloody diarrhea

Intestinal
• Fistulae, abscesses
complicati • Toxic megacolon
• Strictures (bowel obstruction)
ons
RLQ = right lower quadrant.

Inflammatory bowel disease (IBD) encompasses 2 disorders: Crohn disease (CD) and
ulcerative colitis (UC). Both CD and UC present with chronic diarrhea, abdominal
pain, anemia, and elevated inflammatory markers (eg, CRP, ESR). Although not
universally true, pain is predominant in CD, whereas hematochezia is more common
in UC. Severe manifestations of CD include fistulas, strictures,
and abscesses; uncontrolled UC can lead to toxic megacolon.

Diagnosing IBD and distinguishing between CD and UC requires colonoscopy with


biopsies. CD is characterized grossly by a cobblestone appearance, skip lesions (eg,
areas of normal-appearing bowel between inflamed segments), and deep serpiginous
ulcers. On the other hand, UC demonstrates continuous, shallow ulcerations and
pseudopolyps. On histology, chronic inflammation in CD extends beyond the
submucosa and is often transmural; noncaseating granulomas may be present. In UC,
inflammation is usually limited to the mucosa, although it occasionally extends into
the submucosa.

………..
Fecal calprotectin, a stool marker for inflammatory diarrhea, is typically elevated in
patients with IBD. However, it is not specific for IBD and is NOT used to make the
initial diagnosis of IBD; it is sometimes used in patients with established CD who
have symptoms of a flare. An elevated fecal calprotectin would not distinguish
between CD and UC.

****Tissue transglutaminase (tTG) is a marker of celiac disease, whereas loperamide


and the low FODMAP (Fermentable Oligosaccharides,
Disaccharides, Monosaccharides, And Polyols) diet represent initial steps in the
management of irritable bowel syndrome (IBS). Neither celiac disease nor IBS presents
with an elevated CRP or bloody stool on rectal examination.

A barium enema can be used to characterize colonic inflammation; however, it is less


accurate than a colonoscopy for diagnosing IBD.

CT findings (eg, fat stranding, intestinal wall thickening) in IBD are identical to
those of other types of colitis (eg, infectious, ischemic) and are therefore too
nonspecific to be used for initial diagnosis.

Azithromycin is used to treat travelers' diarrhea, which causes abdominal cramps


and self-limited watery or bloody diarrhea. This patient's travel history is too remote
to be related to his clinical presentation; travelers' diarrhea typically presents within 2
weeks of initial infection.

- Severe Crohn disease: prior need for surgery and her young age at the time of
diagnosis indicate a severe disease. Such patients are at high risk for disease
recurrence; >50% require repeat surgery within 10 years if the disease is not
properly controlled. Most patients with severe Crohn disease are medically
managed with biologic therapy (eg, TNF-alpha inhibitors) and/or
immunomodulators (eg, azathioprine, 6-MP) and require close endoscopic
surveillance to ensure sustained remission.

- Patients should be advised to stop smoking because it is the only major modifiable
risk factor that affects the severity and progression of Crohn disease. Smoking is
strongly associated with an increased need for hospitalizations and intestinal
surgery as well as failure of biologic therapy. Other risk factors for severe
disease include:
1. Young age (<30 years) at diagnosis,
2. Extensive anatomic involvement,
3. Perianal disease,
4. Deep ulcerations,
5. Strictures,
6. Fistulization, and
7. Prior intestinal surgery.
- Both obesity and diabetes mellitus have been associated with colon cancer but
neither increases the risk for complications related to Crohn disease. Obesity is also
considered a risk factor for diverticulosis, but diabetes mellitus does not share this
association.
- Patients with Crohn disease, as well as irritable bowel syndrome, are at increased
risk for mood disorders, including generalized anxiety disorder; however,
concurrent mood disorders do not influence the progression or severity of Crohn
disease.
- The effect of medical marijuana on the progression of Crohn disease is unclear;
therefore, it is not recommended for these patients. Furthermore, marijuana is a
major risk factor for cyclic vomiting syndrome, and in the absence of a known
medical benefit, cessation should be recommended.
CLOSTRIDIUM Antibiotic use (most
DIFFICILE commonly fluoroquinolones,
INFECTION penicillins, cephalosporins, and
clindamycin).
Unexplained leukocytosis in
hospitalized patients should
also raise suspicion for C
difficile, even without diarrhea.

Diagnosis is usually confirmed


with stool studies for C
difficile toxin (eg, polymerase
chain reaction or enzyme
immunoassay). Both tests are highly sensitive and specific, and results are available
within a few hours. Negative laboratory tests are generally not repeated given the high
sensitivity and specificity. Patients with negative studies may require sigmoidoscopy or
colonoscopy with biopsy to document pseudomembranous colitis.

Receiving broad-spectrum antibiotics that would likely treat diverticulitis, making this
less likely.

Laparoscopy is usually reserved for patients with acute peritoneal signs (eg, rebound
tenderness, guarding), toxic megacolon, or severe ileus.

This patient's watery diarrhea, abdominal


distension, leukocytosis, fever, and radiographic
evidence of a markedly distended colon raise
strong suspicion for toxic megacolon due to
Clostridioides (formerly Clostridium) difficile
infection. Major risk factors for C difficile include
advanced age, recent hospitalization,
and antibiotic use (eg, for infected pressure
ulcer).
Most cases of C difficile colitis are marked
by watery diarrhea (≥3 in 24 hours), abdominal
pain, low-grade fever, and nausea. A minority of
patients develop severe complications such
as toxic megacolon, which is characterized by
nonobstructive colonic dilation. It generally presents with the following:

• Severe systemic toxicity (eg, fever, HoTN, tachycardia, lethargy)


• Abdominal distension and pain (diarrhea may cease due to lack of colonic
motility)
• Leukocytosis (usually >15,000/mm3)
• Radiographic evidence of large bowel dilation (eg, >6 cm [60 mm] in
colon, >9 cm [90 mm] in cecum)

Treatment of toxic megacolon includes bowel rest, NG tube, and aggressive


antibiotic therapy against C difficile (eg, PO vancomycin plus IV metronidazole). Any
agent that contributes to a lack of GI motility (eg, opiates, anticholinergics,
antimotility medications) should be discontinued. Lack of response or clinical
deterioration often necessitates further imaging (eg, CT of the abdomen) and,
sometimes, surgical intervention (eg, subtotal colectomy).
……….
Disease severity
• Nonsevere: leukocytosis < 15,000/mm3 and serum creatinine < 1.5 of baseline
• Severe: leukocytosis ≥ 15,000/mm3 OR serum creatinine ≥ 1.5 of baseline
• Fulminant: decreased blood pressure, shock, ileus or toxic megacolon

Treatment
General measures
• Discontinue the precipitating antibiotic.
• Fluid replacement
• Avoid antidiarrheals (e.g., loperamide).
Medical therapy [11]
• If clinical suspicion for CDAD is high, empiric antibiotic treatment may be
initiated before laboratory confirmation of C. difficile. [10]
• Nonsevere cases
o First-line: oral vancomycin
or oral fidaxomicin
o Second-line: oral metronidazole
• Severe cases
o First-line: oral vancomycin
o Second-line: fidaxomicin
• Fulminant cases
o First-line: oral vancomycin and intravenous metronidazole
o In patients with ileus, vancomycin per rectum (enema) may be added.

*C. difficile infection is one of the rare indications for oral administration of vancomycin!

Fecal microbiota transplantation (FMT)


• Indication: recurrent C. difficile infections in a patient who has not responded
to at least two appropriate antibiotic regimens
……….

C difficile is largely noninvasive; pathology is mediated by the release of exotoxins


(toxin A and toxin B) that cause inflammation and mucosal injury and lead to
colonic ulceration. Although perforation can occur due to toxic megacolon, free air is
usually seen on abdominal x-ray.

Colonic pseudoobstruction, a disorder of bowel motility leading to acute colonic


dilatation in the absence of a obstructing lesion, often occurs due to severe illness,
abdominal surgery, or medication. Although abdominal distension and tenderness are
common, fever, tachycardia, leukocytosis, and a history of diarrhea would all be
atypical.
Mechanical bowel obstruction usually occurs due to entanglement of bowel around
fibrous strictures from previous abdominal operations. Patients often
develop crampy abdominal pain and distension, and x-ray generally shows a tapered
transition point (where the distal bowel is pinched by the obstruction). Leukocytosis,
fever, tachycardia, and preceding diarrhea are atypical.
TOXIC - A patient with weeks of lower abdominal pain, bloody diarrhea, and fecal urgency
MEGACOLON likely has undiagnosed inflammatory bowel disease (IBD) (eg, UC). His acute
worsening with fever, abdominal distension, leukocytosis, HoTN, and
tachycardia suggests toxic megacolon. Other manifestations of toxic megacolon
include altered mental status, peritonitis, and electrolyte abn. If due to severe
colitis with massive colonic distension, toxic megacolon may be the first
presentation of IBD and is potentially lethal.
- Key to the diagnosis of toxic megacolon is radiologic evidence of colonic distension
(eg, >6 cm dilation of right colon) with manifestations of severe systemic toxicity.
Consequently, the initial test of choice in suspected cases is AXR, a noninvasive
study that can be performed quickly, confirm the diagnosis, and exclude
perforation. Conservative management (successful in >50% of cases) can initially
be instituted and includes bowel rest, NG suction, and either corticosteroids with
broad spectrum-antibiotics (due to IBD) or antibiotics targeted at Clostridium
difficile.
- Complete endoscopic evaluation
would be appropriate to confirm
IBD if this patient had presented
earlier. However, in the setting of
toxic megacolon, such a procedure
is not recommended due to the risk
of perforation. Limited evaluation
(eg, proctoscopy or sigmoidoscopy
without bowel preparation) can be
considered to help identify the
etiology once the diagnosis has
been confirmed via x-ray.
- Severe cases of toxic megacolon
not responding to medical therapy
may require emergent surgery (eg,
subtotal colectomy and end-ileostomy).
- Opioids are inadvisable in these patients as their antimotility effects can promote
colonic perforation. Other antimotility agents (eg, loperamide) and anticholinergic
agents should also be discontinued.
- In general, parasitic causes of diarrhea (eg, Entamoeba histolytica) do not lead to
sepsis and, in the absence of a travel history, are much less likely than other causes
→ no point of testing stool for ova and parasites
- Toxic megacolon should be confirmed before treatment is initiated; intravenous
corticosteroids are preferred. Sulfasalazine for IBD should be started only once
acute symptoms have resolved.
PLEURAL
EFFUSION

- Pancreatic fistulas are caused by a disruption of the pancreatic ducts, resulting in


leakage of pancreatic digestive enzymes. They are most commonly due to acute or
chronic pancreatitis and typically result in the formation of an internal fistula (ie,
communication with hollow viscera, pleural/peritoneal cavities). Iatrogenic fistulas
can occur after percutaneous drainage of pancreatic cysts or abscesses and
typically result in external fistulas (ie, communication with the skin).
- Fistulas involving the pleural cavity enable pancreatic fluid to enter the pleural
space. The resultant pleural effusion is exudative by Light criteria and amylase-rich
(ie, pleural fluid amylase exceeding upper limit of normal for serum amylase or a
pleural-fluid-to-serum-amylase ratio >1). The pH is typically 7.3-7.5.
Manifestations include cough, dyspnea, dysphagia, and chest pain; however, some
patients can be asymptomatic. Examination demonstrates decreased breath
sounds and dullness to percussion of the affected lung.
- PPFs tend to recur after thoracentesis. Management includes bowel rest to
avoid pancreatic stimulation, enabling the PPF to close. ERCP with
sphincterotomy and/or stent placement may be required to promote drainage
of pancreatic fluid through the ampulla of Vater instead of the fistula. Refractory
cases require percutaneous drainage or surgery.
………………………………………………
- Empyema and pulmonary TB are exudative processes; however, both typically result
in a low pH (<7.30) and a markedly elevated amylase is unexpected. Tuberculous
effusions also typically demonstrate an elevated ADA level.
- An esophageal rupture (Boerhaave syndrome) results in an amylase-rich exudative
pleural effusion; however, pH is typically very low (<6), and undigested food
may be present. In addition, Boerhaave syndrome typically occurs in the setting of
severe vomiting and chest pain is expected.
EOSINOPHILIC
ESOPHAGITIS

- Dysphagia (difficulty swallowing) is categorized as either oropharyngeal (difficulty


initiating a swallow, with coughing/choking after eating) or esophageal (sensation
of food stuck in the esophagus). Esophageal dysphagia to both solids and liquids
indicates a motility disorder, with intermittent (eg, esophageal spasm) or
progressive (eg, achalasia) symptoms depending on the underlying condition.
Esophageal dysphagia to solids generally reflects mechanical obstruction;
symptoms can also be progressive (eg, from solids to liquids), which suggests a
developing stricture or cancer, or intermittent.
- Eosinophilic esophagitis (EoE), which most commonly affects younger men (age 20-
30) and is associated with other atopic
conditions (eg, asthma, eczema),
classically presents with intermittent
solid food dysphagia (often associated
with eating meat) due to extensive
eosinophilic infiltration of the mucosa.
Other common manifestations include
symptoms of refractory
gastroesophageal reflux and
chest/upper abdominal pain. If not
recognized early, EoE can progress to fibrosis, leading to esophageal strictures that
result in progressive dysphagia and food impaction.
- The endoscopic appearance of EoE includes furrowing; small, whitish exudates; and
multiple stacked, ringlike esophageal indentations (trachealization of the
esophagus). The diagnosis is confirmed with esophageal biopsy demonstrating
≥15 eosinophils per HPF. Management includes dietary therapy (eg, allergen
avoidance, elimination diet), proton pump inhibitors, and topical glucocorticoids
(eg, fluticasone, budesonide).
……………………….
- Achalasia results from the degeneration of myenteric plexuses → Dysphagia is
typically progressive (rather than intermittent) and involves both solids and liquids
(VS EoE, for solids only).
- Esophageal cancer typically causes a slowly progressive, rather than an intermittent,
dysphagia. In addition, it is typically associated with unintended weight loss and is
rare in patients in their 20s.
- Herpes esophagitis is more commonly associated with odynophagia, retrosternal
chest pain, and fever than it is with dysphagia; typically occurs in
immunocompromised patients (eg, HIV, organ transplant); and often causes
concurrent oral ulcerations or herpes labialis.
- Myasthenia gravis can affect the bulbar muscles, which can lead to oropharyngeal
rather than esophageal dysphagia.
- Ibuprofen is associated with pill esophagitis, which can result in dysphagia, but also
typically causes retrosternal chest pain and severe odynophagia. Patients may be
unable to swallow saliva due to pain.
PROLONGED
POSTOPERATIVE
ILEUS

Prolonged postoperative ileus—delayed return of bowel function >3-5 d after surgery.


- Ileus is a functional defect in bowel motility without an associated physical
obstruction. Manifestations include N, V, abdominal distension, failure to pass
flatus or stool (obstipation), and hypoactive bowel sounds. Some degree of ileus
occurs following most abdominal procedures; however, persistence of the signs and
symptoms (>3-5 days
postoperatively) is termed
prolonged (or "pathologic")
postoperative ileus (PPI).
Risk factors include:
• Surgical
complications (eg,
ureteral injury),
• Bowel manipulation
(eg, laparotomy), and
• Longer surgery
duration due to
increased intraabdominal inflammation and elevated SANS tone.
Contributors to PPI include
increased splanchnic nerve
sympathetic tone following
peritoneal instrumentation, local
release of inflammatory mediators,
and postoperative opiate analgesic
use. Techniques to prevent PPI
include epidural anesthesia,
minimally invasive surgery, and
judicious perioperative use of
intravenous fluids (to minimize
gastrointestinal edema). Diagnosis
is clinical, although abdominal x-
rays revealing dilated loops of
bowel with no transition point
support the diagnosis.
- Patients with postoperative ileus
have temporary bowel paralysis (ie,
no forward peristalsis), causing
backup of gastric secretions and
gas that results in abdominal distension and vomiting (eg, V, dehydration
[elevated Cr]). Other signs include decreased bowel sounds and absent flatus.
- Diagnosis is primarily clinical, although imaging can help differentiate ileus from
SBO. Abdominal x-rays in patients with ileus typically reveal uniformly dilated
bowel loops due to generalized bowel paralysis throughout both small and large
bowel. In contrast, patients with SBO have air-fluid levels and a discrete transition
point (ie, the obstruction) that results in dilated small bowel proximal to the
obstruction and decompressed large bowel (ie, absent rectal gas) distally.
- Postoperative ileus typically self-resolves as bowel motility gradually returns;
therefore, patients are conservatively managed with antiemetics, bowel rest, and
serial examinations. Opiates, which further decrease bowel motility, should be
avoided if possible.
- Adhesions, which begin to form immediately following abdominal surgeries, are the
most common cause of mechanical bowel obstruction (MBO) in the United States.
Manifestations of MBO are similar to those of PPI. However, MBO typically causes
hyperactive (not hypoactive) bowel sounds. In addition, patients with postoperative
MBO often have a temporary return of bowel function prior to symptom onset
………………………………………………….
- A rectal enema can treat constipation by increasing pressure against impacted
stool. It does not treat ileus and may worsen symptoms (ie, cause more bowel
distension) by increasing intraluminal pressure.
- Small bowel follow-through can be used to diagnose SBO by demonstrating
differences in the flow of contrast proximal and distal to an obstruction. It has
limited utility in patients with postoperative ileus because bowel dysmotility is
generalized.
- Urgent surgical exploration is indicated in patients with an acute abdomen (eg,
perforated bowel). It is not indicated for postoperative ileus unless there are
changes between serial examinations (eg, new rebound/guarding).
- Following cholecystectomy, bile is secreted into the duodenum via the intact
common bile duct and then stored in the upper small intestine during fasting.
Occasionally, postcholecystectomy diarrhea can occur due to bile acid
malabsorption.
- Metoclopramide is a DA antagonist with promotility effects whereas
ondansetron, a serotonin receptor antagonist, can contribute to constipation.
- Common antibiotics are not known to directly inhibit intestinal motility, although
many can cause GI upset and D.
WARFARIN - Therapeutic anticoagulation with an INR between 2 and 3 is appropriate for the
chronic management of a-fib; however, if not corrected preoperatively, therapeutic
anticoagulation can predispose patients to intra- and postoperative bleeding
complications.
- Patients taking warfarin who require anticoagulation reversal (eg, urgent surgery
with a high risk of bleeding, significant hemorrhage) should be managed as follows:
1. Warfarin should be discontinued immediately.
2. Patients (particularly those with an INR >2) should receive PCC, a concentrate of
vitamin K-dependent cofactors. Three-factor PCC contains factors II, IX,
and X with small amounts of protein C and S, whereas 4-factor PCC, the
preferred agent, also contains therapeutic quantities of factor VII. PCC
rapidly normalizes the INR, typically in ≤10 minutes; however, the effects are
transient (hours).
3. IV vitamin K should also be administered to provide substrate for synthesis of
new vitamin K-dependent clotting factors by the liver (particularly given the
transient effects of PCC).
- If PCC is unavailable, FFP can be administered; however, it is less effective and
typically requires transfusion of multiple units, increasing the risk of volume
overload.
- Colloids include FFP and albumin. FFP can be used if PCC is unavailable, whereas
albumin is indicated in the treatment of hepatorenal syndrome or spontaneous
bacterial peritonitis.
- Desmopressin is given preoperatively to patients with mild hemophilia A
- Platelet counts greater than 50,000/mm3 provide adequate hemostasis for most
invasive procedures.
HEMORRHOIDS
- Hemorrhoids are the MCC
of low-volume rectal bleeding,
especially in patients who do
not have red-flag features (eg,
passage of dark blood or
blood MIXED with stool,
fever, abdominal pain,
change in bowel habits) to
suggest more serious causes.
The diagnosis is usually
obvious based on clinical
findings. Anoscopy is useful for those
whose hemorrhoids are not palpable
on examination, but further diagnostic
testing is not usually required for
young, otherwise healthy patients
with typical symptoms.
- Initial management of
uncomplicated hemorrhoids includes
increased intake of fluid and fiber (eg,
psyllium husk), reduction in fat and
alcohol intake, and regular exercise.
Additional measures may include
phlebotonics (eg, calcium dobesilate),
topical hydrocortisone, astringents
(eg, witch hazel), and local anesthetics
(eg, benzocaine, lidocaine). More
aggressive management, including
rubber band ligation and surgical
hemorrhoidectomy, is generally
advised only for patients with
refractory symptoms or prolapsed
hemorrhoids that cannot be reduced
manually (ie, grade IV hemorrhoids).
………………………………..
- Biopsy is warranted for
pedunculated or sessile colorectal
polyps, which typically present as pink,
red, or flesh-colored papules or masses.
- Colonoscopy is recommended for patients who have profuse rectal bleeding or
who are at high risk for CRC. High risk features include:
1. Prior adenomatous colon polyps,
2. Age ≥50 (unless the patient has had a negative colonoscopy within the last
2-3 years),
3. Age 40-49 with a 1st-degree relative with CRC at age <60,
4. A familial colon cancer syndrome (eg, Lynch syndrome).
Younger patients, with low-volume bleeding (eg, blood on the surface of the stool,
drips in the toilet) and a benign explanation (eg, hemorrhoids) may be monitored
without colonoscopy.
- Topical nifedipine is used to reduce anal sphincter pressure in patients with anal
fissures. Fissures can cause bright red rectal bleeding, but are typically associated
with severe pain and would not present as a purplish mucosal bulge.
Anal & perianal masses

• Dusky/purple lump or polyp


External hemorrhoid • Itching, bleeding
• Thrombosis: acute enlargement with pain

• Itching, painless bleeding, leakage of stool


Internal hemorrhoid • Detected with digital rectal exam or anoscopy

• Gradual onset
Perianal abscess • Fluctuant mass/swelling with erythema
• Fever

• Chronic onset
Anogenital wart • Soft papules, plaques, or cauliflower-shaped masses
• Mild itching, bleeding

• SCC MC
Anorectal cancer • Bleeding, pain
• Ulcerating, enlarging mass

A patient has acute anal pain and a large, tender mass consistent with a thrombosed
external hemorrhoid. Hemorrhoids represent abnormal dilation of the inferior
hemorrhoidal plexus and are
categorized by their position with respect
to the dentate line as internal (proximal)
or external (distal). Risk factors include
abnormal stool consistency or volume
(eg, diarrhea, constipation), abnormal
defecation (eg, straining, prolonged
sitting on the toilet), and increasing
age. Hemorrhoids can also be induced by
compression of pelvic veins in pregnancy
and portal hypertension in patients with
liver disease.

Common symptoms of external


hemorrhoids include itching, bleeding (eg, bright red blood on the stool surface,
drops of blood in the toilet), and anal discharge (fecal or mucoid). Thrombosis is
more common with external than internal hemorrhoids and manifests as a feeling of
fullness in the perirectal area, acute pain, tenderness, and a visible mass (eg, purplish
mass at anal verge). The diagnosis is confirmed with direct visualization and DRE;
anoscopy can make visualization easier and is useful if the diagnosis is in doubt.
Initial management of thrombosed hemorrhoids includes sitz baths, stool softeners
(eg, docusate), and topical anesthetics (eg, lidocaine). Incision or surgical excision
can be considered, but most cases improve spontaneously.

……..

Like external hemorrhoids, anal fissures can cause bleeding and severe pain with
defecation. Some fissures are associated with a small sentinel skin tag at the external
end, but a large, purple mass is more consistent with a hemorrhoid.

 Condyloma acuminata (anogenital


warts) present as fleshy, cauliflower-
shaped masses. They are more
common in individuals who engage in
anal intercourse, although this
association has low specificity.
Symptoms are chronic and include
itching, scant bleeding, and impaired
anal hygiene.

Condyloma lata is a manifestation of


secondary syphilis. It presents as flat,
gray papules that occasionally form
raised masses. Patients typically
have systemic symptoms (eg, fever,
LAD) and a macular skin rash.

Perianal abscess presents as


tenderness and swelling with erythema and induration of the overlying skin. A boggy
mass may be palpable but is typically less distinct than the discrete mass of a
thrombosed hemorrhoid. The onset is usually less abrupt than a thrombosed
hemorrhoid, and fever is often present.

Acute thrombosis in an external hemorrhoid can cause severe pain due to stretching
of somatic pain receptors in the overlying epithelium, whereas internal hemorrhoids
receive visceral innervation and typically cause only mild pain. Prolapsed
internal hemorrhoidsappear as pink or purple masses protruding through the anal
verge.
Anal Fissures http://www.uworld.com/media/L28459.jpg

Most fissures are related to chronic constipation with high anal pressures and passage
of hard stools. They can also be seen with frequent diarrhea or anal sexual
intercourse. In some cases, the pain may be so severe that patients withhold bowel
movements, exacerbating the constipation. Chronic fissures can also be accompanied
by an external skin tag (sentinel pile) at the distal end.

Surgical intervention (eg, lateral sphincterotomy, fissure excision) is indicated for


fissures that are refractory to medical management. Gradual dilation of the sphincter
can provide a wider aperture for the passage of stool and interrupt the spasm but can
lead to fecal incontinence and possible recurrent fissures.
ZOLLINGER
ELLISON
SYNDROME (ZES)
- ZES is usually sporadic but is found in conjunction with MEN-1 in 20% of cases.
Gastrinomas usually occur in patients age 20-50 with dyspepsia, reflux symptoms,
abdominal pain, weight loss, diarrhea, or frank GI bleeding. Endoscopy often
shows thickened gastric folds, multiple peptic ulcers, refractory ulcers despite
PPI use, or ulcers distal to the duodenum in the jejunum (suggesting excess
gastric acid that cannot be fully neutralized in the duodenum).
- Fasting serum gastrin level (off PPI therapy for 1 week) should be checked in
suspected gastrinoma; a level <110 pg/mL rules it out, and a level >1000 pg/mL is
diagnostic. If gastrin is elevated, gastric pH should also be measured as gastrin
may also be elevated due to failure of gastric acid secretion (achlorhydria). A
gastrin level of 110-1000 pg/mL is non-diagnostic and requires a follow-up
secretin stimulation test. Secretin stimulates the release of gastrin by
gastrinoma cells. Normal gastric G cells are inhibited by secretin; therefore,
secretin administration should not cause a rise in serum gastrin concentrations in
patients with other causes of hypergastrinemia.
- The calcium infusion study is usually reserved for patients who have gastric acid
hypersecretion and are strongly suspected of having gastrinoma despite a
negative secretin test. Calcium infusion can lead to an increase in serum gastrin
levels in patients with gastrinoma.
- Serum chromogranin A is a marker for well-differentiated neuroendocrine tumors.
It is elevated in several conditions such as carcinoid tumors, hyperthyroidism,
chronic atrophic gastritis, and even chronic PPI therapy. For this reason, it is
less SN and SP than measurement of serum gastrin level. However, it may be
used as a confirmatory test in non-diagnostic cases.
A patient with multiple peptic ulcers
and diarrhea has clinical features
of Zollinger-Ellison
syndrome (ZES). ZES is due to
a gastrin-producing tumor, which
usually occurs in
the pancreas or duodenum. Most
cases are sporadic, but ZES also
occurs frequently in the context of
MEN1. Uncontrolled gastrin secretion
leads to parietal cell hyperplasia, with
excessive production of gastric
acid. Multiple duodenal (and
sometimes jejunal) ulcers are typical,
and the ulcers can be refractory to
standard acid-reducing
medications. The excess gastric acid in the small intestine can cause diarrhea and
steatorrhea due to inactivation of pancreatic enzymes and injury to the mucosal
brush border.
The diagnosis of ZES is suggested by a markedly elevated serum gastrin level
(>1000 pg/mL) in the presence of acidic gastric pH (<4). Endoscopy usually reveals
ulcers (>90% of patients with ZES) and occasionally will identify a primary duodenal
gastrinoma. CT, MRI, and somatostatin receptor scintigraphy can be used
to identify pancreatic tumors and metastatic disease. If gastrinoma is confirmed,
patients should be screened for MEN1 with assays for PTH, ionized calcium, and
prolactin.

…..

Small intestinal bacterial overgrowth (SIBO) can also cause mucosal injury, possibly due
to bacterial toxins. However, duodenal and jejunal ulcerations are not typical in this
condition.

It would be unusual for Crohn disease to present with multiple duodenal ulcers.
MESENTERIC - AMI is most commonly due to abrupt arterial occlusion from either of the following:
ISCHEMIA 1. Cardiac embolic events in the setting of atrial fibrillation, valvular disease (eg,
infective endocarditis), or cardiovascular aneurysms
2. Acute thrombosis due to peripheral arterial disease or low cardiac output
states
- AMI typically presents with
sudden-onset, severe, poorly
localized (visceral)
midabdominal pain
accompanied by nausea and
vomiting. Urge to defecate is
common. In early-stage
ischemia, physical examination
is typically unremarkable (eg,
mild diffuse tenderness)
despite patients having severe
pain out of proportion to the
examination findings. If bowel
infarction occurs, patients may develop more focal abdominal tenderness (due to
local inflammation/infarction), peritoneal signs (eg, guarding, rebound tenderness),
rectal bleeding, and sepsis. Leukocytosis, hemoconcentration, elevated amylase,
and metabolic acidosis (lactate) are frequently seen on laboratory testing.
Patients with evidence of bowel infarction should undergo immediate operative
evaluation; otherwise, diagnosis can be confirmed radiologically by
CTA. Treatment includes open embolectomy with vascular bypass or endovascular
thrombolysis. In addition, patients should be started on broad-spectrum antibiotics
and, in the absence of active bleeding, anticoagulation to reduce the risk of clot
expansion.
- Acalculous cholecystitis usually occurs in critically ill, hospitalized patients. Patients
often have jaundice as well as pain and/or a mass in the upper right quadrant.
- Alcohol withdrawal frequently presents with restlessness, diaphoresis, and
tachycardia. In moderate-to-severe cases, it may also present with seizures,
hallucinations, and altered mental status. Acute, severe abdominal pain is less
likely.
- Peptic ulcer perforation does not commonly localize to the lower right quadrant
and is not typically accompanied by metabolic acidosis unless there is
hemodynamic instability with hypovolemia, hypotension, and impending shock.
- Intraabdominal abscesses usually present subacutely with fever, focal abdominal
tenderness, and weight loss.
- Opioid withdrawal commonly presents with gastrointestinal manifestations.
However, these are also usually accompanied by flulike symptoms and signs of
sympathetic nervous system activation (eg, mydriasis, agitation, anxiety); bowel
sounds are typically increased rather than decreased.
PILONIDAL SINUS - Subacute pain over the midline sacrococcygeal with mucoid and bloody
(PNS) drainage is consistent w/ pilonidal disease (PD). PD most frequently affects
individuals age 15-30, particularly young males, obese individuals, those with
sedentary lifestyles or occupations, and those with deep gluteal clefts. The MC
presenting manifestations include a painful, fluctuant mass 4-5 cm cephalad to
the anus in the intergluteal region with associated mucoid, purulent, or bloody
drainage. Pain is frequently worsened by activities that stretch the overlying
skin (eg, bending down).

- PD develops when an edematous, infected hair follicle


in the intergluteal region becomes occluded. The infection
spreads subcutaneously and forms an abscess, which can
rupture and create a pilonidal sinus tract. As the patient sits
or stands, hair and debris are forced into the sinus tract,
resulting in recurrent infections and foreign-body reactions.
Treatment is drainage of abscesses and collected debris
followed by excision of sinus tracts. Despite longer healing
times, open closure is preferred due to decreased
recurrence rates.

- A perianal abscess presents with fever, malaise, anal pain, and a tender,
erythematous bulge at the anal verge. PNS patients are afebrile and pain and
swelling are located superior to the anus over the coccyx

……………………………….
- Although folliculitis is part of the initial
pathophysiology of PD, it most frequently
presents with multiple small, pruritic pustules,
not a fluctuant, tender mass.

- Hidradenitis suppurativa (HS), PD,


dissecting folliculitis of the scalp, and acne
conglobata are members of the follicular
occlusion tetrad. Patients with HS typically have
multiple, recurrent, painful nodules in the
axillae, inguinal folds, and perineal areas as
opposed to a single, fluctuant mass. Moreover,
the lesions of PD are usually midline in the
gluteal clefts, whereas HS can occur at any
location in intertriginous areas.

COLOVESICAL - Can be a complication


FISTULA of acute diverticulitis.
The mechanism is
usually due to direct
extension of a
ruptured
diverticulum or
erosion of a
diverticular abscess
into the bladder.
Patients typically develop fecaluria (stool in the urine) or pneumaturia (air in the
urine) that usually occurs at the end of urination as the gas collects at the top of
the bladder. Patients can also develop recurrent UTIs (sometimes due to mixed
flora with coliform organisms) or other nonspecific symptoms that can sometimes
delay the diagnosis. Colovesical fistula can also occur in patients with Crohn
disease or malignancy (usually of the colon).

- Abdominal CT scan with oral or rectal (not IV) contrast can confirm the
diagnosis by showing contrast material in the bladder with thickened colonic
and vesicular walls. Colonoscopy is usually recommended to exclude colonic
malignancy. Treatment is typically surgical after resolution of the infection.
………………………………………..
- Acute bacterial prostatitis typically presents with acute onset of systemic findings
(eg, fever, chills, malaise), irritative urinary symptoms (eg, dysuria, frequency),
pelvic/perineal pain, and cloudy urine. Prostatic abscess, a focal purulent
collection, is a complication of acute bacterial prostatitis. However, both
conditions usually cause an exquisitely tender prostate on DRG.
- Bladder cancer classically presents with irritative urinary symptoms and painless
hematuria.
- Emphysematous cholecystitis (typically in
patients with diabetes) can present with abrupt
or gradual onset of fever, chills, flank or
abdominal pain, and N/V. Absence of fever and
flank pain makes this less likely.

- Staghorn calculi, which are typically due


to struvite stones filling the entire intrarenal
collecting system, can lead to recurrent UTIs.
Patients can develop irritative urinary
symptoms, flank pain, or hematuria. However,
staghorn calculi are not usually associated with
pneumaturia.

APPENDICITES Most patients with acute appendicitis will seek medical care within the first 24-48
hours of symptoms. Typical features include migratory (vague periumbilical to sharp
*** right lower) abdominal pain, fever, N, V, and anorexia. These patients may have signs
of irritation of the parietal peritoneum (ie, rebound tenderness, involuntary
guarding, abdominal rigidity) due to impending appendiceal rupture or completed
rupture with diffuse peritonitis. Urgent appendectomy is indicated in these
patients.

Patients who have a delayed presentation with a longer duration of symptoms (>5
days) often have appendiceal rupture with a contained abscess. These patients will
generally have significant fever and leukocytosis, but findings on anterior palpation
of the abdomen may be unrevealing. In such cases, maneuvers that assess the deep
abdominal spaces (eg, psoas sign, obturator sign, rectal examination) may be more
informative. In particular, the psoas sign suggests the presence of an abscess
posterior to the appendix adjacent to the psoas muscle (or possibly an unruptured
retrocecal appendix). CT imaging can confirm the diagnosis in these cases.
Patients with a contained
appendiceal abscess have a
very high complication rate
from immediate surgery due
to the mass of inflamed,
infected, and friable debris
and adhesions. If they are
otherwise clinically stable,
these patients should be
managed with IV antibiotics,
bowel rest, and possibly
percutaneous drainage of
the abscess. They can return
in 6-8 weeks for
appendectomy on an elective
basis ("interval
appendectomy").
- Besides fever and
abdominal pain, inflammatory bowel disease usually presents with prolonged
episodes of diarrhea. Patients with ulcerative colitis typically also have gross rectal
bleeding.
- Parasitic colitis typically presents more acutely after travel to an endemic area, with
many episodes of diarrhea (the predominant symptom) and fever.
A high clinical suspicion of appendicitis was historically considered sufficient to
proceed to appendectomy, and a negative appendectomy (ie, removed appendix was
noninflamed on pathology) rate (NAR) of approximately 10% was deemed
acceptable. However, given its widespread availability and rapidity, diagnostic
imaging is now typically performed when appendicitis is suspected (eg, modified
Alvarado score ≥4). Imaging decreases the NAR and directs definitive management.
For avoiding ionizing radiation
Modified Alvarado score
exposure, TAUS is the initial imaging
modality used in children and Points
Clinical feature
pregnant women, and MRI may be (each)
necessary if ultrasound is
nondiagnostic. However, CT scan • Migratory RLQ pain
• Anorexia
of the abdomen and pelvis (with
1 • Nausea or vomiting
contrast) is the preferred study for • Fever >37.5 C (>99.5 F)
evaluation of suspected appendicitis • RLQ rebound tenderness
in nonpregnant adults. CT has high
• RLQ tenderness
diagnostic accuracy and is less likely 2 • Leukocytes >10,000/mm3
to yield an indeterminate result (ie,
nonvisualized appendix). In addition, 9 Total possible score
CT is less operator dependent than
0-3: Appendicitis unlikely.
ultrasound and is quicker, cheaper,
≥4: Evaluate for appendicitis.
and better tolerated by patients than
MRI. RLQ = right lower quadrant.

Management can be directed by CT findings:

• Normal appendix: Evaluate for other diagnoses (avoids unnecessary


appendectomy).
• Nonperforated appendicitis: Treat with antibiotics and appendectomy
within 12 hours.
• Perforated appendicitis: Treat with antibiotics and either percutaneous
drainage (for stable patients with a contained RLQ abscess [ie, contained
perforation]) or emergency appendectomy (for patients with diffuse
intraabdominal contamination [ie, free perforation]).

Several trials have studied antibiotic-based nonoperative management of patients with


nonperforated appendicitis. Although most patients avoided appendectomy when
treated with antibiotics only, there was no reliable way to predict which patients would
fail treatment and which would require rescue appendectomy. At present,
appendectomy remains the treatment of choice for nonperforated appendicitis.

………………

Plain abdominal (eg, supine, lateral decubitus) x-rays can be helpful in diagnosing
some intraabdominal conditions (eg, air-fluid levels on lateral decubitus view in
bowel obstruction). However, plain films do not visualize the appendix and cannot
detect appendiceal inflammation.
SCURVY
Scurvy (vitamin C deficiency)

• Due to insufficient dietary intake (lack of citrus


Epidemiology fruits/vegetables)
• MC in alcoholics, IVDU & severely malnourished

• Cutaneous: petechiae, follicular hemorrhage, bruising,


coiled hairs
• Gingival: bleeding/receding gums & dental caries
Manifestations • Constitutional: arthralgias, weakness, malaise,
depression
• Impaired wound healing
• Vasomotor instability (if severe/prolonged)

• PO/injectable vitamin C resolves most symptoms within


days
Tx • Toxicity (eg, abdominal pain, D, N) can occur with
excessive supplementation

Hx of alcohol abuse, gingival bleeding, gingival recession, and ecchymosis suggest his
poorly healing wound is due to scurvy (vitamin C deficiency). Vitamin C acts as a
reversible reducing agent and plays a crucial role in fatty acid transport and
collagen synthesis. Most cases of vitamin C deficiency arise in the setting of severe
malnutrition, which is frequently due to alcoholism, drug abuse, or psychiatric illness.
Manifestations of scurvy usually occur within 3 months of a diet deficient in vitamin
C. Common findings include:

• Cutaneous manifestations – follicular hyperkeratosis, perifollicular


hemorrhage, ecchymosis, petechiae, coiled hairs
• Gingivitis – receded gums that bleed easily and dental caries
• Impaired wound healing
• Systemic symptoms – arthralgias, malaise, weakness

As the syndrome progresses, hemolytic anemia and edema also commonly occur.

Diagnosis is made by plasma or leukocyte vitamin C levels.


Treatment with PO or injectable vitamin C rapidly improves constitutional symptoms,
but gingival changes may take up to 2 weeks to resolve.

Approximately 80% of alcoholics have thrombocytopenia due to alcohol-induced


BMS. Anemia and leukopenia can also occur. However, myelosuppression is unlikely
to account for poor wound healing or gingivitis.

Although chronic liver disease can cause thrombocytopenia, anemia, ecchymosis, and
lower-extremity edema, it is not typically associated with poor wound healing and
gingival changes. In addition, there are no other common signs of liver disease (eg,
ascites, jaundice, muscle cramps, spider hemangiomas, splenomegaly) in scurvy pts.
Chronic venous insufficiency is marked by edema, pain, skin changes, and
ulceration. Although this condition can cause poor wound healing, it is not typically
associated with gingivitis or scattered ecchymosis.
*This patient completed his course of antibiotics and does not currently have local (eg,
warmth, pain, purulent discharge) or systemic (eg, fever, leukocytosis) signs of
infection. The mild erythema at the border of the wound (without warmth or
tenderness) is much more likely to represent healing tissue than infection.

HIATAL HERNIA

****

- Most (>90%) hiatal hernias are sliding hernias, which occur when the
gastroesophageal junction and proximal stomach slide into the chest. Sliding
hernias are usually asymptomatic or result in reflux symptoms (eg, heartburn).
- The distal esophagus is attached circumferentially to the diaphragm at the
esophageal hiatus by the phrenoesophageal membrane. Sliding hiatal hernias
occur due to laxity of this membrane, typically in the setting of age-related
degeneration or repetitive stress (eg, vomiting, coughing).
- Most sliding hiatal hernias are discovered incidentally during imaging performed
for other reasons. Plain radiography typically reveals a retrocardiac opacity (often
with an air/fluid level) within the thoracic cavity. Although sliding hiatal hernias
predispose patients to developing GERD (eg, heartburn, dysphagia, chest pain),
asymptomatic patients require no further evaluation beyond observation. Patients
with reflux symptoms should be medically managed (eg, proton pump inhibitors).
Those with refractory GERD symptoms can be considered for antireflux surgery (eg,
Nissen fundoplication)
- In contrast, PEHs occur when the gastric fundus migrates into the thoracic cavity;
larger defects can result in the subsequent herniation of the surrounding stomach
and intraabdominal organs (eg, bowel, spleen). This results in compression of the
stomach and surrounding organs (eg, esophagus, lungs), leading to more severe
symptoms. Common manifestations include nausea and vomiting, postprandial
fullness, dysphagia, and epigastric and/or chest pain. As the stomach advances into
the thoracic cavity, there is risk of respiratory compromise and gastric volvulus.
………………………………
- The presence of a retrocardiac air-fluid level (due to the stomach bubble within the
thoracic cavity) suggests a PEH, although it may also be seen in sliding hernias. The
diagnosis is confirmed with barium swallow or upper endoscopy. Whereas
symptomatic sliding hernias are generally managed with medical treatment of
reflux symptoms, PEHs often require surgical repair.
- 24-hour esophageal pH monitoring can be used in the evaluation of GERD;
esophageal manometry is indicated to evaluate suspected esophageal motility
disorders; upper gastrointestinal endoscopy is used to visualize and intervene in a
variety of gastrointestinal disorders. These procedures, as well as barium swallow
(the most SN test for diagnosis of hiatal hernia), may be performed for further
evaluation prior to surgical intervention in patients with refractory reflux
symptoms. However, they are not indicated in asymptomatic patients.
GROIN HERNIA - Although inguinal hernias are much more common, femoral hernias occur more
frequently in elderly women and are much more likely to lead to complications.
Incarceration occurs when
hernia contents become
trapped within the hernia
sac, which can result in
SBO; reduced venous
outflow eventually leads to
ischemia and necrosis
(strangulation).
- Femoral hernia:
femoral ring (medial to the
femoral artery and lateral
to the inguinal ligament).
Risk factors include
chronic cough (eg,
COPD), constipation, and
smoking. Most cases
arise in older women and
present as a nonpulsatile
mass in the groin. When a
bowel loop is present within the hernia, it is often tympanitic to percussion.
- Because femoral hernias pass through a narrow orifice, they are associated with a
substantial risk of incarceration (trapping of abdominal/pelvic contents within the
hernia) and strangulation (constriction of blood flow with subsequent
ischemia/necrosis). Therefore, asymptomatic femoral hernias are generally referred
for elective surgical repair to prevent potentially life-threatening complications and
subsequent high-risk emergency surgery, which is associated with an increased risk
of morbidity (eg, bowel resection) and mortality.
- In contrast, inguinal hernias (hernia above the inguinal ligament) are associated
with a lower risk for incarceration and strangulation because hernia contents pass
through a wider orifice. Therefore, most asymptomatic inguinal hernias can be
managed with reassurance and watchful waiting. Observation can also be
considered for patients with chronic (>3 months), stable femoral hernias, but is not
recommended for most patients with femoral hernias due to the risk of
incarceration.
- Groin hematomas can cause extrinsic compression resulting in an SBO, and are
more common in anticoagulated patients. However, patients typically have a
history of trauma, and anemia and ecchymosis would be expected.
- Small bowel adhesions and strictures typically occur as a complication of Crohn
disease or intraabdominal surgery. These can cause SBO but are unlikely in this
patient without a history of surgery or IBD.
- Small bowel intussusception can cause SBO but usually occurs in children and is
very rare in adults. Hematochezia is common, and patients typically have a history
of intermittent abdominal pain (due to recurrent telescoping of the bowel).
- Watershed bowel hypoperfusion typically occurs with chronic mesenteric
ischemia and affects the splenic flexure and rectosigmoid junction of the colon. It
usually occurs in patients with hypotension and/or significant atherosclerotic
disease. Patients classically have abdominal pain out of proportion to examination
findings, and occult blood may be present in the stool.
A patient's nontender groin bulge is located below the inguinal ligament, raising strong
suspicion for femoral hernia, a displacement of abdominal or pelvic contents through
a widened or laxed femoral ring (medial to the femoral artery and lateral to the
inguinal ligament). Risk factors include chronic cough (eg, COPD), constipation,
and smoking. Most cases arise in older women and present as a nonpulsatile mass in
the groin. The mass generally worsens with increased abdominal pressure (eg,
standing, Valsalva maneuver, coughing) and improves with decreased abdominal
pressure (eg, lying down). When a bowel loop is present within the hernia, it is often
tympanitic to percussion.
Because femoral hernias pass through a narrow orifice, they are associated with
a substantial risk of incarceration (trapping of abdominal/pelvic contents within the
hernia) and strangulation (constriction of blood flow with subsequent
ischemia/necrosis). Therefore, asymptomatic femoral hernias are generally referred
for elective surgical repair to prevent potentially life-threatening complications and
subsequent high-risk emergency surgery, which is associated with an increased risk of
morbidity (eg, bowel resection) and mortality.

In contrast, inguinal hernias (hernia above the inguinal ligament) are associated with
a lower risk for incarceration and strangulation becausehernia contents pass through
a wider orifice. Therefore, most asymptomatic inguinal hernias can be managed with
reassurance and watchful waiting.
Observation can also be considered for patients with chronic (>3 months), stable
femoral hernias, but is not recommended for most patients with femoral hernias
due to the risk of incarceration.
……

CTA can be used to diagnose femoral artery aneurysm, which is associated with
a pulsatile groin bulge that does not change with alterations in abdominal pressure.

Diverticulitis usually presents with lower abdominal pain and, occasionally, a tender
lower abdominal mass. Treatment of uncomplicated diverticulitis generally includes
oral antibiotics; percutaneous drainage is sometimes required when diverticular
abscess is present. Other infections that can develop in the groin include cutaneous
abscesses or lymphangitis. This patient's mass is nontender and not associated with
erythema or fever, making infection less likely.
UMBILICAL
Congenital umbilical hernia
HERNIA
Pathophysiology • Incomplete closure of abdominal muscles

• Soft, nontender bulge at umbilicus


Clinical features • Protrudes with increased abdominal pressure
• Typically reducible

• Observe for spontaneous closure


Management
• Elective surgery around age 5

A congenital umbilical hernia is due to incomplete closure of the abdominal muscles


around the umbilical ring at birth. It is commonly associated with hypothyroidism and
Ehlers-Danlos, Beckwith-Wiedemann, and Down syndromes, but also occurs
frequently in normal newborns. Physical examination shows a soft,
nontender bulge covered by skin that protrudes with increased abdominal pressure
(eg, crying, straining). The hernia may contain omentum or portions of the small
intestine. Most umbilical hernias are reduced easily through the umbilical ring with
very low risk of incarceration and strangulation.
Small umbilical hernias typically close spontaneously as the rectus abdominis muscles
grow together and fascial layers fuse. Spontaneous closure is less likely with large
(>1.5 cm diameter) hernias or in patients with underlying medical
problems. Surgery is recommended around age 5 for persistent hernias, or sooner if
complications occur (e.g., progressive enargement after age 1-2y, strangulation).

Abdominal ultrasound is often performed in an older child or adult when an acquired


umbilical hernia is suspected. This patient has a congenital hernia, which is diagnosed
clinically.

Immediate surgery is performed for gastroschisis.


Although umbilical hernia is more common in patients with Down syndrome, an
umbilical hernia alone does not warrant further workup.

An umbilical granuloma presents in newborns after the umbilical cord has separated
with a moist, red, pedunculated, friable umbilical mass. Treatment is with silver
nitrate.

INCISIONAL
Incisional hernia
HERNIA
Pathogenesis • Breakdown of prior fascial closure

• Obesity
• Tobacco smoking
Risk factors • Poor wound healing (eg, immunosuppression, malnutrition)
• Vertical or midline incision
• Surgical site infection

• Abdominal mass that enlarges with Valsalva


Clinical
• Palpable fascial edges in nonobese patients
features • Possible delayed presentation (months-years)

• Clinical
Diagnosis • CT scan of abdomen

A patient has a slowly enlarging abdominal mass at her prior surgical incision site,
consistent with an incisional hernia. Incisional hernias occur due to fascial closure
breakdown. They are common in patients with obesity (elevated intraabdominal
pressure) and a prior vertical or midline incision (increased tissue tension). These 2
factors lead to gradual fascial breakdown; presentation may be delayed for months
to years, as in this patient.

Incisional hernias can be associated with acute


gastrointestinal symptoms if the hernias become
very large (mass effect) or bowel incarceration
occurs (higher risk with smaller hernia
size). However, most patients have
an asymptomatic abdominal mass (ie,
protruding abdominal contents) that slowly
enlarges as the fascial defect expands. The mass
may be present when patients lie supine, and
discrete fascial edges may be palpable in
nonobese patients. In addition, abdominal mass
size increases with Valsalva because greater intraabdominal pressure causes more
protrusion through the defect.

Diagnosis is typically clinical.


Abdominal CT scan may be used to identify unclear anatomy (eg, surgical repair
planning) or if there is concern for an acute complication (eg, bowel incarceration).

………

Rectus abdominis diastasis is a bulge between the rectus muscles due to linea alba
weakening. However, unlike a true hernia, it has no associated fascial defect; therefore,
the abnormality is not palpable while supine.

A rectus sheath hematoma may present as an abdominal mass. However, it is usually


associated with acute trauma and pain rather than with a slowly enlarging,
uncomfortable bulge. In addition, hematoma size does not change with Valsalva.
Lipomas are not typically located in the midline and do not enlarge with Valsalva.

Small-bowel obstruction (SBO)

• Colicky abdominal pain, vomiting


Clinical • Inability to pass flatus or stool if complete (no obstipation if partial)
presentation • Hyperactive → absent bowel sounds
• Distended & tympanitic abdomen

• Dilated loops of bowel with air-fluid levels on plain film or CT scan


Diagnosis • Partial: air in colon
• Complete: transition point (abrupt cutoff), no air in colon

• Ischemia/necrosis (strangulation)
Complications • Bowel perforation

• Bowel rest, nasogastric tube suction, intravenous fluids


Management
• Surgical exploration for signs of complications

A patient with abdominal pain, vomiting, and obstipation (ie, no bowel movements or
flatus) likely has a small bowel obstruction (SBO). In the setting of recent surgery (few
days) and a tender, palpable mass at the level of the incision, her obstruction is likely
caused by an incisional hernia. Incisional hernias can develop at the site of any
abdominal incision and reflect a breakdown in fascial closure. Obesity appears to be an
important risk factor. Although most cases present as an asx abdominal mass,
obstruction can occur, particularly with smaller hernias, because bowel can remain
trapped within the small fascial defect.

SBO typically causes vomiting and abdominal distension due to backup of gastric
secretions and intestinal gas. Colicky pain after meals is also common due to
unproductive bowel peristalsis against the obstruction, with hyperactive bowel
sounds in the early stages. SBO treatment depends on disease severity. Partial SBO, in
which the bowel lumen is narrowed but not completely obstructed, typically improves
with conservative management (ie, bowel rest, NG tube insertion for bowel
decompression, serial abdominal examinations).
In contrast, those with a complete SBO (no flatus, no air in the rectum on x-ray), are at
increased risk of life-threatening complications (eg, bowel ischemia, perforation) that
will not resolve with conservative management. Therefore, in addition to NG tube
insertion for gastric decompression, these patients require emergency laparotomy to
relieve the obstruction and resolve any underlying etiology (eg, incisional hernia,
intraabdominal adhesions).

……..

Further imaging studies (eg, abdominal US, small bowel follow-through) are not
needed in patients with complete SBO diagnosed on AXR (eg, no rectal gas). This
patient requires emergency surgery; additional testing delays treatment and increases
the risk of complications.

Barium enemas and digital fecal disimpaction are used to treat constipation, a large
bowel disorder that appears on AXR as impacted stool in the rectum. Neither
intervention affects the small bowel.

ERCP - ERCP is a minimally invasive endoscopic technique used to visualize and intervene
upon the biliary and pancreatic ducts, is widely used for stone removal, tissue
sampling, stent placement, and sphincterotomy. In some patients, ERCP with
manometry can confirm the diagnosis of sphincter of Oddi dysfunction (SOD),
which typically causes biliary-type pain with no obvious etiology. Post-ERCP
complications can occur; postprocedural abdominal pain is relatively common and
may be due to infection (eg, ascending cholangitis), perforation, or pancreatitis.
- The MC complication is post-ERCP pancreatitis, which occurs in up to 10% of
patients and is particularly prevalent in those with SOD. Classic symptoms of
pancreatitis include nausea, vomiting, and epigastric pain that radiates to the back
and worsens with eating. Fever may occur due to inflammation. Acute pancreatitis
is diagnosed in patients meeting ≥2 of the following criteria:

• Severe epigastric pain (often radiating to the back)


• Amylase or lipase ≥3 times the upper limit of normal
• Characteristic findings of acute pancreatitis on imaging (eg, CT, MRI, US)
- Because the first 2 criteria are sufficient to confirm the diagnosis, serum amylase
and lipase should be obtained as part of the initial workup in all patients with
suspected pancreatitis. These enzymes rise within several hours of the
development of symptoms whereas CT findings may remain normal for up to 48
hours.
- Postcholecystectomy pain due to sphincter of Oddi dysfunction (SOD): SOD,
which can develop following any inflammatory process (eg, surgery, pancreatitis),
encompasses 2 separate physiologic entities: dyskinesia and stenosis of the
sphincter of Oddi. Obstruction of flow through the sphincter may result in
retention of bile, causing a functional biliary disorder that mimics a structural
lesion. Recurrent, episodic RUQ/Epigastric pain, with corresponding AST/ALT
and ALP elevations, is common; visualization of a dilated CBD in the absence of
stones increases the likelihood of SOD. Opioid analgesics (eg, morphine) may
cause sphincter contraction and precipitate symptoms of SOD.

Sphincter of Oddi manometry is the gold standard for the diagnosis of SOD;
sphincterotomy is the treatment of choice in most cases.
- Bile reflux gastritis occurs as a result of an incompetent pyloric sphincter (eg,
following gastric surgery), which allows retrograde flow of bile-rich duodenal fluid
into the stomach and esophagus; it would result in vomiting, frequent heartburn,
and abdominal pain.
- The presence of gallstones in the CBD is called choledocholithiasis. Patients have
RUQ pain and jaundice due to biliary obstruction; laboratory studies show elevated
direct bilirubin and elevated transaminases. It’s unlikely if there is no gallstones
on US.
-
……………………
- CT scan of the abdomen is the most SN method for diagnosing perforation (eg,
bowel wall, pancreatic/biliary duct), and it demonstrates intra-abdominal or
retroperitoneal free air. CT scan is less useful in the evaluation of post-ERCP
pancreatitis as findings may initially be normal.
- MR cholangiopancreatography (MRCP) is used to evaluate the biliary and
pancreatic ducts for stones, structural abnormalities (eg, biliary duct
perforation), and pancreatitis. However, MRCP is expensive and timely, and these
complications may be more easily assessed with CT, ultrasound, or laboratory tests.
HEMATEMESIS

- Boorehave: The leaked gastrointestinal contents cause severe chest/abdominal pain


and a systemic inflammatory response (eg, fever, tachycardia) that can quickly
progress to septic shock and death; the leaked air (pneumomediastinum) may be
palpated as suprasternal crepitus (subcutaneous emphysema).
These features distinguish esophageal perforation from Mallory-Weiss syndrome.
Intravenous broad-spectrum antibiotics and proton pump inhibitors should be
administered, all oral intake restricted, and emergent surgical consultation
obtained. Most perforations will require operative debridement and repair.

Once appropriate vascular access has been established, fluid resuscitation with
crystalloid should be undertaken while any necessary blood products are prepared
for transfusion (eg, pRBCs transfusion to maintain hemoglobin at least >7 g/dL). In
addition, patients' airway, vital signs, cardiac rhythm, and urine output should be
closely monitored. Patients with UGIB should also receive acid suppression with IV
PPI and have be NPO. If variceal bleeding is suspected, an octreotide infusion should
be considered. Patients with cirrhosis should also receive prophylactic antibiotics.

Once patients are stabilized → upper endoscopy should be performed to identify and
control the source of hemorrhage and to prevent recurrent episodes. In approximately
50% of cases of variceal bleeding, the hemorrhage ceases on its own without further
intervention; this rate is significantly lower than that seen in UGIB due to other
causes, which approaches 90%.

Contrast angiography should be considered in the small number of patients with


UGIB who are not able to be stabilized sufficiently to undergo upper endoscopy OR
in whom upper endoscopy is unsuccessful in controlling hemorrhage.
Balloon tamponade with the Sengstaken-Blakemore, Minnesota, or Linton-Nachlas
tubes can be used for the temporary control of bleeding when early endoscopy is
unavailable or other modalities are unavailable and medical management is
unsuccessful. This method is most often used to control hemorrhage until the
patient can be transferred to a tertiary care hospital; it is not considered first-line
treatment.

GASTROINTESTIN An elderly patient has an occult gastrointestinal bleed (OGIB), characterized by


AL (GI) unexplained IDA and/or positive fecal occult blood test (FOBT) in the absence of
HEMORRHAGE overt bleeding (eg, hematemesis, melena, hematochezia). Patients are usually asx.
Pallor (eg, conjunctivae, oral mucosa) may be seen, and laboratory studies often
demonstrate microcytic, hypochromic anemia (iron deficiency). Pathology may involve
the colon (eg, malignancy, polyps), upper GI tract (eg, peptic ulcer), or, less
commonly, the small bowel (eg, angiodysplasia). A positive FOBT suggests a colonic
bleeding source but does not rule out proximal pathology and/or colonic lesions that
are not actively bleeding.
Initial workup of OGIB includes colonoscopy, particularly in the presence of a positive
FOBT. Colon cancer is a likely cause in patients age >50. Upper endoscopy is also
generally performed, particularly if the patient has upper GI symptoms, if IDA is
present, or if colonoscopy (-) does not reveal a bleeding source.
Approx. 10% of patients with OGIB have simultaneous lesions in the upper GI tract
and colon. If colonoscopy and upper endoscopy both fail to identify a source of
bleeding, small bowel evaluation can be performed with video capsule endoscopy or
deep enteroscopy.

…….
The presence of hemorrhoids does not preclude endoscopic evaluation in patients with
an OGIB, as more serious pathology (eg, malignancy) may also be present. Anoscopy
can be used to visualize internal hemorrhoids or other anorectal pathology, but cannot
evaluate the colon for more proximal lesions. Medical (eg, topical hydrocortisone
cream) and surgical management (eg, hemorrhoidectomy) may eventually be
required in symptomatic patients, but are less of a priority than evaluation of an OGIB.

Treatment of IDA includes iron supplementation and periodic assessment of


hemoglobin to gauge therapeutic response. Although iron supplementation will likely
be required in this patient, it does not identify the source of blood loss. Failure to
evaluate may result in progression of the underlying disease.

PERIANAL - Indurated, erythematous mass near the anal orifice associated with severe, constant
ABSCESS anal pain and a low-grade fever, which is consistent with a perianal abscess. A
perianal abscess is due to occlusion of an anal crypt gland, which allows for
bacterial infection. Abscesses can form relatively acutely following gland
obstruction due to the high levels of bacteria in the area. Risk factors for
abscess development include anoreceptive intercourse and constipation, among
others. Initially, a perianal abscess may cause pain only with defecation and
mild pruritus, but as the infection progresses, the pain becomes constant and
can be associated with systemic manifestations such as fever. Drainage may not
be apparent unless a fistula forms.
Untreated perianal abscesses often progress to form anorectal fistulae,
communications between the abscess and perirectal skin or nearby organs. Early
recognition followed by incision and drainage is essential to avoid such
progression.
- External hemorrhoids originate below the dentate line; thrombosis can cause
the surrounding skin to become inflamed and edematous, resulting in exquisite
pain and tenderness. However, association with a fluctuant mass or fever is
unusual for hemorrhoids.
RECTAL ***** Clinical features of rectal prolapse
PROLAPSE
• Women age >40 with history of vaginal deliveries/multiparity
/////////////////// • Prior pelvic surgery
Risk
• Chronic constipation, diarrhea, or straining
/// factors
• Stroke, dementia
• Pelvic floor dysfunction or anatomic defects
*
** • Abdominal discomfort (not significant pain)
Clinical • Straining or incomplete bowel evacuation, fecal incontinence with mucus
***
presen • Digital maneuvers possibly required for defecation
**** • Erythematous mass extending through anus with concentric rings of rectum (can
tation
***** be intermittent)

• Medical
oConsidered for non–full-thickness prolapse
oAdequate fiber & fluid intake, pelvic floor muscle exercises
Manag
oPossible biofeedback for fecal incontinence
ement
• Surgical
o Preferred for full-thickness or symptomatic prolapse (eg,
fecal incontinence, constipation, sensation of mass)

An older woman has intermittent rectal protrusion of an erythematous mass,


suggesting rectal prolapse. In rectal prolapse, a mucosal or full-thickness layer of
rectal tissue slides through the anal orifice. It is MC in women age >40. Risk factors
include multiparity, vaginal delivery, pelvic surgery, pelvic floor dysfunction, chronic
constipation or straining, dementia, and stroke.

Rectal prolapse usually presents with stool incontinence or constipation; a palpable


mass is often present during defecation and may require manual reduction. Rectal
irritation can produce a mucous discharge that may be tinged with
blood. Although abdominal discomfort can occur, significant pain is not typical and
suggests another diagnosis. Physical examination often demonstrates a patulous
anus, and maneuvers that increase intraabdominal pressure (eg, Valsalva) can induce
prolapse, revealing a painless, erythematous mass with concentric rings extending
through the anus. The diagnosis is made clinically, and management includes increased
fiber intake, pelvic floor muscle exercises, and, possibly, surgical repair.

…………

polyps can rarely cause rectal prolapse; however, the examination would likely
demonstrate a prolapsed polyp (pedunculated mass) in addition to the
rectum. Colonic polyps are more commonly associated with increased risk of colonic
malignancy.

Crohn disease is associated with increased risk of perianal disease (eg, fissure, abscess,
fistula), not rectal prolapse.
HPV can lead to anogenital warts (cauliflower-shaped, fungating, skin-colored lesions)
and squamous cell carcinoma of the anus (ulcerating lesion often associated with
bleeding), not rectal prolapse.

Portal hypertension increases the risk of hemorrhoids. Although internal


hemorrhoids can occasionally prolapse, examination demonstrates bulging, purplish
veins, not a ringed mass. In addition, hemorrhoids typically result in bleeding during
defecation, not abdominal discomfort or passage of mucus.

DUMPNG Clinical features typical of early


SYNDROME dumping syndrome (DS) include
/////////////////// GI (eg, N, D, abdominal cramps)
and vasomotor (eg, palpitations,
**** diaphoresis) symptoms. DS is a
common postgastrectomy (e.g.,
partial distal gastrectomy after GI
bleeding) complication
occurring in up to 50% of
patients. It is caused by loss of
the normal action of the pyloric sphincter due to injury or surgical bypass and leads to
rapid emptying of hypertonic gastric contents into the duodenum and small
intestine. This causes fluid shifts from the intravascular space to the small
intestine, leading to HoTN, stimulation of autonomic reflexes, and release of
intestinal vasoactive polypeptides.

The diagnosis of DS is primarily based on clinical features, although an UGI x-ray


series or gastric emptying studies may be helpful if the diagnosis is unclear. Most
patients can be managed with dietary modification:
• Consume frequent, small meals and eat slowly
• Avoid simple sugars
• Increase fiber and protein
• Drink fluids between rather than during meals
- Symptoms of DS usually diminish over time. A minority of patients with
refractory symptoms may benefit from a trial of octreotide or require
reconstructive surgery, but this is not usually needed
………….
Endoscopy would confirm the postsurgical anatomy but would not otherwise
contribute to the diagnosis of DS.

This patient has no fever or leukocytosis to suggest antibiotic-associated colitis. The


postprandial onset with resolution of symptoms overnight is more consistent with DS
ESOPHAGEAL
Mallory-Weiss tear (MWT)
TEAR
• Sudden increase in abdominal pressure (eg, forceful retching,
**** BAT)
Etiology • Mucosal tear in esophagus OR stomach (submucosal arterial or
venous plexus bleeding)
• Risk factors: Hiatal hernia, alcoholism

• Vomiting, retching
Clinical
• Hematemesis
presentation • Epigastric pain

Diagnosis • Longitudinal laceration on endoscopy

• Most heal spontaneously


Treatment • Endoscopic therapy for persistent bleeding

A patient with alcohol abuse, alcoholic hepatitis (2:1 ratio of AST:ALT), esophagitis, and
gastritis now has acute, bright red hematemesis following multiple episodes of
vomiting. This presentation is highly suggestive of a Mallory-Weiss tear (MWT). MWT
is characterized by longitudinal tears in the mucosa near the GE junction, with
bleeding due to injury to the submucosal arteries or veins. It typically occurs due to a
sudden increase in intraabdominal pressure (eg, retching, blunt abdominal trauma
[BAT]).

MWT is a common cause of UGI hemorrhage and is often seen in association


with alcohol abuse and hiatal hernia.

The diagnosis can be confirmed on endoscopy.

Tx: Bleeding stops spontaneously in 90% of patients. Those with ongoing bleeding can
be treated endoscopically with:

1. Electrocoagulation or
2. Local injection of epinephrine.
………………

Esophageal rupture can be caused by severe retching (Boerhaave syndrome),


penetrating trauma, or as a complication of endoscopy. It is characterized by acute
chest pain, subcutaneous emphysema, and often a left-sided pleural effusion. When
esophageal rupture occurs due to endoscopy, it is usually apparent shortly after the
procedure.

Stress ulcers develop in the setting of severe and prolonged physiological stress. GI
bleeding from stress ulcers is usually seen in patients in an intensive care or burn unit
setting.

STEATORRHEA chronic pancreatitis may initially be asymptomatic, but disease progression ultimately
leads to malabsorption, steatorrhea, and postprandial epigastric Pain that can become
continuous. Symptomatic
improvement can be
achieved with alcohol
cessation and pancreatic
enzyme
supplementation.

Antimicrobial therapy
(eg, rifaximin) should be
considered in patients
with small intestinal
bacterial overgrowth
syndrome, which can
present with abdominal
pain/bloating and
symptoms of
malabsorption. However,
this condition is typically
associated with
anatomical abnormalities (eg, surgical blind loop) or motility disorders (eg,
scleroderma) of the gastrointestinal tract.
Small intestinal bacterial overgrowth (SIBO)

• Anatomical abnormalities (eg, strictures, surgery)


Etiology • Motility disorders (eg, diabetes mellitus, scleroderma)

• Abdominal pain, diarrhea, bloating, flatulence


S/S
• Malabsorption, weight loss, anemia, vitamin deficiency

• Jejunal aspirate & culture showing >105 organisms/mL


Diagnosis • Carbohydrate breath testing (eg, lactulose, glucose)

Organisms • Streptococci, Bacteroides, Escherichia, Lactobacillus

• Antibiotics (eg, rifaximin, amoxi-clav)


• Avoid antimotility agents (eg, narcotics)
Treatment • Dietary changes (eg, high-fat, low-carbohydrate)
• Promotility agents (eg, metoclopramide)
A patient has abdominal bloating, steatorrhea (elevated fecal fat), and mAcrocytic
anemia. In the setting of a Roux-en-Y gastric bypass, this presentation
suggests small intestinal bacterial overgrowth (SIBO). The proximal small intestine
normally contains minimal bacterial colonization due to gastric acidity and peristalsis;
however, gastric bypass procedures result in a blind loop of intestine that allows for
excessive bacterial growth. Conditions that alter intestinal motility (eg, systemic
sclerosis, DM), anatomy (eg, strictures), or gastric/pancreatic secretions (eg, atrophic
gastritis, chronic pancreatitis) also predispose to SIBO.

Patients with SIBO typically develop bloating, flatulence, abdominal discomfort, and
watery diarrhea; in severe cases, malabsorption (eg, steatorrhea) and weight loss
are seen. Laboratory studies are frequently normal but may demonstrate nutritional
deficiencies (eg, vitamin B12, fat-soluble vitamins) and associated macrocytic
anemia.

Diagnostic tests include carbohydrate breath tests (eg, lactulose, glucose) that
measure the production of hydrogen by intestinal flora. Endoscopy with jejunal
aspirate and culture showing increased bacterial burden are also used.

Management involves correction of the underlying abnormality, dietary modification,


and a short course of empiric antibiotics (eg, rifaximin).

Dumping syndrome (DS) is a complication of gastric bypass and occurs when high-
carbohydrate foods are rapidly emptied into the small bowel, leading to osmotically
driven fluid shifts from the plasma to the intestine. Although patients have abdominal
pain and diarrhea soon (<30 min) after meals, the fluid shifts result in sympathetic
activation, leading to tachycardia, diaphoresis, and flushing. Hypoglycemia may also
occur. DS is not associated with steatorrhea or vitamin B12 deficiency (ie, macrocytic
anemia).

Patients with Giardia lamblia infections commonly have steatorrhea and diarrhea,
flatulence, fatigue, and significant weight loss. Affected patients frequently have a
history of immunosuppression or recent travel.

Mesenteric angioplasty should be considered in patients with chronic mesenteric


ischemia, which can present with postprandial abdominal pain (intestinal angina)
and weight loss due to food aversion. However, steatorrhea is not characteristic.

ABDOMINAL Abdominal surgery can result in a variety of postoperative wound complications (eg,
SURGERY infection, dehiscence). These complications are more common after procedures with:
COMPLICATIONS
1. Large incisions
2. Patients with risk factors for poor wound healing (eg, ca, chronic
corticosteroids, hypoalbuminemia) or
3. Increased intraabdominal pressure (eg, cough, obesity).
Postoperative abdominal wounds can be categorized based on fascial involvement:
• Superficial wound dehiscences are separations of the skin and subcutaneous
tissue with intact rectus fascia. These typically develop within the first postop
week due to an abnormal subcutaneous fluid buildup (eg, seroma) and can
often be managed conservatively with careful dressing changes.

• Deep (fascial) wound dehiscences involve the rectus fascia (ie, nonintact) and
result in exposure of the intraabdominal organs to the external environment.

wound dehiscence complicated by evisceration. Additional clinical features include


heavy drainage of peritoneal fluid (pink/straw/serosanguinous), local bulging, and a
popping sensation (from fascial suture failure). Fascial dehiscence is typically seen in
the first 1-2 weeks postop (up to 30 days). Patients with fascial dehiscence,
particularly with evisceration, require emergency surgery because the eviscerated
bowel is at risk for continued herniation and strangulation.

Abdominal binders may be placed briefly in preparation for surgery in patients with
fascial dehiscence to prevent evisceration. They are C/I in patients with bowel
evisceration because they can cause strangulation and perforation.
In cases of suspected wound dehiscence not apparent on examination, an ultrasound
or CT scan may be indicated.
Wound packing with saline gauze dressings is appropriate for management of
superficial wound dehiscences. Negative pressure dressings are used to prevent
dehiscence in high-risk wounds.

RETROPERITONE
AL HEMATOMA
SPLENIC ABSCESS This patient who recently underwent laparoscopic cholecystectomy for cholecystitis
now has high fever and tender splenomegaly, raising strong suspicion for splenic
abscess, a rare, potentially life-threatening complication of bacteremia from a distant
infection (eg, infective endocarditis, cholecystitis). Risk is greatest in those with
immunocompromise from HIV, hematologic malignancy, or diabetes mellitus.
Patients usually have persistent fever and LUQ pain (sometimes radiating to the back),
with or without splenomegaly. Anorexia and weight loss may also be seen. Labs will
show leukocytosis with left shift, and CXR frequently reveals an elevated left
hemidiaphragm (and/or left pleural effusion).

Diagnosis is made with CT scan of the abdomen;


Tx: Antibiotic therapy PLUS splenectomy is generally required because many patients
fail percutaneous aspiration (due to the presence of occult microabscesses).
- Pancreatic pseudocyst, a walled-off collection of fluid around the pancreas
following pancreatitis, is generally asymptomatic but may occasionally become
infected, leading to fever and epigastric pain that radiates to the back. However,
splenomegaly would be atypical.
- Non-Hodgkin lymphoma can present with splenomegaly, weight loss, and
waxing/waning fevers. However, high fevers are uncommon, and significant splenic
tenderness/pain rarely occurs.
- Splenic infarction usually presents with left upper quadrant pain. Although fever
and splenomegaly can occasionally occur, most patients with splenic infarction
have an underlying hypercoagulable disorder, a source of embolic disease (eg,
atrial fibrillation), a myeloproliferative neoplasm, or hemoglobinopathy (eg, sickle
cell disease).
- Splenic venous thrombosis usually occurs in the setting of portal hypertension
from liver disease or pancreatitis. Although splenomegaly can occur, patients
would not typically have high fever and weight loss.

SUBPHRENIC A patient underwent a laparoscopic


ABSCESS appendectomy 10 days ago and now has
RUQ pain, fever, leukocytosis, and
pulmonary manifestations (SOB, hiccups,
right-sided effusion), suggesting
a subphrenic abscess.
Infections are the MC complication of
appendectomy, and risk of intra-abdominal
abscess is significantly greater with
laparoscopic appendectomy than
laparotomy. Manifestations typically include
recurrent fever and abdominal symptoms
(eg, pain, vomiting) several days after an abdominal operation. Diagnosis usually
requires CT scan of the abdomen. Most patients are treated with drainage and
intravenous antibiotics.

Manifestations typically include fever, RUQ pain (which may be severe), leukocytosis,
and elevated LFTs. The diagnosis is confirmed with abdominal imaging; CT scan
classically demonstrates a well-defined, hypoattenuating, rounded lesion, often
surrounded by a peripherally enhancing abscess membrane.

Management includes blood cultures, antibiotics, and percutaneous


aspiration and drainage, which is both diagnostic (eg, culture, Gram stain) and
therapeutic; catheter placement is often required to drain large lesions. Surgical
drainage can be considered for abscesses not amenable to percutaneous intervention.

Malaria can present with fevers, anemia, and hyperbilirubinemia; however, it would not
be expected to cause right upper quadrant pain or a hypoattenuating liver lesion on CT
scan. It would also be unexpected in a patient who has not traveled recently.

Echinococcus: Cysts are typically visible on a CT scan as thin-walled, septated


lesions, occasionally with calcification.
Entamoeba histolytica is a major cause of hepatic abscess; however, most patients with
amebic liver abscess have a history of exposure in endemic areas (eg, India,
Mexico). This diagnosis should be considered in patients without other, more common
causes of abscess (unlike this patient with recent diverticulitis). Testing options include
serology or stool analysis (eg, ova and parasite).
C difficile infection is characterized by persistent, watery diarrhea and would not be
expected to cause abscess formation.

HAP: most cases occur ≤5 days postop

Postoperative atelectasis is common up to 5 days after abdominal procedures and


often presents with hypoxemia and dyspnea. Fever, significant leukocytosis, abdominal
pain, and reduced bowel sounds are not typically associated with this condition.
ANORACTAL
Anorectal fistula (fistula in ano)
FISTULA (fistula
in ano) • Perianal abscess
• Crohn disease (CD)
Causes • Malignancy, radiation proctitis
• Infection (eg, lymphogranuloma venereum [LGV])

• Perirectal pain, discharge


Clinical
• Inflammatory papule/pustule
manifestations • Palpable fistula tract

• Assess extent of fistula


o Gentle probe
Management o Imaging (endosonography, fistulogram, MRI)
• Surgery (eg, fistulotomy)

A patient with an inflamed perianal lesion associated with pain on defecation and
chronic discharge has an anorectal fistula (fistula in ano). Anorectal fistulas are most
often due to rupture of a perianal abscess with formation of a residual sinus
tract. They may also occur as a complication of Crohn disease, radiation proctitis,
atypical infections (eg, lymphogranuloma venereum), or trauma.

Examination of an anorectal fistula often reveals an external terminus and an indurated


tract leading to the rectum. An internal terminus can sometimes be identified on
anoscopy or by cautiously passing a probe through the fistula from the external
opening. The diagnosis is often apparent on clinical grounds, but endoscopic
sonography, MRI, or fistulogram can be used in complex cases to assess the extent of
fistula formation.

Management of an anorectal fistula requires surgical intervention (eg,


fistulotomy). Fistulas can often be assessed more fully (eg, exploration with a soft
probe) while patients are under anesthesia at the time of surgery to delineate the
extent of the fistula. The entirety of the fistula must be addressed because residual
fistula tracts can lead to persistent symptoms and fecal incontinence.
Anorectal manometry can provide information regarding anorectal neurologic
dysfunction (eg, decreased relaxation of puborectalis and anal sphincter) and is
indicated for evaluation of chronic constipation and fecal incontinence. Patients with
anorectal fistula can have leakage of stool, but neurologic function is typically normal.

Flexible sigmoidoscopy is occasionally needed to identify the internal fistula


opening if it is not apparent on anoscopy. However, colonoscopy is typically only
necessary for:

1. Complex,
2. Recurrent,
3. Atypical fistulas or
4. Fistulas associated with IBD.

Some perianal fistulas associated with Crohn disease can be managed with antibiotics
and medical treatment of the underlying inflammatory bowel disease. For fistulas
arising from a perianal abscess, nonsurgical treatment has a high failure rate..

ISCHEMIC This patient has developed ischemic hepatitis (shock liver) resulting from myocardial
HEPATITIS infarction, ventricular tachycardia, and cardiogenic shock. Ischemic hepatitis usually
occurs in the setting of global hypoperfusion and/or hypoxemia; in addition to
cardiac insults, other common causes include hypovolemic and septic shock as well as
respiratory failure. Histologically, hepatocyte necrosis is found predominantly
in zone 3 of the liver lobule, ie, surrounding the central veins where cells are farthest
from the oxygen supply coming from the hepatic artery (~40% of the liver's oxygen
supply) and the portal vein (~60% of the liver's oxygen supply as partially
deoxygenated blood).
Patients often develop aminotransferase levels >1000 U/L shortly after the inciting
incident. Although the bilirubin level can be unaffected, it usually rises several days
later. Prognosis is poor as patients are typically critically ill and often have
accompanying MOF. In those who survive, the hepatic injury tends to be self-limited;
aminotransferase elevations usually resolve within 7 days, and bilirubin levels
normalize over the course of several weeks.

Acalculous cholecystitis usually occurs as a complication of shock or other critical


illness. Mild elevations in total bilirubin, ALP, and ATs are typical; however, ATs
elevation >1000 U/L would not be consistent.

Like ischemic hepatitis, acute viral hepatitis can cause elevations in aminotransferases
>1000 U/L, most commonly from acute infection with hepatitis A or B. Acute hepatitis
A, which is transmitted fecal-orally, tends to present with fever/N/ V. Patients with
acute hepatitis B often have identifiable risk factors, including IVDU, recent blood
transfusions, or unprotected sexual intercourse.
A patient was exposed to amiodarone, which can cause hepatic steatosis and
cholestasis. This would cause a rise in bilirubin, but marked aminotransferase
elevations >1000 U/L are not typical. Acute liver failure due to amiodarone toxicity
is rare.

AST is present in cardiac myocytes, skeletal myocytes, and intestinal smooth


muscle, and serum levels can rise following injury to any of these tissues. However,
alanine aminotransferase (ALT) is specific to hepatocytes and would not increase in the
absence of hepatic injury.

///////////////////RHEUMATOLOGY/ORTHOPEDICS & SPORTS///////////////////


CLAVICLE
Signs of traumatic arterial injury
FRACTURE
• Distal limb ischemia (6Ps; eg, paralysis, pain, pallor,
*** poikilothermia)
Hard signs
• ABSENT distal pulses (6Ps)
(require immediate • Active hemorrhage or rapidly expanding hematoma (active
surgery) internal hrg)
• Bruit or thrill at site of injury

• DIMINISHED distal pulses


Soft signs • Unexplained HoTN
(require further • Stable hematoma
imaging) • Documented hemorrhage at time of injury (no active hrg)
• Associated neurologic deficit

Clavicle fracture is a common


sports-related injury and can
occur following a blow or fall
on the shoulder or
outstretched arm (FOOSH). The
vast majority are uncomplicated
and carry a favorable prognosis.
However, the middle third of
the clavicle overlies the brachial
plexus and the subclavian
artery and vein in the thoracic
outlet. Therefore, clavicular
fractures require a careful
neurovascular assessment.

The presence of "hard


signs" (eg, absent distal pulses,
bruit at injury site) following a
fracture indicate definite arterial injury and require IMMEDIATE surgical intervention.
However, the presence of a stable hematoma, reduced pulse, documented
hemorrhage at the time of injury, unexplained hypotension, or an associated
neurologic deficit also suggest an arterial injury may have occurred. These "soft
signs" require vascular imaging to confirm and localize the site of injury. CTA is the
diagnostic modality of choice due to its high sensitivity and specificity and rapid
procedure time.

……………

Uncomplicated fractures of the middle third of the clavicle, which account for most
clavicular fractures, are treated nonoperatively with rest, ice, and either a sling
or figure of eight bandage. Fractures of the distal third of the clavicle may require
open reduction and internal fixation (ORIF) to prevent nonunion.

A clavicle fracture may rarely injure the brachial plexus. Features suggesting
neurologic injury include numbness, paresthesia, and upper extremity
weakness. Clinical examination is usually adequate to rule out significant brachial
plexus injury in the acute phase. If a chronic thoracic outlet syndrome develops, nerve
conduction studies could be considered later.

Upper extremity venous duplex imaging is indicated for venous thrombosis or


obstruction; signs of venous obstruction include upper extremity edema or
cyanosis. Acute hematoma is more consistent with an arterial injury. Although
arterial duplex doppler can be used to evaluate arterial injury, it is time
consuming and less sensitive than CTA.

ROTATOR CUFF
Rotator cuff tendinopathy & tear

Rotator cuff • Pain with abduction, external rotation


• Subacromial tenderness
impingement or
• Normal range of motion with positive impingement
tendinopathy tests (eg, Neer, Hawkins)

• Similar to rotator cuff tendinopathy


Rotator cuff tear • Weakness with abduction & external rotation
• Age >40
This patient with impaired abduction at the
shoulder following a fall has a rotator cuff tear.
The rotator cuff is formed by the tendons of the
supraspinatus, infraspinatus, teres minor, and
subscapularis muscles. The supraspinatus is
MCly injured due to degeneration of the tendon
with age and repeated ischemia induced by
impingement between the humerus and the
acromion during abduction.

A complete supraspinatus tear causes weakness


of abduction, which can be appreciated in
the drop arm test. In this test, the patient's arm
is abducted above the head and the patient is
asked to lower the arm slowly. With a complete
tear, the patient will be unable to lower the arm
smoothly and it will drop rapidly around mid-
descent. Although the supraspinatus is the
primary muscle responsible for initiating the first
15 degrees of abduction, the loss of smooth
adduction in the drop arm test typically occurs
when the humerus is near the horizontal plane.

MRI can confirm the diagnosis,

Treatment usually requires surgery.

....
 Rupture of the tendon of the long head of the biceps occurs in
forceful flexion of the arm. It produces sudden pain with an
audible pop and a visible bulge ("Popeye sign") where the biceps
muscle retracts into the medial arm.

Fracture of the surgical neck of the humerus may cause axillary


nerve injury with paralysis of the deltoid and teres minor
muscles (TMn) and sensory loss over the lateral upper
arm. However, patients typically have bony tenderness, swelling,
ecchymosis, or crepitus over the fracture. In addition, the drop
arm test is more reflective of supraspinatus function. Although the deltoid is
responsible for shoulder abduction between 15-90 degrees, deltoid weakness is best
appreciated at extreme extension rather than abduction.
……….
Injury to the long thoracic nerve causes weakness of the serratus anterior with
impairment at extreme abduction (>90 degrees) due to inability to rotate the scapula
upward. It is usually caused by penetrating trauma or medical/surgical procedures
(eg, chest tube insertion).

The lower (inferior) trunk of the brachial plexus originates from the C8 and T1 cervical
roots and can be injured in sudden upward traction on the arm (Klumpke
palsy). Muscles supplied by the ulnar nerve (eg, intrinsic muscles of the hand) are
commonly affected, which results in weakness and atrophy of the hypothenar and
interosseous muscles and "claw hand" deformity.

Common causes of shoulder pain

Rotator cuff • Pain with abduction, external rotation


• Subacromial tenderness
impingement or
• Normal range of motion with positive impingement
tendinopathy tests (eg, Neer, Hawkins)

• Similar to rotator cuff tendinopathy


Rotator cuff tear • Weakness with abduction & external rotation
• Age >40

Adhesive capsulitis • Decreased passive & active ROM


(frozen shoulder) • Stiffness ± pain

• Anterior shoulder pain


Biceps tendinopathy
• Pain with lifting, carrying, or overhead reaching
or rupture • Weakness (less common)

• Uncommon & usually caused by trauma


Glenohumeral
• Gradual onset of anterior or deep shoulder pain
osteoarthritis • Decreased active & passive abduction & external rotation

A patient with subacute shoulder pain on abduction most likely has rotator cuff
tendinopathy (RCT). RCT results from repetitive activity above shoulder height (eg,
painting ceilings) and is common in middle-aged and older individuals. Chronic tensile
loading and compression by surrounding structures can cause microtears, fibrosis,
and inflammatory calcification in the rotator cuff tendons (especially supraspinatus).
Pain may also emanate from the subacromial bursa and the tendon of the long head
of the biceps.

On flexion or abduction of the humerus, the space between the humeral head and
acromion is reduced, causing pressure on the supraspinatus tendon and subacromial
bursa. Impingement syndrome, a characteristic of RCT, refers to compression of these
soft tissue structures. Impingement can be demonstrated with the Neer test: With the
patient's shoulder internally rotated and forearm pronated, the examiner stabilizes the
scapula and flexes the humerus. Reproduction of the pain is considered a positive test.

Untreated, chronic RCT can increase the risk for rotator cuff tear. Patients with a tear
typically present with weakness of abduction following a fall or other minor trauma.
………..

Adhesive capsulitis ("frozen shoulder") is characterized by fibrosis and contracture of


the glenohumeral joint capsule. It can occur in patients with underlying shoulder
pathology (including RCT), but examination would show decreased passive ROM.

Acromioclavicular arthritis is common in patients with chronic overuse of the


shoulder. It presents with pain in the shoulder when lifting the arm overhead or across
the body, and frequently coexists with other overuse injuries (including
RCT). However, patients typically have tenderness, swelling, or deformity in the
acromioclavicular space.

Pain can be referred to the shoulder from cervical nerve root impingement,
diaphragmatic irritation (eg, intrathoracic tumor), myocardial ischemia, or
hepatobiliary disease. Cervical radiculopathy typically presents with pain and
paresthesias of the neck and arm along with upper extremity weakness. However,
referred pain would not cause impingement signs and would not be exacerbated by
shoulder movement.

Rotator cuff tendinopathy & tear

Rotator cuff • Pain with abduction, external rotation


• Subacromial tenderness
impingement
• Normal range of motion with positive impingement
or tendinopathy tests (eg, Neer, Hawkins)

• Similar to rotator cuff tendinopathy


Rotator cuff tear • Weakness with abduction & external rotation
• Age >40

This patient has post-traumatic shoulder pain with limited active abduction, most
likely due to a rotator cuff tear. Rotator cuff tears are MC in patients age >40, often
after a fall on an outstretched arm (FOOSH). Patients typically have pain at the lateral
shoulder that is worsened with raising the arm overhead or with external rotation and
abduction. Examination shows the drop arm sign (inability to smoothly control
shoulder adduction) and weakness with external rotation (passive range of motion is
normal). Rotator cuff tendinopathy (tendinitis) may also present with pain on
abduction, but tendinopathy without tear does not cause weakness.

X-ray of the shoulder in a rotator cuff tear is often performed


as an initial study to rule out fracture; in large, chronic tears
it may demonstrate migration of the humeral head, but it has
low sensitivity for confirming the diagnosis of an acute
tear. MRI can visualize the soft tissues around the humeral
head and can accurately diagnose a rotator cuff
tear. Treatment of an acute tear usually involves surgery,
with best results if performed within 6 weeks of the injury.
……….

Bone scans are most often used to diagnose infectious (eg, osteomyelitis) or metastatic
disease. However, these conditions would not usually present with acute post-
traumatic pain and decreased range of motion.

Lung cancer involving the brachial plexus (Pancoast syndrome) could cause shoulder
pain. However, this would typically present with radicular pain radiating to the wrist,
atrophy of the muscles, and additional compressive symptoms (eg, edema).

CT of the shoulder is effective in diagnosing fractures and other disorders involving


cortical bone but is less sensitive than MRI for visualization of tendons and other soft
tissue structures.

Signs of narrowed subacromial space and of impingement


• Painful arc: pain with abduction of the arm between 60–120°
• Neer test: The internally rotated and outstretched arm is passively elevated, while
the scapula is simultaneously stabilized; the movement causes pain.
HUMERUS A patient with arm pain, deformity, and swelling and evidence of radial nerve injury (eg,
FRACTURE limited extension at wrist joint) has a midshaft humeral fracture, confirmed by x-ray.
Fractures of the humeral shaft are generally seen following a direct blow to the arm, a
fall, an automobile injury, or a crushing injury. The radial nerve is commonly stretched
in such fractures, leading to limited wrist extension and associated sensory deficits;
however, most cases represent neurapraxia that resolves after management of the
fracture.
Although most cases of isolated diaphyseal (midshaft) humeral fracture can be
treated nonsurgically (eg, closed reduction followed by arm immobilization with a
coaptation or sugar-tong splint), open reduction and surgical exploration are
indicated with open fractures, significant displacement (eg, arm
shortening), neurovascular compromise (eg, asymmetric radial pulses, as in this
patient), polytrauma, and pathologic fractures.

…….

A neutral wrist splint alone might help radial nerve symptoms but would not
provide definitive management of the fracture. A long arm hanging cast (with the
cast hung from the neck and the elbow flexed at 90 degrees) can be used in some
cases of midshaft humeral fracture, but neurovascular compromise is an indication
for open reduction and surgical exploration.
Although an EMG can evaluate nerve injuries, radial nerve neurapraxia due to
midshaft humeral fracture resolves in up to 90% of cases, including with nonsurgical
management. EMG can be considered if radial nerve function does not recover
following surgery.
External fixation (eg, with external pins) is used in cases of polytrauma; severe,
comminuted open fractures with defects in skin, bone, or soft tissue; and some burn-
related injuries.

Supracondylar fractures of the humerus are the most common elbow fractures in
children. The mechanism typically involves a fall onto an outstretched hand (FOOSH)
with a hyperextended elbow. The supracondylar area is relatively thin and weak due
to physiologic remodeling during childhood, and a supracondylar fracture MCly results
in posterior displacement of the distal humerus fragment, as seen in this x-ray. The
anteriorly displaced proximal fragment can entrap the median nerve and brachial
artery, which are located anterior
to the humerus.

Distal perfusion should be


monitored closely because
brachial artery injury results in
diminished brachial and radial
pulses. Motor and sensory
function should also be assessed
due to the risk of median nerve
injury that can present with a
weak "OK" sign (ie, inability to
approximate tip of thumb and
index finger).

Diagnosis is based on x-ray, which


reveals a fracture line (red arrows) and often a posterior fat pad (blue arrows)
indicative of elbow effusion. Treatment involves analgesia and immobilization.
Neurovascular injury often resolves after orthopedic alignment and immobilization.
Patients who are treated promptly have a good prognosis.
………

The axillary artery and nerve can be injured during proximal humerus fractures but not
with supracondylar fractures.

Brachial plexus compression is a complication of neonatal clavicle fracture or high-


impact trauma to the neck and shoulder area, not the elbow.

**Ulnar nerve injury is a complication of a rare type of supracondylar fracture that


occurs upon impact of a FLEXED (rather than hyperextended) elbow. This
mechanism of injury causes anterior displacement of the distal humerus fragment,
which can impinge on the ulnar nerve located posterior to the medial epicondyle of
the humerus.

LONG BONE Forearm fractures are common in children and most


FRACTURE frequently involve the distal radius and/or ulna after
a fall onto an outstretched hand (FOOSH). Young
children (age <10) have porous bone and thick
periosteum, which allows for compression of the
distal metaphyseal cortex after an injury. This results
in a buckle fracture, which is considered a stable
and incomplete fracture because it involves only one
side of the bone.

Buckle fractures present with pain and tenderness


over the fracture site. Wrist range of motion may be
intact or slightly limited, and swelling is often
absent. X-ray is required to confirm the diagnosis
and exclude a greenstick or complete
fracture. Rather than a fracture line, a cortical
bulge (blue and red arrows) is visualized.

Treatment involves pain control and prevention of reinjury


(eg, splint placement). Most buckle fractures heal within a few weeks
without complication.
……….

 Avascular bone necrosis most


commonly involves the hip (Legg-
Calvé-Perthes disease) in children
and presents with chronic joint pain
and decreased range of motion. X-
ray findings with prolonged disease
include
bony fragmentation and sclerosis.

 Dietary vitamin D deficiency can lead to


impaired bone mineralization (rickets), which can
result in bowing of the long bones
and fraying/cupping of the distal radius and ulna
with widening of the wrist growth plate. These
findings are not seen on this patient's x-ray.

 Hematogenous osteomyelitis, a bone infection,


typically presents with fever and bony tenderness,
swelling, and erythema. X-ray is often normal in
the first few days to weeks of symptoms but later
shows soft tissue swelling
and periosteal elevation.
 Osteosarcoma is the most common primary bone
malignancy in children and typically presents with localized
bone pain and a softtissue mass.

SALTER HARRIS
FRACTURE

This patient has an ankle injury with diffuse swelling and an inability to bear weight;
initial imaging with x-ray is appropriate, although a CT scan may be needed to fully
visualize a fracture. The CT scan reveals a combination of: a distal tibial
epiphysis fracture; and a growth plate (ie, physeal) fracture, as suggested by growth
plate widening. This combination of fractures is known as a Salter-Harris type
III (juvenile Tillaux fracture).

The distal tibial growth plate


typically closes around age 14 in
boys (age 12 in girls), and fractures
involving the maturing, partially
fused, physis (eg, Salter-Harris type
III and IV fractures) carry an
increased risk for long-term
complications. In particular, injury
to the growth plate can
cause growth arrest and lead to
persistent limb-length
discrepancy. Other complications
include formation of physeal bars
(bony bridges across the growth
plate), premature osteoarthritis,
and decreased ROM.

...........
Acute limb ischemia is most often
due to arterial occlusion. When
seen with limb fractures, it typically
occurs soon after high-velocity
femoral fractures and would be
accompanied by signs of vascular
impairment (eg, loss of distal
pulses, reduced sensation, pallor,
cyanosis).
Post-fracture AVN is most common
in areas of bone with limited blood
supply such as
the scaphoid, femoral head, or metadiaphyseal junction of the FIFTH metatarsal,
but it is uncommon in distal tibia fractures.

Compartment syndrome results from increased pressure within nondistensible fascial


planes. It is uncommon in ankle fractures because the bulk of the leg muscle bellies
are proximal to the injury.
Traumatic foot drop is typically caused by injury to the common peroneal nerve
where it wraps around the lateral neck of the proximal fibula. Distal fractures of the
tibia or fibula would not cause foot drop.
COLLES
FRACTURE

This patient with significant wrist deformity


has a distal radius (Colles) fracture, which
commonly occurs after falling on an
outstretched hand (FOOSH), particularly in
athletes (high-impact falls) or elderly
patients with osteoporosis (low-impact falls).
Characteristic manifestations include pain,
swelling, and dinner fork deformity of the
wrist. In addition, severely displaced
fractures may result in neurovascular
compromise; therefore, careful assessment
of pulse, capillary refill, and sensation is indicated.

DORSAL (VS Smith) displacement of the radius (as occurs in Colles fracture) can
result in compression of the MEDIAN nerve, which enters the wrist through the carpal
tunnel and provides sensation to the lateral 3½ digits and motor innervation to the
thenar muscles (eg, opponens pollicis, abductor pollicis brevis). Compression results in
acute carpal tunnel syndrome (CTS) symptoms, including paresthesia of the affected
digits and impaired thumb abduction by abductor pollicis brevis. Furthermore, if
compression occurs proximal to the tunnel, the palmar cutaneous branch of the
median nerve may be affected, leading to decreased sensation over the anterolateral

hand.

……..

**Adduction of the index, 4th digit, and 5th digit (toward the 3rd digit) is mediated by
the palmar interossei (PAD), which are innervated by the deep branch of the ulnar
nerve. Sensation in the 5th digit (and medial aspect of the 4th) is carried by the ulnar
nerve. This nerve can be injured at the elbow due to external compression (eg, hitting
the "funny bone") or at the wrist due to a HAMATE fracture or external
compression (eg, bicycle handlebar). However, it is not typically affected by a radius
fracture.
Injury to the radial nerve at the elbow can cause weakness of hand/finger extensor
muscles and sensory loss over the posterior forearm/dorsolateral hand. This is typically
seen in supracondylar humerus fractures. Compared to the median nerve, the radial
nerve is less likely to be injured in a Colles fracture.

DEQUERVAIN’S
TENOSYNOVITIS

This patient has lateral wrist pain consistent with de Quervain tendinopathy
(DQT). DQT is an overuse syndrome involving the tendons of the abductor pollicis
longus (AdPL)and extensor pollicis brevis (EPB) and occurs most prominently at the
point where the tendons pass under the extensor retinaculum in the first dorsal
compartment. It most commonly occurs in women age 30-50 and is seen in higher
frequencies 4-6 weeks
postpartum, possibly because
of repetitive thumb abduction
and extension when lifting the
infant.
The diagnosis of DQT is based
on clinical features.
Examination typically shows
tenderness at the radial side
of the wrist at the base of the
hand. In addition,
the Finkelstein
test (adduction of the wrist with the fingers closed over the thumb in a fist) causes
passive stretching of the tendons over the radial styloid; reproduction of the pain in
this maneuver is strongly suggestive of DQT. Conservative management with NSAIDs
(eg, ibuprofen) and thumb spica splinting is usually adequate.

...........
Flexor carpi radialis (FCR) tenosynovitis is associated with excessive wrist flexion and
characterized by pain and tenderness at the anterior aspect of the wrist. It is relatively
uncommon; overuse of this muscle more commonly causes symptoms at the proximal
origin (ie, medial epicondylitis).

OA of the first MCP joint causes pain over


that joint, not at the CMC joint. It would
be uncommon in a young patient without
a history of significant trauma.

Scaphoid fractures typically occur during


forceful hyperextension of the wrist (eg
FOOSH). Pain and tenderness typically
localize to the anatomic snuffbox.
Stenosing tenosynovitis ("trigger
thumb") results in pain over the palmar
aspect of the first MCP joint; the pain is
associated with a catching sensation
during movement or locking of the
thumb in flexion.
SCAPHOID Scaphoid is the most commonly
FRACTURE fractured carpal bone. Scaphoid
fractures typically present with pain
at the radial wrist proximal to the
base of the thumb. Examination
shows tenderness in the shallow
depression at the dorsoradial wrist
bounded medially by the tendon of
the extensor pollicis longus (EPL)
and laterally by the tendons of the
abductor pollicis longus (APL) and
extensor pollicis brevis (EPB;
"anatomic snuffbox"). Scaphoid
fractures carry a significant risk
of osteonecrosis because the blood
supply enters at the distal pole and
flows proximally, and can be disrupted by the fracture.
Initial x-rays can be normal in nondisplaced scaphoid fractures. If scaphoid fracture is
suspected, CT scan or MRI can confirm the diagnosis, or repeat x-rays can be
performed in 7-10 days.

Displaced fractures should be considered for surgical intervention. Wrist


immobilization with a cast can be considered for nondisplaced fractures, but patients
should be monitored with serial x-ray to rule out osteonecrosis of the proximal
segment and nonunion of the fracture.

…………..
Bacterial tenosynovitis typically occurs following penetrating injury to the hand (eg,
cat bite) or from hematogenous spread of a distant infection (eg, N gonorrhoeae).

Fat embolism is most common following major fracture of long bones (eg, femur) or
extensive SOFT-tissue injury (eg, burns).
Synovial cysts of the wrist (ganglion cysts) are MC in individuals age 15-40 and are
thought to arise in most cases due to repetitive stress or inflammation.

LUNATE dislocation following a FOOSH can cause compressive neuropathy of the


median nerve (VS hamate # → ulnar n.). Nerve injury is not common following
scaphoid fracture.
Scaphoid fractures are the MC carpal bone
fractures. They frequently result from FOOSH
that cause axial compression or wrist
hyperextension. The arterial supply to the
scaphoid (from the radial artery) enters through
foramina in the bone's distal pole before
proceeding to the proximal pole; scaphoid
fracture can disrupt flow to the proximal
segment, leading to AVN and nonunion.

If initial x-rays are negative, CT scan or MRI of


the wrist is recommended to confirm the
fracture. As an alternative, the wrist can
be immobilized briefly in a thumb spica splint,
followed by repeat imaging in 7-10
days. EITHER approach can be considered;
repeat x-ray is associated with lower up-front
costs, whereas MRI may allow for earlier return
to normal activity.
………
Corticosteroid injections are used in the short-term treatment of joint inflammation
and bursitis but are not appropriate for fractures as they can impede healing.

Tendon injuries are typically acquired in penetrating injuries or extreme loading of a


digit (eg, "jamming" a finger while trying to catch a ball). FOOSH is unlikely to cause
tendon injury.

Synovial fluid analysis is appropriate for evaluating inflammatory, crystalline, or


infectious arthropathies. It is rarely useful in acute traumatic injury.

GANGLION CYST This patient, with a mobile,


nontender swelling at the dorsal
surface of the wrist, has a ganglion
cyst. Ganglion cysts are generally
harmless lesions that are MC in
patients with underlying joint
disorders or prior joint injury. They
can occur in both upper and lower
extremities but are MC at
the wrist. Ganglion cysts are
typically filled with mucinous fluid
and often communicate with the underlying joint via a hollow pedicle; fluid is
postulated to flow from the joint space to the cyst, and a one-way valve mechanism
prevents return flow to the joint.
The diagnosis of ganglion cyst is usually obvious on inspection and can be confirmed
on transillumination of the mass.

If in doubt, US or MRI can confirm the diagnosis, but


imaging is rarely necessary. Most ganglion
cysts resolve spontaneously and require no
treatment.

For the few patients with painful and persistent


symptoms, options include aspiration and surgical
excision.

 Epidermoid cysts are small (<1 cm), freely mobile


nodules, typically with a central punctum; they develop most commonly on the palmar
surface of the digits.

 Lipomas are soft, fleshy masses found in


subcutaneous tissues; they are usually on the trunk or
shoulders.

 Rheumatoid nodules are firm, fibrotic masses typically found


in subcutaneous tissues overlying pressure points (eg,
olecranon, dorsal forearm). Unlike ganglion cysts, these lesions
do not transilluminate.

Tendinous xanthomas are smooth, firm growths that are most


commonly seen in patients with severe hyperlipidemia or lipid
storage disorders. They are typically multifocal (MC on the
Achilles tendon), move with the tendon, and do not
transilluminate.
SCOLIOSIS
///////////////////

***

Adolescent idiopathic scoliosis (AIS) refers


to lateral curvature (ie, Cobb angle >10
degrees) of the spine without a specified
etiology. The deformity typically presents in
children age ≥10 during periods of rapid
growth.

Although poor posture may be noted, AIS is


usually asymptomatic and identified during
school scoliosis screening or annual physical
examination. Inspection of the back may reveal
asymmetry of the shoulders, scapulae, or iliac
crest. Forward bend test shows asymmetric
thoracic or lumbar prominence.

The first step in evaluating clinically evident


spinal asymmetry is x-ray of the spine. Full-back PA and lateral radiographs confirm
the diagnosis, measure the severity of disease (ie, Cobb angle), and assess skeletal
maturity.

Management of AIS depends on


risk of progression (eg, degree of skeletal maturity) and severity (ie, degree of
curvature on x-ray). Scoliosis typically progresses
slower with increasing degrees of bone
ossification. Therefore, patients are at increased
risk for progression in early puberty (eg,
premenarche).
Most curves are mild (ie, Cobb angle 10-30
degrees) and can be monitored clinically every 6
months.
A thoracolumbosacral spinal brace is indicated in
a child with growth potential remaining and
Cobb angle ≥30 degrees to help reduce curve
progression.

Surgical fixation is considered for severe


curvature (ie, Cobb angle ≥40-50 degrees).

……….
Scoliosis due to pathologic causes (eg, mass, dysraphism) may present with
neurologic symptoms, severe pain, rapid progression, or vertebral abnormalities
on x-ray. MRI of the spine should be performed in such cases but is not indicated in
this asymptomatic patient.

VERTEBRAL
Clinical features of vertebral compression fracture
COMPRESSION
FRACTURE • Trauma
/////////////////// • Osteoporosis, osteomalacia
Etiologies • Bone metastases
• Metabolic (eg, hyperparathyroidism)
***** • Paget disease

Acute
• Low back pain & decreased spinal mobility
• Pain increasing with standing, walking, lying on back
• Tenderness at affected level
Clinical presentation Chronic/gradual
• Painless
• Progressive kyphosis
• Loss of stature

• Increased risk for future fractures


Complications • Hyperkyphosis, possibly leading to protuberant abdomen, early
satiety, weight loss, decreased respiratory capacity

An older patient developed acute back pain associated with vertebral point tenderness
after a nontraumatic event, suggesting an acute vertebral compression
fracture (VCF). VCF is most commonly due to decreased bone
density from osteoporosis. The risk is greatest in postmenopausal women and men
age >65. Uncommon etiologies include osteomalacia, vertebral osteomyelitis,
malignancy with bone metastases, metabolic abnormalities (eg, hyperparathyroidism),
and trauma.

Osteoporotic VCF can be chronic and painless, or may occur acutely due to twisting,
lifting, coughing, or minor trauma (eg, fall from standing height), leading
to severe midline back pain that is typically localized. The pain often worsens
with movement, coughing, or straining and may persist at night. Examination
typically demonstrates vertebral point tenderness; however, spinal cord or nerve root
compression is rare, and patients usually do not have neurologic
deficits. Multiple/recurrent VCF is common and can lead to height loss and kyphosis;
complications of kyphosis can include decreased respiratory capacity and abdominal
compression (eg, early satiety, weight loss). The diagnosis is confirmed with
plain radiography
……………

Apophyseal joint arthritis can occur in spondyloarthropathy (eg, ankylosing spondylitis),


which typically presents with chronic, progressive pain that is worse at night and with
prolonged rest; symptoms usually begin at age <40.
Ligamentous back sprain can occur following exertion. However, the pain is usually
relieved with rest, and tenderness would be seen in the paraspinal tissues rather than at
the midline.
Nerve root demyelination (eg, Guillain-Barré syndrome) presents with paresthesia and
weakness corresponding to the affected nerve root. It typically is not associated with
acute back pain or vertebral tenderness.

PELVIS FRACTURE This trauma patient has


shock (eg, severe HoTN
and tachycardia) in the
setting of severe pelvic
fracture (eg, unstable
pelvis on palpation, pelvic
ring disruption on x-ray)
with accompanying signs
of retroperitoneal
hemorrhage (eg, flank
ecchymosis). To prevent
exsanguination during
ongoing resuscitation
efforts and evaluation, a
pelvic binder should be
immediately placed.

Patients with severe pelvic fractures, such as this open-book pelvic fracture (ie, pelvic
ring disruption with anterior widening), are at risk for life-threatening
hemorrhage. Hemorrhage is typically caused by the tearing of the thin-walled venous
plexus (presacral, lumbar) but rarely may involve the iliac vessels. Resultant
bleeding can both fill the expanded pelvic volume (from widening of the pelvic ring)
and extend into the retroperitoneum. Therefore, rapid, external stabilization with
a pelvic binder should be performed ASAP in hemodynamically unstable (ie, SBP <90
mm Hg) patients, such as this one; this decreases pelvic volume and
promotes tamponade of venous bleeding.
…………..

The eventual workup of this patient will likely include CTAP to assess for
intraabdominal injury and provide 3D visualization of the pelvic fractures. Retrograde
cystourethrogram will also be indicated to assess for urethral injury, especially given
the finding of blood at the urethral meatus. However, because this patient is currently
hemodynamically unstable, hemorrhage control (eg, pelvic binder) and resuscitation
should be performed first to stabilize the patient prior to any diagnostic testing that
requires transport away from the trauma bay.
This patient with a left lower leg deformity will likely undergo plain radiographs of the
left tibia and fibula and reduction of the lower leg deformity (eg, temporary splinting)
following stabilization. However, hemorrhage control (eg, pelvic binder) and
resuscitation should be performed first. Patients are more likely to lose large amounts
of blood into the pelvis (eg, up to the entire blood volume) or thigh (eg, up to 1-2 L)
compared to the lower leg.

HIP FRACTURE This patient presents after a


fall with a left femoral neck
*** fracture. Hip fractures are
classified by anatomic
location and fracture type
into either intracapsular
(eg, femoral neck and head)
or extracapsular (eg,
intertrochanteric,
subtrochanteric).

Intracapsular fractures
have a higher chance of
AVN; extracapsular
fractures have a greater
need for implant devices
(eg, nails, rods). The specific
surgical intervention required depends largely on the details of the individual case.
In general, surgical repair should be done as soon as feasible to relieve pain,
minimize complications, and reduce length of hospital stay. However, surgery may be
delayed up to 72 hours if needed to address unstable medical comorbidity. This
patient has a number of concerning features. The uncertain details surrounding his fall
suggest a possible syncopal episode. His tachycardia and palpitations can be due to
arrhythmia (a-fib); decreased breath sounds on the right side could be due to pleural
effusion; cough, dyspnea, and left basilar crackles also suggest pulmonary edema or
pneumonia. The patient should therefore be evaluated with CXR, ECG, and cardiac
markers to determine the etiology of his fall and assess preoperative risk prior to
surgical correction of his fracture. If there is evidence of pulmonary edema and pleural
effusion, an echocardiogram is needed. Once it is determined that he is otherwise
medically stable, he may be cleared for surgical intervention.

………

Before starting intravenous fluids, pulmonary edema (heart failure) and pleural effusion
should be ruled out with chest x-ray. This patient is not hypotensive and likely has
an intracapsular fracture, which has a relatively low risk of acute blood loss and
need for intravenous fluids.
Skeletal traction has been advocated as an interim measure for patients with hip
fracture who require delayed surgical repair. However, recent studies have found that
traction does not relieve pain or improve surgical outcomes.
Surgery may reasonably be delayed up to 72 hours to allow for stabilization of acute
medical problems, but longer delays increase the risk of medical complications and
prolong the return to premorbid functional status.
An elderly woman has an acutely
shortened, externally rotated leg
following a fall. This presentation is most
consistent with either a femoral neck or
an intertrochanteric fracture, which are
the MC hip fractures in older adults,
typically occurring due to mechanical
falls.The classic pattern of shortening
and external rotation of the leg
compared with the contralateral side is
primarily due to contraction of the psoas
and iliacus without the normal acetabular
counterforce. This pattern is also seen in
anterior hip dislocation, which is
significantly less common than fracture
and typically occurs following severe

trauma (eg, industrial accident,


MVA). An x-ray generally confirms the
diagnosis of hip fracture.

Hip fractures are classified as either


intracapsular (femoral head and neck) or
extracapsular (intertrochanteric or
subtrochanteric). Intracapsular fractures
typically present without significant
ecchymoses and have a higher risk of
AVN.
Extracapsular fractures are at higher risk
for displacement and usually have
visible ecchymosis. Both types generally require surgical correction with
either arthroplasty or ORIF. In stable patients, surgery within 48 hours is associated
with lower mortality and a lower risk of pressure ulcers and pneumonia.

……..

 Femoral shaft fracture in elderly patients


can present with shortening of the leg, often
with angulation.

Pubic ramus fracture can occur from minor (or


no) trauma in elderly patients and also can
cause shortening of the ipsilateral leg.
However, external rotation is more typical of
femoral neck fracture.

Posterior hip dislocation


typically presents with adduction
and internal rotation at the hip. It
usually occurs from an axial force
on the femur (eg, dashboard
injury), and some patients have
neurologic manifestations due to
involvement of the sciatic nerve.

 Isolated fracture of the greater trochanter can


occur due to a ground-level fall in older
patients. Typical features include pain with
abduction and tenderness at the trochanter.
However, leg length is not affected.
SLIPPED CAPITAL
Slipped capital femoral epiphysis (SCFE)
FEMORAL
EPIPHYSIS (SCFE) • Obesity
Risk factors • Adolescence

• Dull hip pain


Clinical • Referred knee pain
presentation • Altered gait
• Limited internal rotation of hip

Diagnosis • Posteriorly displaced femoral head on x-ray

• Non–weight bearing
Treatment • Surgical pinning

• Avascular necrosis (AVN)


Complications • Osteoarthritis

Slipped capital femoral epiphysis (SCFE) is characterized by anterolateral and


superior displacement of the proximal femur along the physis (growth plate). It most
commonly presents in obese
adolescents with chronic hip
pain. However, 15% of patients with SCFE
have only referred thigh or knee pain. Due
to the referred nature of the pain, knee
examination is normal. Characteristic
findings on hip examination, however,
include limited internal rotation, which
causes the foot to point laterally, and
thigh abduction and external rotation
with passive hip flexion.
Bilateral hip x-rays are diagnostic for SCFE
and show a posteriorly displaced femoral
head. The patient should be non–weight
bearing to avoid further slip and referred
for immediate surgical screw fixation of the physis.
…….

Osgood-Schlatter disease (OSD), or


osteochondritis of the tibial tubercle, is a
common cause of chronic knee pain that is
worse with running and jumping in active
adolescents. However, tenderness over the
tibial tubercle would be present
on examination.

Osteoarthritis of the hip presents with


chronic hip pain and stiffness. It is more
common in obese patients but typically does
not occur until age >50.

Osteoid osteoma is a benign bone tumor


that typically occurs at the proximal femur
and causes nighttime leg pain in
adolescents. In contrast to this patient's
presentation, pain associated with osteoid
osteoma is unrelated to activity and
relieved with ibuprofen.

Transient synovitis of the hip typically


presents in children age 3-8 with acute hip
pain, often after a viral illness.

A 12-y-o obese (BMI >95th percentile) patient with dull thigh pain and decreased
range of motion of the hip likely has SCFE. SCFE occurs when excessive shearing at the
proximal femoral physis (growth plate) weakens the growth plate, causing
anterosuperior displacement of the proximal femur diaphysis and giving the
appearance of a posteriorly displaced femoral head. Obese adolescents are at
highest risk due to increased mechanical strain on the hip.
Symptoms most commonly include chronic (>3 weeks), intermittent, dull pain/ache
of the hip, thigh, or knee that is exacerbated by activity. The leg is typically externally
rotated (foot pointed laterally), and gait may be altered due to pain. On examination,
decreased ROM (particularly internal rotation) and tenderness of the hip may be
present. Abduction and external rotation of the thigh during passive flexion of the
hip is characteristic. Diagnosis is confirmed by visualization of a posteriorly slipped
femoral head on x-ray.
…….

Septic arthritis, a bacterial joint infection, presents with acute fever, joint pain, inability
to bear weight, and a swollen, warm joint. In contrast, this patient is afebrile with
chronic symptoms and no joint swelling.

Expansion of malignant cells in the BM is characteristic of leukemia, which can present


with chronic bone pain. Systemic findings (e.g., fever, weight loss, LAD, and
splenomegaly) are typical. This patient is otherwise asx and has LNs of normal size.

Transient synovitis is characterized by self-limiting, intraarticular inflammation and


presents in children age 3-8 with acute hip pain and limp following a viral illness. This
patient's age and chronic symptoms make SCFE more likely.

Vasoocclusive crisis due to red cell sickling causes bone pain in patients with sickle cell
disease. Undiagnosed sickle cell disease is unlikely in this adolescent because pain
crises typically present in early childhood.
ILIOTIBIAL BAND
Common overuse injuries of the knee
SYNDROME
• Women much more than men
PFPS • Poorly localized anterior pain
• Pain reproduced with patellofemoral compression during extension

Iliotibial band • Poorly localized lateral pain


syndrome • Tenderness at lateral femoral epicondyle with flexion & extension

Pes anserine • Highly localized medial pain


bursitis • Point tenderness at the pes anserine bursa

• Common in jumping sports (eg, basketball, volleyball)


Patellar
• Moderately localized pain at & below patella
tendinitis • Tenderness at inferior margin of patella

• Common in patients who work on their knees ("housemaid's knee")


Prepatellar
• Bogginess & tenderness at inferoanterior aspect of knee
bursitis • Propensity to secondary infection with S aureus
The iliotibial band (tract) is a thickened
fibrous structure in the deep fascia of the
lateral thigh. It contributes to the insertion of
a number of hip and thigh muscles (eg,
gluteus maximus) and stabilizes the lateral
thigh during flexion and extension of the
knee. Iliotibial (IT) band syndrome is a
common overuse injury characterized by
poorly localized lateral knee
pain originating in the region where
the distal IT band passes over the lateral
femoral epicondyle (LFE). It is often seen in
inexperienced runners starting a new or
more strenuous training regimen.

The diagnosis of IT band syndrome is based


primarily on clinical findings. Examination
may show tenderness proximal to the
lateral joint line near the LFE; the pain can often be reproduced with pressure over the
IT band just proximal to the LFE during flexion of the knee (eg, Noble test).

Initial management includes activity modification and lower extremity stretching and
strengthening exercises.
........

Lateral collateral ligament sprains are uncommon and typically caused by a blow to
the medial aspect of the knee. Examination shows joint line tenderness and varus
laxity.

In young patients, lateral


meniscus injuries are
typically caused by acute
trauma rather than
overuse. Associated
symptoms include catching,
popping, or locking during
movement and sometimes a
give-way sensation. Small
effusions are common, and
tenderness, if present, is at
the joint line.
 Patellofemoral pain
syndrome is a common cause of knee pain in young, physically active women, but it
typically presents with anterior pain that is reproduced by compression of the patellar
complex during extension.

 Pes anserinus pain syndrome,


often termed pes anserine
bursitis, causes medial (not
lateral) knee pain. Tenderness in
this disorder is distal to the joint
line in the region where
the conjoined tendon of the
semitendinosus, gracilis, and
sartorius inserts on the tibia.

BAKER CYST

A popliteal cyst is due to extrusion of synovial fluid from the knee joint into the
gastrocnemius or semimembranosus bursa through a communication between the
joint and the bursa. Excessive synovial fluid formation (eg, due to OA or RA) and
positive pressure in the knee during extension can cause passage of fluid into the
bursa and gradual enlargement of the cyst. Popliteal cysts are often asymptomatic
and present as a chronic, painless bulge behind the knee. The diagnosis is usually
apparent on examination, with a soft mass in the medial popliteal space that is most
noticeable with knee extension and less prominent with flexion.
Rupture of a popliteal cyst (eg, following strenuous exercise) can cause posterior
knee and calf pain, with tenderness and swelling of the calf resembling DVT. An arc of
ecchymosis is often visible distal to the medial malleolus ("crescent sign").
Ultrasound can rule out DVT and confirm the popliteal cyst.

……….

Pes anserinus pain syndrome (anserine bursitis) presents with subacute medial knee
pain. Examination shows a well-defined area of tenderness over the medial tibial
plateau below the joint line.

Stress fractures of the tibia present with progressive pain at the shin and focal
tenderness over the fracture site.

Tears of the medial collateral ligament are caused acutely by a twisting injury or blow
to the lateral knee. Examination findings include tenderness at the medial joint line and
valgus laxity.

Tears of the medial meniscus result from a twisting force on a fixed foot. Patients
report a popping sound followed by acute pain. Typical examination findings include
a small effusion and crepitus, locking, or catching with ROM.

The onset of symptoms with exercise, posteromedial tenderness, and ecchymosis at the
medial malleolus make ruptured popliteal cyst more likely than DVT.

OSTEOARTHRITIS

Risk factors for osteoarthritis include obesity, prior joint injury, and abnormal joint
alignment. Initial management of OA of the knee includes lifestyle measures such as
weight loss and regular moderate activity (eg, walking).

In addition, a dedicated exercise program (often with assistance from a physical


therapist) to strengthen the quadriceps muscles can improve function and protect
the articular cartilage from further stress. Quadriceps strength declines with age, and
the pain of osteoarthritis may induce patients to decrease activity, leading to disuse
atrophy of the quadriceps complex. This can cause abnormal loading of the articular
cartilage and accelerate degeneration of the joint. Simple analgesics (eg, NSAIDs)
may also be helpful.
…….

Therapeutic arthrocentesis is usually used for inflammatory or rapid-onset effusions


(eg, septic arthritis, hemarthrosis). It is rarely helpful in the small effusions of OA.

Intraarticular injections of glucocorticoids or hyaluronic acid can be considered for


osteoarthritis of the knee but usually provide little long-term benefit; they are
generally recommended only when noninvasive measures fail. Arthroplasty is also
highly effective for advanced OA but also is recommended only when less aggressive
measures fail.
Pes anserinus pain syndrome (sometimes called anserine bursitis) is characterized by
tenderness over the medial aspect of the knee just distal to the joint line, at the
insertion of the sartorius, gracilis, and semitendinosus tendons. The pain is
exacerbated by pressure from the contralateral knee while lying on the side; it can be
relieved by placing a pillow between the knees during sleep.

If conservative management is
inadequate, injectable glucocorticoids or
possibly hyaluronic acid may relieve
symptoms. However, if appropriate measures
have failed, the patient is a candidate for
surgical intervention with total knee
arthroplasty, an elective surgery consisting
of replacement of the articular surface of the
knee with prosthetic components. Total knee
arthroplasty has been associated with
significant improvements in pain level,
function, and QoL.

Indications for totak knee arthroplasty?

.......
Arthroscopic meniscectomy is indicated for symptomatic tears of the meniscus.
Long-term complications include increased risk of OA in the operated knee. In
general, arthroscopic intervention is not helpful in patients with established OA.
Systemic glucocorticoids (eg, methylprednisolone) are occasionally considered for
mild to moderate OA of the knee, but their side effect profile makes them an
unattractive option. They are relatively contraindicated in a patient with comorbid
diabetes, due to possible hyperglycemia.

Hinged unloader (valgus) knee braces transfer load from the medial to the lateral
joint compartment and are useful for patients with medial unicompartmental OA
associated with varus deformity. A patient with a more generalized OA would be
unlikely to benefit from the use of a brace.

Chronic treatment of OA with long-acting opioids does not appear to provide


substantial improvements in pain or function; it also carries significant risk.

The pain of hip osteoarthritis is typically


felt in the groin, buttock, or lateral hip
region, and can radiate to the lower
thigh. Patients may have mild pain and
brief stiffness with prolonged rest, but
the worst pain usually occurs with activity
and weight bearing. Examination
findings often include decreased
rotational ROM with no synovitis
(redness, warmth).

………
AVN (osteonecrosis) is characterized by reduced perfusion of the femoral head and
collapse of the periarticular bone and can present with hip pain and reduced
ROM. Most atraumatic cases are associated with glucocorticoid or alcohol use. X-ray
findings include subchondral lucency and loss of the normal spherical contour of
the femoral head, but the joint space is preserved and osteophytes are not present.
Fracture of the femoral neck is most common in elderly patients following an acute
fall. Examination typically shows a shortened and externally rotated leg, with severe

pain on ROM. X-ray reveals shortening and angulation of the femoral neck.
Greater trochanteric pain syndrome (formerly trochanteric bursitis) is caused by
friction of the tendons of the gluteus medius and tensor fascia lata over the greater
trochanter of the femur. Pain is localized to the lateral hip and is worsened by direct
pressure. X-ray is typically normal.
Dislocation of the hip is uncommon in adults and is typically seen only in the context
of major trauma.

Osteosarcoma is most common in children and adolescents, although it is occasionally


seen in adults age >65. The femur is a common location in pediatric patients, but in
adults, osteosarcoma more commonly affects the AXIAL skeleton. X-ray shows
destruction of trabecular and cortical bone with formation of new periosteal bone.
Medial collateral ligament (MCL) tear is a common knee injury caused by severe
valgus stress (eg, blow to the lateral knee) or twisting injury. Examination findings may
include local swelling, ecchymosis, and joint line tenderness at the medial knee.
Appreciable laxity when the leg is forced into abduction (valgus stress test) is helpful
for diagnosis but may be MASKED by swelling and muscle spasm. Acute
effusion/hemarthrosis is uncommon unless there is concurrent injury to the ACL.
MRI is the most SN test for diagnosis but is generally reserved for patients being
considered for surgical intervention. Patients with uncomplicated MCL tears can be
managed nonoperatively with rest, ice, compression, and elevation (RICE measures)
and analgesics with progressive return to activity as tolerated.
…………

Injuries to the lateral collateral ligament are uncommon but are occasionally seen
following high-velocity trauma. The knee will have varus laxity.
Meniscal tears often occur when patients pivot on a flexed knee while the foot is
planted.

Patellar tendonitis ("jumper's knee") is a chronic overuse injury characterized by


anterior knee pain and tenderness. Stress testing of the ligaments is normal.

Tibial plateau fractures can occur due to contact sports, falls, or MVAs. However,
most patients with such fractures are unable to bear weight on the knee.
SEPTIC
Septic arthritis
ARTHRITIS
• Abnormal joint (eg, RA, prosthetic joint)
• Age >80
Risk factors • DM
• IVDU, alcoholism

• Acute monoarthritis: hot, swollen, decreased ROM


Clinical
• Fever
features • Elevated ESR & CRP

• Blood cultures
Diagnosis • Synovial fluid analysis: leukocytosis (>50,000/mm3), Gram stain, culture

• Joint drainage: needle aspiration (eg, knee, ankle, wrist), arthroscopy


Initial
(eg, hip, shoulder), or open arthrotomy
treatment • Intravenous (IV) antibiotics

CRP = C-reactive protein; ESR = erythrocyte sedimentation rate; ROM = range of motion.

Initial treatment of septic arthritis includes IV antibiotics and, as with other purulent
infections in closed spaces (eg, abscess), adequate drainage of purulent material.

For easily accessed joints (like the knee), needle aspiration may be performed.
However, for deep joints (eg, hip) or when needle aspiration does not facilitate
adequate drainage, arthroscopic irrigation or open surgical drainage may be
needed. Regardless of drainage technique, serial procedures (eg, daily needle
aspirations) are often required to completely clear the infection.

……….
SLE (with a positive ANA) can cause synovitis of the knee, but it is usually
polyarticular and symmetric; the synovial fluid leukocyte count is relatively low,
often in the noninflammatory range (eg, <2,000/mm3). Even if a patient's Gram stain is
negative, the severe synovial leukocytosis and single joint involvement are more
consistent with septic arthritis.

For the first few days of treatment, splinting the infected joint to reduce motion may
facilitate relief of pain. However, placement of a cast can limit clinical access to the
joint and is not advised.

MRI or CT scan can often be helpful for assessing effusions involving deep (eg, hip) or
atypical joints in septic arthritis. However, for the knee, plain film x-ray is typically
adequate.
Systemic glucocorticoids (eg, prednisone) are sometimes used as an adjunctive
measure for reducing joint damage in pediatric septic arthritis. However, they are not
well studied in adults and are relatively contraindicated in a patient with comorbid
DM.
Arthroplasty is sometimes needed in chronic management (after resolution of
infection) for patients with severe joint injury and impaired mobility. However,
arthroplasty would not be considered for this patient with an active infection.

KNEE
DISLOCATION

This patient with severe


knee pain and deformity after a hard tackle has a
posterior knee dislocation (displacement of the
proximal tibia posterior to the distal femur). This
typically results from a direct blow to the proximal
tibia and disrupts multiple ligaments (eg, cruciate,
collateral), with possible injury to the
neurovascular structures within the popliteal fossa.
Injury to the popliteal artery is the most feared
complication of any knee dislocation because the
resulting lower leg ischemia can cause irreversible
injury, requiring above-the-knee amputation.
Management begins with immediate reduction of
the dislocated knee. Given the risk of vascular (eg,
popliteal artery) injury, this should be followed by
a meticulous vascular examination that includes:

• Palpation of the popliteal and distal pulses


• Measurement of the ankle-brachial index (ABI)
• Duplex USG (if available)

Ankle-brachial index (ABI)

ABI = SBP of dorsalis pedis or posterior tibial artery ÷ SBP of brachial artery

≤0.9 Diagnostic of peripheral artery disease

0.91-1.3 Normal
>1.3 Suggests calcified & uncompressible vessels*

*Other testing should be considered.


SBP = systolic blood pressure.
Because pulse examination alone is of limited accuracy in diagnosing vascular injury,
obtaining and documenting the ABI are critical. The combination of normal pulses
with an ABI >0.9 virtually excludes clinically significant vascular injury. Any signs
(eg, diminished pulse, ABI ≤0.9) of vascular injury warrant emergency imaging (eg,
CTA) and vascular consultation.
……….

Following reduction of the dislocated knee and vascular examination, temporary


stabilization of the joint is indicated to prevent redislocation. However, a removeable
knee immobilizer is typically applied because a long leg cast obstructs serial
vascular examinations and can become dangerously tight (eg, compartment
syndrome) because of acute postinjury limb swelling.

This patient has no signs of compartment syndrome (eg, tense compartments, pain on
passive stretch); therefore, invasive compartment pressure measurement is not
indicated.

Arthrocentesis (ie, joint fluid aspiration) with synovial fluid analysis is commonly
performed to evaluate for infection or crystals in an inflamed joint. It may also be
performed for pressure relief of a severe effusion. Although this patient's knee is
swollen from trauma, he has none of these indications for arthrocentesis.

Peroneal nerve injury occurs in 20%-25% of knee dislocations. When signs of injury
(eg, foot drop) are present, NCS may help determine injury severity and direct
treatment. This patient has no such signs.

KNEE TRAUMA
Special tests for knee examination

Valgus stress test


MCL injury • Stabilize lateral thigh; apply abduction force to lower leg
Laxity indicates MCL injury

Anterior drawer test


• Patient supine with knee flexed
• Grip proximal tibia with both hands & pull anteriorly
Lachman test
ACL injury • Place knee at 30 degrees flexion
• Stabilize distal femur with 1 hand & pull proximal tibia anteriorly with the
other
Laxity of tibia indicates ACL injury

Thessaly test
Meniscal
• Patient stands on 1 leg with knee flexed 20 degrees
tear • Patient then internally & externally rotates on flexed knee
McMurray test
• Passive knee flexion & extension while holding the knee in internal or
external rotation
Pain, clicking, or catching indicates meniscal tear

ACL = anterior cruciate ligament; MCL = medial collateral ligament.

Tears of the medial meniscus often result from twisting force with the foot in a fixed
position. Patients generally report a popping sound followed by acute
pain. Associated symptoms can include reduced extension, a sensation of instability,
and knee effusion (slow; gradual). Because the meniscus is not directly perfused,
rapid-onset hemarthrosis does not occur and any effusion that develops is typically not
immediately apparent. The medial meniscus is more commonly injured than the lateral.

Examination reveals tenderness at


the joint line on the affected side. Patients may also have palpable locking or
catching when the joint is rotated or extended while under load; characteristic
maneuvers include the Thessaly and McMurray tests.
X-ray may reveal OA in older patients with tears caused by chronic degeneration of
the cartilage, but plain films are usually normal in young patients with traumatic
tears.

Diagnosis is confirmed with MRI or arthroscopy.

.........
ACL tears are caused by forceful hyperextension of the knee or torsional injury during
deceleration. Rapid-onset hemarthrosis is typical, and patients have anterior laxity of
the tibia relative to the femur (eg, Lachman test, anterior drawer sign).

MCL injury can occur by a blow to the lateral knee or forceful leg abduction. It would
cause medial knee tenderness but would not be associated with catching during
extension or rotation.

Patellar dislocation occurs after quick lateral movements around a FLEXED knee.
Examination reveals a flexed knee with the patella displaced laterally. The patellar
tendon is rarely ruptured, although tears can occur with extreme loading in knee
flexion. Rupture of the patellar tendon causes impaired knee extension.
PFPS causes chronic anterior knee pain. Patients experience pain worsened by activity
or prolonged sitting (due to sustained flexion) and may also have crepitus with
motion of the patella.

Features of anterior cruciate ligament injury

Injury • Rapid deceleration or direction changes


mechanisms • Pivoting on lower extremity with foot planted

• Pain: rapid onset, may be severe


• A "popping" sensation at the time of injury
Symptoms • Significant swelling (effusion/hemarthrosis)
• Joint instability

Examination • Anterior laxity of tibia relative to femur (anterior drawer test, Lachman
findings test)

Diagnosis • MRI

• RICE (rest, ice, compression, elevation) measures


Treatment • ± Surgery

Anterior cruciate ligament


(ACL) tears are common in
young athletes, especially
women participating in
sports requiring rapid
direction changes
or pivots on the lower
extremity (eg, soccer,
basketball, tennis). ACL
tears can also occur following
a blow to the knee or
significant twisting
force. Patients with ACL tears
typically experience an acute
"popping" sensation in the
knee, followed by rapid
onset of effusion due to hemarthrosis and a feeling of instability when bearing
weight.

Examination in an ACL tear shows laxity of anterior motion of the tibia relative to the
femur. Two such maneuvers—the Lachman and anterior drawer tests—are highly
sensitive (>90%) and specific for ACL injuries.

The diagnosis can be confirmed on MRI.


…….
Medial collateral ligament (MCL) tears may also occur in pivoting/twisting injuries or if
the knee is struck from the lateral side with the foot planted. Examination shows medial
knee tenderness and laxity when the foot is gently forced into abduction with the knee
stationary (valgus stress test). However, MCL injuries are not usually associated with
a significant joint effusion or hemarthrosis unless there is a concurrent ACL injury.
Posterior cruciate ligament (PCL) injuries are often caused by hyperextension or by a
blow to the anterior aspect of the proximal tibia. They are less common than ACL tears
and are usually associated with little pain or alteration in ROM.

Stress fractures of the tibial plateau may occur in female athletes, especially following
abrupt initiation or changes in exercise regimen. However, stress fractures typically
present with subacute pain and are much less common than ligament injuries.

Meniscal tears

• Younger patients: rotational force on planted foot


Etiology
• Older patients: degeneration of meniscal cartilage

• Acute "popping" sensation


Symptoms • Catching, locking, reduced ROM
• Slow-onset joint effusion

• Joint line tenderness


Examination
• Pain or catching in provocative tests (Thessaly, McMurray)

• MRI
Diagnosis • Arthroscopy

• Mild symptoms, older patients: rest, activity modification


Management • Persistent symptoms, impaired activity: surgery

Patients may have a sensation of instability. Examination (eg, Thessaly test, McMurray
test) have high specificity for meniscal injury, sensitivity, however, is variable, and
patients may have normal findings at the time of examination.
Patients with short-term symptoms and minimal limitations in activity, or older
patients with chronic tears due to degeneration of the cartilage, can be managed
conservatively with rest and NSAIDs.

Young patients who have mechanical limitations or recurrent effusions lasting >3-4
weeks should be considered for confirmatory MRI, which can clearly visualize the soft
tissues of the knee. If a significant tear is found, surgical consultation is
recommended.

………..

Radionuclide bone scans are used to evaluate malignant, infectious, or inflammatory


bone diseases. These scans can also identify occult fractures, but MRI is superior for
imaging the soft tissues of the knee and would identify an occult fracture as well.
BURSITIS
Greater trochanteric pain syndrome (trochanteric bursitis)

• Age ≥50
• Women > men
Risk factors • Obesity
• Low back & lower extremity disorders (eg, scoliosis, osteoarthritis,
plantar fasciitis)

• Chronic lateral hip pain


Symptoms • Pain worse with hip flexion or lying on affected side

• Focal tenderness over trochanter


Diagnosis • X-ray to rule out hip joint pathology
• Ultrasound: degeneration of tendons, tendinosis

• Exercise, physical therapy, activity modification


Treatment • Nonsteroidal anti-inflammatory drugs
• Corticosteroid injection

Greater trochanteric pain syndrome


(GTPS) is an overuse syndrome
involving the tendons of the gluteus
medius and minimus where they run
over the greater trochanter. GTPS is
sometimes termed "trochanteric
bursitis," although the bursae are not
the source of pain in most
patients. GTPS typically presents with
chronic lateral hip pain that is
worsened with repetitive hip flexion
(eg, climbing stairs, walking uphill) or
lying on the affected side.

This diagnosis of GTPS is based


primarily on clinical findings. Physical
examination shows local tenderness
over the greater trochanter during flexion. Hip ROM is normal, although abduction
may aggravate the pain. X-ray is commonly performed to rule out concurrent hip
joint disorders; US may show chronic degeneration/tendinosis in the affected tendons.
Initial treatment of GTPS includes local heat,
activity modification, and NSAIDs. Oral NSAIDs
(eg, ibuprofen) are typically used, although topical
NSAIDs (eg, diclofenac) are also effective. Physical
therapy is often helpful as well. Patients with
persistent symptoms despite conservative therapy
often benefit from local corticosteroid injection,
which is generally safe and provides rapid pain
relief.

...........

Evidence to support the use of opioids (eg,


hydrocodone) in GTPS is negligible, and these
agents carry significant risks (eg, dependence,
sedation).

Pregabalin, an alpha-2 delta ligand, is indicated


for the treatment of chronic pain due to
fibromyalgia and neuropathic disorders (eg,
diabetic neuropathy, postherpetic neuralgia).
Fibromyalgia can be associated with tenderness over the greater trochanter, but
the pain and tenderness are typically widespread.

Arthroscopic or open surgical intervention is reserved for patients who have


significant tendon rupture or who fail less invasive management.
Although lidocaine is often injected with a corticosteroid to give immediate pain relief,
topical lidocaine has no anti-inflammatory effects and is used only occasionally for
GTPS.

GTPS typically presents with chronic lateral hip pain that is worsened with repetitive hip
flexion (eg, climbing stairs, walking uphill, getting out of a car) or lying on the
affected side.

This diagnosis of GTPS is based primarily on clinical findings. Physical examination


shows local tenderness over the greater trochanter during flexion. Hip range of
motion is normal, although abduction may aggravate the pain. X-ray is commonly
performed to rule out concurrent hip joint disorders; ultrasound may show chronic
degeneration/tendinosis in the affected tendons.

Initial treatment of GTPS includes local heat, activity modification, and


NSAIDs. Physical therapy is often helpful as well. Patients with persistent symptoms
despite conservative therapy may benefit from local corticosteroid injection.

……….
Stress fracture of the femoral neck can cause lateral hip pain. However, this condition
is typically caused by prolonged, strenuous exercise (eg, a runner training for a
marathon) and is otherwise uncommon in patients without underlying bone disease.
Hip osteoarthritis usually causes pain in the deep, medial aspect of the joint.

Iliotibial band syndrome is a common overuse syndrome that arises where the iliotibial
band passes over the lateral femoral condyle. It presents with lateral knee, not hip,
pain and tenderness at the condyle, just proximal to the knee joint.

Meralgia paresthetica is caused by compression of the lateral femoral cutaneous nerve


where it passes under the inguinal ligament. The pain and paresthesia typically occur in
a VARIABLE region of the midlateral thigh.

Osteoid osteoma is a benign tumor that commonly affects the proximal femur, and
can cause nocturnal pain. However, it typically occurs in children, adolescents, and
young adults and is rare in older individuals. Also, although children have FOCAL
TENDERNESS, adolescents and older patients often have no focal examination
abnormalities.
This patient has acute knee pain and tenderness,
with an x-ray showing swelling anterior to the
patella. This presentation is consistent with
acute prepatellar bursitis, which is common in
occupations requiring repetitive kneeling (eg,
landscaping/gardening, plumbing).
Bursae—synovial sacs alleviating friction at bony
prominences and ligamentous attachments—are
vulnerable to acute injury or chronic repetitive
pressure. Acute prepatellar bursa inflammation
(bursitis) may be due to infection (ie, septic
bursitis), crystalline arthropathy (eg, gout), or
autoimmune conditions (eg, RA). Because the
bursa is located in an exposed position, exquisite
tenderness, swelling and erythema, and painful active ROM are often present.
Passive motion may be normal because it causes less pressure on the inflamed bursa.
Bursitis is generally noninfectious, but
~1/3 cases of acute prepatellar bursitis
are due to infection with skin bacteria
(eg, due to microtrauma). The affected
side depends on individual patient
factors (eg, favoring kneeling on one
knee rather the other).

Bursal fluid aspiration for cell count,


Gram stain, and culture is required. If
infection is ruled out, patients may be
managed with NSAIDs; otherwise,
drainage and systemic antibiotics are
required.

……..

Bursal fluid in prepatellar bursitis should also be examined for urate and CPPD
crystals. However, crystalline arthropathy (ie, gout, pseudogout) more commonly
affects the joint space rather than the bursa, leading to diffuse swelling at the joint
rather than focal swelling anterior to it. Also, x-ray in pseudogout typically
demonstrates chondrocalcinosis.

Patellar fracture presents with acute swelling, tenderness, and inability to extend the
knee. It is usually caused by a direct blow or a sudden force under load (eg, fall from
a height).
Infectious (septic) arthritis presents with acute pain, joint effusion, and fever. Unlike
in septic bursitis, the swelling in septic arthritis involves the joint space proper rather
than the anterior tissues, and patients have pain with both active and passive ROM.
Patellar tendinitis causes episodic pain at the inferior patella and patellar tendon. It
is usually seen in athletes in jumping sports or in occupations with repetitive,
forceful knee extension. Although careful inspection of x-ray films may reveal
thickening of the patellar tendon, the anterior soft tissues would be normal.

PFPS: Swelling is generally absent; x-rays are usually normal and are done primarily to
exclude other causes of knee pain.
PATELLOFEMORA
L SYNDROME

A patient with poorly localized anterior knee pain has typical features
of patellofemoral pain syndrome (PFPS). PFPS is one of the MCCs of chronic knee
pain in young women. It is usually related to chronic overuse but can also occur
acutely following trauma. Symptoms are typically worse during weight bearing with
the knee flexed (eg, squatting, ascending/descending stairs) or when running or
sitting for extended periods. The precise source of pain is variable and not well
understood, but generally involves the track of the patella in the trochlear groove of
the femur.

The diagnosis of PFPS is primarily based on characteristic history and examination


findings; however, no individual finding has high clinical utility, and the examination is
often normal. In general, the pain is provoked by maneuvers (eg, squatting) that
involve tonic contraction of the quadriceps with the knee in flexion. The knee should
also be examined for rotational or angular (varus/valgus) malalignment and
dynamic abnormalities (eg, weakness or atrophy of the quadriceps or hip abductors).
The patellofemoral compression test (reproduction of pain when the patella is
compressed into the trochlear groove) is often helpful but may generate significant
discomfort for the patient.

The initial management of PFPS includes activity modification (eg, reduced intensity
of exercise) and NSAIDs. Patients also should be counseled on stretching and
strengthening exercises with an emphasis on the quadriceps and hip abductors.
..........

Soft patellofemoral braces and customized taping can be considered for patients
with PFPS, but the benefits are inconsistent. In contrast, hinged knee orthotic braces
are primarily used for the treatment of acute ligamentous injuries.

Intraarticular glucocorticoid injections are occasionally used in PFPS but are not
likely to provide better long-term results than exercise and carry a small but clinically
significant risk of complications (eg, tendon injury).
Rigid knee immobilizers are used for patellar fractures and tears of the quadriceps
or patellar tendon, which present following major trauma with inability to extend the
knee. Functional weakness (eg, buckling, giveway sensations) does NOT indicate
tendon rupture but can occur in a wide variety of tendon, ligament, and joint
disorders, including PFPS.
Nonweight-bearing ("3-point") crutches are typically used for acute fractures and
sprains of the lower extremity. Although patients with PFPS should temporarily
reduce the intensity of exercise and avoid activities that aggravate their symptoms,
nonweight-bearing crutches are not necessary.

QUADRICEPS
TENDON
RUPTURE

At full activation, the quadriceps


muscle—comprised of the rectus
femoris, vastus lateralis, vastus
medialis, and vastus
intermedius—generates extreme
force across the knee
joint. Sudden, forceful
contraction, as can occur in
deceleration from a fall or in
certain athletic activities, can
cause rupture of the
quadriceps–patellar tendon
complex. Tendon injuries can be
categorized in relation to the
patella:
• Quadriceps tendon tears (proximal to the patella in the rectus femoris
tendon): the patella rides low , indicating an intact connection to the tibia,
with a palpable defect above the patella

• Patellar tendon tears (distal to the patella): the patella rides high, often with
a palpable defect below the patella (the nomenclature can be confusing as the
patellar tendon is actually a ligament).
A patient, who is unable to bear weight after landing on flexed knee, has a large knee
effusion and a low-lying patella; this presentation suggests quadriceps tendon rupture
(proximal to the patella). Typical symptoms include an audible pop, rapid swelling,
and inability to actively extend the knee against gravity. Risk factors include chronic
kidney disease (CKD), HPTH, and anabolic steroid abuse. Exposure to
fluoroquinolone antibiotics may also increase the risk. The diagnosis and extent of the
injury can be assessed on MRI. Management of complete tears of the quadriceps
tendon requires surgical intervention.

.............
Tears involving the ACL are common in middle-aged athletes and can cause rapid-
onset hemarthrosis. Extensive swelling may MASK ligamentous instability in the
acute phase, but knee extension against gravity is intact.

Meniscal injuries typically present with slow-onset swelling, with locking or catching
during ROM testing. Weight bearing and knee extension are intact.

PATELLAR This patient's x-ray shows a high-


TENDON riding patella pulled proximal to
RUPTURE the femoral condyles, consistent
with a tear of the patellar
tendon. Rupture of the
quadriceps–patellar tendon
complex can occur due
to sudden, forceful
contraction of the quadriceps
(which includes the rectus femoris,
vastus lateralis, vastus medialis, and vastus intermedius), as can occur in deceleration
from a fall. Tears can also occur in patients with tendinous fragility (eg, CKD,
exposure to fluoroquinolone antibiotics).

Rupture of the quadriceps–patellar tendon complex can be categorized in relation to


the patella:

• Quadriceps tendon tear: Occurs proximal to the patella in the rectus femoris
tendon. The patella rides low, indicating an intact connection to the tibia, with
a palpable defect above the patella.

• Patellar tendon tear: Occurs distal to the patella. The patella rides high,
often with a palpable defect below the patella. The patella is midline or may
be pulled laterally due to the greater strength of the vastus lateralis versus
the vastus medialis.

Typical symptoms include an audible pop, rapid swelling, and inability to support
weight or actively extend the knee against gravity. The diagnosis is often suggested on
examination and x-ray findings, and the extent of the injury can be assessed on
MRI. Management of complete tears of the tendon complex requires surgical
intervention.

…..

Meniscal injuries typically present with slow-onset swelling and either locking or
catching during range-of-motion testing. Posterior cruciate ligament )PCL) injury is
uncommon and is typically caused by hyperextension or by a blow to the anterior
aspect of the proximal tibia with the knee flexed. Patients with these injuries are
usually able to ambulate (albeit with a sense of instability), and patellar position is
generally unaffected.

Tibial plateau fractures can occur due to contact sports, falls, or motor vehicle
accidents. Stress fractures of the tibial plateau are often related to abrupt initiation or
changes in exercise regimen and typically present with progressively worsening
pain. X-ray alone may miss these injuries.

PATELLAR
Patellar dislocation
DISLOCATION
• Age <20
• Joint laxity
Risk • Misaligned lower extremity
factors • Tight iliotibial band
• Patellar subluxation (excess lateral movement within the trochlea).
• Competitive sports, dance, military training

Clinical • Quick, twisting motion around a flexed knee


presentati • Feeling of knee giving way, severe pain, popping noise
on • Examination: lateral dislocation of patella, decreased extension

Patellar dislocation usually occurs after quick, lateral movements on a flexed knee
and is MC in young athletes (eg, soccer or ice hockey players, gymnasts), dancers, and
military trainees. Patients may feel the knee giving way, followed by a popping noise
and severe pain. The quadriceps muscles normally exert a lateral force on the
patella, especially during quadriceps contraction; therefore, in cases of patellar
dislocation, lateral displacement with associated tear of the medial patellofemoral
ligament (MPFL; which normally provides an opposing medial force) is most
common. Superior and medial dislocations are rare.

Examination shows a flexed knee with reduced ROM and lateral displacement of the
patella (seen as a large, immobile deformity) out of the trochlea, which may be
palpable as a depression at the anterior knee. Other findings can include
hemarthrosis and tenderness along the medial patella (reflecting tear of the
MPFL). X-rays may not be needed for diagnosis in clinically apparent dislocation but
are performed after reduction to rule out additional injuries.
Patellar dislocation often reduces spontaneously. However, closed reduction is
required for patients without spontaneous resolution. Follow-up care includes
splinting and rehabilitation for muscle strengthening.
………

ACL tears are typically due to rapid deceleration or pivoting at the knee with the
foot planted. The medial tibial spine (intercondylar tubercle) is the insertion of the
ACL; avulsion fractures can occur in young patients and have a similar mechanism as
ACL tears. Acute hemarthrosis is common in these injuries and may limit extension
of the knee, but the patella remains seated in the trochlea.

MCL tears are caused by severe valgus stress (eg, blow to the lateral knee) or twisting
injury. Ligamentous laxity may be masked (like ACL tears) in the acute phase by
swelling and muscle spasm, but ROM is preserved, and the patella would not be
displaced.
Meniscal tears often occur from pivoting on a flexed knee while the foot is
planted. Examination typically shows joint line tenderness, a small joint effusion, and
crepitus, locking, or catching with range of motion.

Patellar tendonitis ("jumper's knee") is a chronic overuse injury characterized by


anterior knee pain that worsens with activity. Examination may show tenderness at
the inferior pole of the patella, but ROM and patellar position are normal.

ANKLE TRAUMA
///////////////////

*****
This patient has a lateral ankle sprain. Lateral ankle sprains are typically caused
by forceful inversion of the foot during
running, walking on uneven surfaces (eg,
descending stairs), or a fall from
height. The anterior talofibular
ligament (ATFL) is MCly injured, but the
calcaneofibular ligament can also be
injured in more severe sprains.

Examination in a lateral ankle sprain


shows swelling about the lateral ankle
and tenderness over the ligaments. If the
ligaments are intact (grade I sprain), the
ankle will be stable and weightbearing is
only minimally affected. However,
partial (grade II) or complete (grade III) ligament tears may be associated with
ecchymosis, anterior or inversion laxity, and impaired weightbearing.

If the patient has tenderness only over the ligaments distal to the lateral malleoli and
can bear weight, conservative management (eg, compression bandage or brace, ice
packs, crutches to reduce weightbearing) without imaging is appropriate.
However, if there is tenderness over the posterior edge or tip of the malleolus OR
the patient cannot bear weight, x-ray of the ankle is indicated to rule out a distal
fibular fracture.

.............

Surgical intervention for ankle sprains is rarely indicated. It is primarily considered


for patients with severe (grade III) sprains who have occupations (eg, professional
athlete) or other factors that increase the risk of repeat injury. It is not clear whether
immediate surgery is superior to delayed surgery.
Intraarticular corticosteroid injections are primarily performed for inflammatory
arthritides (eg, gout, rheumatoid arthritis) and are not indicated for acute trauma.

If the patient can bear weight and has no malleolar tenderness; x-ray is not indicated
for acute management. MRI is rarely needed for ankle sprains, although it may be
considered to rule out a concomitant talar fracture in patients who do not improve
as expected in 6-8 weeks.

A patient has a lateral ankle injury caused by forceful inversion of the foot, which
commonly occurs with running, walking on uneven surfaces (eg, descending stairs), or
a fall from a height. Most ankle injuries are caused by sprains, in which the ligaments
are injured but the bones remain intact. Uncomplicated sprains are typically
managed conservatively; however, orthopedic intervention may be required if a bony
fracture is present.
The Ottawa ankle rules were developed to determine which patients require imaging
to rule out an ankle fracture. These rules have high SN (up to 99%) for detecting
fracture and can help reduce the number of unnecessary radiographs. Plain
radiographs of the ankle are indicated in patients with pain in the area of the
malleolus in association with either:

• Point tenderness over the posterior margin or tip of the malleolus


OR
• Inability to bear weight after the injury AND for 4 steps during medical
evaluation

Further management depends on presentation and x-ray findings. Immediate


orthopedics consult is indicated for any open fracture OR in those with evidence of
neurovascular impairment.

……..

NSAIDs are effective for acute ankle pain, whether due to sprain or fracture. However,
if the patient has clear indications for ankle x-ray, simple splinting may not be adequate
management.

If x-ray reveals a distal fibular fracture, most cases can be managed with a hard (eg,
plaster, fiberglass) or soft (eg, air cushion) splint. Surgical intervention is considered
for some proximal fibular fractures as well as for complicated (eg, comminuted,
spiral) distal fibular fractures.
Ankle aspiration is usually done to evaluate suspected septic or crystalline
arthritis. Typical features include erythema, swelling, warmth, and fever.
MRI can visualize the ligaments of the ankle, but it is expensive and rarely needed for
acute evaluation of ankle sprains. MRI may be considered to rule out a concomitant
talar fracture in patients who do not improve as expected in 6-8 weeks, but x-ray is
the preferred initial imaging study.
PLANTAR
Plantar fasciitis
FASCITIS
• Pes planus
Risk factors • Obesity
• Working or exercising on hard surfaces

• Pain at plantar aspect of heel & hindfoot


Symptoms • Worse with weight bearing (especially after prolonged rest)

• Tenderness at insertion of plantar fascia


Diagnosis • Pain with dorsiflexion of toes
• Presence of heel spurs on x-ray has low SN & SP

• Activity modification
Treatment • Stretching exercises
• Heel pads/orthotics

A patient with heel pain that is worse with weight bearing has plantar fasciitis. Plantar
fasciitis is MC in obese, middle-aged adults, but it may also be seen in younger, well-
conditioned individuals who participate in high-impact exercise or spend extended
periods in bare feet. Plantar fasciitis is characterized by inflammation and
degeneration of the plantar aponeurosis (deep plantar fascia), a thick, fibrous band
that extends from the calcaneus to the toes and supports the longitudinal arch of the
foot. It is thought to be due to chronic overuse and repetitive microtrauma to the
aponeurosis and its insertion point at the calcaneus.

The diagnosis of plantar fasciitis is based on clinical factors. The pain is often worst
when first standing from rest; it may lessen with activity but may again worsen after
long periods of standing or walking. Physical examination typically shows tenderness
at the insertion of the plantar fascia at the calcaneus, especially with the toes passively
dorsiflexed. Patients frequently have preexisting abnormalities of the arch (eg, pes
planus, high arch).
X-ray may show calcifications in the proximal plantar fascia ("heel spurs"), but this
finding is neither sensitive nor specific, and imaging is done primarily to rule out
other causes of pain. Initial management includes activity modification (eg, avoiding
walking barefoot, reducing high-impact exercise), stretching exercises, and
padded heel inserts. However, the plantar aponeurosis recovers slowly; the long-term
outcome is generally good, but symptoms may require months or even years for
complete resolution.

.........

Tarsal tunnel syndrome is caused by entrapment and compression of the tibial nerve
where it passes under the flexor retinaculum at the medial aspect of the ankle. It
presents with numbness and paresthesias at the toes and distal sole. The pain is
provoked by percussion at the tarsal tunnel (not palpation of the heel).

Chronic exertion compartment syndrome is related to transient relative ischemia


due to muscle expansion within a fixed fascial compartment. It is typically seen in
distance runners and presents with episodic muscle pain and paresthesias during
strenuous exercise. However, it does not cause pain when standing up from rest or
tenderness to palpation of the heel.

Achilles tendinopathy is a common cause of exercise-induced heel pain. However,


the pain and tenderness are most prominent at the posterior heel, often with palpable
thickening of the tendon. Also, the pain is typically reproduced by passive dorsiflexion
of the ankle rather than palpation of the heel.

Calcaneal stress fractures are typically seen in inexperienced athletes after initiation
of a high-impact exercise program. Like plantar fasciitis, they cause pain at the heel
that is worse with weight bearing. However, the pain can often be elicited by firm
palpation at the SIDES of the heel (squeeze test).

Differential diagnosis of heel pain

• Pain maximum upon first stepping out of bed


Plantar fasciitis • Local point tenderness with dorsiflexion of the toes

• Sudden-onset pain
Ruptured plantar
• Loss of height of the arch
fascia • Visible swelling or ecchymosis

Bone infection/ • Constant throbbing pain


metastasis • Nocturnal worsening

Calcaneal stress • Worse with activity


fracture • Palpation of the bone elicits tenderness

• Pain, paresthesia & numbness on the sole of the foot


Tarsal tunnel
• Percussion tenderness over the posterior tibial nerve in the tarsal
syndrome tunnel

The plantar aponeurosis (deep plantar fascia) is a fibrous band that extends from the
calcaneus to the toes and supports the longitudinal arch of the foot. Plantar fasciitis is
characterized by inflammation and degeneration of the aponeurosis due to overuse
and repetitive stress. It is common in middle-aged adults, especially those who
are obese. The pain is typically worst during the first steps of the day or after long
periods of standing or walking.

Examination in plantar fasciitis shows tenderness at the insertion of the plantar fascia
on the calcaneus, which is worse during passive dorsiflexion of the toes. X-ray may
show calcifications in the proximal fascia (heel spurs), but this is neither sensitive nor
specific. Initial management includes activity modification (eg, avoiding walking
barefoot), stretching, and padded heel inserts. Symptoms may take months for
complete resolution.

……..

Achilles tendinopathy causes posterior heel pain and thickening of the tendon. The
pain is reproduced by passive dorsiflexion of the ankle (which stretches the tendon)
rather than the toes (which stresses the plantar fascia).
Calcaneal stress fractures can occur after initiation of a high-impact exercise
program. They cause heel pain with weight bearing, but the pain is elicited by pressure
at the sides of the heel rather than at the insertion of the plantar fascia.

Charcot joint (neurogenic arthropathy) presents with deformity and sensory deficits of
the foot. Because it relates to recurrent trauma due to impaired proprioception and
pain sensation, the pain is typically mild.
STRESS Stress fracture
FRACTURE
• Repetitive activities (eg, running, gymnastics)
• Abrupt increase in physical activity
• Inadequate calcium & vitamin D intake
• Decreased caloric intake (e.g., anorexia)
Risk factors
• Female athlete triad: low caloric intake, hypomenorrhea/amenorrhea, low
bone density
• Improper footwear
• Biomechanical abnormalities (eg, weak calf muscles, high-arched feet)

• Insidious onset of localized pain


Clinical
• Point tenderness at fracture site
presentation
• Possible negative x-ray in the first 6 weeks

• Reduced weight bearing for 4-6 weeks


Management • Referral to orthopedic surgeon for fracture at high risk for malunion (eg,
ANTERIOR tibial cortex, 5th metatarsal)

A patient has subacute leg pain consistent with a tibial stress fracture, which most
commonly occurs in athletes (eg, runners, dancers) or other individuals who suddenly
increase their activity level. These types of fractures are caused by repeated tension
or compression without adequate rest, and are especially common in women who
have low bone density associated with low caloric intake and
hypomenorrhea/amenorrhea (female athlete triad).
In addition to the tibia, stress fractures can also be seen in the metatarsals, tarsal
bones (eg, navicular, calcaneus), and, less commonly, the femur and pelvis. The
diagnosis is based primarily on clinical factors, including localized activity-related
pain, swelling, and point tenderness on palpation. X-rays are frequently normal
(especially in the first few weeks) but may reveal periosteal reaction at the site of the
fracture. MRI and bone scan are more sensitive but not usually necessary.

..........

Injury to the interosseous ligaments (high ankle sprain) presents with acute
anterolateral ankle pain, typically following rotational force on a dorsiflexed
ankle. Patients frequently have an associated fibular fracture.
Nerve entrapment presents with radiating pain and is not reproducible with local
palpation.
Osteoid osteoma is a benign bone-forming tumor of the femur and other long bones
that occurs in adolescence and early adulthood. Focal tenderness is common, but the
pain is not activity related, and x-rays will reveal areas of lucency with sclerotic
margins.

Medial tibial stress syndrome (shin splints) causes anterior leg pain resembling that
of a stress fracture. It is usually seen in casual runners and is characterized by a
DIFFUSE area of tenderness (not point tenderness, as seen in stress #). Shin splints
are more common in overweight than underweight individuals.

A patient has a nondisplaced stress fracture of the second metatarsal. Stress fractures
are caused by repeated tension or compression without adequate rest and most
commonly occur in athletes and military recruits who suddenly increase their activity.
The second metatarsal, which is subjected to significant extremes of loading during
gait, is the MCly involved metatarsal. Typical findings include subacute, activity-
related pain and point tenderness over the fracture. The diagnosis is primarily based
on clinical findings. X-rays may initially be normal but can show a hairline lucency
or local periosteal thickening. MRI, CT, and scintigraphic bone scans are more SN,
especially for early fracture, but are not usually needed.
Fractures of the second, third, and fourth metatarsals are managed conservatively as
the surrounding metatarsals act as splints and nonunion is uncommon. Initial
interventions include rest and simple analgesics (eg, acetaminophen). Some experts
advise against NSAIDs due to a possible delay in healing times. Patients who continue
to have pain despite activity restrictions may additionally be managed with a wide,
hard-sole podiatric shoe.

………..
Stress fractures of the fifth metatarsal shaft are at increased risk for nonunion and
are usually managed with casting or internal fixation. However, stress fractures of the
middle (ie, second, third, and fourth) metatarsals usually heal well and do not require
casting or surgery unless there is severe pain, displacement, or other complicating
factors.

FAT EMBOLISM
Fat embolism syndrome

• Fracture of marrow-containing bone (eg, femur)


Etiology • Orthopedic surgery
• PANCREATITIS

• 24-72 hours following inciting event


• Clinical triad
Clinical presentation o Respiratory distress (RD)
o Neurologic dysfunction (eg, confusion)
o Petechial rash

Diagnosis • Based on clinical presentation

Prevention & • Early immobilization of fracture


treatment • Supportive care (eg, mechanical ventilation)

Fat embolism syndrome (FES): The mechanism of FES is not completely understood,
but it is postulated that fat enters the venous circulation following an inciting event (eg,
femur fracture) and causes mechanical disruption of capillary blood flow (fat
microemboli may be small enough to pass through the pulmonary circulation to the
systemic circulation) or leads to a systemic inflammatory response through the
production of toxic intermediaries. Patients classically have a triad of
severe respiratory distress (eg, hypoxemia, dyspnea, tachypnea, tachycardia), which
can mimic ARDS; neurologic dysfunction (eg, confusion, visual field defects); and
a petechial rash (present in up to half of cases). Low-grade fever and
subconjunctival hemorrhage may also be present. CXR is typically unremarkable at
the time of symptom onset but reveals bilateral pulmonary infiltrates within 24-48
hours.

The diagnosis of FES is based on compatible clinical presentation. Treatment is


supportive; approximately half of patients with FES due to long bone fracture require
mechanical ventilation. Death can occur, but the majority of patients make a full
recovery.

….

Blood transfusion-related pulmonary capillary damage describes transfusion-related


acute lung injury (TRALI). Patients typically demonstrate respiratory distress, but
crackles should be present on lung auscultation and chest x-ray should reveal
bilateral infiltrates (due to pulmonary edema) AT symptom onset.

Postoperative partial lung collapse (atelectasis) may lead to respiratory distress and
hypoxemia; however, it is ruled out by chest x-ray revealing clear lung fields.
Chest trauma can lead to myocardial or pulmonary contusion, both of which typically
manifest with respiratory distress due to pulmonary edema. Absence of external
evidence of chest trauma (eg, ecchymosis) and a clear CXR make these diagnoses
unlikely.

A marked decrease in blood alcohol level can lead to alcohol withdrawal in patients
with chronic alcohol abuse. Patients typically present with agitation, tachycardia,
confusion, and/or hallucinations within 24-48 hours of hospital arrival. Marked
hypoxemia and a petechial rash are not typical.

Thromboembolic occlusion of the pulmonary arteries describes pulmonary


thromboembolism. Patients often present with respiratory distress; however, the
presence of a petechial rash makes FES more likely.

COMPARTMENT
Clinical features of compartment syndrome
SYNDROME
• Pain out of proportion to injury
*** • Pain ↑ on passive stretch
Common • Rapidly increasing & tense swelling
• Paresthesia (early)

• ↓ Sensation
• Motor weakness (within hours)
Uncommon • Paralysis (late)
• ↓ Distal pulses (uncommon)
Compartment syndrome (CS) can be caused by direct trauma or prolonged
compression of an extremity or after revascularization of an acutely ischemic limb.
Patients typically have excruciating pain that is worsened on passive ROM and does
not respond well to narcotics. There is considerable variation in associated signs and
symptoms, and a high index of suspicion is needed to make a diagnosis. Paresthesia
from sensory nerve ischemia is usually an early finding. Neurologic deficits (eg,
sensory loss, motor weakness) may be present but develop later in the course of the
disease. Pallor is the result of arterial occlusion but is uncommon and not required
for diagnosis. Similarly, arterial pulses are present in a majority of patients, especially
in early stages of CS.

Diagnosis can be confirmed by measuring compartment pressures in the affected


extremity.

However, in high-risk patients (eg, those with limb revascularization, other


examples?) who develop symptoms suggestive of CS (pain, swelling, sensory loss), the
diagnosis may be made on clinical grounds ALONE!! (no need for measuring
compartment pressure). Time to fasciotomy is the most critical prognostic indicator
and should be performed without delay.
…….

Supportive measures should be taken to maintain perfusion pressure in the extremity,


which includes keeping the leg AT the level of the torso (not higher or lower). Leg
elevation and ice would be recommended if leg edema were secondary to
inflammation (eg, cellulitis).
Increased analgesic dosages may be needed to control increasing pain. However,
surgical intervention is of utmost importance to preserve limb function. Neurologic
symptoms and tense swelling highly suggest developing CS rather than simple
postsurgical changes.

A patient has severe lower extremity pain


worsened on passive stretch of the calf
muscles. In light of the acute trauma and
concurrent antiplatelet medications, which
may cause occult bleeding in tissues, this
presentation is concerning
for compartment syndrome (CS). CS is a
limb-threatening condition caused by
increased pressure within an
enclosed fascial space that limits perfusion
of muscle and nerve tissues. Although
frequently associated with trauma
with fracture, other common causes of CS
include crush injury, severe burns, and
arterial reperfusion procedures; patients
who are on anticoagulation or have a bleeding diathesis are at increased risk. In
addition to severe pain (classically out of proportion to the injury), other possible
features include paresthesias, loss of sensation, motor weakness, and diminished
pulses.

However, although CS can often be a clinical diagnosis, not all features may be present
early in the course, and pain medications may mask the symptoms. Clinicians must
maintain a high index of suspicion. Surgeons often choose to confirm the diagnosis by
measuring compartment pressures (eg, needle manometry); a delta pressure (DBP
− compartment pressure) ≤30 mm Hg is suggestive of CS. Definitive management
includes urgent fasciotomy (although some patients with mild CS may be
managed conservatively with close observation if compartment pressures are falling
and delta pressures are rising). Compartment pressure measurement can also help
avoid unnecessary fasciotomies in patients in whom CS is one of the differential
diagnoses under consideration.
……….
Compression wraps of the calf are typically used for edema caused by chronic venous
insufficiency, especially when associated with additional complications (eg,
stasis dermatitis).

CT scan of the lower extremity is unlikely to be helpful acutely in a patient with no


fracture noted on x-ray

AMPUTATION
Post-amputation pain
/////////////////
• Tissue & nerve injury
*** Acute stump pain
• Severe pain lasting 1-3 weeks

• Swelling, skin discoloration


Ischemic pain • Wound breakdown
• ↓ Transcutaneous oxygen tension

• Weeks to months after amputation


Post-traumatic
• Focal tenderness, altered local sensation
neuroma
• ↓ Pain with anesthetic injection

• Onset usually within 1 week


Phantom limb pain • Increased risk in patients with severe acute pain
• Intermittent cramping, burning felt in distal limb
A patient has intermittent stump pain despite good wound healing following lower
extremity amputation. The pain is elicited by focal palpation near the operative site,
which suggests a post-traumatic neuroma. Post-traumatic neuromas are initiated by
the transection of nerve fibers, which leads to an inflammatory reaction and
formation of a tangled mass of unmyelinated nerve endings. These nerve endings
have decreased depolarization thresholds that cause pain signals spontaneously or
in response to nonpainful stimuli.

Post-traumatic neuromas form over several weeks to months following injury or


amputation. Because the pain is worsened by local pressure, neuromas can complicate
fittings for prosthetic devices. The diagnosis is based primarily on clinical grounds, but
injection of a local anesthetic can provide transient pain relief and confirm the
diagnosis. Management typically involves excision of the neuroma; TCAs and
antiepileptic medications can be used for pain management prior to surgical
intervention.
........

Heterotopic ossification is characterized by abnormal soft tissue calcification. It is


more common in traumatic amputation, in which the amputation line passes through
a region of damaged tissue, than in surgical amputation. Common associated findings
include swelling, erythema, fever, and skin ulceration.
Chronic osteomyelitis is a common complication of amputation, especially in patients
with comorbid diabetes. It is typically associated with swelling, chronic wounds, or
formation of weeping sinus tracts. However, minor bogginess at the amputation site is
normal, and this patient has no other signs of osteomyelitis.
Phantom limb pain is common following amputation and has a highly variable
presentation. However, it is typically apparent in the first week following amputation,
causes sensations felt in the distal amputated limb, and is less likely than neuroma to
be elicited by point palpation.

Stump hematoma typically presents with swelling, ecchymosis, and skin breakdown
in the first several days following amputation. Formation of a chronic, expanding
hematoma may occur but is rare and would not cause point tenderness with only
minor soft tissue swelling.

A patient has acute low back pain consistent with lumbar strain. She has no
complicating factors (eg, neurologic deficits) or red flag features (eg, fever, weight loss,
history of malignancy) that warrant imaging or laboratory testing. Most patients with
acute (ie, <4 weeks), uncomplicated low back pain will have spontaneous resolution;
therefore, initial management emphasizes short-term, symptomatic relief.

Patients with acute back pain should be advised to continue moderate


activity. Extended periods of bed rest or other significant reductions in activity are
associated with prolonged pain and stiffness and should be avoided. Nonopioid
analgesics are preferred as initial management; a short course of NSAIDs is adequate
for most patients. Acetaminophen is less effective but can be considered for those who
cannot take NSAIDs. Nonpharmacologic adjunctive measures include heat and spinal
manipulation.
……..
Inflammatory markers (eg, ESR, CRP) are usually combined with imaging during the
evaluation of infection (eg, vertebral osteomyelitis), vertebral metastases, or
inflammatory back pain (ie, pain and stiffness that lessen with activity and worsen at
night and with rest, as seen in conditions like ankylosing spondylitis).

Patients who do not improve with NSAIDs may benefit from a nonbenzodiazepine
muscle relaxant (eg, cyclobenzaprine). Short courses of opioid analgesics (eg,
hydrocodone) are sometimes used for refractory back pain, but they are less effective
than NSAIDs and carry a significant risk profile (eg, sedation, respiratory
depression, abuse).
MRI is indicated to evaluate patients with neurologic deficits, signs of infection (eg,
fever), or cauda equina syndrome (eg, saddle anesthesia, urine retention). X-ray can be
used to evaluate for spinal metastasis in patients with a history of malignancy and can
also identify compression fractures in patients with osteoporosis. However, imaging
does not improve outcomes for low-risk patients with uncomplicated acute back pain.

A patient who required below-knee amputation following trauma now has persistent
pain in the absent portion of the limb. This presentation is most consistent
with phantom limb pain (PLP) and should prompt initiation of a multimodal pain
regimen that includes both pharmacotherapy and adjuvant therapies.
PLP is a complex and incompletely understood pain syndrome that is
extremely common (~50%-85%) among amputees. The pain, which is perceived in
the absent portion of the limb, is usually neuropathic (eg, shooting, burning) and
may have seemingly unrelated, innocuous triggers (eg, urination, defecation).
Severe PREOPERATIVE pain (eg, due to devascularizing trauma) and/or
postoperative pain likely increase the risk of developing PLP.
Treatment of PLP is typically multimodal and may include:
• Pharmacotherapy: antidepressants (eg, TCAs), antiepileptics (eg, gabapentin),
NMDA receptor antagonists (eg, ketamine), and analgesics (eg,
acetaminophen, opioids).

• Adjuvant therapies: biofeedback, CBT, and mirror therapy (ie, using a mirror
to watch the reflection of the residual limb moving at the location of the absent
limb).

..............

Regrowth of transected nerve fibers into a tangled mass of unmyelinated nerve


endings can cause posttraumatic neuroma with neuropathic pain. Neuroma pain
would be temporarily relieved (and, therefore, diagnosed) by local anesthetic injection;
however, in contrast to this patient's pain, it would be located in the distal amputation
stump and exacerbated by palpation or percussion.
MRI can help diagnose lumbosacral radiculopathy (eg, L5, S1), which may cause
shooting pain that radiates into the foot. However, radiculopathy typically has
associated back pain, and ROM testing at the hip (ie, similar to the straight leg raise
test) often exacerbates symptoms.

Surgical stump revision may be necessary for a patient whose stump fails to heal
postoperatively or cannot tolerate a prosthesis (eg, due to awkward stump shape or
inadequate soft tissue padding). In contrast, this patient has a well-healed surgical scar
and adequate soft tissue padding.

A patient sustained a traumatic amputation of his dominant thumb. Thumb


replantation will likely be attempted, especially given that his profession (ie, mechanic)
requires manual dexterity and that the thumb provides an estimated 40% of hand
function. Optimal care of the amputated part during transport improves the chances
of replantation success.

Care of an amputated part, regardless of the level of injury (eg, digit, hand, forearm),
begins with gentle removal (eg, saline irrigation) of gross contamination (eg, sawdust,
debris). Following this, the part should be wrapped in sterile saline-moistened
gauze and sealed in a plastic bag. The bag should be then placed in a container of ice
water to be transported with the patient. Cooling of the amputated part decreases
tissue metabolism and oxygen demand, thereby minimizing ischemic damage and
prolonging the window of viability for replantation. An ice water bath (temperature ~0
C [32 F]) provides adequate cooling while minimizing the risk of frostbite, which can
occur if the amputated part, or even the bag containing the part, is placed directly on
ice.

………
The amputated part should be kept moist, rather than open to air, to prevent
desiccation of the exposed tissues, which can cause further tissue damage.

The amputated part should be kept moist but should not remain submerged in any
solution during transport. Prolonged submersion in water can injure digital vessels,
making microsurgical vessel repair more difficult or impossible. Prolonged
submersion in antiseptic solution can cause chemical injury. Instead, to decrease
infection risk, the amputated part should be thoroughly cleansed (eg, saline irrigation)
and prepped (eg, betadine) at the same time in the operating room prior to
attempted replantation.
BONE TUMOR

A patient with Paget disease of bone now


has a destructive femoral lesion, raising strong suspicion for osteosarcoma. Most cases
occur in a bimodal distribution, as follows:

• In children and adolescents, osteosarcoma usually develops at the metaphysis


of long bones, where cellular turnover is high (primary osteosarcoma).
• In adults age >40, osteosarcoma usually develops at sites of damaged bone,
particularly due to Paget disease of bone, irradiation, or prior benign bone
tumor (secondary osteosarcoma).

Paget disease of bone is associated with increased bone remodeling and bone
overgrowth. Although most cases are asymptomatic, the risk of osteogenic
sarcomatous transformation (at sites of bone overgrowth) is several-thousand times
greater than the general population. Manifestations of malignant transformation
generally include worsened localized pain and soft tissue swelling. Imaging usually
shows a destructive bone lesion with a mixture of radiodense and radiolucent
areas, a sunburst periosteal pattern, and/or Codman triangle (periosteal elevation).
Adults who develop osteosarcoma have a poor prognosis.
……….
Lyme arthritis is a manifestation of Borrelia
burgdorferi infection usually characterized by
monoarticular arthritis of the knee. However, it
typically develops months to years after tick exposure.
In addition, although erosion of the joint cartilage or
bone can sometimes occur, cortical bone destruction
with periosteal elevation would be atypical.
 Osteoarthritis, characterized by inflammatory
destruction of articular cartilage, often involves several
joints (eg, knees, hips). Imaging generally shows
thickening of subchondral bone, joint space narrowing,
and formation of osteophytes.

 Stress fracture is an overuse injury to bone caused by


repetitive stress (eg, running on pavement). Although
stress fracture can be associated with periosteal
elevation, cortical thickening, and sclerosis, the presence
of bone destruction with radiolucent areas suggests a
neoplastic process. In addition, stress fracture is rare in the
femur; most cases occur in the tibia and fibula.

 CPPD is common in older patients.


Symptomatic disease often presents with acute
arthritis in the knee (>50% of cases). However,
imaging shows cartilage calcification (eg, menisci,
pubic symphisis) and degenerative changes in
the joint that are similar to OA (eg, subchondral
cysts, osteophytes, bone/cartilage
fragmentation).
A patient has progressive knee pain
with a lytic bone lesion consistent
with giant cell tumor of
bone (GCTB). GCTB is a benign
but locally destructive neoplasm
that is MC at the epiphysis of long
bones. The incidence is highest
in young adults who have reached
skeletal maturity, but it can also
occur in older adults, typically
associated with PAGET disease of
bone. The course is variable; most
patients have only local manifestations (eg, pain, swelling, stiffness, pathologic
fracture), but pulmonary metastasis and malignant transformation may occur.

X-ray in GCTB characteristically shows an eccentric lytic lesion, which is often described
as resembling soap bubbles. CT imaging can show greater detail of the mass, and MRI
is often indicated to visualize the adjacent soft
tissues. The diagnosis is confirmed with biopsy,
which shows multinucleated giant
cells resembling osteoclasts interspersed with
sheets of mononuclear stromal cells.

Surgery (eg, intralesional curettage or excision)


is first-line treatment.
Monoclonal antibodies (eg, DENOSUMAB)
against the receptor activator of nuclear factor
kappa-B ligand (RANKL), which is overexpressed in the stromal cells, can also be used
to shrink the tumors.

……..
Baker (popliteal) cyst is caused by extrusion of fluid from the knee joint into the
semimembranosus/gastrocnemius bursa. It is typically seen in patients with
underlying knee disorders (eg, osteoarthritis) and presents as an asymptomatic bulge
in the popliteal fossa or with pain and swelling in the posterior calf. X-ray findings in
OA include joint space narrowing, subchondral sclerosis, and osteophyte formation, but
the epiphyseal bone is normal. A young patient with no history of joint disease is
unlikely to have osteoarthritis or a Baker cyst.
Tibial tubercle apophysitis (Osgood-Schlatter disease [OSD]) causes pain at the
insertion of the patellar tendon. It usually occurs
in adolescent and preadolescent patients and
frequently follows a rapid growth spurt. X-ray
may show avulsion of the tubercle, but the femur
is normal.

Patients with advanced HPTH can develop


increased resorption in cortical bone with
subperiosteal thinning and cystic degeneration
(osteitis fibrosa cystica). Although x-ray may show
lytic lesions, most patients have multifocal
involvement, and either symptoms of
hypercalcemia (eg, constipation, fatigue; in
primary HPTH) or a history of CRF (in secondary
HPTH).
 Osteoid osteoma is another benign bone
tumor that appears on imaging as a small, round
lucency. It typically causes nocturnal pain that is
quickly relieved by NSAIDs (eg, ibuprofen).

This patient with a history of retinoblastoma has a lytic


bone lesion in the distal femur with "sunburst"
periosteal reaction (concentric layers of reactive
bone), raising strong suspicion for osteosarcoma.
Most cases arise in children or adolescents at long-
bone metaphyses (site of greatest bone proliferation)
and present with unilateral pain and swelling; x-rays
generally show a lytic bone lesion, sunburst periosteal
reaction, and/or Codman triangle (raised edge of
ossified periosteum).

Risk of osteosarcoma is greatest in those with


inherited genetic mutations to the tumor suppressor
genes RB1 (linked to hereditary retinoblastoma)
and TP53 (linked to the congenital cancer disorder Li
Fraumeni syndrome). Therefore, patients often have a
previous history of other cancers, such as
retinoblastoma (RB1) or leukemias, breast tumors,
brain tumors, or adrenocortical tumors (TP53).
Osteosarcoma can also arise in older adults (usually
age >60) in areas of previous bone damage due to
Paget disease (MC), irradiation, or benign bone
tumors; these individuals are less likely to have inherited genetic mutations.
……..

Brodie abscess is a form of chronic osteomyelitis that


typically occurs in the femoral or tibial metaphysis in
the second decade of life. X-ray typically shows a
radiolucent, elongated lesion with surrounding
reactive sclerosis. A history of retinoblastoma and
sunburst periosteal reaction make osteosarcoma more
likely.

 Langerhans cell histiocytosis (LCH) is a rare disorder


most often seen in children age <3 years. Single or
multiple "punched-out," lytic bone lesions are
common. This patient's age, history of retinoblastoma,
and sunburst periosteal reaction make
osteosarcoma more likely.

 Myositis ossificans usually develop at


the site of muscle injury (eg, quadriceps
contusion); they are marked by an
eggshell-appearing periosteal reaction
around the area of the muscle injury. A
lytic bone lesion would not be seen.

 Unicameral bone cysts are simple bone


cysts that generally occur in individuals age
<20 years. X-rays usually reveal a full-
thickness lytic bone lesion with well-
defined (not irregular) borders and
surrounding reactive sclerosis.
HYPERKALEMIA Succinylcholine is a depolarizing neuromuscular blocker that works by binding to
(HrK) postsynaptic acetylcholine receptors to trigger influx of sodium ions and efflux of
potassium ions through ligand-gated ion channels; depolarization occurs and
temporary paralysis ensues (delayed repolarization of the skeletal muscle
membrane). Succinylcholine is often used during rapid-sequence intubation as it has
a rapid onset (45-60 seconds) and offset (6-10 minutes) of action. However, in
certain patients it can cause life-threatening cardiac arrhythmia due
to severe hyperkalemia.

A patient has experienced an extensive skeletal muscle CRUSH injury, which places
him at risk for hyperkalemia due to skeletal muscle cell lysis (rhabdomyolysis). In
addition, skeletal muscle injury leads to upregulation of postsynaptic acetylcholine
receptors, which can result in massive efflux of potassium following administration of
succinylcholine. Other relevant conditions that cause upregulation of acetylcholine
receptors include BURN injury, disuse muscle ATROPHY, and DENERVATION (eg,
stroke, GBS, critical illness polyneuropathy).
To avoid life-threatening hyperkalemia in such
settings, nondepolarizing neuromuscular blocking agents (eg, vecuronium,
rocuronium) should be used as they do not affect postsynaptic ligand-gated
ion channels.
……….

Halothane can lead to acute liver failure due to production of hepatoxic intermediary
compounds; therefore, it is now rarely used. Adult women are at greatest risk of
developing halothane-associated liver toxicity.
Etomidate inhibits 11β-hydroxylase and can lead to adrenal insufficiency. The
elderly and patients with critical illness (eg, sepsis) are typically susceptible.

Nitrous oxide (N2O) inactivates vitamin B12, leading to inhibition of methionine


synthase activity; subsequent neurotoxicity (eg, peripheral neuropathy) can result in
patients with preexisting vitamin B12 deficiency.
Severe HoTN due to myocardial depression is a common adverse effect of
propofol. Therefore, propofol should be avoided or used with extreme caution in
patients with ventricular systolic dysfunction.
OSTEONECROSIS
Avascular necrosis (AVN)

• Steroid use
• Alcohol abuse
• SLE
• APLAS
Etiology • Hemoglobinopathies (eg, sickle cell)
• Infections (eg, osteomyelitis, HIV)
• Renal transplantation
• Decompression sickness

• Groin pain on weight bearing


Clinical manifestations • Pain on hip abduction & internal rotation
• No erythema, swelling, or point tenderness

• Normal WBC count


Laboratory findings • Normal ESR & CRP

• Crescent sign seen in advanced stage


Radiologic imaging • MRI is most SN modality

CRP = C-reactive protein; ESR = erythrocyte sedimentation rate.


A patient with progressive hip pain in the setting of chronic glucocorticoid use
(dorsocervical fat pad, moon facies, glucocorticoid myopathy) has typical features
of osteonecrosis (AVN) of the femoral head. Osteonecrosis occurs in disorders that
disrupt the circulation of bone through micro-occlusion, abnormal endothelial
function, or increased intraosseous pressure. It is a common complication of long-
term glucocorticoid use, possibly due to effects on osteocytes or abnormal plasma
lipid levels causing microemboli. Osteonecrosis causes bone and BM infarction.
Abnormal bone remodeling subsequently results in trabecular thinning and collapse
over months to years.

Osteonecrosis of the femoral head is characterized by pain in the groin, thigh, or


buttock that is worsened by activity and relieved by rest. Progression of the disease can
lead to reduced range of motion (usually abduction and internal rotation), rest pain,
and joint instability. In the first few months, x-rays are often normal, and MRI is a
more sensitive test.

………

Septic arthritis can occur in the hip from hematogenous seeding or direct extension of
a regional infection (eg, diverticulitis). Most patients have systemic symptoms (eg,
fever) and an acute course over days.

Osteoarthritis of the hip is uncommon in patients age <40 unless there is antecedent
trauma or a systemic disorder that causes premature joint degeneration (eg,
hemochromatosis).
A patient has features typical of osteonecrosis (aseptic necrosis) of the femoral
head, including progressive hip pain, limited internal rotation and
abduction, unremarkable x-rays (can remain normal for months), and normal
inflammatory markers (eg, ESR). Osteonecrosis is caused by occlusion of end
arteries supplying the femoral head, leading to necrosis and collapse of the
periarticular bone and cartilage.

Osteonecrosis is common in patients with sickle cell disease due to disruption of


microcirculation in the bone by sickling as well as increased intraosseous pressure due
to bone marrow hyperplasia. The femoral head has 2 main sources of blood - the
ascending arteries and the foveal artery, which lies within the ligamentum
teres. The foveal artery is patent early in life, but may become obliterated in older
patients. For this reason, aseptic necrosis of the femoral head is uncommon in children
but the risk rises in older patients.

….

Osteomyelitis is a recognized complication of sickle cell disease, and is usually due to S


aureus or Salmonella. It is MC in children, often multifocal, and typically accompanied
by fever, malaise, and elevated ESR.

N gonorrhoeae can cause acute purulent arthritis (usually without associated skin
lesions or fever). However, involvement of the distal large joints (eg, knees, wrists,
ankles) is more common, and it would usually be associated with elevated ESR.

Features that suggest drug-seeking behavior include "lost" or "stolen"


medication, premature refill requests, and pain inconsistent with examination findings
or known pathology.

OSTEOPOROSIS
Secondary causes of osteoporosis

• HypERthyroidism
• HPTH
Endocrine • HypERcortisolism
• HypOgonadism

Metabolic/ • Calcium &/or vitamin D deficiency


nutritional • Eating disorders

Gastrointestinal/ • Malabsorption (eg, celiac disease, Crohn disease)


hepatic • Chronic liver disease

• Chronic kidney disease (CKD)


Renal • Renal tubular acidosis (RTA)

• Glucocorticoids
Medications • Phenytoin, carbamazepine (CBZ)
• Proton pump inhibitors (PPIs)
• Inflammatory disorders (eg, RA)
• Multiple myeloma (MM)
Other • Alcoholism
• Immobilization

This patient has a distal radius (Colles) fracture after falling on his outstretched hand
(FOOSH). Significant fracture due to a ground-level fall is uncommon and often
indicates underlying bone fragility due to osteoporosis. The MCC of osteoporosis is
declining estrogen levels in women after menopause; the risk is significantly lower in
men, and osteoporosis in a young or middle-aged man suggests the presence of a
secondary cause.

Bone loss is very common in patients with celiac disease, largely due
to malabsorption of vitamin D (impaired absorption of fat-soluble vitamins) and
resultant secondary HPTH (SHPTH). Bone disease may present as (otherwise) asx
osteopenia/osteoporosis or as osteomalacia with bone pain, muscle weakness, and
impaired ambulation. Laboratory evaluation shows low serum 25-hydroxyvitamin D and
elevated PTH levels. ALP may be elevated, and hypocalcemia and
hypOphosphatemia may be observed in severe cases.
……….
The prevalence of osteoporosis increases with age, but in men it is usually seen at age
>65. The risk in middle-aged men is low.
The risk of osteoporosis is increased in patients with low (NOT NORMAL) body weight
(BMI <18.5 kg/m2), likely due to decreased mechanical stress on the skeleton and
comorbid conditions that contribute to both weight loss and osteoporosis (eg,
smoking, anorexia nervosa).

The prevalence of both hypertension and osteoporosis increases with age, and the
conditions often coexist. However, hypertension does not contribute to osteoporosis,
and thiazide diuretics, commonly used in hypertension, may decrease the risk by
reducing renal excretion of calcium.

Hyperthyroidism can increase the risk of osteoporosis due to accelerated bone


turnover. Hypothyroidism is not a major risk factor unless thyroid hormone is
chronically over-replaced.
ANOREXIA
Anorexia nervosa
NERVOSA
Clinical • BMI <18.5 kg/m2
features • Fear of weight gain, distorted body image

• Decreased bone mineral density, osteoporosis


• Amenorrhea
• Lanugo, hair loss, dry skin
Medical
• Gastroparesis, constipation
complications • Enlarged parotid glands (if binge/purge type)
• Hypotension, hypothermia, bradycardia
• Cardiac atrophy, arrhythmias

Low body weight (BMI <18.5 kg/m2), stress fracture, and distress in response to the
recommendation to limit his physical activity are concerning for anorexia nervosa
(AN). Although AN is more common in female patients, male patients are also
affected and at risk for bone loss. Decreased bone mineral density, which is caused
by a number of factors (eg, endocrine abnormalities, hypercortisolism, growth hormone
resistance), results in an increased risk of bone fractures. Other medical complications
associated with AN include bradycardia, hypotension, and cardiac atrophy.

The most important next step in management of this patient is to obtain


a comprehensive dietary history. Caloric intake and meal patterns, attitudes about
food and weight, and history of bingeing and efforts to control weight with
compensatory behaviors (eg, exercise, fasting, self-induced vomiting, misuse of
laxatives, diuretics) should be assessed. In addition, AN in male patients may present
with a focus on muscularity rather than thinness. In these cases, dietary history may
reveal dietary restrictions aimed at muscle building and the use of protein
supplements at the expense of balanced nutrition.
……
Advising participation in another sport would not treat an underlying eating disorder.
Although exogenous steroid use (eg, androgen) can predispose to decreased bone
mineral density and fractures, it is typically associated with increased muscle strength
and mass, and other typical signs of steroid use could be seen (eg, hypertension, acne,
gynecomastia, testicular shrinkage). Obtaining a dietary history would take priority
in a patient with significantly low body weight and no other signs of steroid use.
Hyperthyroidism can result in decreased bone mineral density and weight loss.
However, these symptoms are typically accompanied by increased appetite and signs
of sympathetic hyperstimulation such as tachycardia, hypertension, heat intolerance,
tremor, and hyperreflexia.
The initial management of small tibial stress fractures includes rest, analgesia, and
stabilization with a splint or brace if necessary. Surgical intervention could be
considered for severe fractures or for lack of healing or severe pain despite
conservative treatment.

OSTEOMYELITIS
Chronic osteomyelitis

• Hematogenous seeding
Mechanisms • Extension from infection in adjacent structure (e.g., diverticulitis)
• Direct inoculation (eg, compound fracture, bite wound)

• S aureus (MC)
Pathogens • Coagulase-negative staphylococci (CoNS)
• Polymicrobial

• Surgical hardware/instrumentation
Risk factors • DM
• Impaired circulation or sensation in the limb

• Persistent s/s (e.g., pain, swelling, chronic wound)


Clinical
• Sinus tract formation
features • Nonunion of fracture

• Positive probe-to-bone test


• Imaging: lytic lesion with loss of cortical & trabecular bone, surrounding
Diagnosis sclerosis, periosteal thickening
• Bone biopsy for culture

• Surgical debridement
Management
• Antibiotics (prolonged course)

A patient has a tibial fracture complicated by nonunion, defined as failure of a fracture


to achieve radiographic or clinical evidence of union within an adequate time frame
(eg, 6 months for long bone shaft fractures). Risk factors for nonunion include
impaired blood flow (eg, smoking, arterial atherosclerosis), certain medications (eg,
systemic glucocorticoids), metabolic disorders (eg, DM, hypothyroidism), and
infection (eg, osteomyelitis).

Acute osteomyelitis typically presents with local pain, erythema, and


fever over a few days or weeks. However, if the bony infection is not
completely eradicated, it can lead to a chronic, indolent infection
(ie, chronic osteomyelitis) with necrosis and fragmentation
(sequestrum) of the bone. Chronic osteomyelitis is a common
complication of high-velocity tibial fractures because the bony
fragments are often exposed and the blood supply may be disrupted by
trauma.

A sinus tract with a persistently draining wound, strongly suggests


chronic osteomyelitis. Intermittent pain and swelling may occur;
however, fever and leukocytosis are usually less prominent. The
diagnosis is suggested by this patient's x-ray, which
shows fragmentation of the bone and ragged, irregular fracture lines. ESR and CRP
are often elevated.

OPEN bone biopsy is preferred over percutaneous aspiration for microbiologic


sampling.

Culture of sinus tracts generally has poor SN and SP.


Surgical debridement of the infected and necrotic bone is required for definitive
management.

………..
Amputation is often required for diabetic patients with ulcers that do not heal due to
impaired blood flow. However, this patient's sinus tract is caused by chronic
osteomyelitis, and aggressive treatment of the infection may allow limb salvage.
A CTA to assess vascular status is indicated for patients with fractures associated with
absent distal pulses. Although violent fractures can cause disruption of the tibial blood
flow despite palpable distal pulses, other vascular signs (eg, pallor or mottling of the
skin, decreased sensation) are usually present.

Surgical fixation with insertion of additional hardware without first eliminating the
infection would likely result in infection of the hardware. Once the infection is fully
eradicated, additional orthopedic fixation could be considered.

Silver sulfadiazine and skin grafting are often used for burn wounds. These superficial
interventions would not treat the deep infection.
Osteomyelitis in children

• Hematogenous spread
Pathogenesis • S aureus MCC

• Fever, irritability
Clinical features • Limited function (eg, limp)
• Bony tenderness, swelling

• Elevated ESR, CRP, WBC count


• Blood culture
Diagnosis • X-ray (often normal), MRI
• Definitive: Bone biopsy/culture

Treatment • Antistaphylococcal antibiotic (eg, vancomycin)

CRP = C-reactive protein; ESR = erythrocyte sedimentation rate;


WBC = white blood cell.
Fever, leg pain, and refusal to bear weight are concerning for acute osteomyelitis.
Although trauma (eg, surgery, injury) can sometimes introduce infection to bone, in
children osteomyelitis is MCly caused by hematogenous spread. Bacteria (typically S
aureus) travel through the bloodstream and seed the well-vascularized metaphysis of
long bones (eg, femur), resulting in focal BM infection. Inflammation compromises
blood flow, allowing the infection to penetrate the bony cortex and cause necrosis.

Signs of bony inflammation include fever and pain (which may initially be referred to
the knee) as well as swelling, erythema, and point tenderness over the affected
area. Inflammatory markers are typically elevated and blood culture can help
determine the infectious etiology. Radiographs may be normal initially or
reveal periosteal elevation (due to subcortical purulence) or
cortical thickening. Although inflammation on MRI typically confirms the
diagnosis, biopsy and culture of the infected bone is the gold standard for diagnosis.
Treatment is with antistaphylococcal antibiotics (eg, vancomycin).

………………

AVN of the femoral head, or LCP disease, presents with hip (or referred knee) pain and
a limp. Patients are afebrile and have limited range of motion of the hip without soft-
tissue swelling on examination.

Decreased mineralization of bone is seen in rickets and can lead to generalized, chronic
bone pain and bowing. Patients are at increased risk for fracture, not osteomyelitis.

Ewing sarcoma is a malignant degeneration of bone commonly found in the femoral


diaphysis. Findings include localized pain and swelling; however, symptoms typically
present over weeks to months and are often worse at night.

A patient has vertebral osteomyelitis. Injection drug users, patients with HbS,
and immunosuppressed patients are at highest risk for osteomyelitis. The spine is a
frequent site of osteomyelitic infection in IVDU. In this group, S aureus is the
MC pathogen, but infections with gram-negative organisms also occur.
Most cases of vertebral osteomyelitis are chronic (>6 weeks) and insidious with
minimal symptoms. Many patients have back pain unrelieved by rest; fever is present in
<50%. Physical examination often shows few findings, but tenderness to gentle
percussion over the spinous process of the involved vertebra can be an important
clue.
WBCs may be elevated or normal. Platelet count is often high as a marker of
inflammation/stress. The ESR is often significantly elevated (>100 mm/hr). MRI is the
most SN diagnostic study.

Treatment is long-term IV antibiotics with or without surgery.

…….

Although prostate cancer is a possibility, a young age, localized pain on palpation, lack
of night pain, and lack of urinary symptoms make this a less likely diagnosis.

Vertebral compression fractures can cause local tenderness to palpation, but this
usually occurs in elderly patients with a history of osteoporosis. (Mets → no
tenderness??)

CHARCOT JOINT
Charcot joint (neurogenic arthropathy)

• Diabetic neuropathy
• Other peripheral neuropathy (eg, vitamin B12 deficiency)
Risk factors • Spinal cord injury
• Syringomyelia
• Tabes dorsalis (tertiary syphilis)

• Impaired sensation & proprioception


Pathogenesis • Altered weight bearing & recurrent trauma
• Acute inflammatory response

• Impaired ambulation
• Foot & ankle deformity (swelling, collapsed arch, decreased range of
Clinical
motion)
manifestations • Mild pain
• X-ray: bone & joint destruction, fragmentation, subluxation/dislocation

• Mechanical offloading & correction of joint mechanics (eg, casting,


Management orthotics)
A patient has destruction and collapse of the ankle joint and arch consistent with
neurogenic arthropathy (Charcot joint). Neurogenic arthropathy is due to impaired
sensation and proprioception in the foot, leading to altered weight bearing,
mechanical stresses, and recurrent trauma. It is most commonly seen in diabetic
peripheral neuropathy, other neuropathies (eg, vitamin B12 deficiency), syringomyelia,
spinal cord injury, and tabes dorsalis.
Neurogenic arthropathy causes difficulty in weight bearing and ambulation. Patients
may also have acute inflammation (eg, redness, swelling) following minor
trauma. However, pain is mild due to the underlying loss of sensation. Examination
shows skeletal deformity, skin calluses, and neuropathic ulcers.

Laboratory tests in neurogenic arthropathy, including blood counts, inflammatory


markers (eg, CRP), and synovial fluid analysis, are typically normal. X-rays show bony
destruction (eg, deformity, fragmentation), decreased bone mass, osteophyte
formation, and loss of joint spaces. Management
involves casting and orthotic footwear to assist weight bearing and decrease
further trauma.

………
 Septic arthritis causes acute fever, redness, swelling, and joint
pain. Osteomyelitis can present as a chronic, indolent infection,
but is typically seen in association with a neuropathic ulcer with
sinus tracts or exposure of the underlying bone; x-ray shows bone
necrosis with surrounding sclerosis and periosteal thickening,
rather than collapse of the joint.
RA typically presents as a symmetric, polyarticular arthritis, and
most patients have upper extremity (eg, hands, wrists)
involvement. X-rays reveal periarticular osteoporosis, joint
erosions, and joint space narrowing.
 Gout

////////////////////////////////OPHTHALMOLOGY////////////////////////////////
GLAUCOMA
Angle-closure glaucoma (ACG)

Clinical • Symptoms: headache, ocular pain, nausea, decreased visual acuity


features • Signs: conjunctival redness; corneal opacity; fixed, mid-dilated pupil

• Tonometry (measures intraocular pressure)


Diagnosis • Gonioscopy (measures corneal angle)

• Topical therapy: multidrug topical therapy (eg, timolol, pilocarpine,


apraclonidine)
Treatment • Systemic therapy: acetazolamide (consider mannitol)
• Laser iridotomy

A patient has acute angle-closure glaucoma (ACG) presenting with headache, ocular
pain, nausea, and decreased visual acuity. ACG is characterized by narrowing or
closure of the anterior chamber angle, leading to decreased aqueous outflow and
elevated intraocular pressure (IOP). Examination findings include conjunctival
injection; corneal edema; palpable firmness of the eyeball; and a fixed, mid-dilated
pupil. The diagnosis is confirmed by gonioscopy to visualize the corneal angle and/or
tonometry to measure IOP.

Initial management is directed at rapidly lowering IOP. Combination therapy with


multiple topical agents is recommended; a typical regimen includes timolol (which
reduces aqueous production), apraclonidine (which decreases aqueous production
and increases outflow), and pilocarpine (which causes ciliary muscle contraction to
open the trabecular meshwork at the corneal angle) eye drops. In addition, oral or
intravenous acetazolamide is recommended to rapidly reduce further production of
aqueous humor. Subsequently, laser iridotomy can facilitate aqueous outflow and
provide definitive management.
……………

Pupillary dilation (eg, dilated funduscopic examination) narrows the corneal angle and
can precipitate or worsen ACG. Therefore, mydriatic drops (eg, phenylephrine,
cyclopentolate) should not be given in patients with ACG.

Fluorescein dye can be instilled onto the surface of the eye to facilitate inspection of
the cornea (eg, foreign body, corneal abrasion, keratitis). Systemic administration of
fluorescein can be used to inspect the retinal circulation (ie, fluorescein
angiography). These techniques are not helpful in the diagnosis of ACG.

High-dose systemic glucocorticoids (eg, methylprednisolone) are indicated for the


treatment of anterior ischemic optic neuropathy (AION) in giant cell (temporal) arteritis
(GCA). They are also used in patients with optic neuritis (eg, due to MS). Although
these conditions can cause various combinations of headache, ocular pain, and vision
loss, this patient's conjunctival redness, mid-dilated pupil, and associated nausea are
more consistent with ACG.
Ophthalmic zoster (which is treated with valacyclovir) presents with eye pain,
conjunctivitis, and corneal ulcers. It is usually associated with a vesicular rash in the
distribution of the cutaneous branch of the first division of the trigeminal nerve [CN V].
Headache and retching are more typical of ACG.
A patient has developed impaired
vision while receiving glucocorticoid
eyedrops (ie, prednisolone), for
anterior uveitis, suggesting possible
steroid-induced open-angle
glaucoma (OAG). OAG is a type of
optic neuropathy associated with
increased intraocular pressure (IOP).
Topical glucocorticoid
eyedrops and systemic
glucocorticoids can raise IOP due to
decreased outflow of aqueous
humor via drainage channels in the
anterior chamber, possibly worsened
by the swelling of structures (eg,
choroid, lens) in the posterior
chamber.

OAG is characterized by insidious loss of peripheral vision related to atrophy of the


optic nerve head; signs include enlargement of the optic cup and increased cup/disc
ratio (ie, "cupping" of the optic disc), but this is a late finding and may not be
immediately apparent. However, patients with steroid-induced OAG may also develop
central blurriness early in the course due to corneal edema. Chronic use of
glucocorticoids can also cause subcapsular cataracts, although this is not yet seen in
this patient.

Elevated IOP in OAG can be documented with tonometry. Tonometry measures


deformation of the cornea in response to applied (mechanical or pneumatic)
pressure. Patients with suspected OAG should also receive assessment of peripheral
fields using automated test devices (manual visual field testing by opposition has
low sensitivity for OAG).
………

Fluorescein dye can be instilled onto the surface of the eye to facilitate inspection of
the cornea in patients with an ocular foreign body, corneal abrasion, or
keratitis. These conditions are typically associated with eye pain, irritation, tearing, or
redness.

MRI of the brain can detect signs of MS. However, optic neuritis presents with ocular
pain, acute vision loss, and an afferent pupillary defect.

Untreated steroid-induced OAG can lead to permanent vision loss and therefore needs
careful confirmation. Observation without testing is not appropriate.

HYPERTHYROIDIS
M

This patient has clinical features


of hyperthyroidism (ie, weight
loss, tachycardia) with proptosis
and impaired extraocular
motion (decreased
convergence, diplopia)
consistent with Graves ophthalmopathy. Other common symptoms include irritation
(eg, gritty or sandy sensation), redness, photophobia, pain, and
tearing. Ophthalmopathy in Graves disease is typically diagnosed at the same time as
hyperthyroidism but may occur before or after the onset of hyperthyroidism. Risk
factors include female sex, advancing age, and smoking.

In Graves disease, thyrotropin (TSH) receptor autoantibodies (TRAB) stimulate thyroid


hormone production, resulting in hyperthyroidism. Thyroid hormone increases
sensitivity to catecholamines, and thyrotoxicosis of any etiology may cause lid lag
and retraction due to sympathetic activation and contraction of the superior tarsal
muscle. However, true exophthalmos with impaired extraocular motion is seen only in
Graves disease and is due to T cell activation and stimulation of orbital fibroblasts and
adipocytes by TRAB, resulting in orbital tissue expansion and lymphocytic infiltration.
……..

Brain stem dysfunction can cause diplopia but not proptosis. Furthermore, because
many neural tracts are tightly packed in the brain stem, other neurologic findings (eg,
weakness, sensory deficits, altered sensorium) are usually present.

Myasthenia gravis (MG): Ocular involvement may cause fluctuating diplopia and
ptosis. Lambert-Eaton myasthenic syndrome (LEMS) is a paraneoplastic syndrome that
may also cause diplopia. However, ocular involvement by LEMS is less common than
involvement of the proximal limb muscles, and neither MG nor LEMS would cause
ocular irritation, painful movement, or proptosis.

////////////////////////////////NERVOUS SYSTEM/////////////////////////////////
Malignant
Malignant hyperthermia
hyperthermia
• Genetic mutation alters control of intracellular calcium
Epidemiology • Triggered by volatile anesthetics, succinylcholine, excessive heat

• Masseter muscle/generalized rigidity


• Sinus tachycardia
• Hypercarbia resistant to increased minute ventilation
Manifestations • Rhabdomyolysis
• Hyperkalemia
• Hyperthermia (late manifestation)

• Respiratory/ventilatory support
Treatment • Immediate cessation of causative anesthetic
• Dantrolene

This patient developed tachycardia, dyspnea, generalized muscle rigidity, and dark
urine soon after general anesthesia, raising strong suspicion for malignant
hyperthermia (MH). MH is an autosomal dominant or sporadic skeletal muscle
receptor disorder marked by excessive calcium release following exposure to
succinylcholine or a volatile anesthetic (eg, halothane).

In MH, sustained muscle contraction leads to:

• hypercarbia (due to increased levels of cellular metabolism) that does not


improve with increased minute ventilation (tachypnea)
• sinus tachycardia
• masseter/generalized muscle rigidity
• myoglobinuria, as seen in this patient with dark urine (due to muscle
breakdown)
• hyperthermia (late manifestation due to the sustained contractions generating
more energy than the body can dissipate; not usually present initially).
Most cases arise shortly after induction or during maintenance of anesthesia, but
symptoms can occur soon after anesthetic cessation. Urgent treatment
with dantrolene (a skeletal muscle relaxant) and supportive care are required to
prevent death.

DIC can be due to ABO-incompatible blood transfusion but is most common with
infection, malignancy, trauma, and obstetric complications. It generally presents with
bleeding symptoms such as oozing from catheters, drains, or incisions. Muscle rigidity
would be atypical.

Neuroleptic malignant syndrome (NMS) is often associated with rigidity,


myoglobinuria, tachypnea, and tachycardia. However, it is caused by administration of
a neuroleptic agent (eg, haloperidol, fluphenazine), not by surgery or surgical
anesthesia. In addition, most cases progress over a few days, not minutes.

PE is a common complication of surgery and is frequently associated with sinus


tachycardia and dyspnea. However, most cases occur >24 hours after surgery (not
immediately) due to immobility.
TRALI usually presents during or shortly after blood transfusion with acute dyspnea,
hypoxia, and lung infiltrates.

CNS TUMORS GBM: MC brain malignancy. The CT/MRI findings of a butterfly


appearance with central necrosis is classic for glioblastoma, and the heterogenous,
serpiginous contrast enhancement is typical of a high-grade (eg, grade IV) astrocytoma
such as a glioblastoma.

………………
Patients with brain abscess usually have fever, acute onset of symptoms, and evidence
of a systemic infectious process. On neuroimaging, there is often thick and diffuse
contrast enhancement resulting in a ring (an inflammatory capsule due to blood-brain
barrier breakdown).
Patients with brain metastases usually have a duration of symptoms of less than 2
months. The site of metastasis is the gray-white junction or watershed zones. Brain
metastases are usually multifocal and spherical in shape.

Patients with low-grade (rather than high-grade) astrocytoma usually present with
seizures, and have a longer duration of symptoms. Furthermore, contrast enhancement
is less likely. The diagnosis of meningioma is confirmed intraoperatively and the
treatment of choice in symptomatic patients is typically complete resection as this
leads to a cure in most individuals.

CTX is not considered first-line treatment for meningioma. Chemotherapy may be


coupled with surgical resection and radiation in patients with highly malignant primary
brain tumors (eg, glioblastoma multiforme, medulloblastoma). It can also be
considered in patients with highly chemosensitive metastatic brain disease (eg, due to
testicular germ cell tumor).

Whole brain radiation is not indicated for the treatment of meningioma; rather, it is
usually considered for the treatment of diffuse metastatic brain disease. Focused
tumor radiation (stereotactic radiosurgery) may be considered for partially resected or
unresectable meningiomas.

Meningioma: benign, middle age to elderly women.


Pituitary apoplexy: typically involves sudden hemorrhage into an enlarged pituitary
adenoma (eg, prolactinoma). Most cases occur without a clear precipitating cause,
although size appears to be a risk factor, possibly due to the expanding mass
outgrowing or compressing its blood supply.

Bleeding into the tight anatomic space of the sella turcica can cause sudden-onset,
severe headache as well as visual disturbances (eg, diplopia, bitemporal hemianopsia)
due to compression of the oculomotor nerves or optic chiasm. In addition, there is
typically loss of all pituitary function; in the acute setting the loss of ACTH is most
important and can lead to adrenal crisis (acute adrenal insufficiency).
Cortisol helps maintain vascular tone by potentiating the vasoconstrictive effects of
catecholamines (eg, norepinephrine). In the absence of ACTH, the adrenal
cortex ceases production of cortisol and severe hypotension and distributive
shock can develop. Hypoglycemia may also be present. Treatment involves rapid
volume resuscitation and prompt administration of IV glucocorticoids (eg,
hydrocortisone, dexamethasone). Neurosurgical intervention may also be necessary.
DA agonists (eg, cabergoline, bromocriptine) are used as initial treatment for
prolactinoma. These drugs are not known to precipitate pituitary apoplexy.

Neurogenic shock results from loss of sympathetic tone due to spinal cord
injury. Patients develop severe hypotension, but it is usually accompanied by
bradycardia rather than tachycardia. Neurogenic shock is extremely unlikely in a
patient with normal strength and sensation in all extremities.
TRAUMATIC A patient presents after a trauma with obtunded mental status and multiple
BRAIN INJURY ecchymoses on the chest and
(TBI) abdomen. All trauma patients should
/////////////////// be first assessed using the GCS,
which estimates the severity of the
***** patient's neurologic injury for
triage. The GCS can also give some
prognostic information when used
in conjunction with the patient's age
and presence of concomitant
adverse clinical findings, such as
hypoxia, cardiovascular
compromise, increased ICP, and
radiographic evidence of a midline
shift of the brain. alculation of GCS
score is shown below.

The GCS is used to predict the


prognosis of coma and other medical conditions, such as bacterial meningitis, TBI,
and SAH. However, the GCS is not used to diagnose coma in a patient. Findings used
to diagnose coma include impaired brainstem activity (e.g., disruption of the pupillary
light, extraocular, and corneal reflexes), motor dysfunction (e.g., decorticate or
decerebrate posturing), and impaired LOC.

…………

Exaggerated DTRs can be seen in locked-in syndrome, which mimics coma but is due
to an ischemic or hemorrhagic stroke of the brainstem area. Patients have total
paralysis of the limbs and an inability to speak, retain cognition and alertness, and
can only communicate with their eyes.

The gag reflex is tested to evaluate for proper cranial nerve function and swallowing
mechanism to prevent foreign objects from entering the pharynx, larynx, or trachea. Up
to 20% of the normal patients can have an absent gag reflex. This is not used as a
test in the GCS.

SPINAL CORD Neurogenic shock: Acute spinal cord injury is generally associated with an initial period
INJURIES (SCI) (several minutes) of massive sympathetic stimulation (leading to hypertension and
tachycardia) due to release of norepinephrine from the adrenal glands.

Quickly thereafter, sympathetic tone plummets due to injury to the descending spinal
tracts that carry signals from the brainstem to the preganglionic sympathetic neurons
in the lateral horn of the spinal cord (at levels T1-L2). This results in unopposed
parasympathetic stimulation (carried by the intact vagus nerve), leading
to hypotension and hypothermia from peripheral vasodilation and bradycardia.
Neurogenic shock often lasts 1-5 weeks before improving, but patients with spinal cord
injury are at increased risk for hypotension, bradycardia, and hypothermia due to long-
standing reduced sympathetic tone.

This patient with C6 paraplegia had severe hypertension, flushing, diaphoresis, and
bradycardia in the setting of urinary retention (due to an occluded urinary
catheter). This presentation suggests autonomic dysreflexia, a potentially life-
threatening complication of spinal cord injury (SCI) above T6. In an intact spinal cord,
sympathetic activity is modulated by higher-level neurons; however, SCI results in loss
of modulatory activity below the lesion. Noxious stimuli (eg, urinary retention,
constipation, pressure ulcers) can precipitate an unregulated sympathetic response,
leading to vasoconstriction and severe hypertension. Above the lesion, a
compensatory parasympathetic response causes diaphoresis,
flushing, bradycardia, and nasal congestion; vasodilation occurs but cannot overcome
the sympathetic drive to normalize systemic pressure.

Severe disease can result in intracranial hemorrhage or progressive bradycardia with


cardiac arrest. Management includes close monitoring of blood pressure and placing
the patient in an upright position to encourage orthostatic blood pressure
reduction. A search for precipitating events should ensue; tight-fitting clothes should
be removed, and the patient should be evaluated for urinary retention, fecal impaction,
or pressure sores. Short-duration antihypertensives (eg, nitrates, hydralazine) may be
indicated for blood pressure control.
……………

Carotid sinus hypersensitivity, which manifests as significant drops in heart rate (ie,
pauses) and blood pressure (>50 mm Hg) following carotid massage, typically presents
with lightheadedness or syncope after carotid manipulation in older patients with
atherosclerotic disease. .

Pheo: patients are typically tachycardic rather than bradycardic, and perspiration would
occur throughout the body, not just above the SCI.

Peripheral autonomic neuropathy is common in patients with diabetes; depending on


the organs involved, hyperhidrosis, hyperthermia, skin changes, gastroparesis, and
bladder dysfunction can occur. However, cardiovascular involvement typically causes
tachycardia and orthostatic hypotension; additionally, this patient does not have a
history of diabetes.

Spinal shock occurs immediately after spinal cord injury and is characterized by flaccid
paralysis with loss of reflexes; HoTN and bradycardia may also occur. It typically lasts
days to weeks, then progresses to the spastic paraparesis more common in SCI.
- Central cord syndrome
(CCS) typically occurs
with hyperextension injuries in elderly
patients with pre-existing degenerative
changes in the c-spine. CCS is
characterized by weakness that is more
pronounced in the upper
extremities than the lower. This unique
motor deficit occurs because the motor
fibers serving the arms are closer to the
central part of the corticospinal tract. A
patient with CCS may also have occasional
selective loss of pain and temperature
sensation in the arms due to damage to
the spinothalamic tract.
All trauma patients should be evaluated for cardiorespiratory stability and have the
spine immobilized until spinal injury has been ruled out. Airway protection and
mechanical ventilation are not needed if the patient is awake and has a normal
respiratory examination. Additional intravenous (IV) access, such as a femoral line, is
not required in hemodynamically stable patients if adequate peripheral IV access is
established.
In the absence of obvious pelvic injury and blood at the urethral meatus, such patients
should have a urinary catheter placed to assess for urinary retention and prevent
possible bladder injury from acute distension (which is likely causing the abdominal
discomfort in this patient). Imaging is then performed to diagnose and evaluate spinal
cord damage. Surgical intervention is indicated in patients with acute cord
compression with neurologic defects or unstable vertebral fracture/dislocation.

IV cefazolin is commonly used for antimicrobial prophylaxis before surgery to prevent


wound infections. It is usually given within 60 minutes of the procedure.

Nasogastric tube placement is indicated for bowel obstruction, enteral nutrition, and
gastric lavage but is not otherwise recommended for routine use. This patient is alert,
can swallow, and does not have an immediate need for nasogastric tube placement.
Syringomyelia: It is most commonly
seen with Arnold-Chiari type 1
malformations (extension of the
cerebellar tonsils into the foramen
magnum) but can also occur as a
sequela of meningitis, inflammatory
disorders, tumors,
and trauma. Symptoms may present
months to years after the initial insult.
It occurs MCly in cervical and thoracic
segments. Expansion into the motor fibers in
the ventral horns may result in progressive
weakness and flaccid paralysis within the
affected area. Over time, central pain,
incontinence, and lower extremity
manifestations may also develop.

The diagnosis is confirmed by MRI, which


shows an intramedullary cavity. Management
usually requires surgical intervention (eg, shunt
placement).

A patient who fell onto his head resulting in an inability to move his legs and hands
likely has cervical spine trauma. In addition, his tachypnea, use of accessory muscles,
and increasing somnolence are concerning for hypercapnic respiratory failure due
to diaphragmatic paralysis. After a high c-spine injury, patients are at risk for
IMMEDIATE diaphragmatic paralysis because the diaphragm is innervated by cervical
nerve roots from C3-C5. However, if the spinal cord is injured lower than C5, a
DELAYED diaphragmatic paralysis may occur due to ascending edema.
As in all trauma patients, primary survey includes assessment of the airway (A; in
addition to B & C). In patients with impending respiratory failure, the best next step
in management is orotracheal intubation, even with evidence of an unstable
cervical spine injury, because this procedure can be rapidly performed and is relatively
noncomplex. Typically, this is done with manual stabilization of the cervical spine to
minimize neck movement during intubation.
………………..

Nasotracheal intubation may be performed over a fiberoptic laryngoscope, or more


often, the endotracheal tube is blindly passed through the nose and then direct
laryngoscopy is performed to visualize the tube passing through the larynx. However,
this adds unnecessary complexity (and increased time) to this patient's management,
does not decrease neck extension, and does not result in a safer airway than
orotracheal intubation.

Needle cricothyroidotomy can be performed when other, less invasive techniques of


airway management have failed (or are likely to fail). It is used only temporarily until a
more secure airway can be placed because it offers no airway protection from
secretions, may lead to CO2 retention (and is particularly inappropriate in patients
with hypercapnic respiratory failure), and gives less feedback about respiratory
function (eg, ventilatory volumes, pressures, gas exchange).

Noninvasive positive pressure ventilation (NIPPV; CPAP) is used primarily to improve


oxygenation. This patient is likely hypercapnic due to diaphragm paralysis and requires
improved ventilation rather than simply oxygenation. Noninvasive positive pressure
ventilation also does not provide airway protection.

Orotracheal intubation carries less risk (eg, bleeding, tracheal stenosis) and can be
performed quicker than surgical tracheostomy; therefore, it is first-line management.
In addition, the neck is often extended during tracheostomy, so this procedure would
not necessarily prevent cervical spine movement.

A patient who fell headfirst down a flight of stairs and landed on his head may have
a cervical spine (CS) injury.

Patients at high risk of CS injury (eg, high-energy mechanism, trauma causing


concomitant closed head injury) automatically require CS imaging. Other patients,
such as this one, who have a less severe trauma mechanism but some risk for CS
injury are managed according to clinical decision rules such as the National
Emergency X-Radiography Utilization Study (NEXUS) low-risk criteria. According to
these criteria, CS imaging is necessary if any of the following is present:

• Neurologic deficit
• Spinal tenderness
• Altered mental status (AMS; e.g., somnolence)
• Intoxication
• Distracting injury

This patient is both somnolent and intoxicated; therefore, CS imaging is indicated. A CT


scan of the CS without contrast is the preferred screening test to evaluate for CS
injury, especially for fracture, because it is significantly more sensitive than plain
radiography (ie, ~98% vs ~52% sensitivity). If none of the NEXUS criteria are present,
then imaging is not required; a normal neurologic examination is sufficient to rule
out CS injury and remove the collar.
………….
Flexion and extension x-rays are lateral views taken during CS flexion and extension
to assess for ligamentous injury. However, these views are not performed until after
a CS fracture has been ruled out (eg, with a negative CT scan) and can be performed
safely only in alert, cooperative patients (vs this somnolent, intoxicated patient).

MRI is not commonly performed as the primary screening test for CS injury; instead,
it is typically performed when a CT scan suggests ligamentous or spinal cord injury,
in order to better visualize the soft tissues. CT scan is quicker, cheaper, and better
tolerated by patients than MRI.

This patient's CT scan of the cervical spine


demonstrates anterior subluxation of the C6
vertebral body relative to C7, which is
consistent with unilateral C6 facet
dislocation. Cervical facet dislocation typically
occurs with forced flexion of the cervical
spine (eg, falling onto a flexed neck). A single
facet is usually dislocated and results
in radiculopathy of the corresponding nerve
root. The following are the most commonly
affected vertebral bodies:

• C5/C6: Results in C6 radiculopathy


with weakness of wrist extension and
numbness of the forearm and thumb.
• C6/C7: Results in C7
radiculopathy with weakness of
triceps extension and wrist flexion, as well as numbness of the index and
middle finger.

Bilateral dislocations are less common and usually present with significant spinal
cord compression, often resulting in paralysis and UMN signs (eg, upgoing plantar
reflex). Management includes spinal precautions (eg, neck stabilization) and surgical
intervention.

………

Although a burst fracture could cause radiculopathy and may often follow significant
trauma, it typically occurs at L1. In addition, a burst vertebral body is visible on CT
scan.

Cervical strain would cause neck pain but would not likely cause the FNDs or anterior
dislocation of vertebral bodies seen on imaging.
Cervical stenosis can cause radiculopathy and symptoms may be preceded by trauma;
the anterior dislocation of C6 on CT scan is consistent with cervical facet dislocation,
not cervical stenosis, which would demonstrate narrowing of the spinal canal.
Syringomyelia (fluid-filled cavity in the spinal cord) can occur as a result of trauma but
typically presents months to years after the initial insult; symptoms typically include
a capelike loss of pain and temperature sensation, which may be associated with
flaccid paralysis.

The ASA is particularly dependent on blood supply from the radicular arteries that
originate from the thoracic aorta, such as the artery of Adamkiewicz. Thoracic aortic
surgery can result in reduced blood flow through the radicular arteries (eg, from aortic
cross-clamping and/or systemic hypotension) and consequently lead to anterior spinal
cord infarction.

Lumbar
plexopathy is a
peripheral
neuropathy
characterized by
asymmetrical focal
weakness,
numbness, and
paresthesias due
to involvement of
multiple adjacent
nerve roots.

SPINAL CORD
Spinal cord compression
INJURIES
• Spinal injury (eg, motor vehicle crash)
Causes • Malignancy (eg, lung, breast, prostate cancers; myeloma)
• Infection (eg, epidural abscess)

• Gradually worsening, severe local back pain


• Pain worse in the recumbent position/at night
Signs & • Early signs: Symmetric lower extremity weakness, hypoactive/absent deep-
symptoms tendon reflexes (spinal shock)
• Late signs: Bilateral Babinski reflex, decreased rectal sphincter tone,
paraparesis/paraplegia with increased DTRs, sensory loss

• Emergency MRI
Management • IV glucocorticoids
• Radiation-oncology & neurosurgery consultations

A patient with advanced prostate cancer presents with subacute back pain that has
now profoundly changed in severity. Examination shows lower extremity motor
weakness, hyperreflexia, and bladder dysfunction, raising concern for epidural
spinal cord compression (ESCC). The thoracic (60%) and lumbosacral (30%) spine
are the MC locations for ESCC. Cancers that often metastasize to the spine include
lung, breast, and prostate, as well as multiple myeloma.
Early diagnosis of ESCC is crucial in preserving neurologic function; in fact, the primary
determinant of outcome is the degree of neurologic involvement at the time of
intervention. Pain is typically the first symptom, often present for 1-2 months before
additional symptoms appear. Motor findings (bilateral weakness) and ataxia are
common as the disease progresses. Bowel and bladder dysfunction are late findings.

Although imaging (MRI) plays an important role in the diagnosis of ESCC, IV


glucocorticoids should be given without delay in a patient with suspected
ESCC. Glucocorticoids decrease vasogenic edema (caused by obstructed epidural
venous plexus), help alleviate pain, and may restore neurologic function. Once
imaging confirms ESCC, neurosurgical consultation is typically required.

………..
Analgesics and baclofen are used for muscular spasm. Alpha blockers may help relieve
bladder obstruction from an enlarged prostate. This patient is manifesting signs of
significant bilateral motor nerve involvement, making a muscular cause much less likely
than a critical neurologic cause.
Radiation to the spine is a useful intervention for painful spinal metastases. It may
play a role in this patient's treatment but requires imaging and
scheduling. Glucocorticoids should not be delayed.

A radionuclide bone scan and skeletal survey (x-rays of major bones) can detect
metastatic bone disease but do not provide useful information about compression of
the thecal sac; therefore, they are not recommended for diagnosis of ESCC. In general,
glucocorticoids should not be delayed for imaging in this patient as the pretest
probability of ESCC is quite high (given patient's neurologic findings, history of
prostate cancer, and symptoms).

CAUDA EQUINA Cauda equina syndrome (CES). Most cases arise when a large, central lumbar disc
SYNDROME (CES) herniation at the L4-S1 level bulges into the lumbar cistern and compresses multiple
spinal nerve roots in the cauda equina.

Injury to the cauda equina, which is composed of nerve roots for L2-L5, S1-S5, and the
coccygeal nerve, usually results in severe back pain radiating into one or both legs
and loss of lower extremity motor strength, sensation, and reflexes in the affected
dermatomes. Saddle anesthesiais highly suggestive of CES and reflects impingement
of S2-S4 nerve roots; bladder (eg, urinary straining) and rectal sphincter paralysis can
also occur due to damage to S3-S5 nerve roots. Loss of ankle reflex indicates damage
to S1-S2 nerve roots.
Straight leg raise testing (worsened sciatic pain when straight leg is raised to 30-70
degrees) is positive in >90% of those with acute lumbar herniation and may aid
diagnosis. Confirmation requires
urgent MRI of the lumbosacral
spine. Because patients with CES
(particularly in the setting of
bladder or rectal dysfunction) are
at high risk for permanent
neurologic sequelae, surgical
decompression is then usually
necessary.

Acute lumbosacral radiculopathy


is usually caused by disc herniation
and is marked by impingement of
≥1 lumbosacral nerve
roots. Although patients often
have low back pain that radiates
into one or both legs (and may
occasionally have dermatomal
motor weakness or loss of reflex),
other neurologic issues such as
saddle anesthesia and bowel/bladder dysfunction are generally absent (in contrast to
CES). Acute lumbosacral radiculopathy is typically managed with nonsteroidal anti-
inflammatory drugs, acetaminophen, and activity modification to lessen pain. Physical
therapy with graduated exercises may be recommended in some cases; prolonged
bed rest and inactivity is not recommended and may worsen outcomes.
SUBARACHNOID Subarachnoid hemorrhage (SAH) usually results from rupture of a saccular
HEMORRHAGE aneurysm and classically presents with a sudden-onset, severe headache, often
(SAH) called a thunderclap headache. The location of the headache can be highly variable,
but it is commonly accompanied by vomiting and photophobia, and some patients
may experience a brief loss of consciousness. Neck pain (or stiffness) and low-
grade fever are also common due to blood-induced meningeal irritation.

SAH is associated with high morbidity and mortality, and recognition is complicated by
a wide range of clinical presentations. Although nearly 20% of patients die before
reaching the hospital, approximately 40% present in a normal state of alertness
without any neurologic deficit. Noncontrast CT scan of the brain is the initial
diagnostic step of choice as it has high sensitivity for detecting blood pooling in the
basal cisterns or sulci of the subarachnoid space.
If CT scan is unremarkable and there is still reasonable suspicion for SAH, lumbar
puncture is performed to evaluate for red blood
cells or xanthochromia. Xanthochromia is a
pink or yellow tint of the CSF caused by
hemoglobin degradation products (eg,
bilirubin) that appear as soon as 2 hours
following a subarachnoid bleed and persist in
the CSF for weeks.

…………….
Cervical spine x-ray is appropriate to evaluate
for fracture following cervical spine trauma, but
it is not indicated for spontaneous onset of
severe headache.

ARTERIOVENOUS A patient with a right-sided headache developed nausea, vomiting, and a decreased
MALFORMATION level of consciousness, suggesting an intracerebral hemorrhage (ICH). The
(AVM) presentation of ICH varies with the location of bleeding; this patient's left-sided
hyperreflexia suggests a possible right hemispheric lobar hemorrhage. Symptoms tend
to progress over minutes to hours as the hemorrhage expands. Elevated intracranial
pressure can result in headache, nausea/vomiting, and altered mental status.

Although ICH can be caused by a variety of mechanisms, this patient's young age and
history of right-sided headaches suggests a ruptured arteriovenous
malformation (AVM). AVMs occur when an artery directly anastomoses with the veins
without an interposed capillary bed. The high-pressure system predisposes patients to
aneurysm formation and/or spontaneous bleeding. Most AVMs are congenital,
although de novo formation may occur. Patients are typically diagnosed before age
40, most commonly after developing an ICH. Other presenting symptoms may
include recurrent headache, seizure, or focal neurologic deficit (due to compression).

……………………..

Cerebral amyloid angiopathy (CAA) and hypertensive vasculopathy are both potential
causes of ICH. However, CAA typically presents in elderly patients and is associated
with dementia, whereas hypertensive vasculopathy would be unexpected in a patient
without a prolonged history of hypertension. Although uncontrolled hypertension can
cause headaches, recurrent unilateral headaches would be unexpected.

Dural venous sinus thrombosis can present with headache and signs of intracranial
hypertension. However, rapid progression to coma with focal neurologic deficits is
more consistent with ICH. In addition, venous sinus thrombosis typically occurs in the
setting of hypercoagulability (eg, pregnancy, malignancy), infection, or head injury.

SPINAL It is especially common in older adults taking antithrombotic medications, even


EPIDURAL when such drugs are appropriately discontinued periprocedurally. Additional risk
HEMATOMA factors include other coagulopathies (eg, thrombocytopenia), spinal structural
(SEH) abnormalities, and osteoporosis.

Bleeding within the spinal canal results in cord compression, leading to progressive
motor and sensory dysfunction in the distribution of the affected nerve root. Due to
the typical insertion site of spinal needles in the lower lumbar spine, cauda equina
syndrome with bowel and bladder dysfunction may occur. There is often localized
back pain and point tenderness, and some patients may also develop radicular
pain. The bleeding source is typically venous; therefore, symptoms are usually slowly
progressive (ie, days). Management includes an urgent MRI of the spine to evaluate
for blood within the canal. SEH is considered a surgical emergency and typically
requires urgent decompression (eg, laminectomy).
………………

Opiates can cause urinary retention

Nerve conduction studies (NCSs) can be used to evaluate for nerve injury that occurred
intraoperatively. Although common peroneal nerve injury causes foot drop and
numbness of the dorsal foot, it would not cause weakness with plantar flexion, urinary
retention, or bilateral sensory deficits.

Mild pain at the epidural site is common and does not require further evaluation.

Carpal tunnel
Pathophysiology of carpal tunnel syndrome (CTS)
syndrome (CTS)
Cause Pathophysiologic features

Idiopathic/overuse • Swelling & fibrosis of tendons & soft tissue

Hypothyroidism • Soft tissue enlargement (mucopolysaccharides)

• Soft tissue enlargement


DM • Microvascular insufficiency & neovascularization

RA • Extrinsic compression from joint deformity

Pregnancy • Edema/fluid accumulation

• Amyloid & calcium phosphate deposition


End-stage renal
• Access related (bleeding, venous hypertension during HD,
disease (ESRD) vascular steal)

• Tendon enlargement
Acromegaly • Synovial edema

Gout • Compression from tophi

HD = hemodialysis.
A patient has myalgias, constipation, impaired concentration, dry skin, mild edema, and
hyporeflexia, suggesting hypothyroidism.

In addition to slowed metabolic activity, many symptoms of hypothyroidism


(eg, myxedema, dry skin) are due to deposition of mucinous material composed of
glycosaminoglycans, hyaluronans, and mucopolysaccharides within tissue. Direct
infiltration of the median nerve and nerve sheath with mucinous material results in
swelling and a localized ischemic neuropathy. Deposition on the tendons within the
carpal tunnel results in swelling and compression on the nerve, leading to symptoms of
CTS.
……………….

Patients with hypothyroidism are more likely to develop bilateral CTS, and their
symptoms may be more severe. The risk does not appear to correlate with the severity
of the underlying thyroid disease, although it is greater in patients with elevated
BMI. Treatment of hypothyroidism with levothyroxine often leads to improvement of
CTS symptoms.
Patients with ESRD on hemodialysis frequently develop CTS due to a variety or reasons,
MCly due to the accumulation of dialysis-related amyloid (inflammation stimulate the
formation of beta-2-microglobulin). Other contributing factors include increased
venous pressure during hemodialysis, blood tracking (through fascial planes) from
the fistula into the carpal tunnel during vascular access, and deposition of calcium
phosphate in the tunnel, all of which worsen extrinsic compression of the nerve. In
addition, vascular steal through the fistula may lead to ischemic neuropathy. When CTS
occurs, it is more common in the arm used for vascular access (eg, access graft,
arteriovenous fistula), suggesting a contribution of vascular steal or venous
hypertension. Pts may also have referred pain throughout the hand).

The diagnosis is usually made clinically (H&P, Phalen and tinel tests) but can be
confirmed with electrodiagnostic studies.
………..
Subclavian steal: s/s involve the entire arm.

Uremic polyneuropathy is common in patients with ESRD but causes progressive pain
and paresthesia in the feet, not the hands. Because it is due to uremia, the
polyneuropathy typically resolves when dialysis is initiated.
HEMATOMAS EPIDURAL HEMATOMA (EDH):

It MCly occurs following traumatic injury to the sphenoid bone, resulting in laceration
of the middle meningeal artery. Failure to treat EDH emergently may result in
worsening intracranial hypertension and uncal herniation. On examination, patients
with uncal herniation have dilation of the pupil on the ipsilateral side of the lesion (due
to oculomotor nerve compression) along with ipsilateral hemiparesis (due to
contralateral crus cerebri compression). Emergent craniotomy should be performed in
patients with focal neurologic deficits to prevent brain herniation and death.

……………….

Cerebral contusion is a bruise of brain tissue resulting from coup/contre-coup


injuries. These are less likely to cause uncal herniation with focal neurologic deficits
and typically heal without medical intervention.

← Diffuse axonal injury (DAI) is the result of


traumatic acceleration/deceleration shearing
forces that diffusely damage axons in the
brain. Patients typically have coma, and head
CT scan may show diffuse small bleeds at the
grey-white matter junction.

Concussion is a form of mild traumatic brain


injury typically associated with confusion and
amnesia. It may involve loss of
consciousness; however, elevated intracranial
pressure and focal neurologic deficits are not
typical and neuroimaging is usually normal.

SDH: Symptoms of headache and confusion


occur gradually (over 1-2 days) compared to
those of EDH.
Cerebral angiography is a useful diagnostic tool for identifying cerebral aneurysms and
AVMs.

MRI of the brain without contrast can be used as an adjunct to CT scan in the
evaluation of an epidural hematoma if there is strong clinical suspicion of epidural
hematoma but no clear evidence on CT scan.

Close observation and serial (every 6-8 hours) CT scans may be appropriate in patients
with a small epidural hematoma and minimal neurologic signs.

EDH: often occur with fracture in the pterion region (the junction of the frontal,
parietal, temporal, and sphenoid bones) and most commonly affect young adults.

Although patients classically present with loss of consciousness followed by a lucid


interval, only 20%-50% of patients develop this characteristic presentation and some
initially remain alert. However, rapid hematoma expansion soon results in neurologic
decompensation with signs ofelevated intracranial pressure (eg, headache,
nausea/vomiting, altered mental status); symptoms typically develop within minutes
to hours.

………..
Diffuse axonal injury (DAI) can occur from blunt force injury but typically results in
immediate coma. In addition, DAI is usually diagnosed with MRI; CT scan has low
sensitivity and is often normal.

SDH: This typically occurs in older adults and in those on antithrombotic agents. In
addition, many patients develop coma from the time of onset in acute SDH, whereas
chronic SDH results in the insidious onset of confusion, headache, and somnolence
over a period of days to weeks.

Cerebellar hemorrhage

• Hypertension
Risk factors • Antithrombotic therapy (eg, warfarin, aspirin)
• Cerebral amyloid angiopathy

• Headache, nausea, vomiting


Manifestations • Ipsilateral ataxia, dysarthria, vertigo, nystagmus
• Cranial neuropathies

• Reversal of anticoagulation
• Blood pressure management
• ICP management (eg, head of bed elevation, mannitol)
Management • Surgical decompression indicated with:
o Hemorrhage >3 cm
o Neurologic deterioration (eg, impaired consciousness)
o Brainstem compression, obstructive hydrocephalus

ICP = intracranial pressure.


Cerebellar hemorrhage accounts for approximately 10% of cases of ICH. Risk factors
for ICH include hypertension, antithrombotic therapy (eg, warfarin, aspirin), and
cerebral amyloid angiopathy. Symptoms of ICH are progressive and worsen over
minutes to hours. Headache, nausea, and vomiting are common, and meningismus
can occur if the bleed extends into the intraventricular space. Further manifestations
depend on the location of the bleed; cerebellarhemorrhage can result in vertigo,
dysarthria, nystagmus, and ipsilateral truncal (cerebellar vermis) or limb (cerebellar
hemispheres) ataxia. Unlike other forms of stroke, hemiparesis and sensory loss are
rare.

Management includes stabilization (eg, airway, breathing, cardiovascular


system), reversal of anticoagulation, blood pressure management, and elevation of the
head of the bed to decrease ICP. However, extension of cerebellar hemorrhage can
result in obstructive hydrocephalus (due to compression of the fourth ventricle), with
elevated intracranial pressure (ICP) or brainstem compression leading to stupor, coma,
and death. Therefore, urgent surgical decompression and clot evacuation are
indicated in cerebellar hemorrhage with:

• signs of neurologic deterioration (eg, progressive lethargy, obtundation,


coma).
• radiologic evidence of a bleed >3 cm, brainstem compression, or obstructive
hydrocephalus.

…………….

Anticoagulation is indicated in patients with an uncomplicated ischemic stroke due to


embolic disease; however, it is initiated 24-48 hours after the infarction. This patient
has evidence of hemorrhage stroke on CT scan, and anticoagulation would likely
worsen the bleed.
Glucocorticoids are used to reduce inflammation and peritumor edema due to
intracranial malignancy, but tumors are usually less hyperdense and demonstrate
peritumor edema with central necrosis. Glucocorticoids are not indicated in ischemic
or hemorrhagic stroke.

Osmotic diuretics (eg, mannitol) are occasionally used to acutely reduce ICP due to
hemorrhage or life-threatening mass effect; however, loop diuretics have no role in the
management of either.

Cerebrospinal fluid removal via serial lumbar puncture or an external ventricular drain is
sometimes needed to reduce ICP due to obstructive hydrocephalus, but this is
considered second-line therapy in cerebellar hemorrhage and should be pursued only
if decompressive surgery is not possible.
SHH: Pt w/ cerebral atrophy are at highest risk. Symptoms of acute SDH usually
develop gradually 1-2 days after the initial injury and often include impaired
consciousness, confusion, or symptoms of intracranial hypertension (eg, headache,
nausea/vomiting). Small hematomas are managed conservatively whereas larger ones
causing neurologic deficits require urgent surgical evacuation.
……….
← Cardioembolic strokes

………
← PCA stroke

A tumor originating from the arachnoid cap cells is


called a meningioma. These extra-axial, well-
circumscribed, dural-based masses can be partially
calcified on neuroimaging. .

CERVICAL The MCC of cervical myelopathy in older adults is spondylosis. Spondylosis of the
MYELOPATHY cervical spine is often diagnosed radiographically, and myelopathy is confirmed with
MRI or CT myelogram. Patients may require surgical decompression.
Cervical myelopathy

• Age >55
Epidemiology • Degenerative cervical spine/discs → canal stenosis → cord
compression

• Gait dysfunction - usually first


• Extremity weakness & numbness
Manifestations • LMN signs (arms) - muscle atrophy, hyporeflexia
• UMN signs (legs) - Babinski, hyperreflexia
• ↓ Proprioception/vibration/pain sensation

• MRI of cervical spine


Diagnosis • CT myelogram

• Nonsurgical - immobilization
Treatment • Surgical decompression

LMN = lower motor neuron; UMN = upper motor neuron.

………………

An epidural fluid collection can compress the spinal cord but is usually associated with
fever. Muscle atrophy, a sign of a chronic process, would be unusual.

A parasagittal meningioma can compress the motor strip in the brain and cause lower
extremity weakness with UMN and not LMN findings.
Central cord syndrome, an incomplete acute SCI. Most cases arise when an older
individual with a stenotic cervical spinal canal (eg, due to cervical
spondylosis) experiences a hyperextension injury to the neck (eg, whiplash due
to rear-end collision); this compresses the spinal cord between a hypertrophied
ligamentum flavum posteriorly and a bulging disc/osteophyte complex anteriorly,
leading to damage to the central spinal cord (grey matter).

Patients primarily develop upper extremity manifestations, including:

• Weakness due to damage to the alpha motor neuron cell bodies in the anterior
horn of the spinal cord
• Pain, temperature, and sensory loss in dermatomes at and surrounding the
level of injury due to damage to the posterior grey column
• Reflex loss (eg, tricep reflex) at the level of injury due to damage to fibers as
they cross from the dorsal to ventral horn

Due to the more central location of the lesion, the lateral spinal tracts running to the
sacrum (eg, bowel, bladder) and lower limbs are generally spared.
Workup begins with spinal imaging; x-ray is often normal but may show cervical
spondylosis. Cervical myelogram is generally diagnostic and often demonstrates
persistent cord compression; treatment with glucocorticoids and/or surgery is
usually required.
Acute stress disorder is common after severe motor vehicle collisions and is generally
characterized by agitation, anxiety, and fear. Numbness, weakness, and loss of reflexes
in the upper extremities would be atypical.

Brachial plexus injury (nerve roots C5-T1) is generally characterized by shoulder or arm
pain, upper extremity muscle weakness, atrophy (after weeks), and sensory loss;
however, most cases are unilateral. In addition, traumatic injury to the brachial plexus
usually occurs when there is downward traction on the shoulder and the neck is forced
to the contralateral side (not during hyperextension injury).
Malingering is when a patient simulates an illness to obtain obvious external benefit
(eg, money, medications, work benefits).

Postconcussion syndrome is generally marked by headache, dizziness, cognitive


impairment (eg, loss of concentration/memory), irritability, anxiety, and noise
sensitivity. Although it can occur following whiplash injury, upper extremity weakness,
sensory loss, and reflex abnormalities would be atypical.

Clinical features of RA cervical myelopathy

• Neck pain radiating to occipital region


Symptoms • Slowly progressive spastic quadriparesis
• Painless sensory deficits in hands or feet
• Respiratory dysfunction (eg, from vertebral artery compression)

• Protruding anterior arch of atlas


• Scoliosis with loss of cervical lordosis
Signs • UMN signs (eg, spastic paresis, hyperreflexia, Babinski sign)
• Hoffman sign

RA = rheumatoid arthritis.
A patient developed weakness, paresthesias, and hyperreflexia after intubation; in the
setting of her known rheumatoid arthritis (RA), this presentation suggests spinal cord
compression due to atlantoaxial joint instability. Although joint destruction in RA
most commonly affects the small joints of the hands, the cervical spine can also be
involved and may lead to subluxation (misalignment) of the atlantoaxial
joints. Patients with severe RA (eg, elevated inflammatory markers, rapidly progressive
erosive disease) and peripheral joint subluxation (eg, hand deformities) are at
increased risk. Forced neck extension during intubation can worsen the subluxation,
leading to cord compression and progressive cervical myelopathy; complete
dislocation can result in paralysis and death.

The earliest symptom of atlantoaxial subluxation is typically cervical pain that


radiates to the occiput. Cervical myelopathy classically presents with slowly
progressive, spastic paraparesis characterized by weakness involving the upper and
lower extremities, hyperreflexia, and sensory changes. UMN signs, including Babinski
sign, are present. Hoffman sign (flexion and adduction of the thumb when flicking the
nail of the middle finger) suggests a corticospinal tract lesion and supports the
diagnosis of cervical myelopathy; however, it is nonspecific and may be seen in normal
patients.

Patients with evidence of subluxation should undergo urgent MRI of the cervical spine
which demonstrates separation of C1 and C2. Management involves stiff surgical
collars and neurosurgical intervention (eg, cervical fixation).

………………
Anterior cerebral artery stroke can result in extremity weakness with spastic
paraparesis. However, stroke typically occurs acutely with maximum weakness upon
presentation and the findings are unilateral; dysarthria and aphasia are also common
(MCA?)

Critical illness polyneuropathy is a complication of sepsis characterized by axonal


injury of the peripheral nerves. It is a common cause of weakness after a prolonged
stay in an ICU. However, peripheral nerve injury results in hyporeflexia; UMN signs
would not occur.
GBS: Reflexes are typically decreased or absent, and Hoffman sign would be
unexpected.

Severe hypothyroidism is occasionally associated with polyneuropathy or myopathy,


which can cause weakness and paresthesia. However, the reflexes are diminishe.
NORMAL
Normal pressure hydrocephalus (NPH)
PRESSURE
HYDROCEPHALU • Gait instability (wide-based) with frequent falls
S (NPH) • Cognitive dysfunction
Clinical
• Urinary urgency/incontinence
features • Depressed affect (frontal lobe compression)
• UMN signs in lower extremities

• Marked improvement in gait with CSF removal: Miller Fisher (lumbar tap)
test
Diagnosis • Enlarged ventricles out of proportion to the underlying brain atrophy on
MRI

Treatment • VP shunting

NPH: Most cases are idiopathic and occur in the elderly; however, secondary disease
can occur in those with prior neurologic insults (eg, SAH, trauma, meningitis) that
result in scarring and destruction of the arachnoid granulations responsible for CSF
resorption.

All symptoms may not be present in early disease. A magnetic gait with slow, wide-
based steps that appear as though the patient's feet are stuck to the ground is
characteristic and typically appears early in the disease process. The diagnosis is
confirmed with improvement of symptoms with large-volume lumbar puncture; VP or
VA shunt placement is the definitive therapy.
All 3 features are not required for diagnosis; in fact, only gait dysfunction, typically
the most prominent symptom, is mandatory. In addition, upper-motor neuron signs,
such as lower-extremity spasticity, hyperreflexia, and extensor plantar responses, may
occur. Improvement of gait function after the CSF removal is diagnostic. Alternatively,
a temporary lumbar drain may be placed to assess for gait improvement over a period
of days.

…….

Aqueductal stenosis can also occur after SAH and causes ventricular enlargement;
however, symptoms of elevated intracranial pressure (eg, headache, nausea/vomiting)
predominate.

Cerebellar degeneration (eg, alcoholic cerebellar degeneration) can cause a wide-


based, shuffling gait, but it is also commonly associated with dysarthria and postural
instability. Neuroimaging may show cerebellar atrophy, but ventriculomegaly is
unexpected.
Vasospasm occurs within 8 days of the primary bleed (SAH). Symptoms develop
acutely, correspond to the location of the stroke.

Secondary parkinsonism can rarely result from cerebrovascular disease that interferes
with the striatonigral pathway → shuffling or festinating (quick, short
steps). Rigidity, bradykinesia, and pill-rolling tremors are also expected, but ventricular
enlargement is not common.

PERIPHERAL The glenohumeral joint is the MCly dislocated joint in the body due to the shallow
NERVE INJURY articulation between the humeral head and the glenoid fossa of the scapula. The
shoulder may
dislocate anteriorly
(MC), inferiorly, or
posteriorly. Anterior
dislocations are
typically caused by a
blow to an externally
rotated and
Abducted arm.
When the head of the
humerus is displaced
anteriorly, there
is flattening of the deltoid prominence, protrusion of the acromion, and anterior
axillary fullness (due to the humeral head's displacement into this location); the arm is
abducted and externally rotated. The axillary nerve is the n is the MCly injured by
anterior shoulder dislocations. It innervates the teres minor and deltoid (weakened
shoulder abduction) muscles. It also provides sensory innervation to the skin overlying
the lateral shoulder.

The biceps reflex is mediated by C5 and C6 spinal nerves, with muscular innervation via
the musculocutaneous nerve (lateral cord of the brachial plexus). Traumatic injuries to
this system are uncommon but can occasionally be seen in high-velocity motor vehicle
collisions.

The ulnar nerve may be injured by fracture of the medial epicondyle of the humerus or,
more distally, by deep lacerations of the anterior wrist. Symptoms include "claw hand"
resulting from paralysis of the intrinsic muscles of the hand as well as sensory loss
involving the medial hand.

The long thoracic nerve innervates the serratus anterior muscle.


Femoral N (FN) injury: The femoral nerve innervates the
muscles of the anterior compartment of the thigh (ie,
quadriceps femoris, sartorius, pectineus) and is responsible
for knee extension and hip flexion. It also
provides sensation to the anterior thigh (via the anterior
cutaneous branches of the femoral nerve) and medial leg (via
the saphenous branch).
FN is vulnerable to injury from pelvic fracture, hip dislocation,
or hematoma (eg, iliacus). It can also suffer iatrogenic injury
during prolonged maintenance of the dorsal lithotomy
position (eg, hip/pelvic surgery, childbirth) or vascular
procedures involving the femoral artery or vein.

The obturator nerve innervates the medial compartment of


the thigh (ie, gracilis, adductor longus, adductor
brevis, anterior portion of adductor magnus). Injury would lead to weakness of hip
adduction and decreased sensation at the medial thigh.

The ilioinguinal nerve provides


sensory innervation to the upper
medial thigh and genital region.

Injury to the common peroneal nerve


is typically caused by fracture or
compression at the proximal
fibula. Sensory symptoms occur at
the posterolateral leg and
dorsolateral foot and are often
associated with footdrop due to
weakness of foot eversion,
dorsiflexion, and toe extension.

The tibial nerve is the larger of the


two terminal branches of the sciatic
nerve. Injury at the popliteal fossa
may result in weakness of plantar
flexion (due to denervation of the
gastrocnemius and soleus
muscles). Compression of the nerve
as it passes under the flexor
retinaculum in the medial side of the
ankle can cause numbness and
paresthesias in the sole and distal
toes (tarsal tunnel syndrome).
NEUROFIBROMA
Neurofibromatosis type II
TOSIS (NF)
• AD
Genetics • NF2 gene on chromosome 22 encoding merlin (TSG)
• Variable expressivity, usually at age 20-30

• Bilateral vestibular schwannomas


• Intracranial meningiomas
Clinical
• Spinal tumors (eg, schwannomas, ependymomas)
manifestations • Cataracts
• Cutaneous tumors or skin plaques

• Audiogram
Tumor
• Ophthalmologic evaluation
surveillance • MRI of the brain & spine
Vestibular schwannoma primarily affects the cochlear and vestibular nerves. Cochlear
nerve involvement results in sensorineural hearing loss (~95% of patients). Vestibular
nerve involvement can cause imbalance, particularly when patients are deprived of
visual input (eg, eyes closed when standing on 1 foot, yoga poses). Most patients do
not experience vertigo because the tumor's slow growth allows for central
compensation of gradual loss of input.

Sporadic vestibular schwannomas are typically unilateral and manifest around age
50. In contrast, the presence of bilateral, hereditary vestibular schwannomas in a
young patient, as seen in this case, should raise suspicion for NF2. NF2 is caused by
an autosomal dominantdefect in merlin (a cell-membrane tumor-suppressor protein)
and usually presents at age 20-30. Vestibular schwannomas are the most common
tumor seen; however, patients also may have intracranial meningiomas, spinal tumors,
or peripheral neuropathies. Cataracts and cutaneous tumors also can be seen.

Jervell and Lange-Nielsen syndrome, a congenital LQTS, also causes inheritable


sensorineural hearing loss due to the associated mutations affecting endolymph
production. In contrast to NF2, congenital long QT syndrome typically presents in
childhood with profound bilateral deafness and episodes of arrythmia-induced syncope
during periods of stress.
Mitochondrial disorders are maternally inherited and can present with diverse
symptoms that may include hearing loss or imbalance. However, they typically involve
multiple organ systems and myopathy is often present.

MS can present in young women and may have a hereditary component.

TRACE MINERAL
****** Clinical manifestations of trace mineral deficiencies
DEFICIENCY
/////////////////// Chromium • Impaired glucose control in diabetes

****** • Brittle HAIR


• Skin DEPIGMENTATION
Copper • NEUROLOGIC dysfunction (eg, ataxia, peripheral neuropathy)
• Anemia (microcytic?)
• OSTEOPOROSIS

Iron • Microcytic anemia

• Thyroid dysfunction
Selenium • Cardiomyopathy
• Immune dysfunction

• Alopecia
• Pustular skin rash (perioral region & extremities)
• Hypogonadism
Zinc • Impaired wound healing
• Impaired taste
• Immune dysfunction
A woman with distal extremity paresthesia, loss of proprioception and vibration sense,
and ataxia has peripheral neuropathy. In association with anemia, skin depigmentation,
and edema, this presentation suggests copper deficiency. Acquired copper deficiency
is most often due to malabsorption from prior gastric surgery (eg, bariatric
surgery). Other risk factors include chronic malabsorptive syndromes (eg, IBD, celiac
disease) and excessive ingestion of zinc, which competes with copper for absorption
in the GIT.

Copper is a required cofactor in many enzymes, including those involved with protein
synthesis, cellular division, iron absorption, and normal CNS function. Neurologic
manifestations are often the presenting symptoms and begin several years after
surgery with a slowly progressive myeloneuropathy similar to SCD from B12 deficiency
(eg, paresthesia, numbness, sensory ataxia). Myelopathic symptoms (eg, spasticity,
UMN signs) occur in advanced disease.

Hematologic abnormalities are also characteristic. Hypochromic microcytic


anemia occurs from copper deficiency–induced impairment of iron absorption,
although macrocytic or normocytic anemia is also common. Leukopenia may
occur. Other symptoms include hair fragility, skin depigmentation, HSM, edema,
and osteoporosis. The diagnosis is confirmed with low serum copper and
ceruloplasmin levels; management includes copper supplementation and
discontinuation of zinc.

………….

Thiamine deficiency can occur after bariatric surgery and results in Wernicke
encephalopathy. Although ataxia is common, it typically also causes abrupt onset
oculomotor dysfunction and confusion. Korsakoff syndrome is a late manifestation
characterized by severe anterograde and retrograde amnesia. .

Selenium deficiency causes cardiomyopathy and macrocytosis, as well as immune and


thyroid dysfunction. It does not typically cause neuropathy or microcytic anemia.
- Zinc is a trace mineral that is obtained
from meat, nuts, and fortified cereal in
the diet.
- Characteristic physical examination
findings include alopecia as well as a skin
rash consisting of erythematous pustules
around body orifices (eg, mouth) and on
the extremities.
- Zinc is absorbed mostly in the
duodenum and jejunum, and deficiency
can be due to malabsorption (eg, Crohn
disease, celiac disease), bowel resection,
gastric bypass, or poor nutritional intake.
In addition, patients dependent on
parenteral nutrition (PN) are at risk for
trace mineral deficiency (eg, zinc,
selenium, copper, chromium) due to
inadequate supplementation of
parenteral intake. Comorbid diarrhea and malabsorption further increase the risk
of trace mineral deficiency in patients on PN due to excessive gastrointestinal
losses.
- Clinical manifestations of copper deficiency include fragile hair, skin
depigmentation, neurologic dysfunction that can mimic vitamin B12 deficiency, and
sideroblastic anemia.
Zinc is a trace mineral that is obtained from meat, nuts, and fortified cereal in the
diet. It is an essential component of numerous enzymes in the body and plays an
important role in gene transcription and cell division. Clinical manifestationsof zinc
deficiency in adults include hypogonadism, impaired wound healing, impaired taste,
and immune dysfunction. Characteristic physical examination findings
include alopecia as well as a skin rash consisting of erythematous pustules around
body orifices (eg, mouth) and on the extremities.

Zinc is absorbed mostly in the duodenum and jejunum, and deficiency can be due
to malabsorption (eg, Crohn disease [CD], celiac disease), bowel resection, gastric
bypass, or poor nutritional intake. In addition, patients dependent on parenteral
nutrition (TPN) are at risk for trace mineral deficiency (eg, zinc, selenium, copper,
chromium) due to inadequate supplementation of parenteral intake. Comorbid
diarrhea and malabsorption further increase the risk of trace mineral deficiency in
patients on PN due to excessive gastrointestinal losses.

…………

Clinical manifestations of copper deficiency include fragile hair, skin


depigmentation, neurologic dysfunction that can mimic vitamin B12 deficiency, and
SIDEROBLASTIC anemia. Impaired taste and a pustular skin rash would be unusual.
Gluten avoidance is used to treat celiac disease, which can lead to malabsorption and
deficiency of zinc and other vitamins and minerals. Celiac disease is associated
with dermatitis herpetiformis, an autoimmune-mediated vesicular skin rash that
typically appears on the extremities. Perioral involvement would be unusual. In
addition, celiac disease is less likely in this patient with alternate explanations for zinc
deficiency (eg, Crohn disease, recent PN).

Niacin (vitamin B3) deficiency (rare in developed countries) causes pellagra, which is
characterized by dermatitis, diarrhea, and dementia. The rash associated with pellagra
is hyperpigmented, occurs symmetrically in sun-exposed areas, and is not pustular.

Selenium deficiency can lead to thyroid dysfunction as well as cardiomyopathy.


Absence of heart failure signs (eg, JVD, lower extremity edema) makes selenium
deficiency less likely.
WOUND Most cases occur when Clostridium botulinum spores contaminate a puncture
BOTULISM

wound (eg, injection drug use), germinate in the anaerobic environment, and
generate botulinum toxin in vivo. Botulism can also occur when spores (infant
botulism) or preformed botulinum toxin (foodborne botulism) are ingested.
Exposure to botulinum toxin, which impairs the presynaptic release of acetylcholine at
the neuromuscular junction, generally results in:
• Symmetric, descending motor paresis beginning with the CNs; this often
includes CN III, CN IV, and CN VI (diplopia, ptosis, mydriasis) and CN IX and CN
X (dysphagia, suppressed gag)
• Progressive respiratory compromise due to diaphragmatic paralysis
• Autonomic dysfunction (eg, ileus, orthostatic hypotension, urinary retention)

Because botulinum toxin targets the motor synapse, sensory abnormalities and
confusion are rare (although lethargy from respiratory failure can occur). In contrast
to infant or foodborne botulism, wound botulism is often associated with fever and
leukocytosis and generally presents ~10 days (rather than within hours) after
transmission.

Confirmation requires the isolation of C botulinum in culture or identification of


toxin in serum. However, urgent treatment with equine botulinum antitoxin is
required to prevent progression and should not be delayed for testing in suspected
cases. Respiratory support and wound debridement are also often required.

Intravenous buprenorphine can be used to treat symptoms of opioid withdrawal,


which is usually characterized by flulike symptoms (eg, myalgia, rhinorrhea, diarrhea);
cranial nerve palsies and respiratory failure would be atypical.

////////////////////////////////OPTHALMOLOGY//////////////////////////////////
ENDOPHTHALMI Postoperative endophthalmitis is the MC form of endophthalmitis. It usually occurs
TIS within 6 weeks of surgery. Endophthalmitis refers to a bacterial or fungal infection
within the eye, particularly the vitreous. Patients usually present with pain and
decreased visual acuity. Examination reveals swollen eyelids and conjunctiva,
hypopyon, corneal edema and infection. The vitreous can be sent for Gram stain and
culture. Based on the severity, intravitreal antibiotic injection or vitrectomy is done.

……………………..

Conjunctivitis presents with excessive tearing, burning sensation, mild pain,


conjunctival, and eyelid edema. Vision is not affected.

Corneal ulceration presents as a foreign body sensation, blurred vision,


photophobia, and pain. A history of contact lens use, recent trauma, or ocular
disease may be present. The eye is erythematous, and ciliary injection is
present. Purulent exudates are seen in the conjunctival sac and on the ulcer surface.

Uveitis (ie, viral or parasitic infection within the eye) presents as blurred vision with
moderate pain, conjunctival injection, and constricted pupils. Hypopyon (cell
layering within the anterior chamber) is seen in severe anterior uveitis. Keratic
precipitates ("mutton fat") and iris nodules may be seen. It is associated with HLA
B27-related conditions.
Cavernous sinus thrombosis is characterized by proptosis, ophthalmoplegia, chemosis,
and visual loss. It occurs due to hematogenous spread from an infected, inflamed
sinus.

CHEMICAL BURN Typical symptoms include eye pain, blepharospasm (eg, inability to open the eyelid), a
gritty sensation, and eye erythema; severe cases may lead to ischemia of the
conjunctiva, and the eye may appear white. As in other parts of the body, chemical
burns are categorized as follows:

• Acid burns: cause coagulation necrosis (eg, protein denaturation) that results in
an eschar, thereby preventing further acid penetration and injury
• Alkali burns (eg, bleach): cause liquefactive necrosis (eg, cell membrane
dissolution), which leads to deeper penetration of tissues and therefore more
severe injuries

In either case, the


most important (and
urgent) step in
management is
to restore a neutral
pH by flushing the
eye copiously with
water or saline.
Normalization of
pH typically takes 30-60 minutes but may take greater than >2 hours for alkali
burns. In ideal circumstances, a provider continuously flushes the eye, but eye-wash
devices (eg, Morgan lens) can be used. Topical anesthesia can be administered
to improve patient comfort and tolerance of irrigation. After cessation of irrigation, the
eye's pH should be remeasured with litmus paper to confirm that it remains neutral.

……………………

Cycloplegics (parasympatholytic agents that prevent light-induced miosis) may be


prescribed for long-term management of pain caused by photophobia resulting from
large corneal abrasions. They are not indicated for acute ocular burns.

Topical antibiotics may be used in the management of ocular chemical burns but not
before normalization of the eye's pH is achieved through copious irrigation.

Eye patch application was once a commonly used treatment for corneal abrasions
because it was thought to promote epithelial proliferation and encourage healing. It is
used much less frequently now because it does not seem to provide much
benefit. Patching is not appropriate for the treatment of chemical burns.

Acetic acid is sometimes used for alkali burns on skin, but evidence regarding its
efficacy is limited.
OPEN EYE GLOBE
Open globe injury
INJURY
• Blunt force trauma (rupture)
Risk factors • Penetrating trauma (laceration)

• Extrusion of vitreous (eg, "gush" of fluid)


• Eccentric or teardrop pupil
Clinical features • Decreased visual acuity
• Relative afferent pupillary defect
• Decreased intraocular pressure

• Emergency ophthalmology referral


• Eye shield
Management • CT scan of eye
• Intravenous antibiotics
• Tetanus prophylaxis

Open globe laceration (OGL) with an intraocular foreign body. Small, high-velocity
particles (eg, metal shards) sent airborne by power tools, explosions (including
weapons), lawn mowers, or motor vehicle accidents can cause either OGL or corneal
abrasion. Penetration of the globe most commonly occurs at the cornea.
Large OGL may present with globe deformity, extrusion of vitreous or iris, or a visible
entry wound. Smaller lacerations and abrasions, however may not be visibleon routine
inspection, In the absence of an obvious wound, OGL may be inferred by a peaked
or teardrop pupil (iris stretching due to the laceration or foreign object), increased or
decreased anterior chamber depth, or reduced intraocular pressure. Other potential
manifestations include loss of visual acuity or absent afferent pupillary response. If
OGL is suspected, further assessment should be performed with fluorescein
drops; as aqueous drains through the puncture site, fluorescein may be seen streaming
away from the site or clear fluid may be seen flowing through the fluorescein.

Management of OGL requires shielding of the eye and emergent ophthalmologic


consultation for surgical repair. Chronic complications include infection
(endophthalmitis), posttraumatic cataract, and vision impairment.

Hypopyon is a layering of inflammatory cells in the dependent portion of the anterior


chamber. Perilimbal injection (also called ciliary flush) is characterized by dilation of
the vasculature at the junction of the sclera
and cornea. These findings are seen in a
variety of inflammatory (eg, anterior
uveitis/iritis) and infectious (eg,
endophthalmitis, keratitis)
conditions. Although OGL can lead to
endophthalmitis, these findings would not
be present at the time of initial
presentation.
Periorbital ecchymosis (raccoon eyes) suggests orbital/skull fracture. Orbital fracture
with globe rupture typically occurs with blunt trauma.

Saline irrigation can be used to clear a superficial foreign body but should not be
performed unless OGL has been ruled out and a foreign body identified.

Ocular tonometry is used primarily to identify elevated intraocular pressure associated


with glaucoma. If there is concern for OGL, tonometry is typically avoided as it can
cause further extrusion of aqueous and vitreous humor.

Ocular glucocorticoids are used for certain inflammatory conditions (eg, uveitis) but
would likely slow healing in a patient with a high-velocity injury and may predispose to
infection or corneal perforation.
INFERIOR RECTUS
ENTRAPMENT

All patients who sustain orbital trauma should undergo assessment of visual
acuity and extraocular movements. CT scan is obtained if there is evidence of a
fracture (eg, step-off on palpation), changes in vision or extraocular movements, or the
examination is limited due to soft-tissue swelling or altered mental status.

This patient's coronal CT scan shows a left orbital floor fracture with herniation of
orbital fat into the maxillary sinus and entrapment of the inferior rectus muscle. An
entrapped inferior rectus keeps the affected globe in a downward position, leading
to diplopia on upward gaze despite normal visual acuity. Prolonged entrapment may
cause ischemia, fibrosis, and permanent dysfunction. Therefore, if there is concern for
muscle entrapment either on examination or CT scan, the patient should be seen by a
surgical subspecialist (eg, plastic surgeon, ophthalmologist) within 24 hours.

…………….
Open globe injury can also occur with
facial trauma. However, patients with
this injury have markedly decreased
visual acuity, and CT scan would show
a flattened appearance of the globe.
Orbital cellulitis can present with
swelling, pain, and impaired extraocular
movements. However, patients also
typically have fever and diffuse (rather
than just upward gaze) restriction of
ocular movement.

An orbital hematoma can occur after


facial trauma and present with
diplopia. However, patients with
hematoma have markedly decreased
visual acuity due to pressure-induced
ischemia of the optic nerve, and CT scan
would show intraorbital fluid rather than
an orbital floor fracture.

Trochlear nerve (CN IV) dysfunction can


present with vertical diplopia after
trauma. However, because CN IV innervates the superior oblique muscle, the vertical
diplopia is present on downward (rather than upward) gaze, and examination
shows hypertropia and extorsion of the eye.
SPINAL
EPIDURAL
ABSCESS (SEA)

A patient underwent epidural anesthesia 10 days ago and now has lumbar back pain,
low-grade fever, and lower-extremity neurologic symptoms (eg, weakness, tingling,
numbness), raising strong suspicion for spinal epidural abscess (SEA). Most cases
arise via hematogenous spread of a distant infection, contiguous spread from an
adjacent infection (eg, vertebral osteomyelitis), or following direct inoculation during
spinal/epidural anesthesia.

The epidural space is a vertical, contiguous area that contains fat, arteries, and a venous
plexus; infections tend to affect multiple spinal levels and cause
progressive neurologic impairment due to direct spinal cord compression,
thrombophlebitis of the draining venous plexus, and/or interruption of the arterial
blood supply. Although patients classically have the triad of fever, spinal pain,
and neurologic symptoms, all 3 are present in a minority of cases. Most patients have
fever, malaise, and neurologic symptoms that typically progress in the following
fashion (due to worsening spinal cord compression):

1. Focal back pain →


2. Nerve root pain (eg, shooting, electric-shock sensation) →

3. Motor weakness, sensory changes, bowel/bladder dysregulation →


4. Paralysis

Early diagnosis is crucial to prevent paralysis and death; MRI of the affected spinal
area is the test of choice because it is positive early in the disease course and provides
excellent visualization of the extent of infection. Surgical drainage plus antibiotic
therapy is generally required for treatment.
NSAIDs and early physical activity are important components of treating lumbosacral
radiculopathy, which usually presents with back pain radiating along the affected
dermatome. Although motor and sensory impairment can occur, fever is atypical.
Lower-extremity peripheral nerve injury is a relatively common obstetric complication
due to nerve compression or vascular compromise during labor. However, it rarely
causes fever.

INCREASED
INTRACRANIAL
PRESSURE (ICP)
MANAGEMENT

Partial pressures of O2 and CO2 also play an important role in regulating CBF. In
conditions such as stroke or trauma, these systems are disrupted and interventions may
be required to lower ICP.
HERNIATION Transtentorial (Uncal) Herniation

Lesion Neurologic Signs

Compression of the
contralateral crus cerebri Ipsilateral hemiparesis
against the tentorial edge

Loss of PANS innervation causes mydriasis (occurs early);


Compression of the ipsilateral loss of motor innervation causes ptosis and a down-and-
oculomotor nerve (i.e., CN III) out gaze of the ipsilateral pupil due to unopposed
by the herniated uncus trochlear (i.e., CN IV) and abducent (i.e., CN VI) action
(occurs late)

Compression of the ipsilateral


posterior cerebral artery (i.e., Contralateral homonymous hemianopsia
ischemia of visual cortex)

Compression of the reticular


Altered LeOC; coma
formation
A patient presents after blunt head trauma with signs/symptoms worrisome
for transtentorial (uncal) herniation secondary to a right-sided epidural
hematoma. Epidural hematomas result from rupture of the middle meningeal artery,
and this higher arterial pressure can rapidly expand the hematoma and compress the
temporal lobe. The fluid resuscitation in this patient likely increased the rate at which
the epidural hematoma expanded. The uncus is the innermost part of the temporal
lobe and herniates through the tentorium to cause pressure on the ipsilateral
oculomotor nerve, ipsilateral PCA, and contralateral cerebral peduncle against the edge
of the tentorium. Patients typically present with the focal neurologic signs shown in
the table above.

Abducens nerve (i.e., CN VI) injury from uncal herniation usually occurs later in the
clinical presentation.

Accessory nerve (i.e., CN XI) dysfunction can be due to lesions in the medulla, such as
occlusion of the PICA. The
accessory nerve can also be injured
during surgical procedures
involving the anterolateral
neck. Injuries present with
paralysis of the ipsilateral
sternocleidomastoid and trapezius
muscles.
The glossopharyngeal nerve (i.e.,
CN IX) is most commonly injured
by compression from a nearby
tumor, as in jugular foramen
syndrome (posterior fossa
tumor). A glossopharyngeal nerve
lesion would cause loss of the gag
reflex, loss of taste and sensation
on the posterior one-third of the
tongue, loss of pharyngeal
sensation, and dysfunction of the carotid sinus reflex leading to an increased risk of
syncope.

////////////////////////////////////////ENT////////////////////////////////////////
AIRWAY A patient with an enlarging fluid collection (ballotable neck swelling) developing after
EMERGENCY thyroidectomy likely has an expanding neck hematoma. An expanding neck
hematoma is life-threatening due to the potential for airway obstruction, either from
direct airway compression or from vascular compression causing venous
congestion leading to laryngeal edema. Stridor, dysphagia, voice changes, and
"tripod positioning" are all signs of increasing upper airway obstruction; however,
patients can initially have few or no symptoms (e.g., only mild neck
tightness). Notably, patients often maintain their peripheral blood oxygenation
concentration until rapid (<1 min) decompensation.
The most important step in management is to decompress the airway by
immediately evacuating the hematoma (including at the bedside if necessary). This
is done by reopening the incision and removing the blood collection. The wound
should then be explored in the OR to control the source of bleeding, and the patient's
airway should be evaluated to assess for laryngeal edema and need for protective
endotracheal intubation (until the edema subsides).

…..

Compression via direct pressure or with a compressive dressing is often used to control
bleeding at many sites. However, when bleeding is in the neck, compression can
worsen the airway obstruction.

A CT scan of the neck is not needed to diagnose an expanding neck hematoma and
would delay potentially life-saving treatment.

A fluoroscopic swallowing study may be used to assess for pharyngeal or esophageal


injury (which would not cause rapidly expanding neck swelling) or to evaluate
swallowing function if there is concern that a regular diet may be unsafe (eg, in some
patients with head and neck tumors).

Urgent cricothyrotomy involves an incision through the cricothyroid membrane to


secure an airway for a patient in distress when other forms of airway management are
unsuccessful, unavailable, or contraindicated. If a patient's airway is threatened by a
hematoma; removal may be sufficient to improve the airway.

HEAD AND NECK A patient with persistent


CANCERS hoarseness and a fungating
laryngeal mass has laryngeal
squamous cell carcinoma (SCC),
which accounts for >90% of
lesions in the adult larynx. The
most important risk factors
are smoking and alcohol
use. Hoarseness is often the
presenting symptom, due to
impaired vibration or movement
of the vocal cords. Persistent
hoarseness (>30 days) should
always be evaluated by
laryngoscopy to ensure no delay
in diagnosis of possible cancer.

Other presenting symptoms may include dysphagia or airway obstruction due to a


mass blocking passage of food or air. Patients may have referred otalgia facilitated by
either the glossopharyngeal nerve (CN IX) (which innervates both the base of tongue
and the external auditory canal [EAC]) or the vagus nerve (CN X) (which innervates
parts of the larynx/hypopharynx and the EAC). Hemoptysis may be seen due to
tumor friability (eg, blood crusting). Regional nodal metastases manifest as cervical
adenopathy.
…..

Adenocarcinoma is unlikely in the larynx (<1% of laryngeal malignancies). Lung


adenocarcinoma shares some risk factors (eg, smoking, age) and presenting symptoms
(eg, cough, hemoptysis) with laryngeal SCC but would not metastasize to the larynx.

Aphthous ulcers are common, painful ulcers that typically resolve in <2 weeks and
are usually seen in the oral cavity (eg, tongue, buccal mucosa) rather than the larynx
and would not cause hoarseness.

Laryngeal candidiasis is an opportunistic infection that often occurs concurrently


with oral candidiasis (thrush) in patients using corticosteroid inhalers (eg, due to
COPD). White patches or plaques on the mucosal surfaces are seen; a single, large
laryngeal mass would not occur.

Pemphigus vulgaris is an autoimmune disease characterized by painful blisters of the


skin and mucous membranes. However, the pain is severe, and there would be skin
blistering and multiple ulcerations (not a mass isolated to the vocal cord).

Reflux laryngitis could cause hoarseness and dysphagia but not a laryngeal mass.
Recurrent respiratory papillomatosis is due to HPV. Wartlike proliferations described
as multiple, raised, finger-shaped lesions are seen on the surface of the vocal
cords. Squamous papillomas appear more uniform and do not invade deeper
structures.

Vocal cord polyps often occur due to inefficient or excessive voice use (eg, teachers,
telemarketers). They can cause hoarseness due to impaired vocal cord
vibration. Polyps do not invade or ulcerate.

An ulcerated tonsillar lesion in a patient with a long smoking history is highly


suspicious for oropharyngeal squamous cell carcinoma (SCC). Oropharyngeal SCC may
present with sore throat and odynophagia due to tumor invasion or local irritation.
Lesions often ulcerate and can be friable (eg, bleeding to light touch). Other
manifestations include referred otalgia or an isolated neck mass (representing
regional nodal spread of disease).
Traditional risk factors for oropharyngeal SCC include age >40, tobacco use, alcohol
use, and immunocompromised status. There has also been a dramatic increase in
oropharyngeal cancer due to human papillomavirus (HPV), which may be seen in
younger, nonsmoking patients.
\This patient should undergo biopsy of the tonsil lesion with evaluation of HPV
status as well as neck imaging (eg, CT scan) to characterize the lesion and any
associated nodal metastasis.
Tonsil stones (tonsilloliths)—commonly occurring concretions of food, cellular, and
bacterial debris—are NOT known to be risk factors for oropharyngeal SCC and can
be managed with gentle removal (eg, gargling with salt water). However, frequent
patient manipulation (eg, with toothpicks) can cause mucosal trauma, which may be
mistaken as an ulcerated neoplasm.
…………

A tonsil malignancy is unlikely to be adenocarcinoma. Lung adenocarcinoma shares


some risk factors (eg, smoking, age) and possible presenting symptoms (eg, cough,
hemoptysis) with oropharyngeal SCC, but it does not metastasize to the tonsil.

Although NHL rarely can present in a tonsil, it is much less common than head and
neck SCC. It would also cause tonsillar enlargement rather than ulceration and is
usually seen in patients with immunosuppression or autoimmune diseases.

Small cell carcinoma is a neuroendocrine lung tumor that occurs almost exclusively in
smokers. However, it is not seen in the tonsil.

Otalgia in the setting of a normal ear examination is likely referred pain (pain perceived
at a different site than its true origin). The most common causes of referred
otalgia are dental disease and temporomandibular joint disorders. However, referred
otalgia is commonly a presenting symptom of mucosal head and neck squamous cell
carcinoma (HNSCC), especially in an older patient with a history of smoking, alcohol
use, occupational exposure to welding fumes, and cervical lymphadenopathy.

Referred pain frequently occurs when nerves travel in close proximity, especially if their
cell bodies are located in the same ganglia or they synapse in the same location In
this case, the glossopharyngeal nerve (CN IX), which has afferent fibers innervating
the base of the tongue, also has a branch that provides afferent sensory input from the
EAC; all of these afferent fibers converge in a ganglion just prior to entering the jugular
foramen. Similarly, the vagus nerve (CN X), which innervates parts of the larynx and
hypopharynx, also provides sensory innervation to the EAC.

Therefore, referred otalgia can be due to a tumor at the base of the tongue or from the
larynx/hypopharynx. Flexible laryngopharyngoscopy would likely identify the
primary tumor site.

……..

Referred otalgia is almost never due to central causes (e.g., vestibular schwannoma).

Tympanometry can assess for eustachian tube dysfunction, which can cause ear pain
but would also cause hearing loss, feeling of fullness, and tympanic membrane
retraction or middle ear effusion; tympanostomy tube placement can be considered in
patients who do not respond to medical management (eg, decongestants). Tube
placement is also indicated in some children with recurrent otitis media.
Persistent, enlarged, firm neck mass and ulcerated tonsillar lesion are consistent
with head and neck squamous cell carcinoma (HNSCC), which can also lead to
pharyngitis, dysphagia, and halitosis (bad breath). Traditional risk factors for HNSCC
include older age, tobacco and alcohol exposure, and poor dentition. With the
decrease in smoking rates, there has been a decline in many types of HNSCC; however,
the incidence of oropharyngeal (eg, tonsil, base of tongue) HNSCC has dramatically
increased.
This increase is due to HPV–positive HNSCC, often seen in younger patients with no
tobacco exposure and multiple sexual partners. HPV-positive HNSCC primarily
affects the oropharynx (possibly due to the higher concentration of lymphatic tissue
facilitating viral processing) and often presents with neck LAD. However, it is more
responsive to therapy than HPV-negative HNSCC. The MC causative subtype is
HPV-16, which is covered by the HPV vaccine.

This patient should undergo biopsy of the tonsil lesion with evaluation of HPV status,
neck imaging (eg, CT scan) to characterize the lesion and associated nodal metastasis,
and endoscopic evaluation of the upper aerodigestive tract.

………

Cervicofacial actinomycosis is a rare infection presenting as a slowly progressive,


NONTENDER mass that can form abscesses and draining sinus tracts with
characteristic yellow "sulfur granules." Regional adenopathy is unlikely because the
infection spreads by direct extension, ignoring normal tissue planes.

EBV is also associated with certain lymphomas (eg, Burkitt) and with nasopharyngeal
(rather than oropharyngeal) cancer, all of which are unlikely to cause unilateral
tonsillar ulceration.
*****Fusobacterium necrophorum causes Lemierre syndrome, a life-threatening,
deep neck space infection that progresses to suppurative thrombophlebitis of the
internal jugular vein (IJV). Although it presents with painful pharyngitis and
odynophagia, it also has a more acute (<1 week) and toxic presentation with high
fevers (>39 C [102 F]), rigors, and respiratory distress from associated septic
pulmonary emboli.

Although GAS causes painful pharyngitis, it presents acutely with fever and would not
be expected to cause persistent, unilateral, firm tonsillar enlargement with ulceration
and unilateral enlarged, nontender adenopathy.

Primary syphilis (caused by Treponema pallidum) can present with a


typically PAINLESS oral ulcer (chancre). Chancres on the tonsil are extremely rare
and would not present with adenopathy. In addition, primary syphilis typically occurs
within weeks after the initial exposure (eg, unprotected sexual encounter).
Although leukoplakia (white mucosal lesion) is often a benign, asx
condition, evolving oral leukoplakia that is nonhomogeneous and friable (eg, tasting
blood) is concerning for squamous cell carcinoma
(SCC), especially in a patient who uses smokeless
tobacco.
Most cancers of the head and neck arise from
squamous epithelial cells that undergo stepwise,
premalignant changes (ie, hyperplasia to dysplasia to
carcinoma). Lesions may initially manifest in the oral
cavity as hyperplastic or dysplastic white
(leukoplakia) or red (erythroplakia). Because these
lesions are at risk of malignant transformation,
patients require counseling regarding risk reduction
(eg, tobacco chewing cessation), regular examination
of the oral cavity, and rebiopsy if changes (eg,
thickness, firmness) are seen.

If biopsy results are cancerous, regional metastatic


spread to the cervical lymph nodes is highly likely;
therefore, a CT scan of the neck with contrast should
also be obtained, both to evaluate the extent of invasion and characterize the
metastatic nodal spread.
……….

White patches in the mouth may be the result of hyperkeratotic changes in response
to repeated trauma. Broken teeth and ill-fitting dentures can cause chronic
irritation; therefore, dental evaluation and treatment can improve these lesions.
However, in a patient with a changing area of leukoplakia who uses smokeless tobacco,
malignancy must be ruled out.

Fluconazole therapy can be used to treat oral candidiasis, a common opportunistic


infection. The use of corticosteroid inhalers (likely in this patient with asthma) is a
common risk factor. Although oral candidiasis also presents with white patches or
plaques in the oral cavity, malignancy must be ruled out in a changing lesion exposed
to smokeless tobacco.

Topical glucocorticoids are sometimes used to treat aphthous ulcers, which tend to
be multiple, painful, and short-lasting (<2 weeks). Topical glucocorticoids and topical
tacrolimus can be used to treat oral lichen planus, which often occurs on the buccal
mucosal surfaces and can appear as reticular, white plaques; erythematous, atrophic
patches; or ulcerations. An evolving, persistent, leukoplakic lesion in the setting of
tobacco use is more likely cancer.
SALIVARY GLAND A patient has nontender, bilateral enlargement of the parotid glands, which is
TUMORS consistent with sialadenosis. Sialadenosis is a benign, noninflammatory swelling of
the salivary glands. It can result from overaccumulation of secretory granules in acinar
cells (possibly due to abnormal autonomic innervation) in patients with CHRONIC
alcohol use, bulimia, or malnutrition. It can also result from fatty infiltration of the
glands in patients with diabetes mellitus or liver disease.

Sialadenosis presents with gradual enlargement of the glands that does not fluctuate
and is not associated with eating. Recognition of this condition should prompt an
investigation for underlying disease. Control of the underlying disease may improve
(but usually not fully resolve) the glandular enlargement.

…….

Parotitis due to mumps can present with parotid swelling. However, it is typically
painful and presents a few days after the onset of systemic symptoms (eg, fever,
headache, myalgias).
A pleomorphic adenoma is a benign salivary neoplasm that can present with painless
enlargement of the parotid gland. However, the enlargement would be unilateral, and
a distinct mass is typically palpated.
Salivary stones (sialolithiasis) can block the flow of saliva out of the duct and lead to
swelling due to increased fluid in the gland. However, swelling due to sialolithiasis is
usually fluctuating, painful, and associated with eating (which stimulates saliva
secretion).

Sjögren syndrome can lead to bilateral swelling of the parotid glands due to
lymphocytic infiltration. However, in contrast to sialadenosis, it typically presents in
women and is associated with dry mouth on examination (whereas sialadenosis
patient's mucous membranes are moist).

Vitamin A deficiency often coexists with malnutrition, which can lead to


sialadenosis. However, sialadenosis in these patients is probably related to protein
malnutrition rather than vitamin A deficiency. Vitamin A deficiency most notably
causes ocular manifestations, including impaired night vision, dry eyes, and
keratinization of the conjunctivae and corneas.
NASOPHARYNGE A patient
AL CARCINOMA has nasopharyngeal
carcinoma (NPC), which is
associated with the
reactivation
of EBV. Tumors typically
express EBV DNA, and
EBV assays are often
used to monitor
treatment response and
disease relapse. NPC is
rare in the US but is
endemic to southern
China (and parts of Africa
and the Middle East). Risk
is thought to be higher in
these locations due to a nitrosamine-rich diet (salt-cured food, EARLY EXPOSURE to
salted fish) and genetic predisposition.
NPC tumors obstruct the nasopharynx and invade adjacent tissues, often resulting
in nasal congestion with epistaxis, headache, cranial nerve palsies (eg, facial
numbness), and/or serous otitis media (eustachian tube obstruction). Early metastatic
spread to the cervical lymph nodes may cause a nontender neck mass.

…….

Aflatoxin B1 is a mycotoxin that often contaminates agricultural products and is


associated with an increased risk of HCC, not NPC.
Although NSAIDs may increase the risk of epistaxis, they are not a cause of NPC.

Recurrent bacterial sinusitis may result in nasal polyposis. This may manifest with
nasal congestion and thick rhinorrhea. Recurrent sinusitis is not typically associated
with NPC.
Some forms of vitamin A (eg, beta carotene) in excess may increase the risk of lung
cancer in smokers, but there is no association with NPC.

Nasopharyngeal carcinoma

• Endemic to Asia
Epidemiology • Linked with Epstein-Barr virus reactivation
• Risk factors: Diet (salty fish), smoking, genetics

• Obstruction: Nasal congestion, epistaxis, headache


Manifestations • Mass effect: Cranial nerve palsy, otitis media
• Spread: Neck mass (cervical lymphadenopathy)
Diagnosis • Endoscope-guided biopsy

• Radiation therapy (RTX)


Treatment
• Chemotherapy (CTX)

NPC is a tumor associated with EBV reactivation that is rare in the United States but
endemic in southern China.

The nasopharynx links the nasal cavity and the oropharynx. NPC tumors obstruct this
passage and invade adjacent tissues, often resulting in nasal congestion with
epistaxis, headache, facial numbness (para-cavernous sinus tumor invasion), and/or
serous otitis media (eustachian tube obstruction). Early metastasis to the bilateral
cervical lymph nodes is common.

Diagnosis is made with endoscope-guided biopsy of the PRIMARY tumor, and


treatment usually involves a combination of RTX and CTX. Survival is excellent if the
neoplasm is discovered early, but most patients present with advanced disease.

………
GPA is a vasculitis that commonly causes ear (otitis) and nasal (rhinorrhea, bloody
discharge) symptoms. However, patients usually have systemic manifestations (eg,
fever, malaise, weight loss) and ulcerative lesions, not a soft-tissue mass.

Mucormycosis is a fungal infection primarily seen in patients with DM or significant


immunocompromise. Manifestations typically include a pan-sinusitis with fever and
purulent rhinorrhea. Necrotic destruction of bone may occur, and examination often
reveals black eschars.
Nasal polyps are usually seen in patients with chronic sinusitis, asthma, or aspirin
sensitivity and may obstruct the nasal cavity, resulting in congestion and thick
rhinorrhea. However, cranial nerve involvement is unlikely and examination would
reveal a grey mass with inflammatory material.

RECURRENT Constant (≥1 month) or


RESPIRATORY progressive hoarseness is often
PAPILLOMATOSIS related to a vocal cord lesion
(RRP) and REQUIRES evaluation by
laryngoscopy. This patient
with irregular, exophytic
growths in clusters on the
surfaces of his vocal cords likely
has laryngeal papillomas due to
recurrent respiratory
papillomatosis (RRP). These
lesions are often warty or
grapelike and, as with skin
papillomas, have dark-red punctate areas corresponding to blood vessels. Laryngeal
papillomas are caused by HPV subtypes 6 and 11. Infants typically acquire HPV via
vertical transmission; adults may develop symptoms due to reactivation of
vertically transmitted HPV or from exposure to HPV through sexual contact.
Although benign, RRP is associated with significant morbidity (eg, voice outcomes,
airway obstruction, repeated operative interventions). In addition, the clinical course is
variable and unpredictable with fluctuations in severity. In rare instances, it can
spread beyond the vocal cords to involve the lower respiratory tract. Medical therapy
(eg, interferon, cidofovir) has LIMITED efficacy; therefore, the mainstay of treatment
remains surgical debridement, and patients often require many procedures.

……..

Cigarette smoking can cause a variety of laryngeal lesions including leukoplakia and
SCC. Although smoking can cause malignant degeneration of RRP lesions, this
patient's pathology has no malignant features.
Environmental allergens can cause chronic inflammation of nasal and sinus mucosa,
which can produce thick secretions that cause hoarseness. Inflammation may be seen
but not wartlike growths.
Fungal infection of the larynx typically causes thickened, white patches on an
erythematous base (similar to that seen in oral thrush).

***GER may affect the larynx and cause hoarseness and laryngeal lesions such as
focal swelling, erythema, or granulation tissue. However, it generally has a
fluctuating course (often worse when laying down) and would not cause wartlike
growths.

Polyps and nodules can form on the vocal cords due to chronic irritation from vocal
abuse (eg, yelling or using voice excessively). However, both polyps and nodules tend
to have more smooth edges and would not form in clusters.

THYROID Thyroid nodules are common and may be diagnosed on physical examination or
CANCER noted incidentally when imaging studies are obtained for other reasons. Once a
thyroid nodule is found, cancer risk factors (eg, family history, radiation exposure)
**** should be assessed by history. Physical examination should evaluate the approximate
size, mobility, and firmness of the thyroid nodule and whether enlarged cervical lymph
nodes are present. A serum TSH should be obtained.

If serum TSH is normal, a thyroid ultrasound is the next step to determine nodule
sonographic features and size. Certain sonographic features (eg, microcalcifications,
irregular margins, internal vascularity) carry a much higher risk of malignancy than
others (eg, cystic or spongiform lesions). Thyroid nodules >1 cm with these high-risk
sonographic features should undergo fine-needle aspiration (FNA) biopsy. Thyroid
nodules >2 cm should ALL undergo FNA (unless they are cystic, as they have a low
risk of malignancy).
Pregnant women undergo the SAME evaluation but should not receive radioactive
iodine (for either diagnosis or treatment) because it can lead to congenital
hypothyroidism, intellectual disability (ID), and increased risk of malignancy in the
fetus. In a patient who has a >1-cm nodule with high-risk sonographic features,
workup should proceed with an FNA, which is safe during pregnancy.
………

Thyroid nodules may be found incidentally on an MRI scan; however, ultrasound is the
initial imaging modality of choice for workup of thyroid nodules.

Radionuclide scans may be done for thyroid nodules in the setting of a low TSH
(hypERthyroid) level because those nodules are often hot and less likely to be
cancerous. In a patient with a normal TSH, an FNA is the appropriate next step. In
addition, radioactive iodine should never be used in pregnancy because of the risk to
the fetus.
………….

If a thyroidectomy is needed (eg, due to cancer), it can often be delayed until after
delivery. However, should the workup reveal a more aggressive or rapidly growing
thyroid cancer, the optimal timing of surgery would be during the second
trimester. Therefore, this patient should undergo an FNA to determine if (and when) a
thyroidectomy is needed.

Thyroglobulin is produced by both normal thyroid tissue and differentiated thyroid


cancer cells. Therefore, it is a useful tumor marker to monitor for recurrence after the
thyroid gland has been completely removed.

A patient has medullary thyroid cancer (MTC), a neuroendocrine malignancy arising


from the calcitonin-secreting parafollicular C cells. Most cases of MTC are sporadic,
although approximately 25% are due to germline mutations in the RET proto-
oncogene (eg, MEN2, characterized by: MTC; pheochromocytoma; and either
parathyroid hyperplasia [type 2A] or marfanoid habitus and mucosal neuromas [type
2B]).

MTC most commonly presents as an asymptomatic thyroid nodule. A minority of


patients have diarrhea and flushing (due to hormone secretion by the tumor). Unless
there is concomitant HPTH (eg, MEN2A), serum calcium is usually NORMAL (even
though it produces calcitonin), possibly due to downregulation of calcitonin
receptors. The diagnosis of MTC is confirmed with FNA. Primary treatment is
TOTAL thyroidectomy, and therefore postoperative thyroid replacement therapy (ie,
levothyroxine) is necessary.
MTC is often metastatic at the time of diagnosis, primarily to the cervical LNs and
upper aerodigestive tract. Serum calcitonin levels CORRELATE with the risk of
metastasis and are measured at the time of diagnosis; they also correlate with risk
of recurrence and are therefore measured serially following surgery. CEA also
correlates with disease progression and is typically measured with calcitonin.

………..

The DXM suppression test is used in the evaluation of hypercortisolism (ie, Cushing
syndrome); suggestive manifestations include proximal muscle weakness, facial
plethora, and supraclavicular fat pads. Hypercortisolism is not associated with MEN2 or
sporadic MTC. It is occasionally seen in MEN1 (which is not associated with MTC),
generally due to ACTH-secreting pituitary tumors or adrenal tumors.

***PHPTH is a major manifestation of MEN1 (parathyroid, pituitary, and


enteropancreatic endocrine cell tumors) and MEN2A (unlikely in a patient with
negative RET germline mutation testing). Transient HoPTH can develop after
thyroidectomy; patients are often given calcium supplementation, with the dose
tapered off based on CALCIUM (NOT PTH) measurements.
Thyroglobulin (TG) is a glycoprotein that serves as a substrate for the production of
thyroid hormone. Levels are elevated in differentiated papillary/follicular thyroid
cancer, for which it is used as a cancer marker, but NOT in MTC (which does not
produce thyroid hormone). Thyroglobulin levels are also elevated in goiter, Graves
disease, and thyroiditis. Similarly, thyroid scintigraphy uses radiolabeled iodine to
assess iodine uptake as a marker for thyroid metabolic activity; papillary and follicular
thyroid cancers take up radiolabeled iodine, but MTC does NOT.
Serum thyroxine is used in the initial diagnosis of hypo- and hyperthyroidism.
Thereafter, levothyroxine dose adjustments are primarily based on serum TSH, and
thyroxine measurement has little utility.
Thyroglobulin (Tg) is the precursor to active thyroid hormones (T3 and T4) and is
produced by normal thyroid tissue or differentiated (ie, papillary, follicular) thyroid
cancer. Most Tg is stored in the thyroid gland, but some is released into the
circulation. Patients who have undergone a total thyroidectomy and radioactive iodine
treatment should have no residual normal thyroid tissue; therefore, a rising Tg level in
these patients is likely due to recurrent differentiated thyroid cancer.

Patients who have undergone thyroidectomy for differentiated thyroid cancer


REQUIRE levothyroxine (T4) supplementation for 2 reasons:

• Levothyroxine replaces thyroid gland function.


• Levothyroxine suppresses pituitary release of TSH (negative
feedback). Because TSH stimulates thyroid tissue growth, levothyroxine
supplementation to suppress TSH (ie, by causing a mildly hyperthyroid state)
may help prevent thyroid cancer recurrence.

In some patients, a "stimulated" Tg level can also be obtained to evaluate for


recurrence. This test measures the Tg level after withdrawing levothyroxine
supplementation (to increase pituitary release of TSH) or after giving recombinant
TSH. If there is recurrent cancer, the increased TSH will cause increased Tg release
from the cancer cells.
………
Radioactive iodine causes radiation-induced destruction of thyroid cells and is used as
an adjuvant treatment for thyroid cancer. Because the iodine can be taken up by other
cells, adverse effects include sialadenitis (salivary gland infection), dry mouth, and
pulmonary fibrosis. Tg levels would not be affected.

Tumor neoantigens are attractive targets for cancer immunotherapy. However,


increased immunologic response (ie, increased antibody formation) toward tumor cells
should decrease persistent/recurrent disease and therefore decrease serum Tg.
Excess thyroid hormone replacement, which leads to enhanced negative feedback to
the pituitary, would be expected to decrease, rather than increase, Tg levels in patients
who have Tg-producing cells (including in this patient with likely recurrent thyroid
cancer).

EPISTAXIS A patient who sustained nasal trauma and now has nasal obstruction and fluctuant
swelling of the nasal septum has a septal hematoma, which is an accumulation of
blood between the perichondrium and the septal cartilage.

Although the incidence is rare, the complications of an untreated septal hematoma are
significant. Infection can develop rapidly (2-3 days) and lead to a septal abscess. In
addition, because the septal cartilage has no direct blood supply and receives all
nutrients via diffusion from the perichondrium, a septal hematoma can cause avascular
necrosis (AVN) of the septal cartilage. Destruction of the nasal septal cartilage can
result in septal perforations, external nasal deformities (eg, saddle nose), or internal
nasal valve collapse (ie, nasal obstruction).
Therefore, all patients who sustain nasal trauma undergo examination of the nasal
septum. Palpation can differentiate between a deviated nasal septum (firm) and a
septal hematoma (soft, fluctuant).

Patients with a septal hematoma require prompt incision and drainage (I&D). After
incision and drainage, anterior nasal packing should be placed to compress the
perichondrium to the nasal septum; ice packs and NSAIDs can reduce edema due to
inflammation. Patients are treated with antibiotics and require otolaryngology
evaluation after a few days for packing removal and assessment.

………
Embolization of the sphenopalatine artery is one method to control posterior
epistaxis. This patient does not currently have epistaxis because blood has already
accumulated in the nasal septum.

A CBC is ordered in cases of severe epistaxis that may require transfusion, but it is not
needed in this patient who has had no further bleeding. Coagulation studies are
ordered in patients who are anticoagulated or have recurrent, spontaneous
epistaxis. This patient is healthy and has an obvious cause of epistaxis (trauma).

EPIGLOTTITIS This patient with HIV had a week of progressive sore throat and then developed
acute hypoxemic respiratory failure with tripod position, drooling, and hoarseness,
raising strong suspicion for epiglottitis, a cellulitis of the epiglottis, aryepiglottic folds,
and other adjacent tissue. Swelling of these structures often leads to rapid airway
deterioration; therefore, airway management is the main priority.
In patients unable to maintain adequate oxygen saturations, bag-valve-mask
ventilation (BVM) with 100% oxygen (to keep oxygen saturation ≥~88%) should be
initiated. If BVM does not result in adequate oxygenation (ie, oxygen saturation
remains low), endotracheal intubation using a video laryngoscope (to facilitate
direct visualization of the epiglottis) should be attempted.

However, given the risk of rapid respiratory deterioration, failure of a SINGLE attempt
at endotracheal intubation with a video laryngoscope should immediately prompt the
establishment of a surgical cricothyrotomy by the most experienced provider
available (preferably an otolaryngologist or general surgeon). Cricothyrotomy
establishes an airway below the epiglottal swelling and potential obstruction.

Once the airway is stabilized (ABCs), antimicrobial therapy is needed to clear the
infection. This patient with untreated HIV has evidence of epiglottitis due
to Candida (eg, white plaques on the oropharynx). In addition to antifungal therapy,
he would also likely receive broad-spectrum antiBACTERIAL agents (eg, vancomycin,
ceftriaxone) given the severity of the condition and the possibility of a secondary
bacterial infection.
…….

Laryngeal mask airway should not be used because it is a supraglottic airway device
(above the obstruction).

Bilevel positive airway pressure (BiPAP) is not able to overcome a significant epiglottal
obstruction and is not recommended in patients who are hypoxic and fail an
endotracheal airway. Surgical airway is required.

Multiple attempts at endotracheal intubation with a video laryngoscope are not


recommended because this would delay adequate oxygenation in a patient who is
severely hypoxic. A surgical airway should be established when a single attempt at
video-assisted endotracheal intubation fails. If intubation without a video
laryngoscope fails, intubation with a video laryngoscope could be attempted.
Blind nasotracheal intubation is not recommended because the swollen epiglottis
must be visualized in order to effectively pass an endotracheal tube.

Infectious epiglottitis

• S. pneumoniae, H. influenzae
Epidemiology • Risk reduced with H influenzae vaccination

• Rapidly progressive & life-threatening


• Fever, sore throat, drooling, muffled voice
Clinical • Airway obstruction (stridor, dyspnea)
• Pooled oropharynx secretions
• Laryngotracheal tenderness

• Direct visualization
Diagnosis
• Imaging (lateral neck x-ray) – not necessary

• Early artificial airway (if needed)


Treatment • IV antibiotics (ceftriaxone PLUS vancomycin) – after ABCs

In adults, most cases are caused by S. pneumoniae or H. influenzae, with DM, obesity,
and preceding URI (disruption of the respiratory epithelium creates a portal of entry
for bacteria) increasing infection risk. Whereas children classically have abrupt onset
of drooling, dysphagia, and distress (the "3 Ds"), initial manifestations in adults are
more subtle and include sore throat, fever, and laryngotracheal tenderness to
palpation. With worsening of the swelling, difficulty swallowing, pooled oral
secretions, and respiratory compromise (eg, tachypnea, stridor) can develop.

The diagnosis is confirmed by direct visualization or soft-tissue lateral neck


radiograph, which usually shows an enlarged epiglottis, loss of the vallecular air
space, and/or a distended hypopharynx. Radiographs may also help exclude other
conditions. Respiratory compromise is less common in adults but can progess
quickly; therefore, patients with significant compromise (eg, hypoxia resistant to
noninvasive intervention) require airway establishment prior to considering neck
radiograph.

…….
CXR and sputum Gram stain/culture are used to diagnose pneumonia. Although this
patient has a cough, his pulse oximetry is normal, and his lungs have no evidence of
intrapulmonary pathology (eg, crackles). The presence of stridor and pooled oral
secretions makes an upper airway issue such as epiglottitis far more likely than
pneumonia.
Diphtheria can cause
slowly progressive sore
throat, malaise, cough,
stridor, hoarseness, low-
grade fever, and a thick,
white exudate
(pseudomembrane) on the
posterior pharynx. However,
diphtheria is very rare in
developed countries due to
widespread vaccination;
epiglottitis is more
common.

GAS pharyngitis usually


presents with acute-onset
sore throat, tonsillar
exudates, cervical LAD
(usually anterior), and NO
cough (Centor criteria).

PERITONSILLAR
ABSCESS

A patient has fever, pharyngeal pain, and earache suggesting a possible peritonsillar
abscess (PTA). PTA, also known as quinsy, is an acute bacterial infection of the region
between the tonsil and the pharyngeal muscles. It begins as persistent
tonsillitis/pharyngitis and progresses to cellulitis/phlegmon, with pus collecting into
an abscess within a week of symptom onset. PTA is MC in older adolescents and
young adults, and drug or alcohol use increases the risk.
Examination findings in PTA can include spasm of the jaw muscles (trismus) (which
often limits the physical examination), muffled "hot potato" voice, and swelling of
peritonsillar tissues with deviation of the uvula to the contralateral side.
Treatment involves: Airway mamagement (1st step). needle aspiration or
I&D plus antibiotic therapy to cover GAS and respiratory anaerobes (e.g., IV
clindamycin or ampi-sulbactam), if not responsive, consider MRSA.

Tonsillectomy indicated if:

• Unresponsive to drainage and antibiotics


• Recurrent tonsillitis or peritonsillar abscess, or other complications occur
• Airway obstruction is present

………..

Epiglottitis is characterized by high-grade fever, severe sore throat, and odynophagia


that can rapidly progress to airway obstruction. Pharyngeal findings in epiglottitis
are typically normal.
Uncomplicated tonsillitis is characterized by tonsillar erythema and exudates, often
with tender anterior cervical nodes and palatal petechiae. However, trismus, pooling
of saliva, and uvular deviation are more consistent with PTA.
Enlarged adenoids are usually seen in early childhood and regress with age. They are
unlikely to cause symptoms in an adolescent but may occasionally obstruct the
nasopharynx.

Herpangina is caused by the coxsackie A virus and presents with fever, sore throat,
and odynophagia. It is MC in children and is characterized by vesicles on the tonsils
and soft palate.

Infectious mononucleosis is characterized by the triad of fever, pharyngitis, and


posterior cervical lymphadenopathy. Mononucleosis is not typically complicated by
abscess formation.

Although a three-day history of fever, chills, and sore throat were most likely secondary
to tonsillitis, his muffled or "hot potato voice" and deviation of the uvula suggest
that a peritonsillar abscess has developed as a complication of his tonsillitis. Patients
with a peritonsillar abscess typically have prominent unilateral LAD. This condition can
be fatal secondary to either airway obstruction or spread of the infection into the
parapharyngeal space, which may lead to involvement of the carotid sheath. Initial
treatment consists of aspiration of the peritonsillar abscess and initiation of IV
antibiotics.

Surgical intervention may be necessary if the purulent material cannot be removed


with aspiration alone.
…………

Emergency laryngoscopy is typically required in patients with epiglottitis to ensure


adequate protection of the airway. Epiglottitis can present with difficulty swallowing
and a muffled voice similar to that seen in this patient, but the unilateral
lymphadenopathy and deviation of the uvula are more suggestive of a
peritonsillar abscess. In addition, the epiglottis is located more distal in the airway and
is not adjacent to the uvula.

Cricothyroidotomy is used as a last resort in patients who are unable to protect their
airway. This patient, however, is not complaining of any difficulty breathing at this time.

RETROPHARYNG The retropharyngeal


EAL ABSCESS space is a deep
compartment of the neck
defined anteriorly by the
buccopharyngeal fascia
and constrictor muscles of
the pharynx and
posteriorly by the alar
fascia. It communicates
laterally with the
parapharyngeal space.
A patient has a retropharyngeal abscess with neck pain, odynophagia, and fever
following penetrating trauma to the posterior pharynx. Examination findings can
include NUCHAL rigidity and bulging of the pharyngeal wall. Although deep space
infections of the neck have become less common since the advent of widespread
antibiotic use, they can progress rapidly with potentially fatal complications.
Infection within the retropharyngeal space drains inferiorly to the SUPERIOR
mediastinum.
Spread to the carotid sheath can
cause thrombosis of the internal
jugular vein (IJV) and deficits in
cranial nerves IX, X, XI, and XII (9-
12). Extension through the alar
fascia into the "danger space"
(between the alar and prevertebral
fasciae) can rapidly transmit
infection into the POSTERIOR
mediastinum to the level of the
diaphragm.

Acute necrotizing mediastinitis is


a life-threatening complication
characterized by fever, chest pain,
dyspnea, and odynophagia, and requires urgent surgical intervention.

………..

Extension of infection from the paranasal sinuses through the underlying bone can
lead to subdural empyema. Clinical findings include fever, headache, and mass
effect signs (eg, AMS).

Ludwig angina is a rapidly progressive bilateral cellulitis of the submandibular and


sublingual spaces, most often arising from an infected mandibular molar. Clinical
findings include fever, dysphagia, odynophagia, and drooling.

Cavernous sinus thrombosis (CST) is most often due to contiguous spread of infection
from the medial third of the face, sinuses, or teeth via the valveless facial venous
system. Clinical findings include headache, fever, cranial nerve deficits (eg, diplopia),
and proptosis.

Spinal epidural abscess can be caused by hematogenous dissemination (eg, IVDU),


contiguous spread from vertebral osteomyelitis, or direct inoculation (eg, epidural
anesthesia). Symptoms include fever, focal back pain, and neurologic deficits.
ACUTE PAROTITIS
Suppurative parotitis

• Elderly, dehydrated, postsurgical


• Decreased oral intake (eg, NPO perioperatively)
Risk factors • Medications (eg, anticholinergics)
• Obstruction (eg, calculi, neoplasm)

• Firm, erythematous pre/postauricular swelling


Clinical • Exquisite tenderness exacerbated by chewing and palpation
presentation • Trismus, systemic findings (eg, fever, chills)
• Elevated serum amylase without pancreatitis

• Ultrasound or CT scan (eg, ductal obstruction, abscess)


• Hydration, oral hygiene
Management • Antibiotics
• Massage (ie, milking pus out of gland)
• Sialagogues

An elderly patient with fever, leukocytosis, and severe tenderness and swelling of the
left parotid gland following a recent surgical procedure likely has acute suppurative
parotitis. Suppurative parotitis often occurs in elderly postoperative patients,
particularly those with dementia who are at risk of inadequate fluid hydration and
poor oral hygiene. In these patients, salivary stasis (due to NPO status, poor oral
hygiene, and dehydration) leads to retrograde seeding of bacteria from the oral cavity
(eg, S aureus, oral flora) through Stenson's duct to the parotid gland. Patients then
have rapid-onset and excruciatingly painful swelling of the involved parotid
gland that is aggravated by chewing. Examination shows an exquisitely tender,
swollen, and erythematous gland. Purulent fluid can often be expressed from the
parotid duct.

Although the diagnosis is typically evident after clinical evaluation, imaging (CT scan or
US) should be obtained to assess for salivary stones or neoplasms obstructing the
duct, as well as to differentiate between suppurative parotitis and an abscess.

Treatment includes IV, broad-spectrum antibiotics (narrowed based on culture


results), hydration, sialagogues (increase salivary flow), and massage (expression of
purulence from the duct).

……

Polysaccharide vaccine is given to patients age ≥65 to prevent illnesses caused by S


pneumoniae. However, S aureus and oral flora are much more commonly implicated in
suppurative parotitis.

Tetanus can also present with painful trismus (due to masseter spasm) and sometimes
fever.
Extreme jaw opening (eg, during intubation) can cause anterior TMJ dislocation,
which presents with pain in the preauricular area and difficulty opening or closing the
jaw. However, fever with leukocytosis would not be present.
Malignant hyperthermia occurs in susceptible patients following administration of
volatile anesthetic agents and can present with hyperthermia and masseter muscle
spasms. However, it typically presents within an hour of anesthesia induction; is not
associated with leukocytosis; and would not have localized symptoms.

Incentive spirometry and early ambulation can reduce postoperative pulmonary


complications (eg, pneumonia).

VERTIGO The inner ear contains the endolymphatic fluid-filled semicircular canals (which convey
movement and position of the head) and the cochlea (which is the sensory organ of
*** hearing). Conditions that cause disruption of endolymph flow can present with vertigo
(semicircular canals) and/or SNHL (cochlea).
Perilymphatic fistulas are a rare, but debilitating, complication of head injury or
barotrauma. They cause leakage of endolymph from the semicircular canals and
cochlea into surrounding tissues, resulting in characteristic clinical features:

• Progressive SNHL caused by damage to cochlear hair cells from loss of


endolymph.
• Episodic vertigo with nystagmus triggered by pressure changes in the inner
ear (eg, Valsalva maneuver, elevation changes [eg, riding in elevator]) due to
acutely increased endolymph leakage. This can be demonstrated clinically by
performing a loud clap (ie, pressure change due to sound conduction through
the ossicles) near the patient's ear and observing for nystagmus (Tullio
phenomenon).

Patients are advised to limit activities that increase inner ear pressure; they also
require ENT referral for further management.
………

BPPV vertigo is caused by debris (otoliths) that temporarily alters endolymph flow
through the semicircular canals. Therefore, patients typically have sudden, brief (<1
min) episodes of vertigo triggered by head movement.

Eustachian tube dysfunction can result in fluid in the middle ear space (not the inner
ear vestibular system) and can cause ear popping, cracking, and hearing loss in
response to changes in pressure. However, it also causes a sense of ear fullness or
pain, not episodic vertigo with nystagmus.
Ménière disease is caused by increased endolymphatic fluid volume or pressure in the
vestibular system. It also causes episodic vertigo with hearing loss, but episodes are
accompanied by aural fullness or tinnitus, last 20 minutes to 24 hours, and often lack
specific, identifiable triggers.

Orthostatic hypotension causes lightheadedness, presyncope, or syncope when the


patient assumes a standing position due to cerebral hypoperfusion. It does not involve
the vestibular system and therefore does not cause true vertigo or nystagmus.

This patient's traumatic brain injury puts him at risk for postconcussive syndrome,
which is characterized by headache, confusion, difficulty concentrating, mood
alterations, and sleep disturbances. It does not cause spinning vertigo with nystagmus
triggered by Valsalva maneuvers.

VESTIBULAR
Vestibular schwannoma (acoustic neuroma)
SCHWANNOMA
• Median age 50
Epidemiology
• Usually unilateral (bilateral associated with NF type 2)

Clinical
• SNHL & imbalance (CN VIII Schwann cell tumor)
manifestations & • ± Facial numbness &/or paralysis (CN V & VII compression)
pathophysiology

• Audiogram
Diagnostic studies • MRI with contrast of internal auditory canal (IAC)

• Observation (eg, small tumors, minimal symptoms, older or


infirm patients)
Management • Surgery
• Radiation therapy

This patient with asymmetric SNHL (AC > BC, lateralization to the unaffected ear),
imbalance, and facial numbness likely
has a vestibular
schwannoma (acoustic neuroma), a
benign tumor of CN VIII Schwann cells
that forms in the internal auditory
canal.

CN VIII is comprised of both the


cochlear nerve and vestibular
nerves. Therefore, patients with
schwannoma often have unilateral
hearing loss due to cochlear nerve
compression. Although vertigo would
generally be expected with acute
vestibular nerve damage, most
patients with vestibular schwannoma do not experience vertigo because the tumor's
slow growth allows for central compensation of gradual unilateral loss of input;
however, imbalance can still manifest when patients are deprived of visual input (eg,
walking at night). As the tumor expands from the internal auditory canal into
the cerebellopontine angle, it may compress
CN V (causing facial numbness) and CN VII
(causing facial weakness).

Patients require an audiogram and an MRI


with contrast of the internal auditory canal. In
select patients, observation may be
appropriate; otherwise, treatment is with
surgical resection or radiation therapy.
……….

A cholesteatoma is an erosive, expansile


mass of keratin debris in the middle
ear. Although it may present with unilateral
hearing loss, the hearing loss is usually
conductive (ie, BC > AC, lateralization to the
affected ear), often with otorrhea and a pearly white mass in the middle ear.

Eustachian tube dysfunction may present with asymmetric hearing loss, but it is
conductive rather than sensorineural. It also is accompanied by pain, popping sounds,
and middle ear effusion.
Herpes zoster oticus (reactivation of varicella zoster virus involving CN VIII) can cause
hearing loss and imbalance. However, it also presents with severe pain and a
vesicular rash. Facial paralysis (CN VII) rather than numbness (CN V) is also
characteristic.

Although Ménière disease causes asymmetric sensorineural hearing loss, it is


characterized by discrete episodes of spinning vertigo, hearing loss, and aural
fullness. Subtle imbalance and facial numbness are not typical.

Vertebrobasilar insufficiency could cause imbalance. However, isolated hearing loss


is rare; other classic symptoms include diplopia, perioral (rather than hemifacial)
numbness, dysarthria, and ataxia.
EAR TRAUMA A patient with severe ear
pain during a flight who
now has persistent
hearing loss most likely
sustained barotrauma
of the ear complicated
by rupture of the
tympanic
membrane (TM).

Under normal
conditions,
the eustachian
tube opens intermittently, typically during swallowing or yawning. Failure to open
adequately (eg, functional obstruction due to URI) leads to a pressure difference
between the middle ear and the outside environment. This pressure difference is more
pronounced when there are significant shifts in barometric pressure (eg, during a
plane's ascent and descent), which can lead to stretching of the TM, causing ear pain
and hearing loss.

In severe cases, the TM can rupture, which rapidly equalizes the pressure difference
and relieves the stretching; this relieves the pain but may also result in minor bleeding
(eg, drop of blood on the finger). Patients can have persistent hearing loss because of
the TM rupture. Barotraumatic TM injuries typically heal spontaneously within a few
weeks, so reassurance and follow-up is appropriate initial management.

……
(Aural irrigation can be used to treat cerumen impaction, which can cause unilateral
hearing loss. However, cerumen impaction is typically neither painful nor associated
with flying or URI symptoms.

MRI of the internal auditory canal can detect a vestibular schwannoma, which can
cause unilateral hearing loss. In general, hearing loss due to a vestibular schwannoma
is slowly progressive, but some patients have acute hearing loss. However, a
vestibular schwannoma is not painful, would not bleed, and is not associated with
flying.

**Oral glucocorticoids are often used in patients with idiopathic sudden


sensorineural hearing loss (SSNHL). The cause of SSHL is unknown, but it is not
associated with flying, ear pain, or bleeding from the ear.
Oxymetazoline used prior to air travel might reduce URI-induced obstruction of the
eustachian tube and reduce the risk of barotrauma, but it is unlikely to improve a TM
rupture after it has occurred.
Tympanoplasty is used to repair persistent TM perforations. However, an acute
barotraumatic TM rupture often heals in a few weeks without surgical intervention.

OTOSCLEROSIS
Otosclerosis

*** • Younger (early to mid-30s) Caucasian patients


Epidemiology • More common in women
• Autosomal dominant (AD) with incomplete penetrance

Pathophysiology • Imbalance of bone resorption & deposition → stiffening of STAPES

• Progressive CHL
Clinical • PARADOXICAL improvement in speech discrimination in noisy
manifestations environments
• ± Reddish hue behind tympanic membrane

• Amplification (eg, hearing aids)


Management • Surgery (eg, stapes reconstruction)

A patient has hearing loss that paradoxically improves (with regard to speech
discrimination) in noisy environments, consistent with conductive hearing
loss (CHL). Given her young age and family history of hearing loss at a young age, this
presentation suggests otosclerosis.

Otosclerosis is inherited in an AD pattern with incomplete penetrance. It results from


an imbalance of bone resorption and deposition, causing stiffening and fixation of
the ossicular chain (malleus, incus, stapes), primarily the stapes. Because the ossicles
function as a lever connecting the tympanic membrane to the oval window to amplify
sound energy, this stiffening results in
CHL. As with other causes of CHL, patients
with otosclerosis can have paradoxical
improvement of speech understanding in a
noisy environment (a phenomenon called
the paracusis of Willis), possibly because
CHL dampens the competing background
noise, thereby allowing speech to be heard
more clearly.

Ear examination is generally unremarkable,


although excessive bony resorption can
expose underlying blood vessels, leading
to a reddish hue sometimes seen behind
the tympanic membrane. Management involves hearing amplification or surgical
reconstruction of the stapes.

………..
Degeneration of neuronal cell bodies (which can occur in presbycusis) is seen in older
individuals and presents with bilateral, symmetric SNHL. Unlike CHL, SNHL is
characterized by worse speech understanding with increasing background noise.
A lamellated basement membrane is characteristic of Alport syndrome, which can
cause hereditary SNHL (not CHL) due to damage of the basement membrane in the
cochlea. It typically presents with recurrent hematuria in childhood.

A neoplastic growth involving CN VIII (vestibular schwannoma) would also cause a


unilateral hearing loss and is sometimes hereditary (eg, neurofibromatosis, type 2).
However, it would present with SNHL and a normal otoscopic examination (no
reddish hue).

OSTEONECROSIS
Bisphosphonate-related osteonecrosis of the jaw

**** • High-dose, parenteral bisphosphonates


Risk • Dental procedures (eg, extractions, implants)
factors • Concurrent glucocorticoid use
• Concurrent or previous malignancy

• Chronic, indolent symptoms


Clinical
• Mild pain, swelling
features • Exposed bone, loosening of teeth, pathologic fractures

• Oral hygiene
Management • Antibacterial rinses
• Antibiotics and debridement as needed

A patient has a nonhealing dental extraction site associated with local swelling, mild
pain, and exposed bone. Given her treatment of osteoporosis with zoledronic acid,
these findings most likely represent bisphosphonate-related osteonecrosis of the
jaw (ONJ), which is likely caused, at least in part, by impaired bone remodeling. It is
MC in patients receiving high-dose parenteral bisphosphonates in the context of
cancer therapy, but it can also occur in those taking bisphosphonates for
osteoporosis.

ONJ classically presents as an area of exposed, necrotic bone following tooth


extractions or other dental procedures. It may persist for extended periods (months
to years) and is often asymptomatic for much of the course. The mandible is more
commonly affected than the maxilla.

Treatment is largely supportive, including careful oral hygiene and antibacterial rinses;
oral antibiotics and limited debridement are used only when needed.

Because the course can be intractable, some experts advise dental consultation and
completion of any anticipated dental procedures prior to initiation of
bisphosphonate therapy.
…….
Both aphthous ulcers and herpetic stomatitis present with painful oral
ulcers. However, these conditions cause superficial ulcerations without exposed bone
that heal spontaneously, usually within 7-10 days.
Mucormycosis (acute invasive fungal sinusitis) can present with pain and bone
destruction. However, it occurs almost exclusively in immunocompromised patients
(eg, those with uncontrolled DM) and presents with rapidly progressive fever, facial
pain, nasal congestion, and changes in vision or mentation.

Periodontal abscess can cause pain and swelling of tissues surrounding the affected
tooth, but symptoms are typically acute. Patients usually have associated fever and
lymphadenopathy.

Like ONJ, SCC can present as a nonhealing oral ulceration, and biopsy is often
required for definitive diagnosis. However, this patient has no other clinical features
(eg, cervical lymphadenopathy) or risk factors (eg, tobacco, heavy alcohol use) for oral
cancer. Her history of bisphosphonate use makes ONJ much more likely.

Interpretation of Rinne & Weber tests

Rinne result Weber result

Normal AC > BC in both ears Midline

BC > AC in affected ear, AC


CHL Lateralizes to affected ear
> BC in unaffected ear

Lateralizes to unaffected ear,


SNHL AC > BC in both ears
away from affected ear

BC > AC in affected ear, AC Lateralizes to unaffected ear,


Mixed HL
> BC in unaffected ear away from affected ear

AC = air conduction; BC = bone conduction.


Hearing loss is classified as either conductive (impaired transmission of sound to the
inner ear) or sensorineural (involving the cochlea or auditory nerve), which can be
distinguished with tuning fork examination. The Rinne test compares air conduction
(tuning fork near the ear) versus bone conduction (tuning fork on the mastoid bone).
With normal hearing, air-conducted sound is perceived as louder than bone-conducted
sound. The Weber test is performed by placing the tuning fork on the middle of the
forehead equidistant from both ears. The sound carried by bone conduction in this
manner is normally heard equally in both ears; vibration heard louder in one ear is
abnormal.

• SNHL causes lateralization to the unaffected ear because the inner ear is
unimpaired on that side and therefore can better hear the sound.
• CHL causes lateralization to the affected ear because the conductive deficit
masks the ambient noise in the room, allowing the sound to be better heard.

CHL can be caused by disorders of the external auditory canal (eg, cerumen impaction,
otitis externa), tympanic membrane (eg, perforation), middle ear space (eg, otitis
media, cholesteatoma), or the ossicular chain (eg, otosclerosis).

This young woman with conductive hearing loss and a normal otoscopic examination
likely has otosclerosis. Otosclerosis results from an imbalance of bone resorption and
deposition that leads to stiffening and ultimately fixation of the stapes, which
dampens the transmission of sound energy from the tympanic membrane to the
cochlea. Otosclerosis may progress during pregnancy. It is inherited in an
autosomal dominant pattern with incomplete penetrance. Treatment involves
hearing amplification or surgical reconstruction of the stapes.

………
Although chronic otitis media may cause conductive hearing loss, it is typically
accompanied by ear pain and/or fullness, which were not reported by this patient.
Examination of the tympanic membranes would show retraction of the tympanic
membrane or fluid in the middle ear.
Aminoglycoside antibiotics can cause sensorineural (rather than conductive) hearing
loss. In addition, they are more commonly used in pyelonephritis rather than cystitis.

Although Ménière disease (episodic vertigo, fluctuating hearing loss), presbycusis


(progressive hearing loss in advanced age), and vestibular schwannoma (CN VIII tumor)
would all have normal otoscopic examinations, they would cause SNHL rather than
conductive.

BRANCHIAL
Pediatric neck masses
CLEFT CYST
• Tract between foramen cecum & base of anterior neck
Thyroglossal duct Midl
• Cystic, moves with swallowing or tongue protrusion
cyst ine
• Often presents after URTI

• Cystic mass with trapped epithelial debris


Mid
Dermoid cyst • Occurs along embryologic fusion planes
line
• No displacement with tongue protrusion

• Tract may extend to the tonsillar fossa (2nd branchial


Late
Branchial cleft cyst arch) or pyriform recess (3rd branchial arch)
ral
• Anterior to the SCM muscle

Reactive Late • Firm, often tender


adenopathy ral • Multiple nodules

Mycobacterium Late • Necrotic lymph node


aviumlymphadenitis ral • Violaceous discoloration of skin
• Frequent fistula formation

Post
Cystic hygroma • Dilated lymphatic vessels
erior

A child with a lateral, cystic neck


mass has a branchial cleft
cyst (BCC). The branchial apparatus is
an embryologic structure, composed
of paired arches separated by clefts,
which forms many face and neck
structures. Abnormalities in the
embryologic development of these
structures can lead to cysts, sinuses,
fistulae, and/or cartilaginous
remnants along the paths of
development.

Abnormalities most often arise from the second branchial


arch, which forms structures in the face and upper neck.
Therefore, BCCs are most often located between the
internal and external carotid arteries, anterior to the SCM
muscle and inferior to the mandible. Surgical resection of
the entire cyst and any associated fibrous tract remnants is
curative.

Similar to a thyroglossal duct cyst (TDC), which is also


formed from incomplete involution of an embryologic
remnant, a BCC is often detected when it
becomes secondarily infected after an URI, leading to
erythema, tenderness, and sometimes drainage of fluid from a sinus tract. However,
a TDC is located
in the midline
because the
path of thyroid
descent is from
the base of
tongue to its
location in the
midline lower
neck.
……………

Actinomyces lymphadenitis can present as a submandibular mass with a draining


sinus tract. However, it typically occurs after dental infections or trauma in
immunosuppressed patients and the sinus tracts drain sulfur granules.
A cystic hygroma is formed from dilated lymphatic
vessels and presents as a lateral, cystic neck
mass. However, it occurs most commonly in the
posterior triangle of the neck and is typically
diagnosed at birth or on prenatal ultrasound.

A laryngocele is an outpouching of the laryngeal


mucosa that can be congenital or acquired. If it
protrudes through the thyrohyoid membrane, it
can present as a lateral neck mass. It characteristically enlarges with Valsalva
maneuver due to inflation with air. Acquired laryngoceles are classically seen in
glassblowers or trumpet players due to repeated, intense oropharyngeal pressure.

CSF
Cerebrospinal fluid (CSF) rhinorrhea
RHINORRHEA
• Accidental trauma (MC)
Etiology • Surgical trauma (eg, sinus surgery)
• Nontraumatic (eg, elevated ICP) ?!?!

Clinical
• Unilateral watery rhinorrhea with salty or metallic taste
presentatio • Possible complication: meningitis
n

• Test for CSF-specific proteins (β-2 transferrin, β-trace protein)


Evaluation • Imaging (with intrathecal contrast)
• Endoscopy (± intrathecal fluorescein dye)

• Bed rest, head of bed elevation, avoidance of straining


Manageme
• Lumbar drain placement
nt • Surgical repair

CSF = cerebral spinal fluid.

Clear, unilateral rhinorrhea that increases at times of relatively increased ICP (eg,
bending over, bowel movements) is suspicious for cerebrospinal fluid (CSF)
rhinorrhea. Patients also often report a salty or metallic taste. Most cases occur
after head trauma, especially with fracture of the skull base (eg, cribriform plate,
temporal bone). CSF rhinorrhea can be evident immediately after the trauma or may
have a delayed presentation (days to months) in which rhinorrhea is noted after
acute edema resolves.
Testing the nasal discharge for CSF-specific proteins (eg, beta-2 transferrin, beta-trace
protein) is diagnostic. Imaging or endoscopic nasal examination can be used to
localize the precise site of the defect.
Patients typically require inpatient management for bed rest, head-of-bed elevation,
and frequent neurologic evaluation because they are at risk for meningitis due to
nasal flora contamination of the CSF. If the CSF rhinorrhea does not resolve with these
measures, further interventions (eg, lumbar drain, operative repair) are necessary.

………

A personal or family history of atopy may suggest allergic rhinitis, which would present
with bilateral (rather than unilateral) rhinorrhea and pale or bluish, boggy, edematous
turbinates. Accompanying symptoms of sneezing, itching, and watery eyes are also
typically present.

Intranasal cocaine and nasal decongestants cause vasoconstriction. Overuse can


result in bilateral (rather than unilateral) rhinorrhea and severe "rebound" nasal
congestion (eg, rhinitis medicamentosa). Examination often shows swollen,
erythematous turbinates. Tissue destruction from vasoconstriction caused by intranasal
cocaine typically occurs at the septum (resulting in septal perforations) rather than at
the skull base.
Patients can be exposed to a variety of substances at work that result in allergic,
irritant-induced, or corrosive rhinitis, leading to rhinorrhea. However, such rhinorrhea
would be bilateral, and symptoms would increase during times of exposure (ie, at
work) rather than with head positioning or straining.

DEVIATED NASAL Complications are common following rhinoplasty, and up to one in four rhinoplasties
SEPTUM may need revision. Common complications include patient dissatisfaction, nasal
obstruction and epistaxis. Those that involve the nasal septum are less common but
more serious. The septum is made up of cartilage and has poor blood supply
contrasting sharply with the rich anastomosing blood supply of the nasal sidewall. The
underlying cartilage relies completely on the overlying mucosa for nourishment
by diffusion. Because of the poor regenerating capacity of the septal cartilage, trauma
or surgery on the septum may result in septal perforation. The typical postoperative
presentation is a whistling noise heard during respiration. Following nasal surgery,
septal perforation is typically the result of a septal hematoma though a septal
abscess may also be the cause. Additional conditions that can cause septal perforation
are self-inflicted trauma (nose picking), syphilis, tuberculosis, intranasal cocaine
use, sarcoidosis and GPA (Wegener's).
……….

Nasal polyps are usually seen in patients with asthma and allergic disorders but may
also occur in patients with other inflammatory conditions of the nasal mucosa. They
may cause chronic nasal obstruction and should be surgically removed in
symptomatic patients.
Foreign bodies are common in children. On presentation, patients will have nasal
obstruction and may have a foul odor, halitosis and nasal bleeding. Following
surgery, a retained foreign body such as nasal packing most classically would cause
toxic shock syndrome (TSS).

Allergic rhinitis commonly presents with rhinorrhea, nasal pruritus, cough and
occasionally dyspnea. On examination, the nasal mucosa is edematous and pale, and
polyps may be present.

Nasal furunculosis results from staphylococcal folliculitis following nose picking or


nasal hair plucking. It is potentially life threatening as it can spread to the cavernous
sinus. Patients complain of pain, tenderness and erythema in the nasal vestibule.

BONE TUMOR This patient has a chronic mass on his hard


palate consistent with torus palatinus (TP), a
*** benign bony growth (ie, exostosis) located at
the midline suture of the hard palate. It is
thought to be caused by both genetic and
environmental factors and is more common in
women and Asian individuals. TP can
be congenital or develop later in life. Similar
lesions at the lingual surface of the mandible
are termed "tori mandibulari."
TPs are usually <2 cm in size but can gradually enlarge over time. They are typically
asymptomatic and are frequently ignored by the patient but noted by clinicians (or
family members) when examining the mouth for unrelated reasons. However, the thin
epithelium overlying the bony growth may ulcerate with minor trauma of the oral
cavity and heal slowly due to poor vascular supply. The diagnosis is usually obvious on
clinical grounds.

Surgery is indicated for patients in whom the mass becomes symptomatic, interferes
with speech or eating, or causes problems with the fitting of dentures later in life.

………

Oral SCC can be caused by smoking and excessive alcohol use; it can also be a late
complication of HPV infection. However, oral cancer more commonly presents as a
progressive ulcerating lesion located eccentrically on soft structures (eg, tongue, lips)
rather than the center of the hard palate. Rarely, paranasal sinus infections with
atypical organisms (eg, blastomycosis, mucormycosis) can erode into the palate, but
this typically occurs in patients with immune suppression (eg, uncontrolled DM) and
would present acutely.
Chronic complications of palatal fracture include dental malocclusion, TMJ
dysfunction, and complicated wounds. However, palatal fractures are rare, are usually
associated with other obvious skull fractures, and are not a major cause of TP.
Maxillofacial vascular malformations are congenital abnormalities that are present at
birth. They present as soft, dusky lesions that grow with the patient and may bleed
intermittently.

//////////////////////////////////DERMATOLOGY//////////////////////////////////
NONMELANOMA Basal cell carcinoma (BCC) is the MC form of skin
SKIN CANCER cancer in the US, accounting for approximately 75%
of all skin cancers. Possible features of BCC include:
****
• Persistent open sore that bleeds, oozes,
or crusts (A)
• Reddish patch or irritated area (superficial
BCC; B)
• Pearly or translucent nodule with pink, red, or white color (nodular BCC; C)
• Elevated or rolled border with central ulceration (D)
• Pale scar-like area with poorly defined borders (E).

The cancer usually remains local and only rarely spreads to distant parts of the body,
but it may continue to grow and invade nearby tissues and structures, including the
nerves, bones, and brain.

A B C D

E
The treatment of BCC varies, depending on the size, depth, and location of the
cancer. Low-risk lesions on the trunk or extremities can be managed with
electrodessication and curettage (ED&C). However, ED&C is not recommended for
higher-risk lesions or for
BCC on the face, where the
resulting scarring and
hypopigmentation are
undesirable.

Surgical excision is advised


for this patient's large facial
lesion.

Nodular BCC on the trunk


or extremities may be easily
managed with standard
surgical excision, typically
with 3-5 mm margins.
However, Mohs micrographic surgery is employed more often for the face. Mohs
surgery is characterized by sequential removal of thin skin layers with microscopic
inspection to confirm that the margins have been cleared of malignant tissue. This
technique currently has the highest cure rate for BCC and provides the least
disruption to surrounding tissues, making it ideal for delicate or cosmetically
sensitive areas (eg, perioral region, nose, lips, ears).
…………

Radiation therapy is occasionally used to treat BCC in patients who are unable to
undergo surgical excision. It is otherwise not considered first-line management.
Systemic CTX is used in patients with widely metastatic BCC. However, this
occurrence is rare and few patients with BCC require systemic treatment.
Low-risk SUPERFICIAL (image B) BCC may be managed with topical therapy using
either 5-FU or imiquimod. These agents may also be used in field therapy for patients
with multiple actinic keratoses. However, they are less effective in NODULAR BCC
and are not recommended.
This patient has a cutaneous squamous
cell carcinoma (SCC). SCC typically
forms an enlarging nodule in sun-
exposed areas. The lesions often
become keratinized (with a thickened,
rough surface) or ulcerate with crusting
and bleeding. In addition, SCC can
display early perineural invasion,
causing regional neurologic
symptoms (eg, numbness,
paresthesias).
Risk factors for SCC include sunlight exposure, fair skin, chronic inflammation or scar
formation, and ionizing radiation exposure. SCC is especially common in patients who
have a history of organ transplant and are on chronic immunosuppressive therapy.

SCC in immunosuppressed patients is typically more aggressive, with an increased


risk of local recurrence and regional metastasis.

……….
Basal cell carcinoma (BCC) characteristically presents as a pearly, flesh- or pink-colored
nodule with telangiectatic vessels and is usually found on the head or neck. BCC is the
MC skin malignancy in the general population; however, SCC occurs more
commonly among immunosuppressed patients and more commonly on the
extremities, and it is more likely to cause neural
invasion.
 Disseminated cryptococcosis is usually seen in
immunosuppressed patients. The skin is the MC site. It
presents as multiple, discrete, flesh- to red-
colored papules of varying size with slight central
umbilication.

Kaposi sarcoma is most commonly seen in the setting of coinfection with HIV and
HHV-8. The lesions typically begin as papules and later develop into plaques or
nodules. The color can change from light brown to violet, and patients often have
multiple lesions.
Melanomas are also seen with increased frequency in transplant patients. They are
most often pigmented and are characterized by asymmetry, irregular border,
variegated color, diameter >6 mm, and evolution in size and appearance over time
(ABCDE).

A patient most likely has angiosarcoma secondary to previous breast cancer therapy
and should undergo biopsy for diagnostic confirmation. Angiosarcomas are relatively
rare malignant tumors derived from the internal lining of blood vessels or lymphatic
vessels. PRIMARY angiosarcoma can occur anywhere in the body (eg, liver, breast),
but angiosarcoma SECONDARY to breast cancer therapy is typically confined to the
skin.
The majority of patients received localized radiation therapy, which is the strongest
risk factor. In addition, patients with chronic axillary lymphedema, often precipitated
by surgical lymph node resection, are at increased risk.

Patients with secondary angiosarcoma most commonly present with multiple


ecchymoses or purpuric masses on the skin of the breast, axilla, or upper arm 4-8
years following completion of breast cancer therapy. Lymphedema (nonpitting
edema) is often present in the ipsilateral arm, and the skin of the breast may have a
dimpled (peau d'orange) appearance. As with most sarcomas, early hematologic
metastasis is common, and patients may also have signs of metastatic disease (eg,
dyspnea due to lung involvement). Secondary angiosarcomas are typically high-grade
on pathology and generally carry a poor prognosis (worse than primary
angiosarcomas). However, when possible, surgical resection is potentially curative.

……….

Systemic antifungal therapy (eg, itraconazole, amphotericin) is appropriate treatment


for blastomycosis. Skin involvement with violaceous warts or ulcers is common in
patients with systemic blastomycosis; however, respiratory symptoms (eg, dyspnea,
cough) are typically present as well.

A low-potency topical steroid cream (eg, hydrocortisone) is useful in the management


of psoriasis. Psoriasis plaques are typically erythematous with silvery scale (rather than
purpuric) and occur on the extensor surfaces of the extremities.

Oral valacyclovir can reduce the incidence of postherpetic neuralgia when given
within 72 hours of the onset of herpes zoster (shingles) symptoms. However, the rash
of herpes zoster is typically vesicular in a dermatomal pattern rather than purpuric.
A patient, who has an outdoor occupation with significant sun exposure (farming) and
who has developed a persistent, indurated, ulcerating lower lip lesion, has typical
features of squamous cell carcinoma (SCC). SCC is the MC malignancy of the lip,
with 95% of cases occurring in the lower
lip vermilion, likely due to higher exposure
to sunlight. In addition to ultraviolet (eg,
sunlight) exposure, other risk factors for
SCC include fair skin tone, tobacco use
(smoking or chewing), heavy alcohol use,
chronic inflammation or scar formation,
and immunosuppression.

Diagnosis of SCC is confirmed


with biopsy. Typical pathologic findings
include invasive cords of squamous cells
with keratin pearls. Prognosis is usually
very good for cutaneous SCC, with curative resection seen in >90% of patients. Most
lesions do not metastasize, but untreated SCC can cause extensive local destruction
and may eventually spread to local lymph nodes or distant structures.

Adverse prognostic features include larger size, deeper invasion, and involvement of
regional LNs.

…….

 A Tzanck preparation of vesicular fluid can identify the


characteristic giant cells of herpes simplex virus (HSV) infection. HSV
can cause recurrent labial or peri-oral ulcerating lesions, but these
usually resolve within 2 weeks.

Granulomatous ulcers can be seen in a number of disorders, such as tertiary syphilis,


tuberculosis, or disseminated fungal disease. However, most patients with these
conditions would have additional systemic (eg, fever, night sweats in tuberculosis) or
local (eg, genital lesions in syphilis) symptoms or known immunocompromised states.
Basal cell carcinoma (BCC) is characterized histologically by invasive clusters of spindle
cells surrounded by palisaded basal cells. Like SCC, BCC can form ulcerating lesions and
is associated with sun exposure. However, BCC of the lip is much less common than
SCC and typically affects the upper rather than lower lip.
Aphthous ulcers (canker sores) are characterized by shallow, fibrin-coated ulcerations
with underlying mononuclear infiltrates. They are recurrent, self-limiting ulcerations
of the oral cavity and do not affect surfaces covered by keratinized stratified squamous
epithelium.
This patient has a severe burn associated
with extensive scar formation and a chronic
non-healing wound. Squamous cell
carcinoma (SCC) is most often associated
with ultraviolet (sun) exposure but may also
arise within chronically wounded, scarred,
or inflamed skin. SCC arising within a burn
wound is known as a Marjolin ulcer. In
addition, SCC has been seen within the skin
overlying a focus of osteomyelitis,
radiotherapy scars, and venous ulcers. SCC
arising within chronic wounds tends to be
more aggressive, so early diagnosis with biopsy is important for preventing metastatic
disease.

……….
 BCC usually appears as pearly telangiectatic
papules, often with central ulceration. Like SCC,
BCC and melanoma are associated with chronic sun
exposure, but they are less likely to arise in scars
or burns.

 Cutaneous T-cell lymphoma, sometimes


referred to as mycosis fungoides, most
commonly appears as scaly, pruritic patches
or plaques.

Basal cell carcinoma (BCC)

• Sun/ultraviolet light
• Fair skin, light hair/eye color
Risk factors • Ionizing radiation
• Chronic arsenic exposure

• Slow-growing: locally invasive, rare metastasis


Clinical
• Pink or flesh-colored, pearly papule, ± rolled border
features • ± Central ulceration (rodent ulcer)
Diagnosis • Excisional biopsy with narrow margin

Nodular BCC, which represents up to 80% of cases, is characterized by


a pearly or translucent nodule with an elevated or rolled border and central
telangiectasias.

BCC rarely metastasizes but can invade nearby tissues (eg, nerves, bone). Nodular BCC
is typically managed with surgical excisional biopsy with narrow margins (eg, 3-5
mm for BCC), which is usually adequate for complete removal of the neoplastic
tissue. Excision with narrow margins carries a low (<5%) risk of recurrence while
avoiding the sacrifice of large amounts of normal tissue and minimizing the risk of
wound complications. For the face and other delicate or cosmetically sensitive
areas, Mohs micrographic surgery (sequential removal of thin skin layers with
microscopic inspection to confirm that the margins are clear of malignant tissue) is
more often performed to ensure complete removal with good cosmetic results.

………..

Incisional biopsy, in which only a representative portion of a lesion is removed, is


typically used for large lesions for which excision is impractical or for lesions with a
low risk of malignancy. Complete excision of this patient's lesion is easily accomplished
and preferred for definitive management.

Intralesional corticosteroid injections are typically used for inflammatory or


autoimmune lesions (eg, alopecia areata, discoid lupus erythematosus), psoriasis,
or keloids. They are not indicated for possible neoplastic lesions.

Although wide-margin (eg, >5 mm) excision is indicated for melanoma and high-risk
or recurrent nonmelanoma skin cancer, it is associated with greater risk of wound
complications and significant scarring and is not necessary for uncomplicated BCC.
Topical salicylic acid is useful for treating common warts and many other benign
conditions (eg, acne, psoriasis, corns/calluses). It is occasionally considered in
combination with topical 5-FU for low-risk BCC but has a higher risk of recurrence;
excision is generally preferable for definitive management.

Squamous cell carcinoma (SCC) of skin

• Sun/UV light, ionizing radiation exposure


Risk factors • Immunosuppression
• Chronic scars/wounds/burn injuries

• Scaly plaques/nodules
• ± Hyperkeratosis or ulceration
Clinical features • Neurologic signs (if perineural invasion)
• SCC in situ: slow-growing, red, scaly patches/plaques

Diagnosis • Biopsy: dysplastic/anaplastic keratinocytes


SCC = squamous cell carcinoma; UV = ultraviolet.

Squamous cell carcinoma (SCC) should be suspected


in patients with a rough, scaly nodule or nonhealing,
painless ulcer that develops in the setting of a scar or
chronic inflammatory lesion. Sun exposure is the MCC
of SCC, but other risk factors include radiation
exposure; immunosuppression; and chronic wounds,
burns, or scars.
The diagnosis of SCC should be confirmed with skin
biopsy (punch, shave, or excisional) that includes the
deep reticular dermis to assess the depth of invasion.
Small or low-risk lesions are usually managed with
surgical excision or local destruction (eg,
cryotherapy, ED&C); lesions that are high risk or
located in cosmetically sensitive areas should be
referred for Mohs micrographic surgery.
If left untreated, SCC may eventually cause extensive local destruction and lymphatic
or distant metastases. SCC is not usually metastatic at the time of diagnosis, but SCC
arising in a wound or burn (termed a Marjolin ulcer) is associated with an increased
risk of metastasis.
……..

 Intralesional glucocorticoids are used


for symptomatic keloids, which may form
in the setting of an established
scar. Keloids are not usually ulcerated.

Valacyclovir is used to treat infections due to the herpes family of viruses (eg, varicella
zoster, herpes simplex). Zoster skin lesions are often painful and characterized by
vesicles that quickly ulcerate; episodes are usually of short duration and will generally
resolve prior to 2 months.
Chronic ulcerated lesions are frequently colonized with bacteria. However, topical
antibiotics are only rarely
indicated. They are typically used to
prevent infection in scrapes or
burns.
 Fluorouracil cream is most
commonly used for actinic keratosis,
dry, scaly papules on sun-exposed
areas that may occasionally progress to
SCC. It is also used in Bowen disease
(SCC in situ) and other low-risk SCC
lesions.

MELANOMA
Clinical features of melanoma (ABCDE)

• Asymmetry: When bisected, the 2 sides are not


identical
• Border irregularities: Uneven edges, pigment fading
off
• Color variegation: Variable mixtures of brown, tan,
black & red
• Diameter: ≥6 mm
This patient developed a nodular, • Evolving: Lesion changing in size, shape, or color;
new lesion
hyperpigmented lesion on an
area of skin that likely gets
excessive sun exposure while he surfs, raising strong suspicion for nodular
malignant melanoma.

In contrast to superficial spreading melanoma, which grows horizontally along the


dermal layer, nodular melanoma grows vertically. Therefore, it often presents with
few of the classic ABCDE criteria.
Most cases are marked by a darkly pigmented lesion with uniform color and
symmetric borders; however, suspicion is generally raised due to the presence of ≥1
of the following features:

• The "ugly duckling sign" - patients with several pigmented cutaneous lesions
(nevi) usually have lesions that all appear somewhat similar. When a single
lesion looks significantly different from the others this is called the "ugly
duckling sign"; it has a high sensitivity (up to 90%) for melanoma.
• Elevation from surrounding skin (eg, nodular, pedunculated)
• Firm to palpation
• Continuous growth over a month

The diagnosis is generally confirmed by full-thickness excisional biopsy.


………….

Dermal angiosarcoma (left


image) is a rare disorder that
most commonly occurs in
patients with prior radiation
therapy. Cherry
hemangioma (right image) is a
common, benign lesion
composed of congested
capillaries and postcapillary
venules in the papillary dermis. These disorders are usually
heavily vascular (eg, bright red).

 BCC is a common skin malignancy that typically


appears as an enlarging fleshy nodule with ulceration. It
is associated with sun exposure and is most common on
the face, neck, and extremities.

 Keratoacanthomas are nonpigmented cutaneous


tumors that usually present as dome-shaped nodules
with a central keratinous plug.

 Seborrheic keratosis is a benign pigmented


lesion with a well-demarcated border and a
velvety or greasy surface. It can be nearly flat,
but thickened lesions are often described as
having a "stuck-on" appearance.
Clinical features of melanoma (ABCDE)

• Asymmetry: When bisected, the 2 sides are not identical


• Border irregularities: Uneven edges, pigment fading off
• Color variegation: Variable mixtures of brown, tan, black & red
• Diameter: ≥6 mm
• Evolving: Lesion changing in size, shape, or color; new lesion

Early diagnosis is critical


in melanoma as survival
rates rapidly decrease with
increased tumor thickness
and depth of
invasion. Historical and
examination findings are
used in the evaluation of
moles to determine which
lesions require excision and
which can be safely
observed. Significant
melanoma risk factors
include prior personal or
family (≥2 members)
history of melanoma,
numerous (>100) nevi, prior atypical/dysplastic nevi, fair skin, and history of severe
sunburns.

Gross features favoring the diagnosis of melanoma


include Asymmetry, Border irregularities, Color
variegation (ie, color differences within a single
lesion), Diameter ≥6 mm, and change in appearance
over time (Evolving lesion). Coloration of melanoma
can be highly variable. Lesions can be pigmented or
unpigmented and various shades of brown, black,
red, blue, or pink. Color variegation in particular
strongly suggests malignancy.

……….
Keratoacanthomas are cutaneous tumors that usually present as dome-shaped
nodules with a central keratinous plug. They are generally benign, although rare cases
with malignant transformation and metastasis have been reported.
 Lipomas present as slowly enlarging, mobile subcutaneous
masses. They have a soft or rubbery texture and are often
golf-ball size or larger by the time they are brought to clinical
attention. Lesions with the typical appearance of a lipoma are
rarely malignant and can be safely observed without
biopsy.

 Ring-shaped inflammatory skin lesions


with peripheral scaling are characteristic
of tinea corporis.

 Seborrheic keratosis is a benign pigmented lesion with a


well-demarcated border and a velvety or greasy surface. It can
be nearly flat, but thickened lesions are often described as having
a "stuck on" appearance. An explosive onset of multiple
seborrheic keratoses (Leser-Trélat sign) can indicate internal
malignancy, but isolated lesions are benign.

The initial clinical assessment of pigmented skin lesions should emphasize identifying
those that could be melanoma and therefore warrant biopsy. Biopsy is appropriate in
any of the following situations:

• The lesion has features that suggest melanoma using a validated predictive rule
(eg, lesions displaying ≥1-2 of the ABCDE criteria).
• The lesion has inflammatory changes; itching, crusting, or bleeding; or
sensory changes (eg, paresthesia).
• The lesion is significantly different in appearance
from other lesions on the same patient (ugly
duckling sign).
• The lesion develops thickening or nodularity,
which may indicate vertical growth and increase
metastatic risk.

This patient's lesion is asymmetrical with a somewhat


irregular border and notable color variegation (dark
brown, light brown, and pink). The associated itching
further raises the risk, as does the apparent nodularity.
To rule out melanoma, full-thickness excisional
biopsy with 1- to 3-mm margins of normal tissue is
recommended. Partial excision (eg, deep shave biopsy,
central or peripheral punch biopsy) is generally only
considered for very large lesions or lesions in locations
where complete excision may be problematic (eg, ear,
face); partial excision may not allow adequate sampling for diagnosis or accurate
measurement of the lesion's depth (the most important prognostic indicator in
melanoma).

……

In-situ destruction via cryotherapy or topical fluorouracil is indicated for treatment of


actinic
keratosis. Actinic keratosis presents as
small, roughened papules in sun-
exposed areas (eg, scalp, face, hands)
and typically is not pigmented.

 Periodic surveillance is
recommended for patients with
numerous or atypical nevi that do not
have suspicious features. Suspected
melanomas warrant tissue confirmation without delay.

***** Visual assessment of melanoma

• Asymmetry
• Border irregularity
ABCDE criteria
• Color variation (within lesion or compared to other lesions)
(≥1-2 is suspicious) • Diameter ≥6 mm
• Evolving appearance over time
7-point checklist • Major criteria: change in size (D), shape (E), or color (C) (C-E)
(≥1 major or ≥3 • Minor criteria: size ≥7 mm, local inflammation,
minor is suspicious) crusting/bleeding, sensory symptoms

Ugly duckling sign • One lesion is significantly different from others on the patient

The ABCDE criteria (Table) are useful in visual inspection of pigmented skin lesions to
identify which may represent malignant melanoma. However, although the ABCDE
criteria can assist in evaluation of superficial spreading melanoma, they are less
effective in identifying nodular or atypical melanomas.

Nodular melanomas account for up to 30% of all melanomas but may not meet any
of the ABCDE criteria until they are relatively advanced. To address this shortcoming,
additional visual criteria should be considered in assessing risk of melanoma:

• In a patient with multiple pigmented lesions, a lesion that is substantially


different from the others (eg, shape, color) may represent melanoma ("ugly
duckling sign"). The "ugly duckling sign" has a sensitivity of up to 90% for
melanoma.
• Palpable nodularity often corresponds with vertical growth of a melanoma
and increases metastatic risk. The Breslow depth (distance from the epidermal
granular cell layer to the deepest visible melanoma cells) is the most important
prognostic indicator in malignant melanoma.
• Benign pigmented lesions are usually asymptomatic; biopsy should be
considered for moles that itch, bleed, or are associated with local sensory
symptoms (eg, tingling).

Although this patient's lesion does not have any of the ABCDE characteristics, she has a
pruritic, nodular lesion that is significantly different from her other pigmented lesions.
She should therefore undergo a full-thickness excisional biopsy with initial margins of
1-3 mm of normal tissue.

………..

 Actinic keratoses are roughened papules that


usually occur in sun-exposed areas (eg, face,
hands). They can be treated by destruction in situ
with cryotherapy or topical fluorouracil. Actinic
keratoses are rarely pigmented, and nodularity is
more consistent with melanoma.
 Periodic surveillance is recommended for
patients with numerous or atypical nevi, but
suspected melanomas warrant tissue confirmation
without delay.

Shave biopsy is not recommended for most cases


of suspected melanoma because partial removal
may not provide adequate tissue for diagnosis and
does not allow for accurate depth measurement.

KERATOACANTH
Keratoacanthoma
OMA
• Rapidly growing nodule with ulceration & keratin plug
Clinical features
• Often shows spontaneous regression/resolution

Clinical
• May resemble or progress to SCC
significance

Management • EXCISIONAL biopsy with complete removal of lesion


This patient has a keratoacanthoma, which
presents as a rapidly growing nodule with
ulceration and a central keratin plug. As in this
patient, keratoacanthomas frequently regress and
may resolve spontaneously over several
months. Keratoacanthomas are most common in
fair-skinned individuals and are often seen in
areas of UV exposure (eg, face, forearms) or prior
trauma.
Although keratoacanthomas often have a benign course, there may be a relationship
to SCC, with some lesions occasionally undergoing malignant transformation. For this
reason, keratoacanthomas usually warrant excisional biopsy to confirm the
diagnosis.
………

 BCC often presents as an ulcerated nodule on sun-


exposed skin. However, it typically grows slowly and does
not undergo spontaneous regression or form a central
keratin plug.

 A cutaneous horn is a keratinaceous projection from the


skin that can be seen in association with a number of
benign (eg, seborrheic keratosis) and malignant (eg, SCC)
lesions. Cutaneous horns are typically conical or elongated
and do not generally undergo rapid regression.
 A dermatofibroma is a small, firm, hyperpigmented
nodule, most often located on the extremities. The lesion
has a fibrous component that may cause dimpling in the
center when the area is pinched (dimple or buttonhole
sign), but ulceration and keratin plug formation are not seen.

 Molluscum contagiosum typically presents


as umbilicated nodules, but large ulcers and keratin
plug formation are not present.

PYODERMA
Pyoderma gangrenosum
GANGRENOSUM
• Begins with small papule or pustule
**** Clinical • Rapidly progressive, PAINFUL ulcer with purulent base & violaceous,
features irregular border
• Precipitation of ulceration at site of injury (pathergy)

• Peak onset age 40-60


Epidemiol
• Women > men
ogy
• Association with IBD, inflammatory arthritis (eg, RA), malignancy (e.g., AML)

• Exclusion of other causes of ulceration (eg, infection, malignancy, venous


Diagnosis ulvers)
• Skin biopsy: neutrophilic infiltrate

Treatment • Local or systemic glucocorticoids

This patient's skin lesion is consistent


with pyoderma gangrenosum (PG).
PG starts as an inflammatory papule,
pustule, or nodule and progresses to
form an expanding ulcer with a
purulent base and an irregular,
violaceous border. PG can present as
single or multiple lesions, usually on
the trunk or lower extremities. Nearly
30% of cases are triggered by local
trauma (pathergy).
Most patients with PG have an
associated systemic disorder. Common associations include IBD, inflammatory
arthropathies (eg, RA), and hematologic conditions (eg, acute myeloid leukemia
[AML]).
PG is diagnosed clinically after excluding other diagnoses (eg, infection, venous ulcers,
cutaneous cancers).

Skin biopsy of the ulcer margin is recommended and typically shows a mixed
cellular infiltrate with dermal and epidermal necrosis. Surgical debridement is
usually avoided due to the potential of inducing pathergy; most lesions respond to
treatment with local or systemic glucocorticoids.

……..

 Ecthyma gangrenosum is a skin


infection most frequently due to P
aeruginosa. It presents as
hemorrhagic pustules with
surrounding erythema that evolve
into necrotic ulcers and occurs
most commonly in the setting of
profound neutropenia and
bacteremia. Unlike in PG, patients
typically have signs of systemic
infection (eg, fever), and pain is
significantly less prominent.
Erythema nodosum resolve without scarring after 2-8 weeks.

Hidradenitis suppurativa occurs at the axilla, groin, or scalp. It presents


with inflammatory nodules or abscesses with purulent drainage and chronic scarring.

 Sporotrichosis is a fungal skin infection acquired


through breaks in the skin, typically from rose
bushes or other plant materials. It presents as a
small papule or nodule at the inoculation site,
usually on exposed skin of the hand or arm, which
then may enlarge, ulcerate, and lead to additional
lesions along the draining lymphatics. Pain, if
present, is mild.

DIABETIC FOOT Patients with long-standing DM who have poor glycemic control are at high risk
for diabetic foot ulcers. Risk is greatest in those with concomitant
**** diabetic neuropathy, which reduces pain sensation and identification of ulcer,
and PVD, which reduces immune recruitment and ulcer healing. Because diabetic foot
ulcers are usually colonized by a wide range of organisms (eg, aerobic gram-positive
cocci, enterococci, Pseudomonas, anaerobes), they are often complicated by adjacent
soft tissue infection and/or underlying osteomyelitis.
Although these infections often occur simultaneously, osteomyelitis can arise without
evidence of soft tissue infection due to neuropathy (diminished pain) and poor
peripheral blood flow (diminished erythema, warmth, and purulence).
Therefore, foot imaging (eg, x-ray, MRI) is generally recommended for all diabetic
foot ulcers that are:

• Deep (eg, exposed bone, positive probe-to-bone testing).


• Long-standing (eg, present >7-14 days).
• Large (eg, ≥2 cm).
• Associated with elevated ESR/CRP.
• Associated with adjacent soft tissue infection.

………..
Although hyperbaric oxygen therapy is sometimes used in patients with poorly
healing diabetic foot ulcers, evaluation for underlying osteomyelitis must occur
prior to treatment because antibiotics and debridement might be required.
A patient with diabetes, hypertension, and hyperlipidemia is at high risk for peripheral
vascular disease (PVD), which can be evaluated by angiography. However, initial testing
for PVD usually consists of noninvasive studies (eg, ABI testing). In addition, although
this patient might eventually need angiography and possibly stents, evaluation for
osteomyelitis is required first.
Although a deficiency of vitamin C or zinc can impair wound healing, supplementation
with these vitamins should not be considered prior to ruling out underlying bone
infection.

Although wound dressings are an important component of the treatment of diabetic


ulcers, topical antibiotics should not be administered until osteomyelitis is ruled
out. The presence of osteomyelitis necessitates systemic, not topical, antibiotics.
THERMAL BURN Severe burns disrupt the skin barrier and create an avascular, immunologically poor,
protein-rich substrate for the growth and proliferation of bacteria and
fungus. Immediately after a severe burn, gram-positive organisms (eg, S aureus) from
hair follicles and sweat glands dominate; after more than 5 days, most infections are
due to gram-negative organisms (eg, P. aeruginosa) or fungi (eg, Candida).

Wound infections are common, and patients with large surface area (>20%) burns are
at highest risk. The earliest sign is usually a change in appearance (partial-thickness
injury turns into a full-thickness injury) of the wound or the loss of a viable skin
graft. Burn wound sepsis can develop rapidly and is associated with some or all of the
following systemic findings:

• Temperature <36.5 C (97.7 F) or >39 C (102.2 F)


• Progressive tachycardia (>90/min)
• Progressive tachypnea (>30/min)
• Refractory HoTN (systolic blood pressure <90 mm Hg)

Oliguria, unexplained hyperglycemia, thrombocytopenia, and mental status


changes are also common.
Diagnosis requires quantitative wound culture (>105 bacteria/g of tissue) and biopsy
for histopathology (to determine tissue invasion depth).

Treatment involves empiric, broad-spectrum intravenous antibiotics (eg,


piperacillin/tazobactam, carbapenem) with the addition of potential coverage for
MRSA (eg, vancomycin) or MDR P. aeruginosa (eg, an aminoglycoside). Local
wound care and debridement are usually necessary.

……….
CO poisoning due to fire exposure can sometimes cause a delayed neuropsychiatric
syndrome (eg, AMS) but would not explain the hypothermia, laboratory abnormalities,
or change in wound appearance.
The metabolic rate drastically increases after a large surface area burn due to the
release of inflammatory mediators; this may cause increased basal temperature (to
38.5 C [101.3 F]), tachycardia, tachypnea, and hyperglycemia. However, this patient
has hypothermia, a change in wound appearance, reduced urine output, and confusion,
making sepsis more likely.

MI can complicate burn injuries, and this patient may be at risk for renal tubular
injury due to sepsis or HoTN. However, change in wound appearance,
thrombocytopenia, and hypothermia are more consistent with sepsis.

Intravenous fluids

Tonicity Fluid type Clinical use

0.9% (normal) saline (NS)


Volume resuscitation
(eg, hypovolemia, shock)
Lactated Ringer solution (LR)
Isotonic
Volume replacement, treatment
Albumin (5% or 25%)* of
SBP or HRS

Dextrose 5% in water (D5W)**


Free water deficit
0.45% (half-normal) saline
Hypotonic
Dextrose 5% in 0.45% (half-normal) saline
Maintenance hydration
(D5 in 45% NS)**

Hypertonic 3% (hypertonic) saline (HTS) Severe, symptomatic HoNa


HRS = hepatorenal syndrome; SBP = spontaneous bacterial peritonitis.
*Colloid solution; all other listed fluid types are crystalloid solutions.
**Dextrose 5% in water (initially slightly hypOtonic) & dextrose 5% in 0.45% saline (initially
hypERtonic) become markedly hypotonic due to metabolism of glucose.

Severe burn injuries often result in rapid development of intravascular volume


depletion and shock due to both insensible fluid loss (due to decreased skin integrity)
and massive fluid shift into the extravascular space (due to increased vascular
permeability). Patients with extensive burns require aggressive fluid resuscitation to
maintain tissue perfusion and reduce mortality. This is accomplished with the infusion
of an isotonic crystalloid solution (eg, normal saline, lactated Ringer solution [LR]),
which have similar osmolality to the blood (~288 mOsm/kg H2O).

Although these agents are equally effective at plasma volume expansion, LR,
a balanced fluid, is preferred for resuscitation in burn victims. It contains near-
physiologic levels of chloride, potassium, and calcium and includes sodium lactate, a
buffer that is hepatically metabolized to bicarbonate, which helps correct acidosis
and maintain normal blood pH.

Normal saline is considered an unbalanced fluid because its chloride concentration is


supraphysiologic (154 vs 103 mmol/L) and can cause a hyperchloremic metabolic
acidosis. It has also been associated with hypocoagulability.

…………….
Dextrose 5% in water (D5W) is initially a slightly hypotonic crystalloid solution that
quickly becomes markedly hypotonic with the metabolism of dextrose; it is
administered to patients with a free-water deficit (ie, hypernatremia). Half-normal
saline is a hypotonic crystalloid that is also useful in treating hypernatremia. It is also
used as a maintenance fluid in those unable to maintain adequate oral hydration and
may be used after the patient has been adequately resuscitated. However, these
solutions must be infused slowly because the low osmolality causes much of the fluid
volume to shift into the intracellular space following infusion, which can result in
cerebral edema.

Fresh frozen plasma (FFP) is a colloid solution that is indicated for replacement of
coagulation factors (eg, diffuse intravascular coagulation [DIC], major bleeding on
warfarin, massive transfusions). Colloids are less effective and more expensive than
crystalloid solutions and not indicated for resuscitation for severe burns.
Clinical features of compartment syndrome

• Pain out of proportion to injury


• Pain ↑ on passive stretch
Common • Rapidly increasing & tense swelling
• Paresthesia (early)

• ↓ Sensation
• Motor weakness (within hours)
Uncommon • Paralysis (late)
• ↓ Distal pulses (uncommon)

Acute compartment syndrome (ACS) occurs when excessive fluid accumulation in a


confined compartment of the body (eg, upper leg, lower leg, upper arm, forearm, hand,
foot, abdomen) causes an increase in compartment pressure to the point that blood
flow is severely impaired. Pain out of proportion to clinical findings is often the
FIRST presenting symptom, and worsening pain may be elicited with passive stretch
of the affected muscle compartment. Tissue tension, pallor, and decreased sensation
may also be present. If elevated compartment pressure is allowed to persist, tissue
ischemia and eventual tissue death will occur. The eschar that results from
a circumferential, full-thickness (third degree) burn often leads to constriction of
venous and lymphatic drainage, fluid accumulation, and resulting distal ACS.

Overall, clinical findings for ACS can be unreliable, and a high clinical suspicion
should be maintained in patients with significant injuries to the extremities, especially
the distal extremities. Serial compartment pressure monitoring is useful in determining
the need for definitive operative management, which consists of compartment
decompression by fasciotomy or, in the case of circumferential burns, escharotomy.

…….
Subcutaneous bacterial invasion may lead to cellulitis or, in the case of Clostridium
perfringens, gas gangrene with myonecrosis. However, cellulitis is typically
characterized by skin warmth and gas gangrene is characterized by crepitus.
Venous thrombosis of the upper extremity can lead to pain and swelling of the hand,
and burn trauma may increase the risk of venous thrombosis development.
However, the combination of swelling, tissue tension, and severe pain in the setting of
a circumferential, full-thickness burn with an eschar is more consistent with ACS.
EPIDERMOID  This patient has an epidermal inclusion cyst (EIC),
CYST also known as an epidermal cyst, a discrete benign
nodule lined with squamous epithelium that contains a
semisolid core of keratin and lipid. An EIC occurs when
the epidermis becomes lodged in the dermis due to
trauma or comedones, or it can arise de novo. EICs can
be seen anywhere on the body, but are MC on the face,
neck, scalp, or trunk. The lesion can gradually increase
in size and may intermittently produce a cheesy white
discharge. Some patients may develop significant
inflammation with rupture and involvement of
surrounding tissue. An EIC will usually resolve
spontaneously but can recur.

Diagnosis is made clinically with examination showing a


dome-shaped, firm, and freely movable cyst or nodule
with a central punctum (small, dilated, pore-like
opening). Excision is typically performed only for cosmetic reasons. Incision and
drainage (I&D) are occasionally needed for infected and fluctuant cysts that are
painful and erythematous (i.e., symptomatic).

……..

 BCC (Above) is the MC type of skin cancer and


usually presents as a pearly papule or plaque with
small telangiectasias on sun-exposed areas. SCC
(below) is the second MC skin cancer and also occurs
primarily on sun-exposed areas. The lesions are
usually firm and scaly papules, plaques, or nodules.

Lipomas are benign, painless subcutaneous masses with normal overlying


epidermis. In contrast to epidermal cysts, lipomas are usually soft to rubbery and
irregular, and do not typically regress and recur.
PRESSURE- A patient has an extended nursing
INDUCED INJURY facility stay with decreased mobility
and evidence of significant
neurologic injury, and now has a
stage II pressure ulcer at the heel and
a stage I lesion at the
sacrum. Besides reduced mobility,
risk factors for pressure ulcers
include malnutrition, abnormal
mental status (eg, dementia),
decreased skin perfusion, and
reduced sensation.
Standard interventions to prevent
pressure ulcers in high-risk patients
include:

• Proper patient positioning


• Mobilization
• Careful skin care
• Moisture control
• Maintenance of nutrition

Patients at risk for pressure ulcers should be cared for on beds with features that
provide pressure redistribution and reduce focal pressure. Effective modifications
include air-fluidized beds, pressure-relieving overlays (eg, sheepskin), higher-
specification foam mattresses, and active repositioning systems or alternating
pressure supports.

In addition, patients should be repositioned by the nursing staff at regular intervals


(eg, every 2 hours), although there is less evidence to support this intervention.
…….
Aspirin and statins do not prevent pressure ulcers.

Compression stockings are useful in the prevention and management of venous


insufficiency ulcers, which usually occur in the setting of venous stasis dermatitis.
Elevation of the head of the bed to >30 degrees can cause the patient to slide in the
bed, causing skin friction and INCREASEING the risk for pressure ulcers.

Tissue overlying bony prominences is particularly susceptible to pressure ulcer


formation. Massaging bony prominences is not recommended as it may further
increase tension on the overlying skin.
Malnutrition is associated with an increased risk for pressure ulcers. In patients with
otherwise normal nutritional status, however, specialized feedings and nutritional
supplements have NOT been shown to reduce the risk for pressure ulcers.
Intermittent pneumatic compression devices are used to prevent DVT in patients
with contraindications to anticoagulant therapy. They do not decrease the risk of
pressure ulcers.
This patient has typical pressure (decubitus)
ulcers. Pressure ulcers are most common
over bony prominences, such as the sacrum,
ischial tuberosities, malleoli, heels, and 1st or
5th metatarsal head. Constant, unrelieved
pressure causes necrosis of overlying skin and
muscle as blood flow to the soft tissues is
impeded. Risk factors include impaired
mobility, malnutrition, abnormal mental status
(eg, dementia), decreased skin perfusion, and
reduced sensation.
Initial management of pressure ulcers includes local wound care, repositioning of the
patient to reduce pressure, pain control, and nutritional support. Shallow ulcers can
be managed with occlusive or semipermeable dressings to maintain a moist wound
environment. Full-thickness wounds may require more complex dressings and
surgical intervention for debridement or closure.
………..
 Arterial ulcers are due to insufficient blood
supply that leads to tissue necrosis. They usually
occur in the most distal parts of the body where
blood flow is lowest, such as the tips of the toes.

Signs of bacterial infection include fever,


blanching erythema, purulence, and foul odor.
Although all pressure ulcers are colonized with
bacteria, only clinically evident infections should be
treated.
Diabetic foot ulcers result from chronic unnoticed trauma due to peripheral neuropathy
and poor wound healing due to
microvascular insufficiency. They
often occur in association
with Charcot deformity and are
MC on the soles of the feet under
the metatarsal heads and at the
tops of the toes.

 Venous stasis ulcers usually


occur in the setting of chronic
lower-extremity edema and stasis
dermatitis. They are most common
at the pretibial area or above the
medial malleolus.

HIDRADENITIS This patient has hidradenitis suppurativa (HS, also


SUPPURATIVA known as acne inversa). It most commonly occurs
in intertriginous areas (eg, axilla; inguinal, perineal
areas) but can occur in any hair-bearing skin. HS is
due to chronic inflammatory occlusion of
folliculopilosebaceous units, which prevents
keratinocytes from properly shedding from the
follicular epithelium. Risk factors include family
history of HS, smoking, obesity, diabetes, and
mechanical stress on the skin (eg, friction,
pressure).

HS initially presents as solitary, painful, inflamed nodules that can last for several
days to months. The nodules may regress or can progress to abscesses that open to
the surface with purulent or serosanguineous drainage. Most patients have a chronic,
relapsing course. Complications include sinus tracts, comedones, and scarring. Severe
scarring can lead to dense, rope-like bands in the skin with strictures and
lymphedema.
The diagnosis of HS is usually made clinically without the need for biopsy or
cultures.
…….

 Acanthosis nigricans presents as hyperpigmented plaques,


often with associated skin tags. It is common in the axillae,
groin, and neck, and is associated with obesity and
diabetes. However, it does not cause painful nodules.

 Acne vulgaris presents as closed or open comedones


that can progress to inflammatory papules, pustules, and
nodules. It most commonly affects the forehead, nose,
and chin, although it can also involve the chest and
back. However, it rarely involves the axilla.

 A furuncle is a skin abscess, usually due to S


aureus. It presents as a painful pustule or nodule,
typically draining purulent material.

 Intertrigo is due to infection with Candida species


and presents as well-defined, erythematous plaques
with satellite vesicles or pustules in intertriginous and
occluded skin areas. These disorders (fruncles,
intertrigo) typically DO NOT follow a chronic course
or cause significant scarring.

///////////////////////////HEMATOLOGY & ONCOLOGY///////////////////////////


RECTUS SHEATH
***Rectus sheath hematoma (RSH)
HEMATOMA
(RSH) • Abdominal trauma, forceful abdominal contractions (eg, coughing)
Risk
• Anticoagulation
factors • Older age, female sex

• Acute-onset abdominal pain with palpable abdominal mass


Clinical
• Blood loss anemia, leukocytosis
features • ± Nausea, vomiting, fever (misleading s/s)
• Hemodynamically stable: serial monitoring of CBC, reverse anticoagulation
Manage
and transfuse blood products when appropriate
ment • Unstable: angiography with embolization, surgical ligation

CBC = complete blood count.

A patient taking apixaban developed acute abdominal pain associated with a palpable
abdominal mass and anemia, suggesting the development of a rectus sheath
hematoma (RSH).

RSH typically occurs due to rupture of the inferior epigastric artery, which supplies
blood to the lower aspect of the rectus abdominis muscle. The rectus sheath, which
contains this muscle, does not extend posteriorly below the arcuate line; therefore,
bleeding below this line (eg, lower aspect of rectus abdominis muscle) is relatively
uncontained and can result in significant hemorrhage with hematoma formation. RSH
is often seen with blunt trauma or forceful abdominal contractions (eg,
severe coughing in asthma exacerbation), particularly in patients
receiving anticoagulation therapy. Other risk factors include older age and female
sex.

Manifestations include acute abdominal pain, often associated with rebound or


guarding, and a palpable abdominal wall mass. Tenderness may worsen with
abdominal contraction (Carnett sign). N, V, and fever may also occur. Laboratory
findings usually include anemia and leukocytosis (likely due to inflammation).
Abdominal CT confirms the diagnosis. Hemodynamically stable patients are usually
managed conservatively (eg, serial monitoring of CBC, reversal of anticoagulation when
appropriate); however, those with significant bleeding or hemodynamic instability (eg,
shock) may require angiography with embolization or surgery.

………..

Diverticular abscess also causes leukocytosis and abdominal pain and may cause a
palpable abdominal mass. However, fever and N/V usually precede the
development of the abscess. The onset is generally less acute, and anemia is
unexpected.
Ovarian torsion is associated with pain and a palpable mass; however, it causes severe
pelvic (rather than periumbilical) pain and an adnexal mass. In addition, it would not
cause anemia and is more common in women in their 20s and 30s.

Sigmoid volvulus presents with abdominal pain and vomiting. However, onset is
typically insidious (days) and is associated with severe, diffuse, abdominal pain and
tympany on examination. Blood counts are usually normal unless perforation has
occurred.
Although paraumbilical hernias can occur after abdominal surgery and may cause a
painful abdominal mass (particularly if strangulation occurs), patients typically have a
history of a palpable bulge that enlarges with increased intraabdominal pressure (eg,
coughing). Furthermore, anemia is unexpected.

VENOUS
**** Upper extremity deep venous thrombosis (DVT)
THROMBOEMBOL
ISM (VTE) • Central venous catheters (CVC)
• Repetitive arm motions (eg, baseball pitching)
Risk factors • Weight lifting
• Malignancy

• Acute arm edema, heaviness, pain & erythema


Manifestatio • Dilated subcutaneous collateral veins in chest/upper extremity (like SVC
ns syndrome but U/L)
• Pulmonary embolism (PE)

Diagnosis • Duplex or doppler USG

• 3 months of anticoagulation
Treatment • Thrombolysis (non–catheter-related)

A young patient with acute right arm pain, swelling, and heaviness likely has upper
extremity DVT. Although the vast majority of upper extremity DVTs arise in the setting
of central catheters (eg, peripherally inserted central catheter), otherwise healthy
patients can occasionally develop spontaneous DVT due to thoracic outlet
abnormalities that compress or injure the axillosubclavian vein. Young, athletic
men are at greatest risk because most cases are linked to hypertrophy of the
scalene/subclavius muscles from weight lifting or damage to the vein from
repetitive overhead arm motions (eg, baseball throwing, rowing). Congenital
abnormalities of the thorax (eg, cervical rib) can also occasionally lead to spontaneous
DVT.

Manifestations tend to arise in the dominant arm within hours of strenuous activity
(eg, pitching a baseball game). Patients usually have acute upper extremity swelling,
heaviness, and pain. The pain often improves with rest and arm elevation to the
level of the heart; arm elevation above the head typically worsens symptoms.

Diagnosis is generally made with Doppler or duplex ultrasonography. Thrombolysis


and/or 3 months of anticoagulation are generally required; surgical thoracic outlet
decompression may also be necessary.

……………….

Although low EF HCM can cause edema, most patients have lower extremity edema
and other signs of cardiac congestion (eg, dyspnea).

Upper extremity lymphedema causes unilateral arm swelling, but most cases arise
following surgical resection of axillary lymph nodes (eg, lymph node dissection for
breast cancer). Lymphedema is usually chronic, not acute.
Superficial thrombophlebitis is generally marked by localized pain and swelling along
the path of the vein. This patient with arm edema, heaviness, and pain that is worse
with exertion more likely has DVT.

////////////////////////MISCELLANEOUS (MULTISYSTEM)/////////////////////////
SHOULDER
DYSLOCATION

Normal Shoulder
This pregnant patient has hypertension, proteinuria, and a generalized tonic-
clonic seizure, findings consistent with eclampsia. Violent muscle contractions, as seen
in a seizure or electrocution injury, are a common cause of posterior shoulder
dislocation.

In a posterior dislocation, the arm is typically held in adduction and internal rotation,
with impaired external rotation, visible flattening of the anterior aspect of the
shoulder, and prominence of the coracoid process. Radiographic findings may
include:

• Loss of the normal overlap between the humeral head and the glenoid;
a normal shoulder x-ray with normal overlap is given for comparison

• Internal rotation of the humeral head, which causes a circular appearance


(light bulb sign) on anterior views
• Widened joint space >6 mm (rim sign) or 2 parallel cortical bone lines on the
medial aspect of the humeral head (trough line sign)

Potential complications include fractures of the proximal humerus, labral injuries,


and tears to the rotator cuff system. Most posterior dislocations are managed with
closed reduction.

………..

Anterior dislocation is the MC form of shoulder dislocation and is usually caused by a


direct blow or FOOS arm. In anterior dislocations, the patient holds the arm slightly
abducted and externally rotated.
Common adverse effects of magnesium sulfate include headache, N, fatigue, and
diaphoresis. Signs of magnesium toxicity include somnolence, loss of DTRs, and
respiratory depression but not focal weakness.
Todd paralysis refers to transient unilateral weakness following a tonic-clonic seizure
that usually spontaneously resolves. Adduction and internal rotation of the arm are
not seen.

Radial nerve compression from trauma can occasionally be seen in shoulder


dislocations but MCly occurs in the forearm. It often results in hand weakness and
decreased handgrip.

///////////////////POISONING & ENVIRONMENTAL EXPOSURE///////////////////


INSECT BITES A clinical presentation, with a small ulcer developing at the site of a recent bite, is
AND STINGS highly suggestive of a brown recluse spider bite. Such bites commonly occur as
patients are putting their clothes on, although they often go unrecognized initially;
over the course of a few days, a deep skin ulcer develops at the site of bite with an
erythematous halo and a necrotic center, which can progress to an eschar.

With basic wound management, the lesion often heals without


scarring. Debridement should be avoided in the early stages of necrosis but may
facilitate healing once the lesion is stable and well-demarcated; severe cases may
require skin grafting.

…………..

By contrast to brown recluse spider bites, black widow spider bites often lead to more
pronounced local and systemic manifestations due to effects of the toxin. These
include muscle pain (a prominent finding), abdominal rigidity (sometimes
mimicking a surgical abdomen), and muscle cramps (seen in >60% of patients).
Wound ulceration is uncommon, and patients commonly develop N/V within hours
of the bite.
Lyme disease: The tick must remain attached for ~48 hours before transmission
occurs.

Hemoglobinuria, bleeding, and muscle paralysis may be seen with snakebites,


which would cause severe local pain, swelling, and discoloration within hours of the
bite.
///////////////PSYCHIATRIC/BEHAVIORAL & SUBSTANCE ABUSE////////////////
ALCOHOL
*** Alcohol withdrawal syndrome
WITHDRAWAL
///////////////// Onset since
Manifestati
Symptoms/signs last drink
**** ons
(hr)

Mild Anxiety, insomnia, tremors, diaphoresis, palpitations, GI upset,


intact orientation
6-24
withdrawal

Seizures Single or multiple generalized tonic-clonic 12-48

Alcoholic
Visual, auditory, or tactile; intact orientation; stable vital signs 12-48
hallucinosis

Delirium Confusion, agitation, fever, tachycardia, HTN, diaphoresis,


hallucinations
48-96
tremens

6-24: mild withdrawal (GI, tremors, palpitations, insomnia


0.25-0.5-1-
12-48: seizures, hallucinations, stable VS
2-4 d
48-96 (2-4d): DTs (fever, hallucinations, elevated VS….)
Delayed onset (>48 hours) of severe delirium, elevated vital signs (eg, hyperthermia,
HTN, ↑HR), and diaphoresis in the setting of known multiple substance use disorders is
consistent with a diagnosis of delirium tremens (DT). DT is a late
complication of alcohol withdrawal that may affect up to 5% of patients who
experience an initial milder withdrawal syndrome. DT does not present before 48
hours after the last drink, but symptoms may take as long as 96 hours to be seen.
These symptoms may persist for up to 5 days and are associated with a mortality
rate of up to 5%.
Treatment of DT is similar to milder forms of alcohol withdrawal in that
benzodiazepines (eg, lorazepam, diazepam) are the pharmacologic treatment of
choice; however, the treatment of DT invariably requires ICU–level supportive
management.

……….

Abdominal compartment syndrome presents with wide-ranging organ dysfunction


(eg, decreased CO, increased ICP, renal impairment) in patients with increased intra-
abdominal pressure secondary to large-volume resuscitation, burns, or other
abdominal conditions (eg, ascites, liver transplantation).
Patients with an exacerbation of schizophrenia are typically not disoriented. This
patient's agitation and bizarre statements may be influenced by the psychopathology
of schizophrenia, but her disorientation, elevated vital signs, and overall clinical picture
are better explained by DT.
Cocaine intoxication may present with euphoria; autonomic hyperactivity (eg,
tachycardia, hypertension); and, at higher doses, psychosis.

Malignant hyperthermia is characterized by rigidity, rhabdomyolysis, hyperthermia, and


tachycardia immediately following the administration of succinylcholine or a volatile
inhaled anesthetic (eg, halothane, sevoflurane) in susceptible individuals
(susceptibility is inherited in an autosomal dominant pattern).

The timing of opioid withdrawal depends on the half-life of the specific opioid being
used (eg, heroin withdrawal begins approximately 12 hours after last use) and consists
of GI distress, myalgias, rhinorrhea, diaphoresis, and mild elevations of HR and BP (flu-
like). The onset of symptoms after 48 hours, delirium, and severity of this patient's
elevated vital signs are more consistent with alcohol withdrawal and DT.

Postoperative encephalopathy is most commonly associated with cardiac surgery


and presents with delirium and/or seizures in the immediate postoperative period.
Pulmonary embolism may present in the postoperative period with dyspnea and chest
pain.

BENZODIAZEPINE Worsening agitation, impaired attention, and disorientation following surgery are
S (BZs) suggestive of delirium. A number of etiologies are possible, including postoperative
delirium, medication effect, and infection. However, tremulousness, hallucinations,
and elevated vital signs (eg, HTN, tachycardia) in the context of a psychiatric history
consistent with a prescription of benzodiazepines (eg, anxiety, insomnia) are suggestive
of delirium due to benzodiazepine withdrawal. Withdrawal can occur if a patient who
has a history of prolonged use of benzodiazepines does not receive continued
medication therapy on admission to the hospital. In severe cases, benzodiazepine
withdrawal can result in seizures.

Benzodiazepines, like alcohol, exert their inhibitory effects via GABA receptors, and
their sudden withdrawal leads to an excitatory state. Sudden discontinuation can lead
to withdrawal symptoms within 24-48 hours; withdrawal symptoms from long-acting
benzodiazepines may develop over a more protracted period. Treatment of withdrawal
includes reinitiation of benzodiazepines (usually a long-acting agent) that can then
be gradually tapered down over weeks to months (and discontinued if necessary).

………….

Brief psychotic disorder is characterized by the presence of psychotic symptoms (eg,


hallucinations, delusions) for >1 day but <1 month. This disorder cannot be
diagnosed when the symptoms are better explained by another psychiatric disorder,
general medical condition, or substance intoxication/withdrawal.
This patient has a history of mild cognitive impairment, which may include some
memory loss. Although she may be at risk for developing dementia, this would not
present with acute onset of agitation and delirium.
Opioids may be used for postoperative pain. However, this patient does not exhibit
symptoms of opioid intoxication such as slurred speech, sedation, or respiratory
depression.
Although infection (eg, pneumonia) may also precipitate delirium, the patient is
afebrile and has no localized symptoms or findings on physical examination (eg, no
crackles on lung examination) that would indicate a localized infection.

Although this patient's history of hypertension and recent surgery may place her at an
increased risk for stroke, she does not have any localizing neurological findings on
physical examination, making stroke unlikely. Her presentation is more consistent with
benzodiazepine withdrawal.
USMLE Telegram Channels
MedQuest Videos: https://t.me/USMLEMedQuest
Kaplan Classroom Videos: https://t.me/USMLEKaplan
USMLE Rx Videos Step 1: https://t.me/USMLERxStep1
Becker Videos: https://t.me/USMLEBecker
Boards & Beyond Videos Step 1: https://t.me/USMLEBoardsAndBeyond
Boards & Beyond Videos Step CK: https://t.me/USMLEBoardsAndBeyondStep2CK
Physeo Videos: https://t.me/USMLEPhyseo
Pixorize Videos: https://t.me/USMLEPixorize
Pathoma Videos: https://t.me/USMLEPathoma
Goljan Audios High Quality: https://t.me/USMLEGoljanAudios
Amboss Group: https://t.me/USMLEAmboss
Lecturio QBank: https://t.me/USMLELecturio
Endpoint PDFs: https://t.me/USMLEEndopoint
USMLE NBMEs: https://t.me/USMLENBME
ONLINE MedED: https://t.me/USMLEOnlineMedED
USMLE World Step 1: https://t.me/USMLEWorldStep1
USMLE World Step 2 CK: https://t.me/USMLEWorldStep2CK
USMLE World Step 3 Qbank: https://t.me/USMLEWorldStep3
USMLE Anki: https://t.me/USMLEAnki
KissPrep: https://t.me/USMLEKissPrep

You might also like